Sie sind auf Seite 1von 308

TZALOA

Revista de la Olimpiada
Mexicana de Matematicas
2010, No. 1
Ano

Comite Editorial:
Anne Alberro Semerena
Ana Rechtman Bulajich
Carlos Jacob Rubio Barrios
Francisco Ruiz Benjumeda

Comite de la Olimpiada Mexicana de Matematicas


Cubculo 201
Departamento de Matematicas
Facultad de Ciencias, UNAM
Circuito Interior s/n
Ciudad Universitaria
Coyoacan C.P. 04510
Mexico D.F.
Telefono: (55) 56-22-48-64
www.omm.unam.mx

Diseno de Portada: Manuel Macas Beckmann


www.rayaenmedio.com

Impreso: Torre y de la Torre Impresos


Aragon no. 134

Col. Alamos,
03400
Mexico D.F.
Telefonos: (55) 55-30-14-82 y (55) 55-38-34-53

c
Queda
estrictamente prohibida la reproduccion parcial o total por cualquier sistema
o metodo, mecanico o electronico, sin autorizacion previa del autor.
Impreso y hecho en Mexico.
Enero de 2010.

Contenido

Presentacion

Artculos de matematicas: Algunas demostraciones del teorema de Pitagoras

Problemas de practica

13

Soluciones a los problemas de practica

21

Problemas propuestos
2010 No. 1
Problemas propuestos. Ano
2009 No. 3
Soluciones a los problemas propuestos. Ano

35
35
36

Concurso Nacional 2009, 23a Olimpiada Mexicana de Matematicas

43

Olimpiadas Internacionales
XXIV Olimpiada Iberoamericana
XI Olimpiada Centroamericana y del Caribe

47
47
48

Problemas y Soluciones de Olimpiadas Internacionales


50a Olimpiada Internacional

51
51

Informacion Olmpica

59

Apendice

61

Bibliografa

65

Directorio

67

IV

Contenido

Presentacion

Tzaloa es una publicacion periodica trimestral de la Olimpiada Mexicana de Matematicas y su objetivo es fomentar el estudio de las matematicas como una disciplina dinamica y creativa. El diseno de las secciones y la cuidadosa seleccion de sus contenidos
buscan apoyar de manera efectiva, con informacion y con materiales de calidad, a estudiantes y profesores de nivel medio superior que cada ano se preparan para participar
en los diferentes concursos de la Olimpiada de Matematicas.
Esta revista, con orgullo, toma su nombre del nahuatl porque esta hecha por y para los
mexicanos. Tzaloa significa aprender y las paginas que la conforman buscan ayudar a
satisfacer la necesidad de contar con espacios adecuados para profesores, estudiantes
y, en general, para todas aquellas personas interesadas en desarrollar e incrementar sus
capacidades para el razonamiento logico matematico y la resolucion de problemas.

2010, Numero

Tzaloa, Ano
1
Con este nuevo numero celebramos el primer cumpleanos de tu revista e iniciamos
nuestras actividades del 2010. Como siempre, hemos incluido secciones con interesantes problemas as como los examenes de los concursos mas importantes que se llevaron
a cabo en el u ltimo trimestre del ano pasado. La participacion de nuestros lectores se
ve reflejada en varias contribuciones que enriquecen notablemente la publicacion.
Ademas, el artculo de esta ocasion trata sobre el Teorema de Pitagoras, que aunque
es bien conocido no deja de seguir teniendo un atractivo especial. El tratamiento que
Ana Retchman hace sobre este tema, incluye aspectos historicos y algunas demostraciones clasicas que estamos seguros seran de interes para muchos. La seleccion de las
demostraciones del teorema que nos presenta el artculo, es muestra de que la solucion
de cualquier problema puede tener tantos caminos como imaginacion y cretividad hay
en la mente humana.
Por u ltimo, mencionamos que se han actualizado todas las secciones con informacion

Presentacion

VI

olmpica as como el directorio del Comite de la Olimpiada Mexicana de Matematicas.


Esperamos que este ano este lleno de logros para todos nuestros lectores y, nosotros,
refrendamos el firme compromiso de seguir poniendo todo nuestro entusiasmo, dedicacion y esfuerzo para que Tzaloa sea una revista u til para todos ustedes.

Mexico y las Olimpiadas de Matematicas


Hace mas de 23 anos que la Sociedad Matematica Mexicana ha venido impulsando
vigorosamente los trabajos de la Olimpiada Mexicana de Matematicas (OMM). Desde
sus inicios, este programa se ha visto fortalecido gracias a la participacion de miles
de jovenes estudiantes y a la entusiasta colaboracion de muchos profesores quienes,
de manera espontanea y altruista, han dedicado sus esfuerzos a mejorar la ensenanza
y elevar la cultura matematica de nuestro pas. Motivados por el movimento olmpico,
en escuelas ubicadas a lo largo de todo el territorio nacional, se han desarrollado innumerables talleres de resolucion de problemas, donde estudiantes y profesores trabajan
con el u nico afan de incrementar sus capacidades para el razonamiento, el analisis y la
creatividad matematica.
En el a mbito internacional, mediante la destacada participacion de las delegaciones
mexicanas en diversos concursos, la Olimpiada Mexicana de Matematicas ha contribuido a elevar el prestigio de la matematica nacional. Pero, mas importante aun ha sido
la contribucion que el movimiento olmpico ha tenido para el desarrollo cientfico del
pas. En muchos casos, la deteccion temprana de jovenes con talento matematico excepcional ha permitido brindarles una formacion adecuada para desarrollar al maximo
todo su potencial. Asimismo, la participacion en los concursos olmpicos ha definido
las vocaciones de muchos otros estudiantes. Universidades de todo el pas se han visto beneficiadas con el ingreso de jovenes ex-olmpicos, mismos que cuentan con una
solida formacion matematica y muchos de los cuales han permanecido en ellas para
dedicar su vida profesional a la docencia y la investigacion.

24a Olimpiada Mexicana de Matematicas


El programa anual de la Olimpiada Mexicana de Matematicas se desarrolla en 3 etapas:
Concursos Estatales.
Concurso Nacional.
Entrenamiento, seleccion y participacion de las delgaciones nacionales que representan a Mexico en concursos internacionales.
En la 24a Olimpiada Mexicana de Matematicas podran participar los estudiantes de
Mexico nacidos despues del 1 de agosto de 1991. Los concursantes deberan estar inscritos en una institucion preuniversitaria durante el primer semestre del ciclo escolar

Presentacion

VII

2010-2011 y, para el 1 de julio de 2011, no deberan haber iniciado estudios universitarios. Para mayor informacion puedes consultar la pagina:
http://www.omm.unam.mx
Para la primera etapa, los participantes deberan inscribirse directamente con el Comite Estatal correspondiente.
El Concurso Nacional de la 24a Olimpiada Mexicana de Matematicas se realizara del
21 al 26 de noviembre de 2010 en Ensenada, Baja California. A los primeros lugares
de este certamen se les invitara a la etapa de entrenamiento y seleccion de las delegaciones que representaran a Mexico en las distintas Olimpiadas Internacionales del ano
2011: la XXIII Olimpiada Matematica de la Cuenca del Pacfico, que se llevara a cabo
en el mes de marzo; la XIII Olimpiada Matematica de Centroamerica y el Caribe, que
se celebrara en el mes de junio; la 52a Olimpiada Internacional de Matematicas, que se
llevara a cabo en julio en Amsterdam, Pases Bajos, y la XXVI Olimpiada Iberoamericana de Matematicas que se realizara en el mes de septiembre en Costa Rica.

VIII

Presentacion

Algunas demostraciones del


teorema de Pitagoras
Por Ana Rechtman Bulajich
Nivel basico

El teorema de Pitagoras es sin duda uno de los mas utilizados en los problemas de
geometra de las olimpiadas. En este texto vamos a presentar varias demostraciones de
dicho teorema, y una de su recproco. Empecemos con un poco de historia.
Pitagoras nacio en la isla de Samos, en Grecia, en el ano 582 antes de nuestra era. Siendo muy joven viajo a Mesopotamia y Egipto para estudiar con los filosofos de la e poca.
Tras regresar a Samos, finalizo sus estudios con Hermodamas de Samos. Ah, fundo su
primera escuela. Posteriormente, abandono Samos y se establecio en la Magna Grecia:
en Crotona, en el sur de Italia, alrededor del ano 525 de nuestra era, donde fundo su
segunda escuela. Las doctrinas de este centro cultural eran regidas por reglas muy estrictas de conducta. Su escuela estaba abierta a hombres y mujeres indistintamente, y
la conducta discriminatoria estaba prohibida (excepto impartir conocimiento a los no
iniciados). Sus estudiantes pertenecan a todas las razas, religiones, y estratos economicos y sociales. Tras ser expulsados por los pobladores de Crotona, los pitagoricos se
exiliaron en Tarento, donde Pitagoras fundo su tercera escuela.
Pitagoras era ciertamente instruido, aprendio a tocar la lira, a escribir poesa y a recitar
a Homero. El esfuerzo para deducir la generalidad de un teorema matematico a partir de su cumplimiento en casos particulares ejemplifica el metodo pitagorico para la
purificacion y perfeccion del alma. El metodo pitagorico ensenaba a conocer el mundo como armona; en virtud de e sta, el universo era un cosmos: un conjunto ordenado
en el que los cuerpos celestes guardaban una disposicion armonica que haca que sus
distancias estuvieran entre s en proporciones similares a las correspondientes a los intervalos de la octava musical. En un sentido sensible, la armona era musical; pero su
naturaleza inteligible era de tipo numerico, y si todo era armona, el numero resultaba

Algunas demostraciones del teorema de Pitagoras

ser la clave de todas las cosas. La voluntad unitaria de la doctrina pitagorica quedaba
plasmada en la relacion que estableca entre el orden cosmico y el moral. Para los pitagoricos, el hombre era tambien un verdadero microcosmos en el que el alma apareca
como la armona del cuerpo.
Los pitagoricos atribuan todos sus descubrimientos a Pitagoras por lo que es difcil determinar con exactitud cuales resultados son obra del maestro y cuales de los discpulos.
Entre estos descubrimientos esta el teorema de Pitagoras, atribuido a los pitagoricos.
Teorema 1 (Pitagoras) Sea ABC un triangulo rectangulo, tal que el a ngulo ABC
mide 90 , entonces AB 2 + BC 2 = AC 2 .
A

El recproco del teorema de Pitagoras tambien es cierto, es decir,


Teorema 2 Sea ABC un triangulo. Si AB 2 + BC 2 = AC 2 , entonces el a ngulo ABC
mide 90 .
Estos dos teoremas se pueden escribir en un solo enunciado,
El a ngulo ABC de un triangulo ABC mide 90 si y solamente si
AB 2 + AC 2 = BC 2 .
Anteriormente a la escuela pitagorica, se conocan, en Mesopotamia y en Egipto, ternas de valores que se correspondan con los lados de un triangulo rectangulo, y se
utilizaban para resolver problemas referentes a los triangulos rectangulos. Este hecho
esta indicado en algunas tablillas y papiros, pero no ha perdurado ningun documento
que exponga la relacion para TODO triangulo rectangulo. La piramide de Kefren, que
data del siglo XXVI a.c., fue la primera gran piramide que se construyo basandose en
el llamado triangulo sagrado egipcio, cuyos lados estan en proporcion 3-4-5. Actualmente, a las ternas de numeros naturales que pueden ser las longitudes de los lados de
un triangulo rectangulo se les llama ternas pitagoricas: una terna de numeros (a, b, c)
es pitagorica si a2 + b2 = c2 .

Demostraciones del teorema de Pitagoras.


El matematico estadounidense E. S. Loomis, catalogo 367 pruebas diferentes en su
libro The Pythagorean Proposition. Aqu vamos a explicar algunas de ellas. Empezaremos por las demostraciones que se les atribuyen a algunos matematicos griegos, y
dejaremos al final las demostraciones graficas que son las mas conocidas actualmente.
Demostracion que algunos autores adjudican a Pitagoras

Algunas demostraciones del teorema de Pitagoras

Esta es una demostracion que utiliza la semejanza de triangulos1. Primero, necesitamos


demostrar la siguiente proposicion basica, que nos sera u til en otras demostraciones.
Proposicion 3 (Suma de los a ngulos internos de un triangulo) La suma de los a ngulos internos de un triangulo es igual a 180 .
Sea EDF un triangulo cualquiera. Tracemos la recta ST paralela a DF que pasa por
E.
S

Entonces, la relacion de los a ngulos entre paralelas2 implica que


EDF + DF E + F ED = SED + T EF + F ED,
que es claramente igual a 180. Esto demuestra la proposicion.
Regresemos ahora a la demostracion del teorema de Pitagoras. Tomemos un triangulo
rectangulo ABC, y sea P la base de la altura del triangulo ABC sobre la hipotenusa.
A
P

Observemos que el a ngulo ABP es igual a BCP , ya que como la suma de los
a ngulos internos de un triangulo es igua a 180,
ABP = 180 (P AB + 90 ) = BCP.
Analogamente,
P BC = 180 (BCP + 90 ) = P AB.
Por el criterio AA de semejanza3 , tenemos que los triangulos ABC, BP C y AP B son
1 Ver

en el apendice la definicion SEMEJANZA.

en el apendice el teorema ANGULOS


ENTRE PARALELAS.
3 Ver en el ap
endice el criterio AA.

2 Ver

Algunas demostraciones del teorema de Pitagoras

semejantes. Entonces,
AC
AB
=
AB 2 = AC AP
AP
AB
AC
BC
=
BC 2 = AC P C.
PC
BC
Luego, AB 2 + BC 2 = AC(AP + P C) = AC 2 .
Otra demostracion utilizando semejanzas de triangulos
Empecemos considerando dos triangulos rectangulos semejantes EDF y EGH.
H
F

Queremos demostrar que la razon entre las a reas de los triangulos es igual al cuadrado
de su razon de semejanza. La razon de semejanza es igual a
ED
DF
=
.
EG
GH
Como los triangulos son rectangulos tenemos que
r=

a rea(EDF )
=
a rea(EGH)

EDDF
2
EGGH
2

= r2 .

Es decir, la relacion entre las a reas de dos triangulos rectangulos semejantes es igual al
cuadrado de su relacion de semejanza.
Regresemos a nuestro triangulo ABC, con P la base de la altura sobre la hipotenusa.
Vamos a volver a demostrar el teorema de Pitagoras.
A
P

Sabemos que los triangulos BP C y AP B son semejantes, luego


2

BC
a rea(BP C)
,
=
a rea(AP B)
AB

Algunas demostraciones del teorema de Pitagoras

entonces
a rea(AP B)
a rea(BP C)
=
.
2
BC
AB 2
Escrito de otra forma,
AB 2 [area(BP C)]
AB 2 [area(BP C) + a rea(AP B)]
a rea(BP C) + a rea(AP B)
AB 2 + BC 2

= BC 2 [area(AP B)]
= [area(AP B)](AB 2 + BC 2 )
a rea(AP B)
a rea(BP C)
=
=
.
AB 2
BC 2

Por otro lado, como los triangulos BP C y ABC son semejantes tenemos que
a rea(BP C)
a rea(BP C) + a rea(AP B)
a rea(ABC)
=
=
.
2
2
AC
BC
AB 2 + BC 2
Como a rea(ABC) = a rea(BP C) + a rea(AP B), concluimos que
AC 2 = AB 2 + BC 2 .

La demostracion de Euclides
El filosofo Euclides, quien vivio alrededor del ano 300 antes de nuestra era, es considerado el padre de la geometra del plano, que en su honor es conocida como geometra
euclidiana. La demostracion que vamos a explicar es la que aparece en su libro Los
Elementos. Para la demostracion necesitamos probar el siguiente resultado, que utilizaremos tambien en otras demostraciones.
Proposicion 4 Un paralelogramo ABCD y un triangulo EBC tienen la misma base
BC y el vertice E esta en la recta que contiene a AD. Entonces el a rea del paralelogramo es el doble que el a rea del triangulo.
A

La demostracion de esta proposicion es muy sencilla. Sea h la distancia que hay entre las rectas paralelas que contienen los segmentos AD y BC. Entonces, el a rea del
paralelogramo es igual a BC h y el a rea del triangulo EBC es igual a BCh
, sin
2
importar donde este el punto E, siempre y cuando este en la recta que contiene AD.
Demostremos ahora el teorema de Pitagoras. Sea ABC un triangulo rectangulo, cuyo
a ngulo recto es ABC. Dibujemos en cada lado del triangulo un cuadrado.

Algunas demostraciones del teorema de Pitagoras

H
D

Como BC es paralela a AD, tenemos que los triangulos ADB y ADC tienen la misma
a rea (esto es una consecuencia de la proposicion anterior). Ahora, si rotamos el triangulo ADC en el vertice A, de forma que el lado AC coincida con AI y AD con AB,
concluimos que los triangulo ADC y ABI son congruentes4. En particular, tienen la
misma a rea. Por lo tanto, ADB y ABI tienen las mis a rea.

Tracemos la recta paralela a AI por B, y sea P su punto de interseccion con AC.


La proposicion 4 implica que los triangulos ABI y AP I tienen la misma a rea, ya
que BP es paralela a AI. Entonces, los trianglos ADB y AP I tienen la misma a rea.
Observemos que el a rea del cuadrado ADEB es igual a dos veces el a rea del triangulo
ADB e igual a AB 2 . Ademas, el a rea de AP QI es dos veces el a rea del triangulo
AP I, que tambien es igual al a rea de ADEB, y que tambien es igual a AB 2 .

4 Ver

en el apendice la definicion CONGRUENTES.

Algunas demostraciones del teorema de Pitagoras

Q
H
A

Haciendo lo mismo para el rectangulo P CHQ, concluimos que su a rea es igual a BC 2 .


Por lo tanto, AC 2 que es igual al a rea del cuadrado ACHI, es igual a la suma de las
a reas de AP QI y P CHQ, que es igual a AB 2 + BC 2 . Es decir, AC 2 = AB 2 + BC 2 .

AB 2
A

BC 2

AB 2
C

B
BC 2

La demostracion de Pappus
Pappus nacio en Alejandra, de ah que se le conoce como Pappus de Alejandra, apro-

Algunas demostraciones del teorema de Pitagoras

ximadamente en el ano 290 de nuestra era y murio alrededor del 350. Es considerado
como el u ltimo de los grandes geometras griegos. La demostracion de Pappus esta tambien basada en la proposicion 4, y la describiremos a continuacion.
Sea ABC un triangulo rectangulo y dibujemos en cada uno de sus lados un cuadrado.
Sea P la base de la altura sobre la hipotenusa, y continuemos esta recta en las dos
direcciones: por un lado hasta el punto Q, que es la interseccion con HI; y por otro
lado hasta T que es la interseccion con la prolongacion de F G. Como EC es paralela
a F G y AF es paralela a DE, el punto T esta en la interseccion de las rectas F G y
DE.
I

Q
H
D

A
P

La proposicion 4 implica que los paralelogramos ADEB y AA T B tienen la misma


a rea. Ademas, los lados el triangulo BET cumplen que AB = BE, BC = ET
y AC = BT . Luego, BT = AA = AC = AI y los paralelogramos AA T B y
AP QI tienen la misma a rea. Analogamente, concluimos que BT C C y P CHQ tienen
la misma a rea. Calculando estas a reas en terminos de los lados del triangulo ABC
concluimos que AB 2 + BC 2 = AC 2 .

Algunas demostraciones graficas

Como siempre tomemos un triangulo rectangulo ABC. Llamemos a, b y c a las longitudes de los lados AB, BC y AC, respectivamente. En la figura, los cuatro triangulos
son congruentes entre si y congruentes a ABC.

Algunas demostraciones del teorema de Pitagoras

b
a
c

Entonces, tenemos que el a rea del cuadrado grande es igual a la suma de las a reas de
los cuatro triangulos mas el a rea del cuadrado pequeno, es decir,

c2

ab
2

+ (b a)2

2ab + b2 2ab + a2 = a2 + b2 .

Lo que demuestra el teorema de Pitagoras.


Dejamos como ejercicio para el lector desarrollar los razonamientos que pueden acompanar las siguientes figuras para demostrar el teorema de Pitagoras.
Ejercicio 1. En cada figura, los cuatro triangulos son congruentes con catetos a, b e
hipotenusa c.
a

b
a

b
c
b

a
b

Ejercicio 2. En cada figura, los cuatro triangulos son congruentes con catetos a, b e
hipotenusa c. Demuestra que las dos figuras tiene la misma a rea (la de la primera es
igual a c2 y la de la segunda a a2 + b2 ).

10

Algunas demostraciones del teorema de Pitagoras

ba

b
a

c
b

Ejercicio 3. En la figura, el punto O es el centro del cuadrado de lado BC (podra ser


otro punto?) y se trazaron por este punto una recta paralela a la hipotenusa y otra perpendicular a la hipotenusa. Si recortas este cuadrado por dichas rectas obtienes cuatro
piezas. Acomoda estas cuatro piezas y el cuadrado de lado AB dentro del cuadrado de
lado AC, para encontrar otra demostracion del teorema de Pitagoras.

B
Ob

Demostracion del recproco del teorema de Pitagoras


Vamos a dar una demostracion del recproco del teorema de Pitagoras, que es la demostracion que aparece en el libro Los Elementos de Euclides.
Sea ABC un triangulo tal que AB 2 + BC 2 = AC 2 , vamos a demostrar que el a ngulo
ABC es recto. Tracemos el segmento DB que mide lo mismo que BC y es perpendicular a AB.

Algunas demostraciones del teorema de Pitagoras

11

Como ABD es un triangulo rectangulo tenemos que AD2 = AB 2 + DB 2 = AB 2 +


BC 2 = AC 2 , es decir AD = AC. Luego, los triangulos ABC y ABD son congruentes pues sus lados miden lo mismo5 . Esto implica que ABC = ABD y el triangulo
ABC es rectangulo.
Ejercicio 4. Encuentra otra demostracion del recproco del teorema de Pitagoras.

Ejercicios
Los problemas de practica 8, 10 y 18 de este numero utilizan el teorema de Pitagoras.
Te invitamos a resolverlos.

Bibliografa
1.- E. S. Loomis, The Pythagorean Proposition. NCTM. Michigan, 1940.
2.- Animacion (en frances): http://www.mathkang.org/swf/pythagore2.html.

5 Ver

en el apendice el CRITERIO CONGRUENCIA LLL

12

Algunas demostraciones del teorema de Pitagoras

Problemas de practica
Como su nombre lo indica, en esta seccion presentamos problemas cuyo objetivo es
brindar material que te permita trabajar en tu preparacion para participar en los concursos. La dificultad de los problemas seleccionados aumentara conforme transcurra el
ano, de manera que la revista te acompanara a medida que tus habilidades y capacidades vayan aumentando.
Decidimos iniciar el ano con 30 problemas presentados con el formato de opcion multiple. La eleccion del formato obedece a que en las primeras etapas de muchos concursos
estatales los problemas se presentan as. De cualquier manera, es importante que no
solo te limites a determinar la respuesta por eliminacion de las opciones incorrectas,
sino que, ademas, intentes encontrar la justificacion o procedimientos que permiten llegar a la solucion. Esto es muy importante porque en etapas mas avanzadas los examenes
no se presentan con este formato y entonces deberas llegar a la respuesta sin la ayuda
que brindan las opciones.
Como se menciono arriba, buscamos que el nivel de dificultad de esta primera seleccion del ano no fuera muy elevado, sin embargo, esto no quiere decir que todos los
problemas vayan a ser faciles de resolver. Te invitamos a poner en practica todas tus
habilidades y a hacer uso de todos tus conocimientos para encontrar la solucion de cada uno. En la siguiente seccion de la revista encontraras las respuestas de todos ellos,
pero te recomendamos que no la consultes sino hasta despues que hayas llegado por ti
mismo a la solucion. Ten en cuenta que la clave para mejorar tus capacidades esta en
la perseverancia y el esfuerzo.
Por u ltimo, te invitamos a contribuir para que esta seccion de tu revista se siga enriqueciendo con la participacion de todos. Estamos seguros que concoces y tienes
problemas interesantes que proponer, por eso ponemos a tu disposicion la direccion
revistaomm@gmail.com, donde con gusto recibiremos tus sugerencias.

14

Problemas de practica

Problema 1. Cuantos caminos hay de A a B siguiendo las lneas de la figura, si no se


permite caminar horizontalmente hacia la izquierda, ni bajar?
B

A
(a) 16

(b) 2

(c) 30

(d) 22

(e) 20

Problema 2. Cual es el menor entero positivo formado solo por dgitos 1 y 0 que es
divisible entre 15?
(a) 10

(b) 110

(c) 1, 110

(d) 11, 110

(e) 111

Problema 3. Se tiene una sucesion de numeros enteros a1 , a2 , a3 , . . . , tal que si un


termino es par, entonces el siguiente es la mitad del anterior y si el termino es impar,
entonces el siguiente es la suma de los 2 anteriores. Si el primer termino de la sucesion
es 2010, que numero esta en el lugar 2011?
(a) 4020

(b) 1005

(c) 2010

(d) 3015

(e) 2011

Problema 4. En la figura vemos una pared de una chimenea de base cuadrada. Si no


se corto ningun ladrillo para construir la chimenea, cuantos ladrillos se utilizaron para
construirla?

(a) 30

(b) 26

(c) 52

(d) 10

(e) 13

Problema 5. Cuanto mide el a ngulo menor que se forma entre las dos manecillas de
un reloj cuando e ste marca las 4:36?
(a) 91

(b) 78

(c) 75

(d) 96

(e) 45

Problemas de practica

15

Problema 6. En un triangulo rectangulo ABC esta inscrito un cuadrado como se muestra en la figura. Si sabemos que AB = 6 cm y BC = 3 cm, cuanto mide el a rea del
cuadrado?
C

A
(a) 6 cm2

(b) 9 cm2

(c) 4 cm2

(d) 8 cm2

(e) 16 cm2

Problema 7. Un numero de telefono es de la forma ABC DEF GHIJ, donde


cada letra representa un dgito distinto. Se sabe que A > B > C; D > E > F ;
G > H > I > J; D, E, F son dgitos pares consecutivos; G, H, I, J son dgitos impares consecutivos y A + B + C = 9. Cuanto vale A?
(a) 5

(b) 6

(c) 7

(d) 8

(e) 9

Problema 8. Sean ABCD un rectangulo con AB < BC y M , N los puntos medios


de los lados CD y DA, respectivamente. Si el a ngulo BN M es recto y AB = 6 cm,
cuanto mide BC?

(a) 6 2 cm

(b) 6 cm

(c) 4 2 + 2 cm

(d) 3 2 cm

(e) 3 + 2 2 cm

Problema 9. Un piso de 8 m 10 m esta cubierto por mosaicos de 50 cm 50 cm,


como el de la figura. Cual es el total de la superficie blanca?

(a) 20, 000(4 ) cm2


(d) 150, 000 cm2

(b) 200, 000(4 ) cm2


(e) 200, 000( 3) cm2

(c) 60, 000 cm2

Problema 10. A partir de un cuadrado se forma un octagono regular cortando un


triangulo rectangulo isosceles de cada esquina. Si cada lado del cuadrado mide 20 cm,
cuanto mide cada lado del octagono?
(a) 12 cm

(b) 20( 2 1) cm

(c) 10 2 cm

(d) 16 cm

(e) 25 2 12 cm

16

Problemas de practica

Problema 11. Cuantos numeros distintos pueden ser expresados como la suma de tres
numeros distintos del conjunto {1, 4, 7, 10, 13, 16, 19}?
(a) 37

(b) 36

(c) 18

(d) 42

(e) 13

Problema 12. Sean A2 y B4 dos numeros de dos dgitos tales que al sumarlos se obtiene un multiplo de 3. Cuantas soluciones se pueden obtener para A y B?
(a) 28

(b) 30

(c) 25

(d) 27

(e) 36

Problema 13. Cual es el menor entero positivo por el cual hay que dividir al numero
108, 675 para que el cociente sea un cuadrado perfecto?
(a) 805

(b) 543

(c) 483

(d) 110

(e) 161

Problema 14. Si a un numero de dos dgitos le sumamos el cuadrado de la suma de sus


dgitos, obtenemos el numero original con los dgitos invertidos. Cual es la suma de
todos los numeros que satisfacen las condiciones del problema?
(a) 72

(b) 37

(c) 63

(d) 36

(e) 27

Problema 15. La base mayor de un trapecio tiene longitud de 15 cm y el segmento de


recta que une los puntos medios de las dos diagonales tiene longitud de 3 cm. Cuanto
mide la base menor del trapecio?
(a) 6 cm

(b) 9 cm

(c) 10 cm

(d) 12 cm

(e) 11 cm

Problema 16. La siguiente sucesion se forma al escribir los dgitos de los numeros
naturales en orden
1, 2, 3, 4, 5, 6, 7, 8, 9, 1, 0, 1, 1, 1, 2, ...
Cual es el dgito en el lugar 2010?
(a) 6

(b) 1

(c) 3

(d) 0

(e) 2

Problema 17. Un crculo de a rea A1 esta contenido en el interior de un crculo mayor


de a rea A1 + A2 . Si el radio del crculo mayor es 3 cm, y si A1 , A2 , A1 + A2 forman
una progresion aritmetica, cuanto mide el radio del crculo menor?
(a)

3
2

cm

(b)

2
8

cm

(c)

3
2

cm

(d)

3 cm

(e)

6
2

cm

Problema 18. Si el a rea del crculo es 1 m2 , cuanto mide el a rea del triangulo ABC?

Problemas de practica

17

O
b

A
(a)

3
2

m2

(b)

3
2

m2

B
(c)

3
4

(d) 2 3 m2

m2

(e)

m2

Problema 19. Supongamos que tenemos 21 monedas, de las cuales 20 son originales
y una es falsa. La moneda falsa tiene distinto peso, pero no sabemos si pesa mas o
menos. Cual es el mnimo numero de pesadas que se deben hacer en una balanza para
saber si la moneda falsa pesa mas o pesa menos? (No es necesario especificar cual es
la moneda falsa, u nicamente queremos saber si pesa mas o menos.)
(a) 1

(b) 2

(c) 3

(d) 4

(e) 5

Problema 20. La longitud de la hipotenusa del triangulo rectangulo ABC es h, y el


radio del crculo inscrito es r. Cual es la razon entre el a rea del crculo y el a rea del
triangulo?
C

r
O

A
(a)

r
h+2r

(b)

r
h+r

B
(c)

r
2h+r

(d)

r 2
h2 +r 2

(e)

r 2
h+r

Problema 21. Si a2 = a + 2, entonces a3 es igual a


(a) a + 4

(b) 2a + 8

(c) 3a + 2

(d) 4a + 8

(e) 27a + 8

Problema 22. Encuentra el valor de


r
1 2 4 + 2 4 8 + 3 6 12 + 4 8 16 +
3
1 3 9 + 2 6 18 + 3 9 27 + 4 12 36 +
(a) 1

(b)

2
3

(c)

3
4

(d)

1
2

(e)

1
4

18

Problemas de practica

Problema 23. En un condominio, todos los numeros de telefono son de la forma


555 abc def g, donde a, b, c, d, e, f y g son dgitos distintos en orden creciente,
y ninguno de ellos es 0 o 1. Cuantos numeros de telefono se pueden formar?
(a) 1

(b) 2

(c) 7

(d) 8

(e) 9

Problema 24. Sea ABCD un rectangulo de base BC = 2 m y altura AB = 1 m. Si


L y M son los puntos medios de AD y BC respectivamente, determina el a rea del
cuadrilatero EF GH.
A

F
E

G
H

(a)

1
12

m2

(b)

1
24

m2

(c)

1
4

C
m2

(d)

1
2

m2

(e)

2
3

m2

Problema 25. Cuantos enteros positivos n menores o iguales que 24 cumplen que
n!
1 + 2 + + n
es un numero entero?
(a) 8

(b) 12

(c) 16

(d) 17

(d) 21

Problema 26. En la siguiente figura se muestra un hexagono regular cuya a rea es a.


Cual es el a rea del triangulo ABC?

B
C

(a)

a
2

(b)

a
3

(c)

a
6

(d)

a
12

(e)

a
8

Problemas de practica

19

Problema 27. En la expresion c ab d, los valores de a, b, c y d son los numeros


0, 1, 2 y 3, aunque no necesariamente en ese orden. Cual es el mayor valor posible de
tal expresion?
(a) 5

(b) 6

(c) 8

(d) 9

(e) 10

Problema 28. Paty tiene 20 monedas de las cuales algunas son de 5 centavos y otras de
10 centavos. Si sus monedas de 5 centavos fueran de 10 centavos y sus monedas de 10
centavos fueran de 5 centavos, ella tendra 70 centavos mas. Cuanto dinero tiene Paty?
(a) $1.15

(b) $1.20

(c) $1.25

(d) $1.30

(e) $1.35

Problema 29. Cuantos cubos positivos dividen a 3! 5! 7!?


(a) 2

(b) 3

(c) 4

(d) 5

(e) 6

Problema 30. En un trapecio ABCD, se tiene que AB es paralela a DC, E es el punto


medio de BC, y F es el punto medio de DA. Si el a rea del trapecio ABEF es el doble
AB
es:
del a rea del trapecio F ECD, entonces el valor de DC
(a) 2

(b) 3

(c) 5

(d) 6

(e) 8

20

Problemas de practica

Soluciones a los problemas de


practica

En esta seccion te presentamos las soluciones que hemos preparado para los 30 problemas de practica que figuran en este numero de tu revista. Date cuenta que no solo se ha
determinado la opcion correcta, sino que ademas, para cada problema se incluye la argumentacion que establece su validez. Observa que, en todos los casos, la justificacion
se basa en resultados conocidos y/o en razonamientos logicos y que en ningun problema la solucion se ha determinado por eliminacion de las otras opciones de respuesta.
Las soluciones que aqu se presentan no pretenden ser u nicas ni tampoco las mejores
o las mas elegantes, tan solo son ejemplos que muestran el tipo de razonamiento que
se busca estimular en los problemas de la olimpiada. Como puedes ver en el artculo
que presentamos sobre el Teorema de Pitagoras, cada problema puede tener un gran
numero de soluciones. Si encontraste una solucion diferente de las que aqu se presentan y no estas seguro de su validez o simplemente quieres compartirla con nosotros te
invitamos para que nos escribas a revistaomm@gmail.com.

Solucion del problema 1. La respuesta es (d). Una manera de contar los caminos es
contar los caminos a cada vertice (o interseccion) y luego ir sumando. Numeremos los
vertices (el del centro es a5 ):

22

Soluciones a los problemas de practica

a7

a8

a4

a6
a1

a2

a3

A cada vertice podemos llegar a lo mas desde tres direcciones: horizontalmente desde
su izquierda, desde abajo a la derecha o desde abajo a la izquierda. Observemos primero
que para llegar a a1 , a2 o a3 tenemos un solo camino (el horizontal). Para llegar a a4
hay dos caminos: el camino directo desde A y el que pasa por a1 . Para contar los
caminos que llegan a a5 tenemos que sumar los que pasan por a4 , mas los que pasan
por a1 , mas los que pasan por a2 . Luego, hay 2 + 1 + 1 = 4 caminos de A a a5 .
Continuando este proceso tenemos que de A a:
a6 hay 6 caminos;
a7 hay 6 caminos;
a8 hay 16 caminos;
B hay 22 caminos.
Solucion del problema 2. La respuesta es (c). Para que un numero sea divisible entre
15 tiene que ser divisible entre 3 y 5 (ver criterios 2 del apendice). Un numero cuyos
u nicos dgitos son 0 y 1 es divisible entre 5 si el dgito de las unidades es 0, y es
divisible entre 3 si el numero de dgitos 1 es un multiplo de 3. Por lo tanto, el menor
numero que cumple con las condiciones del problema es el 1, 110.
Solucion del problema 3. La respuesta es (d). Como a1 = 2010 es un numero par,
entonces a2 = 2010
= 1005. Como a2 = 1005 es impar, entonces a3 = a1 + a2 =
2
3015. Como a3 = 3015 es impar, entonces a4 = a2 + a3 = 4020. Por u ltimo, como
a4 = 4020 es par, entonces a5 = 2010. A partir de este punto la sucesion se vuelve
cclica, por lo que para determinar el valor de cualquier termino de ella, basta con
fijarnos en el valor del residuo que salga al dividir el ndice del termino entre 4. Sea r
el residuo que resulte de dividir n4 .
si r = 1, entonces an = 2010,
si r = 2, entonces an = 1005,
si r = 3, entonces an = 3015,

Soluciones a los problemas de practica

23

si r = 0, entonces an = 4020.
Como 2011 = (4 502) + 3, entonces r = 3 y a2011 = 3015.
Solucion del problema 4. La respuesta es (a). Veamos que cada una de las lneas tiene
6 ladrillos, como en la figura.

Como hay 5 lneas de ladrillos, se utilizaron 6 5 = 30 ladrillos para construir la


chimenea.
Solucion del problema 5. La respuesta es (b). El dial de un reloj esta dividido en 60
marcas correspondientes a los 60 minutos de 1 hora. El arco de crculo entre dos de

estas marcas consecutivas equivale a un a ngulo de 360


60 = 6 . Para facilitar la explicacion, supondremos que para las marcas correspondientes a los minutos multiplos de 5,
la caratula de nuestro reloj tiene la numeracion tradicional, del I al XII.
Es facil ver que cuando el reloj marca las 4:30 (posicion incial), las manecillas estan
formando un a ngulo de 45 , ya que la manecilla de los minutos se encuentra exactamente en la marca VI y la manecilla de las horas esta exactamente a la mitad entre las
marcas IV y V. Notese que entre las marcas V y VI hay un a ngulo de 30 y que el
a ngulo entre la manecilla de las horas y la marca V es de 15 .
A partir de ese momento y hasta las 4:36, la manecilla de los minutos avanzara un
a ngulo de 6 6 = 36 . Por otro lado, la manecilla de las horas avanza mas lento
con un ritmo de 5 6 = 30 en 60 minutos, por lo que en 6 minutos avanzara solo
1

10 30 = 3 . De esta manera, mientras la manecilla de los minutos avanza 36 de la

posicion incial, la de las horas avanza 3 , por lo que en 6 minutos la separacion efectiva
entre ambas es de 36 3 = 33 . De lo anterior se concluye que a las 4:36 el a ngulo
que separa a las manecillas es de 45 + 33 = 78 .
Solucion del problema 6. La respuesta es (c). Llamemos P , Q y R a los vertices del
cuadrado tal y como se muestra en la figura.
C
Q

24

Soluciones a los problemas de practica

Dado que QR es paralela con AB, tenemos que CQR = CAB ya que son a ngulos
correspondientes (ver la definicion 8 del apendice). Como QCR = ACB por ser
un a ngulo comun y ademas QRC = ABC = 90 , por el criterio AA (ver el criterio
17 del apendice), tenemos que los triangulos ABC y QRC son semejantes.
Sean P B = BR = x y RC = 3 x, entonces como los triangulos ABC y QRC
son semejantes, tenemos que sus lados correspondientes son proporcionales, es decir
6
x
3x = 3 , de donde se sigue que x = 2(3 x) y por lo tanto x = 2 cm. Con lo anterior
concluimos que el a rea del cuadrado P QRB es 2 2 = 4 cm2 (ver el teorema 7 del
apendice).
Solucion del problema 7. La respuesta es (d). Como G, H, I, J son impares consecutivos, un solo dgito entre A > B > C es impar y es igual a 1 o 9. Como A+B +C = 9,
tenemos que uno es igual a 1 y la suma de los otros dos es 8. Por otro lado tenemos que
D, E, F son iguales a 0, 2, 4; 2, 4, 6 o 4, 6, 8, respectivamente. En cada caso los dos
dgitos pares entre A, B, C son los dos dgitos restantes, como su suma es 8 la u nica
posibilidad es A = 8, B = 1 y C = 0.
Solucion del problema 8. La respuesta es (a). Como el a ngulo BN M = 90 , el
triangulo BM N es rectangulo.
A

D
3

Denotemos por y a la medida de AN = N D. Utilizando el teorema de Pitagoras (ver


el teorema 11 del apendice) en los triangulos rectangulos ABN y M DN , tenemos que
BN 2
MN2

= 36 + y 2 ,
= 9 + y2.

Utilizando nuevamente el teorema de Pitagoras en los triangulos rectangulos BM N y


BCM , obtenemos
BN 2 + M N 2 =
36 + y 2 + 9 + y 2

BM 2 = BC 2 + CM 2 ,
=
4y 2 + 9,

ya que BC = 2y. Luego,2y 2 = 36 y y = 18 = 3 2 cm y el lado BC mide 6 2 cm.


Solucion del problema 9. La respuesta es (b). Los radios de los arcos de crculo de
cada mosaico miden 25 cm. Observemos que los 4 cuartos de crculo de cada mosaico,
cubren la misma superficie que un crculo de radio 25 cm.

Soluciones a los problemas de practica

25

Luego, en cada mosaico el a rea blanca es igual al a rea del cuadrado menos el a rea del
crculo (ver el teorema 7 del apendice), es decir, es igual a
502 (252 ) = 625(4 ) cm2 .
Ahora bien, como el piso tiene 80 m2 = 800, 000 cm2 y cada mosaico cubre una
superfice de 2, 500 cm2, se necesitan 800,000
2,500 = 320 mosaicos para cubrir el piso. Por lo
tanto, el total de la superficie blanca es igual a 320[625(4)] = 200, 000(4) cm2.
Solucion del problema 10. La respuesta es (b). Llamemos x a la longitud de los catetos
de los triangulos isosceles y y a la longitud de cada lado del octagono. Luego, 2x+ y =
20 cm.

y
x
Entonces, cada triangulo rectangulo isosceles tiene lados x, x, y. Utilizando el teorema

de Pitagoras (ver el teorema 11 del apendice), tenemos que 2x2 = y 2 , o bien y = 2x.
Sustituyendo en la primera ecuacion tenemos que

2x + 2x = 20,

20
20(2 2)
=

= 20 10 2 cm.
x =
2+ 2
(2 + 2)(2 2)
Luego,
y

2x = 2(20 10 2) = 20( 2 1) cm.

26

Soluciones a los problemas de practica

Por lo tanto, cada lado del octagono mide 20( 2 1) cm.


Solucion del problema 11. La respuesta es (e). Observemos que el numero mas pequeno que podemos obtener es el 12 = 1+4+7 y el mas grande es el 48 = 13+16+19.
Veamos cuales numeros intermedios podemos escribir como suma de tres numeros distintos del conjunto {1, 4, 7, 10, 13, 16, 19}.
Los numeros en el conjunto son de la forma 1 + 3n con n = 0, 1, . . . , 6, entonces la
suma de cualesquiera tres de ellos es igual a
(1 + 3n) + (1 + 3m) + (1 + 3k) = 3 + 3(n + m + k) = 3(n + m + k + 1),
con n, m, k {0, 1, . . . , 6} y distintos entre s. Entonces, tenemos que
12 3(n + m + k + 1) 48
3

n+m+k

15.

Observemos que podemos encontrar n, m y k tales que al sumarlos obtenemos todos


los valores enteros entre 3 y 15:
3=0+1+2
4=0+1+3
5=0+1+4
6=0+1+5
7=0+1+6
8=0+2+6
9 = 0 + 3 + 6.

10 = 0 + 4 + 6
11 = 0 + 5 + 6
12 = 1 + 5 + 6
13 = 2 + 5 + 6
14 = 3 + 5 + 6
15 = 4 + 5 + 6

Por lo tanto, todos los multiplos de 3 entre 12 y 48 pueden ser suma de tres numeros
distintos del conjunto, es decir, en total hay 13 valores posibles.
Solucion del problema 12. La respuesta es (d). La solucion se basa en el criterio de
divisibilidad entre 3, el cual establece que un numero es divisible entre 3 si y solo si
la suma de sus dgitos tambien lo es. Como 4 + 2 = 6 es divisible entre 3, entonces
A + B tambien tiene que ser divisible entre tres. Por inspeccion, es facil determinar
que existen 15 parejas de dgitos tales que su suma es divisible entre tres: (1, 2), (1, 5),
(1, 8), (2, 4), (2, 7), (3, 3), (3, 6), (3, 9), (4, 5), (4, 8), (5, 7), (6, 6), (6, 9), (7, 8) y
(9, 9). Quitando las parejas (3, 3), (6, 6) y (9, 9), el resto de ellas debe contarse doble
pues, por ejemplo, de la pareja (1, 2) salen 2 soluciones A = 1, B = 2 y A = 2,
B = 1. Por lo tanto, el numero total de soluciones es 3 + (2 12) = 27.
Solucion del problema 13. La respuesta es (c). Observemos que
108, 675 = 33 52 7 23.
Como en la descomposicion en primos de un cuadrado perfecto todos los exponentes
son pares, entonces el cociente que queremos obtener es 32 52 , ya que es el mayor
cuadrado perfecto que podemos obtener. Luego, se debe dividir entre 3 7 23 = 483.

Soluciones a los problemas de practica

27

Solucion del problema 14. La respuesta es (e). Denotemos por ab al numero de dos
dgitos que satisface las condiciones del problema, es decir,
10a + b + (a + b)2

10b + a,

9(b a).

(a + b)

Como a es distinto de cero, (b a) < 9 y tenemos que (a + b)2 = 9(b a) < 81.
Los u nicos cuadrados perfectos multiplos de 9 que son menores a 81 son: 9 = 32 y
36 = 62 . Veamos cada caso:
1. si (a + b) = 3 entonces b a = 1, tenemos que a = 1, b = 2 y 10a + b = 12;
2. si (a + b) = 6 entonces b a = 4, tenemos que a = 1, b = 5 y 10a + b = 15.
Por lo tanto, los u nicos numeros que cumplen la condicion son 12 y 15 y su suma es
27.
Solucion del problema 15. La respuesta es (b). Sea ABCD el trapecio, donde AB es
paralela a CD, con CD la base mayor. Sea X el punto medio de la diagonal BD y
sea G el punto medio de BC. Entonces por el teorema de Thales (ver el teorema 9 del
apendice), XG es paralela a DC. Extendemos la recta XG hasta que intersecte a AD
en F y llamemos Y a la interseccion con AC.
A

Utilizando nuevamente el teorema de Thales, tenemos que F es punto medio de AD y


Y es punto medio de AC. Por hipotesis, tenemos que XY = 3 cm. Si nos fijamos en
el triangulo BDC, tenemos (ver el lema 12 del apendice) que XG = 12 DC = 15
2 cm.
9
Como XG = XY + Y G = 3 + Y G, tenemos que Y G = XG 3 = 15

3
=
2
2 cm.
Finalmente si nos fijamos en el triangulo ABC, como Y G es paralela a AB y Y G =
1
2 AB, tenemos que AB = 9 cm.
Solucion del problema 16. La respuesta es (a). Observemos que del 1 al 9 ocupamos
9 lugares y del 10 al 99, ocupamos 180 = 2(99 10 + 1) lugares. Luego, del 1
al 99 ocupamos 189 lugares. Del 100 al 199 se utilizan 300 = 3(199 100 + 1)
lugares, entonces del 1 al 699 se utilizan 189 + (6 300) = 1989 lugares. Como
2010 1989 = 21, entonces necesitamos escribir otros 7 numeros de 3 dgitos, es
decir, al escribir el 706 llegamos al lugar 2010 en la sucesion. Por lo tanto, el dgito en
el lugar 2010 es el 6.

28

Soluciones a los problemas de practica

Solucion del problema 17. La respuesta es (d). Denotemos por s al radio del crculo
menor, entonces su a rea es A1 = s2 cm2 (ver el teorema 7 del apendice). Por otro
lado, el a rea del crculo mayor es A1 + A2 = 9 cm2 . Como A1 , A2 y A1 + A2 forman
una progresion aritmetica, tenemos que A1 + r = A2 y A1 + 2r = A1 + A2 para algun
numero r. Luego,
A1 + r = s2 + r = 9 s2 .
Despejando r tenemos que,
r = 9 2s2 cm.

(1)

Por otro lado,


A1 + 2r = s2 + 2r = 9 = A1 + A2 ,
despejando r
r=

9 s2
cm.
2

(2)

Igualando las ecuaciones (1) y (2) y despejando s obtenemos:


9 2s2

18 4s2
9 3s2

=
=

9 s2
2
9 s2
0

=
=

3 cm.

s2
s

Por lo tanto, el radio del crculo menor mide

3 cm.

Solucion del problema 18. La respuesta es (c). Sea r la longitud del radio del crculo.
Como el a rea del crculo es r2 = 1 m2 , entonces r = 1 m (ver el teorema 7 del
apendice).

Ob

Como OC y AB son paralelas, los triangulos OAB y ABC tienen la misma base y la
misma altura, y por lo tanto tienen la misma a rea.
Ahora bien, el triangulo OAB es equilatero de lado 1 m.

Soluciones a los problemas de practica

29

Luego, utilizando el teorema de Pitagoras (ver el teorema 11 del apendice), tenemos


que su altura mide
s

OA2

AB
2

2

1
1

=
4

3
3
= m.
4
2

Entonces, su a rea es
AB
2

3
2

3
2

3 2
m .
4

Solucion del problema 19. La respuesta es (b). En la primera pesada colocamos 10


monedas en cada platillo y nos sobra una. Tenemos dos posibilidades.
si la balanza queda equilibrada entonces la moneda que sobra es la falsa. Tomamos una moneda de cualquier platillo y la comparamos con la falsa para saber si
pesa mas o pesa menos;
si la balanza no esta equilibrada, tomamos las 10 monedas que pesan mas y las
dividimos en dos grupos de 5 para pesarlas. Si la balanza queda equilibrada,
la moneda falsa no esta en este grupo de 10 monedas y esta en el otro, que
peso menos, luego la moneda falsa pesa menos. Si al comparar los dos grupos de
5 monedas la balanza no queda equilibrada, entonces la moneda falsa esta en el
grupo que pesa mas y por lo tanto pesa mas que las demas.
En cualquier caso el mnimo numero de pesadas fueron 2.

Solucion del problema 20. La respuesta es (b). Observemos que los radios que van a
los puntos de tangencias del crculo con el triangulo son perpendiculares a los lados del
triangulo.

30

Soluciones a los problemas de practica


C

r
b

r
A

Los triangulos AF O y ADO son congruentes, ya que el segmento AO es bisectriz del


a ngulo CAB (ver el teorema 19 del apendice), lo que implica que los triangulos tienen
los mismos a ngulos y dos lados correspondientes de la misma longitud. Analogamente,
los triangulos CF O y CEO son congruentes. El a rea del triangulo CAO es hr
2 , luego,
el a rea del triangulo ABC es
 
hr
+ r2 = r(h + r).
2
2
Por lo tanto, la razon entre las a reas es

r 2
r(h+r)

r
h+r .

Solucion del problema 21. La respuesta es (c). Multiplicando por a la igualdad a2 =


a+2, obtenemos a3 = a2 +2a. Pero como a2 = a+2, resulta que a3 = (a+2)+2a =
3a + 2.
Solucion del problema 22. La respuesta es (b). Observemos que
124+248+3612+ = (124)+23(124)+33(124)+
y
139+2618+3927+ = (139)+23(139)+33(139)+
Luego,
1 2 4 + 2 4 8 + 3 6 12 +
1 3 9 + 2 6 18 + 3 9 27 +

=
=
=

Por lo tanto,
r
3

(1 2 4)(1 + 23 + 33 + )
(1 3 9)(1 + 23 + 33 + )
124
139
8
.
27

1 2 4 + 2 4 8 + 3 6 12 +
=
1 3 9 + 2 6 18 + 3 9 27 +

r
3

2
8
= .
27
3

Soluciones a los problemas de practica

31

Solucion del problema 23. La respuesta es (d). Los u ltimos siete dgitos de un numero
de telefono se deben elegir entre los ocho dgitos:
2, 3, 4, 5, 6, 7, 8, 9.
Luego, todos excepto uno de estos 8 dgitos se deben usar. El dgito que no se usa se
puede elegir de 8 formas distintas, y los restantes siete dgitos se ordenan entonces de
menor a mayor para obtener as un posible numero de telefono. Por lo tanto, se pueden
formar 8 numeros de telefono.
Solucion del problema 24. La respuesta es (a). En primer lugar observamos que
ABM L es un cuadrado y que E es el punto de interseccion de sus diagonales AM
y BL, por lo tanto, podemos concluir que E es el centro de dicho cuadrado. Trazamos el segmento EG y observamos que el cuadrilatero EF GH queda dividido en
dos triangulos congruentes EGF y EGH. Lo anterior puede justificarse con facilidad
apoyandose en el teorema de a ngulos entre paralelas y en el criterio ALA (ver la definicion 8 y el criterio 14 del apendice).

F
E

G
H

Ahora, centremos nuestra atencion en el triangulo rectangulo isosceles EGL. Como


EG = GL = 12 m, podemos calcular el a rea de dicho triangulo, misma que es igual a
1
1
22

= 18 m2 (ver el teorema 7 del apendice).


Por otro lado, con base en el teorema de a ngulos entre paralelas y en el criterio de semejanza AA, podemos afirmar que los triangulos LAF y EGF son semejantes. Como
LA
LF
= EG
. Ademas sabesus lados correspondientes son proporcionales, tenemos que EF
LF
1
mos que EG = 2 m y LA = 1 m, por lo que EF = 2 m y entonces LF = 2EF .
Por u ltimo, observamos que el a rea del triangulo EGF es la mitad del a rea del triangulo F GL, ya que sus bases EF y F L estan en razon 1:2 y tienen la misma altura desde
1
el vertice G. De lo anterior podemos concluir que el a rea del triangulo EGF
 es 13 del2
1
1
1
2
a rea de EGL, la cual es 8 m . Por lo tanto el a rea de EGF es igual a 3 8 = 24 m
1
1
) = 12
m2 .
y el a rea del cuadrilatero EF GH es igual a 2( 24
2

Solucion del problema 25. La respuesta es (c). Observemos que si n = 1, la fraccion


es igual a 1 que es un numero entero. Queremos ver para que enteros positivos n, con

32

Soluciones a los problemas de practica

1 < n 24, se cumple que


n!
=
1 + 2 + + n

n!
n(n+1)
2

2n!
2(n 1)!
=
n(n + 1)
n+1

es un entero.
Es facil ver que esta fraccion siempre es un numero entero a menos que n + 1 sea un
primo impar. Como hay 8 primos impares menores o iguales que 24 (dichos primos son
3, 5, 7, 11, 13, 17, 19 y 23), se sigue que hay 24 8 = 16 enteros menores o iguales
que 24 que satisfacen el problema.
Solucion del problema 26. La respuesta es (e). Trazando algunas diagonales del hexagono, e ste queda dividido en 12 triangulos rectangulos congruentes donde cada uno de
a
ellos tiene a rea 12
, como en la figura.

Dividimos nuevamente al hexagono, pero ahora lo hacemos tomando solo 6 de estos


triangulos rectangulos y el resto lo dividimos usando 4 triangulos equilateros congruentes al triangulo ABC.
D

E
A
C

Soluciones a los problemas de practica

33

Ahora es facil encontrar el a rea buscada. Notese que 4 veces el a rea de ABC es igual
a
) = a2 . Sabiendo que 4 veces el a rea de
a 6 veces el a rea de ADE, que es igual a 6( 12
a
ABC es igual a 2 , concluimos que el a rea de ABC es a8 .
Solucion del problema 27. La respuesta es (d). Si d 6= 0, el valor de la expresion
c ab d se puede incrementar intercambiando 0 con el valor de d. Por lo tanto, el valor
maximo debe ocurrir cuando d = 0. Luego,
1. si a = 1, entonces c ab d = c = 2 o 3,
2. si b = 1, entonces c ab d = c a = 6,
3. si c = 1, entonces c ab d = ab que es igual a 23 = 8 o 32 = 9.
Por lo tanto, el valor mayor posible para c ab d es 9 con a = 3, b = 2, c = 1 y
d = 0.
Solucion del problema 28. La respuesta es (a). Como el dinero de Paty se incrementara si se intercambiaran sus monedas de 5 centavos por las de 10 centavos, ella debe
tener mas monedas de 5 centavos que de 10 centavos. Al intercambiar una moneda de
5 centavos por una de 10 centavos, su dinero se aumenta en 5 centavos, de modo que
as de 5 centavos que de 10 centavos. Por lo tanto, Paty
ella tiene 70
5 = 14 monedas m
tiene 21 (20 14) = 3 monedas de 10 centavos y 20 3 = 17 monedas de 5 centavos,
es decir, en total tiene 3(10) + 17(5) = 115 centavos o $1.15.
Solucion del problema 29. La respuesta es (e). Observemos que:
3! 5! 7! = 28 34 52 7.
Luego, un divisor cubo positivo de 3! 5! 7! debe ser de la forma 2p 3q 5r 7s , donde
p, q, r y s son todos multiplos de 3. Tenemos 3 posibles valores para p: 0, 3 y 6. Hay 2
posibles valores para q: 0 y 3. El u nico valor posible para r o s es 0. Por lo tanto, hay
3 2 1 1 = 6 cubos positivos distintos que dividen a 3! 5! 7!. Ellos son:
1 = 20 30 50 70 ,

8 = 23 30 50 70 ,

27 = 20 33 50 70 ,

64 = 26 30 50 70 ,

216 = 23 33 50 70 ,

1728 = 26 33 50 70 .

Solucion del problema 30. La respuesta es (c). Ponemos una copia A B C D de


ABCD a un lado y volteada.

34

Soluciones a los problemas de practica

D C = B

B = C D

A
Entonces AD A D es un paralelogramo y
EF =

1
1
1
F F = (AB + C D ) = (AB + CD).
2
2
2

Como los trapecios ABEF y F ECD tienen la misma altura h, la razon entre sus a reas
es igual a
2

(AB+F E)h
2
(F E+DC)h
2

de donde,
AB + F E
AB + DC
AB +
2
3AB + DC
2
3AB + DC
AB
DC

= 2(F E + DC)


AB + DC
= 2
+ DC
2
= AB + 3DC
= 2AB + 6DC
= 5.

Problemas propuestos

Tzaloa necesita de tu participacion y esta seccion de la revista esta especialmente disenada para hacerlo. En cada numero presentamos 5 problemas nuevos que necesitan
de ti para encontrar una respuesta. Como sabemos que te gustan los retos, los problemas escogidos para esta seccion pueden ser un poco mas difciles que los que aparecen
en la seccion anterior. Sin embargo, esto no debe ser motivo de espanto, debes saber
que aqu tambien seguimos el mismo criterio que consiste en comenzar con problemas
mas sencillos e ir aumentando la complejidad conforme avanza el ano.

Problemas propuestos.
2010 No. 1.
Ano
Para dar tiempo a que lleguen y puedan ser analizadas las contribuciones de todos los
lectores, las soluciones de los problemas propuestos en cualquier numero de la revista,
se publicaran con dos numeros de diferencia. Es as, que en este numero (Tzaloa 1, ano
2010), publicamos las soluciones de los problemas propuestos en Tzaloa 3, ano 2009.
Las soluciones de los problemas propuestos en esta ocasion, se publicaran en Tzaloa 3,
ano 2010, por lo que aun tienes tiempo para preparar y enviarnos tus soluciones.
Recuerda que nuestra direccion electronica es revistaomm@gmail.com y que a
traves de ella estaremos recibiendo con gusto todas las contribuciones que nos lleguen
desde cualquier rincon del pas. Siempre esperamos con ansia la llegada de las valiosas
contribuciones de todos los lectores, por lo que no dudes en mandarnos la tuya.
Problema 1. (Introductorio) Encuentra todas las parejas de numeros enteros (p, q) tales
que la diferencia entre las dos soluciones de la ecuacion x2 + px + q = 0 sea 2010.
Problema 2. (Introductorio) Si n es un entero divisible entre 7 que es igual al producto
de tres numeros consecutivos, cual (o cuales) de los enteros 6, 14, 21, 28 y 42 no es
necesariamente un divisor de n?

36

Problemas propuestos

Problema 3. (Introductorio) Sea A1 A2 A3 A4 A5 A6 A7 A8 A9 un polgono regular de


nueve lados. Cuantos triangulos equilateros se pueden formar tales que al menos dos
de sus vertices esten en el conjunto {A1 , A2 , A3 , A4 , A5 , A6 , A7 , A8 , A9 }?
Problema 4. (Intermedio) Sea ABC un triangulo acutangulo e isosceles con AC =
AB. Sean O su circuncentro e I su incentro. Si D es el punto de interseccion de AC
con la perpendicular a CI que pasa por O, demuestra que ID y AB son paralelas.
Problema 5. (Avanzado) Sea A = {1, 2, 3, . . . , n}. A cada subconjunto B de A se
le asocia su suma alternada SB , definida como sigue: si B = {a1 , a2 , . . . , ak } con
a1 < a2 < < ak , entonces SB = ak ak1 + ak2 a1 . Por ejemplo, si
n = 10 y B = {2, 4, 5, 7, 8} entonces SB = 8 7 + 5 4 + 2 = 4.
Si n es un numero fijo, determina el valor de la suma
X
SB ,
BA

es decir, la suma de todos los numeros SB con B subconjunto de A.

Soluciones a los problemas propuestos.


2009 No. 3.
Ano
Como se menciono al principio de esta seccion, a continuacion publicamos las soluciones de los problemas propuestos en Tzaloa 3, ano 2009. Felicitamos a Luis Brandon
Guzman Navarrete, de Tamaulipas, quien nos envio soluciones para los problemas 2 y
5. Asimismo, queremos felicitar a Mara del Rosario Soler Zapata y a Martn Velasco
Hernandez, de Chiapas, por su solucion para el problema 5. Por u ltimo, tambien nos
da mucho gusto reconocer el trabajo de Irving Daniel Calderon Camacho, del Estado
de Mexico, quien aporto una excelente contribucion al resolver los problemas 1, 2 y 5.
A todos ustedes enviamos un afectuoso saludo y nuestro mas sincero agradecimiento.
Problema 1. (Principiante) Si CA = 2, CB = 3, CAP = 90 , P BC = 90 y
AP B = 60 , cuanto mide P C?
A
2
C

Problemas propuestos

37

Solucion. (Irving Daniel Calderon Camacho, Estado de Mexico). Como CAP +


P BC = 180 , el cuadrilatero P ACB es cclico (ver la definicion 22 y los teoremas
23 y 21 del apendice). Como CAP = 90 , entonces P ACB esta inscrito en la circunferencia de diametro P C.
A
C
O
P

Sea O el centro de la circunferencia, punto medio de P C, entonces tenemos que


AOB = 2(AP B) = 120 . Sea r el radio de la circunferencia circunscrita a
P ACB. Aplicando la ley de cosenos al triangulo AOB (ver el teorema 20 del apendice), obtenemos

AB 2 = 2r2 2r2 cos(120) = 3r2 .


Por otro lado, aplicando la ley de cosenos al triangulo ABC, obtenemos

AB 2 = CA2 + CB 2 2(CA)(CB) cos(120 ).


De las dos ecuaciones anteriores se sigue que

3r2 = 22 + 32 + 2 2 3
por lo que concluimos que r =
q
2 19
3 .

19
3 .

1
2

= 19,

Finalmente, como P C = 2r, obtenemos P C =

Solucion alternativa. Sea D la interseccion de P A y BC. Como el triangulo DP B es


rectangulo y DP B = 60 , entonces BDP = 30 (ver el teorema 10 del apendice).
Luego, el triangulo BP D es la mitad de un triangulo equilatero de lado 2P B, de donde
P B = 13 DB (ver el teorema 11 del apendice).

38

Problemas propuestos

2
C
3

Ahora bien, como los a ngulos del triangulo DCA miden 30 , 60 y 90 , tenemos que
CD = 2CA = 4 y DB = 7, luego P B = 73 . Entonces,
P C 2 = P B 2 + BC 2 =
de donde P C =

76
3

=2

76
49
+9=
,
3
3

19
3 .

Problema 2. (Intermedio) Demuestra que la desigualdad


  X
n
n
X
n
xii
ixi
+
2
i=1
i=1
se cumple para todo entero n 2 y todos los numeros reales no negativos x1 , . . . , xn .
Solucion. (Irving Daniel Calderon Camacho, Estado de Mexico). Se hara una demostracion por induccion sobre n, pero antes demostraremos por casos la siguiente desigualdad, la cual se cumple para cualquier numero real x no negativo y para todo entero
n 2.
(n + 1)x n + xn+1

(3)

Comencemos transformando la desigualdad (3) mediante las siguientes equivalencias:


(n + 1)x n + xn+1

0 n nx + xn+1 x
0 n(1 x) (1 x)(xn + xn1 + + x)

0 (1 x) n (xn + xn1 + + x) .

Para x = 1, se cumple la igualdad. Si x < 1, tenemos que 0 < 1 x y

n > xn + xn1 + + x 0 < n (xn + xn1 + + x),


por lo que concluimos

0 < (1 x) n (xn + xn1 + + x) .

Problemas propuestos

39

Si x > 1, tenemos que 0 > 1 x y


n < xn + xn1 + + x 0 > n (xn + xn1 + + x),
por lo que concluimos

0 < (1 x) n (xn + xn1 + + x) .

Con esto terminamos la demostracion de (3). Ahora, comenzamos nuestra prueba por
induccion. Para n = 2, tenemos que
 
2
x1 + 2x2
+ x1 + x22 0 x22 2x2 + 1 0 (x2 1)2 ,
2
donde la u ltima desigualdad es evidente. Ahora, supongamos que la desigualdad se
cumple para algun entero n 2
  X
n
n
X
n
xii .
ixi
+
2
i=1
i=1
Usando la formula de Pascal (ver el teorema 6 del apendice), tenemos

      
n+1
n
n
n
=
+
=
+n.
2
2
1
2
Sea xn+1 un numero real no negativo cualquiera, usando (3) tenemos
(n + 1)xn+1 n + xn+1
n+1 .
Finalmente, sumando las dos u ltimas desigualdades obtenemos
n+1
X
i=1

ixi


 n+1
 
n+1
n+1
X
X
X
n+1
n
xii .
ixi
+
xii
+n+
2
2
i=1
i=1
i=1

Eso completa nuestra prueba por induccion y la demostracion de la desigualdad.


Solucion alternativa. Para cada numero real no negativo x y para cada entero positivo
k tenemos, por la desigualdad media aritmetica-media geometrica (ver teorema 5 del
apendice), que
p
k
xk + k 1 = xk + 1 + + 1 k xk 1 1 = kx.
| {z }
k1

La igualdad se da, para k 2, si y solo si x = 1.


Luego, utilizando lo anterior para cada xi y sumando, tenemos que
n
X
i=1

ixi

n
X
i=1

((i 1) +

xii )

  X
n
n
xii .
=
+
2
i=1

La igualdad se da si y solo si x2 = = xn = 1 y x1 0 es arbitraria.

40

Problemas propuestos

Problema 3. (Intermedio) En un paralelogramo estan marcados el centro y los puntos


medios de los lados. Considera todos los triangulos cuyos vertices estan en estos puntos
marcados. Ahora, en cada triangulo marca los puntos medios de los lados y los puntos
medios de la medianas. Cuantos puntos marcados habra en total?
Solucion. Denotemos por A, B, C, D y E, a los puntos medios de los lados del paralelogramo y a su centro respectivamente, como se muestra en la figura. Hay 8 triangulos
cuyos vertices estan en estos puntos, cuatro que tienen un vertice en E y cuatro para
los cuales E es punto medio de uno de sus lados.
Ab
M
B

bD

C
Consideremos los triangulos que tienen un vertice en E y marquemos los puntos medios de los lados y las medianas. Al marcar los puntos medios de los lados AB, BC,
CD, AD, AE, BE, CE y DE, obtenemos 8 puntos. Si marcamos ahora los puntos
medios de las medianas, obtenemos 12 puntos mas, pues cada triangulo tiene 3 medianas.
Ahora consideremos los triangulos para los cuales E es punto medio de uno de sus
lados. Por ejemplo, en el triangulo ABD los puntos medios de los lados ya estan marcados. Llamemos M al punto medio del lado AB, entonces el punto medio de la recta
EM esta marcado, ya que EM es mediana del triangulo ABE. Observemos que como
E es punto medio de BD, la recta EM es paralela a AD. Esto implica que el punto medio de la mediana BP del triangulo ABD esta en EM (ver el teorema 9 del apendice).
Ademas, como P es punto medio de AD, el punto de interseccion de EM y BP es el
punto medio de EM . Esto implica que los puntos medios de las medianas del triangulo ABD que pasan por B y D ya estan marcados. La tercera mediana del triangulo
ABD es AE, cuyo punto medio ya esta marcado. Es decir, con el triangulo ABD no
marcamos ningun nuevo punto y lo mismo pasa con los otros tres triangulos para los
cuales E es el punto medio de uno de sus lados.
Por lo tanto, al final habra 5 + 8 + 12 = 25 puntos marcados.
Problema 4. (Avanzado) Con un plano, cual es el maximo numero de caras de un
cubo, un octaedro y un dodecaedro que puedes cortar?
Solucion. Un plano esta definido por tres puntos, es decir, dados tres puntos en el
espacio hay un solo plano que los contiene. Entonces, al cortar un poliedro con un
plano, el plano no puede cortar mas de dos aristas de cada cara (ya que si no el plano
sera la cara). Ademas, si queremos maximizar el numero de caras cortadas, no debe de
cortar todas las aristas que salen de un mismo vertice. Analicemos los tres poliedros.

Problemas propuestos

41

Por los argumentos anteriores, el plano no puede cortar mas de 2(6)


= 6 aristas, ya
2
que un cubo tiene 6 caras. Por lo tanto, no puede cortar mas de 6 caras. En la figura,
presentamos un tal corte del cubo: empezamos con un plano que contiene una diagonal
y lo rotamos un poco para cortar las 6 caras.

Como las caras del octaedro son triangulos, si una cara es cortada por el plano, de las
otras tres caras que pasan por cada uno de sus vertices hay al menos una que no es
cortada por el plano. Por lo tanto, se pueden cortar a lo mas 6 caras, una manera de
hacerlo es la siguiente.

Por los argumentos anteriores, el maximo numero de aristas que se pueden cortar es
2(12)
= 12. Si colocamos el dodecaedro en un plano y cortamos con un plano paralelo,
2
es facil ver que el maximo numero de caras cortadas es 10. Veamos que este es el mejor
corte. Tomemos una cara que es cortada con el plano. Si el plano corta las cinco caras
adyacentes a ella, corta solo estas 6 caras. Entonces, por cada cara cortada, al menos
una de las caras adyacentes no es cortada. Esto implica que el maximo numero de caras
cortadas es 10.
Problema 5. (Avanzado) Determina el numero de enteros n > 1 que cumplen que
a13 a es divisible entre n para todo entero a.
Solucion. (Mara del Rosario Soler Zapata y Martn Velasco Hernandez, Chiapas).
Supongamos que n > 1 cumple la condicion. Entonces, n|213 2 y n|313 3, por lo que
n|313 3213+2, esto es, n|313 213 1. Como 213 2 = 8, 190 = 232 5713 y
313 213 1 = 1, 594, 3238, 1921 = 1, 586, 130 = 235721383, entonces
los numeros primos que aparecen en la descomposicion en primos de n pertenecen al
conjunto A = {2, 3, 5, 7, 13} y tienen exponente a lo mas 1 (ver el teorema 1 del
apendice).
Ahora, si a es par (o impar), entonces a13 es par (o impar), por lo que a13 a es par.

42

Problemas propuestos

Luego, 2|a13 a para todo entero a, y por lo tanto, n = 2 cumple la condicion.


Por otra parte, por el pequeno teorema de Fermat, tenemos que para todo entero a se
cumple que (ver la definicion 3 y el teorema 4 del apendice)
a3
a5
a7
a13

a (mod 3),
a (mod 5),

a (mod 7),
a (mod 13).

(4)
(5)
(6)
(7)

Veamos que si n 6= 2 esta en A, entonces n cumple la condicion, es decir, a13


a (mod n) para todo entero a.
1. De (4) se sigue que a12 a4 (mod 3), luego a13 a5 a3 a (mod 3), por
lo que a13 a (mod 3).

2. De (5) se sigue que a10 a2 (mod 5), luego a13 a5 a (mod 5), por lo que
a13 a (mod 5).

3. De (6) se sigue que a13 a7 (mod 7), luego a13 a (mod 7).
4. De (7) se sigue que n = 13 cumple la condicion.

De lo anterior, se sigue que todos los elementos de A satisfacen la condicion. Como


cualesquiera dos elementos de A son primos relativos, entonces todo entero de la forma
n = 2a 3b 5c 7d 13e (con a, b, c, d, e enteros mayores o iguales que 0 y menores
o iguales que 1) tambien sera divisor de a13 a para todo entero a. Luego, los enteros
n > 1 que cumplen el problema son precisamente los divisores positivos distintos de 1
del numero 2 3 5 7 13. Por lo tanto, la respuesta es 25 1 = 31 enteros.
Solucion alternativa. Sea n > 1 un entero tal que a13 a es divisible entre n para todo
entero a. Tenemos que p2 , con p primo, no divide a n, ya que p2 no divide a p13 p.
Luego, n es producto de primos distintos. Como n debe dividir al numero a13 a para
todo entero a, en particular n debe dividir al numero 213 2 = 2 32 5 7 13. Ahora,
es claro que para todo entero a se cumple que a2 a (mod 2) (ver la definicion 3 del
apendice), ya que a2 a = a(a 1) es producto de dos enteros consecutivos. Luego,
a13 = (a2 )6 a a6 a (a2 )3 a a3 a a4 (a2 )2 a2 a (mod 2).

Analogamente, tenemos que a3 a (mod 3) para todo entero a, pues a3 a =


a(a 1)(a + 1) es producto de tres enteros consecutivos. Luego,
a13 = (a3 )4 a a4 a = a3 a2 a a2 = a3 a (mod 3).

Usando congruencias modulo 5, 7 y 13 es facil demostrar (se deja al lector) que para
todo entero a,
a5 a (mod 5), a7 a (mod 7) y a13 a (mod 13),

y como en los dos casos anteriores se sigue que a13 a (mod 5) y a13 a (mod 7).
Luego, a13 a (mod 2 3 5 7 13) para todo entero a, y por lo tanto los enteros
n > 1 que cumplen el problema son precisamente los divisores positivos distintos de 1
del numero 2 3 5 7 13. Por lo tanto, la respuesta es 25 1 = 31 enteros.

Concurso Nacional 2009


23a Olimpiada Mexicana de
Matematicas
Del 8 al 14 de noviembre de 2009 se llevo a cabo en Campeche, Campeche, el Concurso
Nacional de la 23a Olimpiada Mexicana de Matematicas, con la participacion de 31
estados de la Republica. El estado de Tabasco no participo. Los 17 alumnos ganadores
del primer lugar fueron:
Hernandez Gonzalez Flavio (Aguascalientes)
Arreola Gutierrez Fernando Ignacio (Aguascalientes)
Zhou Tan David (Baja California)
Dosal Bustillos Manuel Enrique (Chihuahua)
Embarcadero Ruiz Daniel (Distrito Federal)
Calderon Camacho Irving Daniel (Estado de Mexico)
Leal Camacho Manuel Alejandro (Jalisco)
Miranda Olvera Jose Luis (Jalisco)
Ortiz Rhoton Juan Carlos (Jalisco)
Belanger Albarran Georges (Morelos)
Perales Anaya Daniel (Morelos)
Anorve Lopez Fernando Josafath (Nuevo Leon)
Roque Montoya Diego Alonso (Nuevo Leon)
Jimenez Reichow Tilman (Oaxaca)
Guardiola Espinosa Jose Ramon (San Luis Potos)
Jimenez Bentez Jose Manuel (San Luis Potos)
Ucan Ake Raul Eugenio (Yucatan)
Los 8 alumnos preseleccionados para la Olimpiada Matematica de Centroamerica y el
Caribe fueron:

44

Concurso Nacional 2009

Garca Gonzalez Hector Benjamn (Colima)


Ortiz Rhoton Juan Carlos (Jalisco)
Gonzalez Cazares Jorge Ignacio (Jalisco)
Arancibia Alberro Mara Natalie (Morelos)
Roque Montoya Diego Alonso (Nuevo Leon)
Anorve Lopez Fernando Josafath (Nuevo Leon)
Daz Calderon Julio Cesar (Oaxaca)

Cervantes Perez Angel


Gustavo (Yucatan)
Aunque la participacion en el Concurso Nacional es individual, es importante destacar
la labor que han llevado a cabo los estados de la Republica apoyando a sus concursantes. Con el proposito de reconocer este trabajo, presentamos el registro de los estados
que ocuparon los primeros 10 lugares en el Concurso Nacional de la 23a Olimpiada
Mexicana de Matematicas.
1. Jalisco
2. Morelos
3. San Luis Potos
4. Nuevo Leon
5. Distrito Federal
6. Yucatan
7. Chihuahua
8. Baja California
9. Aguascalientes
10. Oaxaca
En esta ocasion, el premio a la Superacion Academica se llamo Copa San Francisco
de Campeche y fue ganado por San Luis Potos. El segundo y tercer lugar de este
premio lo ocuparon, Distrito Federal y Nuevo Leon, respectivamente.
A continuacion presentamos los problemas del Concurso Nacional 2009. Los alumnos
tuvieron dos sesiones de cuatro horas y media cada una para resolverlos.
Examen del Concurso Nacional 2009
Problema 1. Sean ABC un triangulo y AD la altura sobre el lado BC. Tomando a D
como centro y a AD como radio, se traza una circunferencia que corta a la recta AB en
P , y corta a la recta AC en Q. Muestra que el triangulo AQP es semejante al triangulo
ABC.
(Sugerido por Jesus Jeronimo Castro)

Problema 2. En cajas marcadas con los numeros 0, 1, 2, 3, ... se van a colocar todos los
enteros positivos de acuerdo con las siguientes reglas:
Si p es un numero primo e ste se coloca en la caja con el numero 1.

Concurso Nacional 2009

45

Si el numero a se coloca en la caja con el numero ma y b se coloca en la caja


con el numero mb , entonces el producto de a y b, es decir ab, se coloca en la caja
con el numero amb + bma .
Encuentra todos los enteros positivos n que cuando se coloquen queden en la caja con
el numero n.
(Sugerido por Jose Antonio Gomez Ortega)

Problema 3. Sean a, b, c numeros reales positivos tales que abc = 1. Muestra que
b3
c3
a3
+
+
1
a 3 + 2 b 3 + 2 c3 + 2

y que

a3

1
1
1
+ 3
+ 3
1.
+2 b +2 c +2

(Sugerido por Jose Antonio Gomez Ortega)

Problema 4. Sea n > 1 un entero impar y sean a1 , a2 , . . . , an numeros reales distintos.


Sea M el mayor de estos numeros y sea m el menor de ellos. Muestra que es posible
escoger los signos en la expresion s = a1 a2 an de manera que
m < s < M.
(Sugerido por Juan Jose Alba Gonzalez)

Problema 5. Considera un triangulo ABC y un punto M sobre el lado BC. Sea P la


interseccion de las perpendiculares a AB por M y a BC por B, y sea Q la interseccion
de las perpendiculares a AC por M y a BC por C. Muestra que P Q es perpendicular
a AM si y solo si M es punto medio de BC.
(Sugerido por Eduardo Velasco Barreras)

Problema 6. En una fiesta con n personas, se sabe que de entre cualesquiera 4 personas,
hay 3 de las 4 que se conocen entre s o hay 3 que no se conocen entre s. Muestra que
las n personas se pueden separar en 2 salones de manera que en un salon todos se
conocen entre s y en el otro salon no hay dos personas que se conozcan entre s.
Nota: conocerse se considera una relacion mutua.
(Sugerido por Pablo Soberon Bravo)

46

Concurso Nacional 2009

Olimpiadas Internacionales

XXIV Olimpiada Iberoamericana


Este ano, Mexico tuvo el privilegio de organizar la XXIV Olimpiada Iberoamericana

de Matematicas. Esta
se llevo a cabo en la ciudad de Queretaro, del 17 al 27 de septiembre de 2009. Mexico ocupo el 5 lugar de entre los 21 pases que participaron.
La delegacion mexicana estuvo integrada por los alumnos: Manuel Guillermo Lopez
Buenfil (Chihuahua), Erik Alejandro Gallegos Banos (Oaxaca), Daniel Perales Anaya
(Morelos), y Cesar Bibiano Velasco (Morelos).
Manuel Guillermo obtuvo medalla de oro, Erick Alejandro y Daniel obtuvieron medalla de plata y Cesar mencion honorfica.
A continuacion presentamos los problemas de la XXIV Olimpiada Iberoamericana.
Los alumnos tuvieron dos sesiones de cuatro horas y media cada una para resolverlos.
Problema 1. Sea n un natural mayor que 2. Supongamos que n islas estan ubicadas en
un crculo y que entre cada dos islas vecinas hay dos puentes como en la figura.
x1

x2

xn

x3

xn1

xj

Comenzando en la isla x1 , de cuantas maneras se pueden recorrer los 2n puentes


pasando por cada puente exactamente una vez?

48

XI Olimpiada Centroamericana y del Caribe

Problema 2. Para cada entero positivo n se define an = n + m donde m es el mayor


m
entero tal que 22 n2n . Determinar que enteros positivos no aparecen en la sucesion
an .
Problema 3. Sean C1 y C2 dos circunferencias de centros O1 y O2 con el mismo radio,
que se cortan en A y en B. Sea P un punto sobre el arco AB de C2 que esta dentro de
C1 . La recta AP corta a C1 en C, la recta CB corta a C2 en D y la bisectriz de CAD
intersecta a C1 en E y a C2 en L. Sea F el punto simetrico a D con respecto al punto
medio de P E. Demostrar que existe un punto X que satisface XF L = XDC =
30 y CX = O1 O2 .
Problema 4. Sea ABC un triangulo con AB 6= AC. Sean I el incentro de ABC y P
el otro punto de interseccion de la bisectriz exterior del a ngulo A con el circuncrculo
de ABC. La recta P I intersecta por segunda vez al circuncrculo de ABC en el punto
J. Demostrar que los circuncrculos de los triangulos JIB y JIC son tangentes a IC
y a IB, respectivamente.
Problema 5. La sucesion an esta definida por:
a1 = 1, a2k = 1 + ak y a2k+1 =

1
, para todo entero k 1.
a2k

Demostrar que todo numero racional positivo aparece exactamente una vez en esta
sucesion.
Problema 6. Alrededor de una circunferencia se marcan 6000 puntos y cada uno se
colorea con uno de 10 colores dados, de manera tal que entre cualesquiera 100 puntos
consecutivos siempre figuran los 10 colores. Hallar el menor valor k con la siguiente
propiedad: Para toda coloracion de este tipo existen k puntos consecutivos entre los
cuales figuran los 10 colores.

XI Olimpiada Centroamericana y del Caribe


Del 4 al 10 de octubre de 2009, se celebro en Girardot, Colombia, la XI Olimpiada
Matematica de Centroamerica y el Caribe. La delegacion mexicana estuvo integrada
por los alumnos: Manuel Enrique Dosal Bustillos (Chihuahua), Jorge Vargas Garza
(Distrito Federal), y Diego Alonso Roque Montoya (Nuevo Leon).
Jorge y Manuel Enrique obtuvieron medalla de oro, y Diego Alonso medalla de plata.
Mexico ocupo el primer lugar de 12 pases participantes.
A continuacion presentamos los problemas de la XI Olimpiada Centroamericana y del
Caribe. Los alumnos tuvieron dos sesiones de cuatro horas y media cada una para
resolverlos.

Olimpiadas Internacionales

49

Problema 1. Sea P (n) el producto de los dgitos no nulos del entero positivo n. Por
ejemplo, P (4) = 4, P (50) = 5, P (123) = 6, P (2009) = 18. Halle el valor de la
suma:
P (1) + P (2) + + P (2008) + P (2009).
Problema 2. Dos circunferencias 1 y 2 se intersectan en los puntos A y B. Considere
una circunferencia contenida en 1 y 2 , tangente a ellas respectivamente en D y E.
Sean C uno de los puntos de interseccion de la recta AB con 1 , F la interseccion de
la recta EC con 2 y G la interseccion de la recta DC con 1 . Sean H e I los puntos
de interseccion de la recta ED con 1 y 2 , respectivamente. Demuestre que F , G, H
e I estan sobre una misma circunferencia.
Problema 3. Se tienen 2009 cajas numeradas del 1 al 2009, algunas de las cuales
contienen piedras. Dos jugadores A y B juegan alternadamente, comenzando por A.
Una jugada consiste en seleccionar una caja i que no este vaca, tomar una o mas
piedras de esa caja y ponerlas en la caja i + 1. Si i = 2009, las piedras que se tomen se
desechan. El jugador que retire la u ltima piedra (dejando todas las cajas vacas) gana.
1. Suponiendo que inicialmente en la caja 2 hay 2009 piedras y todas las demas
cajas (1, 3, 4, 5, . . . , 2009) estan vacas, halle una estrategia ganadora para uno
de los dos jugadores y justifquela.
2. Suponiendo que inicialmente cada caja contiene exactamente una piedra, halle
una estrategia ganadora para uno de los dos jugadores y justifquela.
Problema 4. Se desea colocar numeros naturales alrededor de una circunferencia cumpliendo la siguiente propiedad: Las diferencias entre cada par de numeros vecinos, en
valor absoluto, son todas diferentes.
1. Sera posible colocar los numeros del 1 al 2009 satisfaciendo la propiedad?
2. Sera posible suprimir alguno de los numeros del 1 al 2009, de tal manera que
los 2008 numeros restantes se puedan colocar satisfaciendo la propiedad?
Problema 5. Dado ABC un triangulo acutangulo y escaleno, sea H su ortocentro, O
su circuncentro, E y F los pies de las alturas trazadas desde B y C, respectivamente.
La recta AO corta nuevamente al circuncrculo del triangulo en un punto G y a los
segmentos F E y BC en los puntos X y Y , respectivamente. La recta AH corta a la
tangente al circuncrculo trazada por G en un punto Z. Demuestre que HX es paralelo
a Y Z.
Problema 6. Encuentre todos los numeros primos p y q tales que p3 q 5 = (p + q)2 .

50

XI Olimpiada Centroamericana y del Caribe

Problemas y Soluciones de
Olimpiadas Internacionales

50a Olimpiada Internacional


La 50a Olimpiada Internacional de Matematicas se llevo a cabo del 10 al 22 de julio de
2009 en Bremen, Alemania, con la participacion de 104 pases. Mexico ocupo el lugar
numero 50. La delegacion que represento a Mexico estuvo integrada por los alumnos: Flavio Hernandez Gonzalez (Aguascalientes), Manuel Guillermo Lopez Buenfil

(Chihuahua), Luis Angel


Isaas Castellanos (Colima), Cesar Bibiano Velasco (Morelos), Cesar Ernesto Rodrguez Angon (Distrito Federal) y Erik Alejandro Gallegos
Banos (Oaxaca). Los alumnos, Manuel Guillermo, Cesar Bibiano y Erik Alejandro
obtuvieron medalla de bronce, y Flavio obtuvo mencion honorfica.
A continuacion presentamos los problemas con sus soluciones de la 50a Olimpiada
Internacional. Los alumnos tuvieron dos sesiones de cuatro horas y media cada una
para resolverlos.
Hemos incluido en el problema 6 una solucion de un exolmpico debido a que es una
solucion muy elegante.
Problema 1. Sea n un entero positivo y sean a1 , . . . , ak (k 2) enteros distintos del
conjunto {1, . . . , n}, tales que n divide a ai (ai+1 1), para i = 1, . . . , k1. Demostrar
que n no divide a ak (a1 1).
Solucion. Por hipotesis, para i = 1, . . . , k 1 tenemos que
ai ai+1 ai (mod n),
lo cual implica que
a1 . . . ak1 ak a1 ak1 a1 ak2 a1 (mod n).

50a Olimpiada Internacional

52

Ahora bien, supongamos que ak a1 ak (mod n), entonces


a1 ak1 ak ak a1 ak1 ak a1 ak2 ak a1 ak (mod n).
Luego, a1 ak (mod n). Como a1 y ak son numeros del conjunto {1, . . . , n}, entonces a1 = ak , lo cual es una contradiccion.
Problema 2. Sea ABC un triangulo con circuncentro O. Sean P y Q puntos interiores
de los lados CA y AB, respectivamente. Sean K, L y M los puntos medios de los
segmentos BP , CQ y P Q, respectivamente, y la circunferencia que pasa por K, L
y M . Se sabe que la recta P Q es tangente a la circunferencia . Demostrar que OP =
OQ.
Solucion de Cesar Bibiano Velasco. Como P Q es tangente a , entonces M KL =
LM P = . Como M y L son puntos medios de P Q y QC, respectivamente, tenemos
que M L y P C son paralelas y LM P = AP Q = . Por lo tanto, M KL =
AP Q = . Analogamente AQP = M LK = .
A

L
b

Entonces los triangulos AQP y M LK son semejantes, luego tenemos que


QP
AP
LK = MK , de donde
PC
ML
PC
AQ
2
=
= QB
,
=
AP
MK
QB
2
dado que M L =
ces,

PC
2 ,

AQ
ML

pues M y L son puntos medios y M L es paralela a P C. EntonAQ QB = AP P C.

50a Olimpiada Internacional

53

Sean X y Y los puntos de interseccion de P Q con el circuncrculo del triangulo ABC.


Por potencia desde Q en AXY B, tenemos que AQ QB = XQ QY . Por potencia
desde P en AY CX, tenemos que AP P C = XP P Y . Ahora bien, como AQQB =
AP P C, entonces XQ QY = XP P Y . Si a = XQ, b = QP y c = P Y , entonces
a(b + c) =

(a + b)c

ab + bc =
a =

ac + bc
c,

es decir, XQ = P Y . Pero M es punto medio de P Q, luego M es punto medio de XY ,


entonces M O es perpendicular a XY , y como M P = M Q, entonces OQ = OP .
Problema 3. Sea s1 , s2 , s3 , . . . una sucesion estrictamente creciente de enteros positivos tal que las subsucesiones
ss1 , ss2 , ss3 , . . .

ss1 +1 , ss2 +1 , ss3 +1 , . . .

son ambas progresiones aritmeticas. Demostrar que la sucesion s1 , s2 , s3 , . . . es tambien una progresion aritmetica.
Solucion. Sea D la diferencia comun de la progresion ss1 , ss2 , ss3 , . . . . Consideremos
para n = 1, 2, . . .,
dn = sn+1 sn .
Debemos probar que dn es constante. Primero probaremos que el numero dn esta acotado. De hecho, por hipotesis dn 1 para toda n. Entonces, tenemos que para toda
n
dn = sn+1 sn dsn + dsn +1 + + ds(n+1) 1 = ssn+1 ssn = D.
El hecho de que dn este acotado implica que existen
m = min{dn : n = 1, 2, . . .}
y
M = max{dn : n = 1, 2, . . .}.
Basta probar que m = M . Supongamos que m < M . Escogemos n tal que dn = m.
Si consideramos una suma telescopica de m = dn = sn+1 sn elementos no mayores
que M , obtenemos
D = ssn+1 ssn = ssn+m ssn = dsn + dsn +1 + + dsn +m1 mM,
donde la igualdad se da si y solo si todos los terminos de la suma son iguales a M .
Ahora, escogemos n tal que dn = M y consideramos una suma telescopica de M
elementos no menores que m, obteniendo
D = ssn+1 ssn = ssn+M ssn = dsn + dsn +1 + + dsn +M1 M m,

50a Olimpiada Internacional

54

donde la igualdad se da si y solo si todos los terminos de la suma son iguales a m. Las
desigualdades anteriores implican que D = M m, de donde
dsn

dsn

dsn +1 = = ds(n+1) 1 = M si dn = m,
dsn +1 = = ds(n+1) 1 = m si dn = M.

Por lo tanto, dn = m implica que dsn = M . Observemos que sn s1 + (n 1) n


para toda n, ademas sn > n si dn = n, dado que en el caso en que sn = n tendramos
m = dn = dsn = M en contradiccion con el hecho de que supusimos que m < M .
De la misma manera dn = M implica que dsn = m y sn > n. En consecuencia, existe
una sucesion estrictamente creciente n1 , n2 , . . . tal que
dsn1 = M, dsn2 = m, dsn3 = M, dsn4 = M, . . .
La sucesion ds1 , ds2 , . . . es la sucesion de las diferencias por pares de ss1 +1 , ss2 +1 , . . .,
que tambien es una sucesion aritmetica. Por lo tanto, m = M .
Problema 4. Sea ABC un triangulo con AB = AC. Las bisectrices de los a ngulos
CAB y ABC cortan a los lados BC y CA en D y E, respectivamente. Sea K el
incentro del triangulo ADC. Supongamos que el a ngulo BEK = 45 . Determinar
todos los posibles valores de CAB.
Solucion. Veamos que CAB = 60 o CAB = 90 son posibles valores para el
a ngulo y que son los u nicos posibles valores.
Como KC es bisectriz de ACD e IC es bisectriz de ACB, entonces I, K y C son
colineales.
A

E
F
I

b
b

K
B

50a Olimpiada Internacional

55

Como DK es bisectriz de ADC = 90 , entonces IDK = CDK = 45 . Sea F


el pie de la perpendicular de I sobre AC. Como I equidista de BC y AC, entonces
ID = IF . Luego, los triangulos IDC e IF C son congruentes (ambos son triangulos
rectangulos y tienen dos lados iguales). Entonces F K es bisectriz de IF C, de donde
IF K = 45 , luego IEK = IF K, entonces, IEF K es cclico.
Sea A = 2. Entonces, BAD = CAD = , luego ABI = DBI =
DCI = ACI = 45 2 , de donde BIC = 90 + , y por lo tanto,
EIK = 90 .
Como BEK = 45 , tenemos tres posibles casos:
1. Si E esta entre F y C, entonces F IC > EIC, de donde, 45 + 2 > 90 ,
es decir, 30 < . Como IF EK es cclico, entonces EF K = EIK = 45 ,
luego EIK = 45 = 90 , de donde = 45 y CAB = 90 .
2. Si F esta entre E y C, entonces < 30 . Como IF EK es cclico, tenemos que
EIK = 180 EF K = 45 , pero EIK = 90 , luego = 45 , lo
cual es una contradiccion pues < 30 .
3. Si F = E, entonces BI es perpendicular a AC, luego BI es altura y bisectriz,
de donde el triangulo ABC es isosceles con AB = BC, pero AB = AC, luego
ABC es equilatero y CAB = 60 .
Por lo tanto, CAB = 60 o CAB = 90 .
Ahora bien, si CAB = 60 , entonces el triangulo ABC es equilatero. Mas aun
tenemos que F = E, luego BEK = IF K = 45 . Por lo tanto, 60 es un valor
posible para CAB. Ahora consideremos el caso en que CAB = 90 , entonces
CBA = ACB = 45 y EIK = EF K = 45 . Luego, BEK = 45 . Por lo
tanto, tambien 90 es un posible valor para CAB.
Problema 5. Determinar todas las funciones f del conjunto de los enteros positivos en
el conjunto de los enteros positivos tales que, para todos los enteros positivos a y b,
existe un triangulo no degenerado cuyos lados miden
a, f (b) y f (b + f (a) 1).
(Un triangulo es no degenerado si sus vertices no estan alineados).
Solucion. Veamos que la funcion identidad, f (x) = x es la u nica funcion que satisface
las condiciones del problema.
Si f (x) = x para todo entero positivo x, entonces las medidas de los tres lados son:
x, y = f (y) y z = f (y + f (x) 1) = x + y 1. Como x 1 y y 1, tenemos
que z max{x, y} > |x y| y z < x + y. Luego, tenemos que existe un triangulo
no degenerado cuyos lados miden, x, y y x + y 1. Ahora bien, veamos que no existe
otra funcion.

50a Olimpiada Internacional

56

1. Demostraremos que f (1) = 1. Supongamos que f (1) = 1 + m > 1. Considerando el triangulo cuyos lados miden 1, f (y) y f (y + m), tenemos que
1 + f (y) > f (y + m) > f (y) 1,
lo cual implica que f (y + m) = f (y). Luego, f es m-periodica y por lo tanto es
acotada. Sea B una cota, es decir f (x) B para toda x. Si escogemos x > 2B,
entonces
x > 2B f (y) + f (y + f (x) 1),
lo cual es una contradiccion, ya que el triangulo de lados x, f (y) y f (y+f (x)1)
sera degenerado. Por lo tanto, f (1) = 1.
2. 2. Demostraremos que para todo entero z tenemos que f (f (z)) = z. Tomando
el triangulo cuyos lados miden z, f (1) = 1 y f (1 + f (z) 1) = f (f (z)), se
deduce que f (f (z)) = z para todo z.
3. Demostraremos que para todo entero z 1, se cumple que f (z) z. Supongamos que existe alguna z tal que f (z) > z. Sea f (z) = w + 1. Por el Paso
1 sabemos que w z 2. Sea M = max{f (1), f (2), . . . , f (w)} el mayor
valor de f para los primeros w enteros. Primero demostraremos que no existe un
entero positivo t con
z1
t + M,
(8)
f (t) >
w
de lo contrario descomponemos el menor valor t como t = wr + s, donde r es
un entero y 1 s w.
Por reduccion al absurdo, supongamos que (8) es valida y tomemos t el mnimo
numero que cumple (8). Por la definicion de M tenemos que t > w. Tomando el
triangulo con lados z, f (t w) y f ((t w) + f (z) 1) tenemos que
z + f (t w) > f ((t w) + f (z) 1) = f (t).
Entonces,
f (t w) f (t) (z 1) >

z1
(t w) + M,
w

en contradiccion con que t es el mnimo. Por lo tanto, la desigualdad (8) falla


para toda t 1, y hemos probado que
f (t)

z1
t + M.
w

(9)

Ahora bien, utilicemos la desigualdad (9) para terminar de demostrar el Paso 3.


Como z w, tenemos que z1
w < 1 y podemos escoger un entero t suficientemente grande que cumpla la condicion


z1
w

2

t+


z1
+ 1 M < t.
w

50a Olimpiada Internacional

57

Aplicando dos veces la desigualdad (9) tenemos




z1 z1
z1
f (t) + M
t+M +M <t
f (f (t))
w
w
w
en contradiccion con el Paso 2. Por lo tanto, para todo entero z 1, tenemos
que f (z) z.
4. Finalmente, por el Paso 2 y el Paso 3, obtenemos que
z = f (f (z)) f (z) z.
Por lo tanto, f (z) = z para todo entero positivo z.
Problema 6. Sean a1 , a2 , . . . , an enteros positivos distintos y M un conjunto de n 1
enteros positivos que no contiene al numero s = a1 + a2 + + an . Un saltamontes
se dispone a saltar a lo largo de la recta real. Empieza en el punto 0 y da n saltos hacia
la derecha de longitudes a1 , a2 , . . . , an , en algun orden. Demostrar que el saltamontes
puede organizar los saltos de manera que nunca caiga en un punto de M .
Solucion de Pablo Soberon Bravo. La demostracion la haremos por induccion fuerte
sobre n. Para eso ordenemos los pasos a1 < a2 < < an y los elementos de M ,
b1 < b2 < < bn1 . Sea s = a1 + a2 + + an1 . Si quitamos a an y a bn1
tenemos dos casos.
1. s no esta entre los primeros n 2 elementos de M . En este caso por induccion
ordenamos los primeros n 1 saltos para llegar a s . Si en algun momento llegamos a bn cambiamos ese u ltimo paso por an y luego seguimos de cualquier
manera para llegar a s.
2. s es uno de los primero n 2 elementos de M . Si esto sucede, entonces como
s = san esta entre M , entre los 2(n1) numeros de la forma sai , sai an
con 1 i n 1 hay a lo mas n 2 en M . Con esto hay un numero i tal que
s ai y s ai an no estan en M . Notemos que entre s ai an y s debe
haber al menos 2 numeros de M , por lo que por induccion podemos usar los
otros n 2 saltos para llegar a s ai an , luego usar ai y luego an para llegar
a s.
Solucion alternativa. Representemos una ruta del saltamontes por la secuencia de ndices (i1 , i2 , i3 , . . . , in ), si realizo los saltos consecutivos ai1 , ai2 , ai3 , . . . , ain . Demostraremos por induccion. Los casos n = 0, 1 son triviales. Ahora supongamos que para
todos los valores menores que n el saltamontes puede organizar los saltos de manera
que nunca caiga en un punto de M . Podemos suponer que a1 < a2 < a3 < < an .
Sea d = min M .
1. Supongamos que d < an .
Si an no pertenece a M , entonces por hipotesis de induccion el saltamontes
puede saltar de an a s, no pasando por los puntos de M \{d}, utilizando los

50a Olimpiada Internacional

58

saltos a1 , . . . , an1 . Luego, si ponemos a an al inicio de la secuencia de saltos


obtendremos la ruta deseada.
Ahora supongamos que an pertenece a M . Consideremos n conjuntos ajenos
por parejas {an }, {a1 , a1 + an }, {a2 , a2 + an }, . . . , {an1 , an1 + an }. Uno de
ellos no intersecta a M , digamos que es {ai , ai + an }. Entonces
|M [ai + an , s]| n 3,
pues d < an < ai + an . Luego, el saltamontes puede saltar de ai + an a s
utilizando todos los saltos excepto ai y an . Entonces, poniendo ai y an al inicio
de la secuencia de saltos obtendremos la ruta deseada.
2. Supongamos que d an .

M = M \{d}. Por hipotesis de induccion, el saltamontes puede saltar de an a s,


sin pasar por los puntos de M , utilizando los saltos a1 , . . . , an1 y la correspndiente secuencia de ndices (i1 , i2 , i3 , . . . , in1 ). Si la ruta no pasa tambien por d
(en particular, se cumple si d > an ), entonces la secuencia U = (n, i1 , . . . , in1 )
provee la ruta deseada. De lo contrario, esta ruta contiene exactamente un punto de M (el punto d), y tenemos que an + ai1 + + aik = d para algun
0 k n 1.
En el u ltimo caso, consideremos la ruta representada por
(i1 , . . . ik+1 , n, ik+2 , . . . , in1 ).

Como ai1 + + aik+1 < ai1 + + aik + an = d, el saltamontes no puede


caer en los puntos de M durante los primeros k + 1 saltos. Por otro lado, durante
el resto del recorrido cae en los mismos puntos que en la ruta U (y estos puntos
no son menores que ai1 + + aik+1 + an > d), de modo que tampoco caera en
los puntos de M . Por lo tanto, hemos encontrado las rutas deseadas en todos los
casos.

Informacion Olmpica
A continuacion presentamos las actividades programadas por el comite organizador de
la Olimpiada Mexicana de Matematicas para el primer trimestre del ano 2010.
Segunda quincena de enero
Envo de material a los estados: primer numero de la revista Tzaloa, convocatoria, trptico y nombramiento de delegado.
Del 21 al 31 de enero de 2010 en Colima, Colima
Entrenamientos para los seleccionados nacionales y aplicacion de tres examenes
de entrenamiento.
Primera quincena de marzo
Envo a los estados del primer examen de practica propuesto por el Comite Organizador de la Olimpiada Mexicana de Matematicas.
Del 4 al 14 de marzo en Guanajuato, Guanajuato
Entrenamientos para los seleccionados nacionales y aplicacion de dos examenes
de entrenamiento y del examen de la XXIII Olimpiada de la Cuenca del Pacfico.
19 y 20 de marzo
Aplicacion, en los estados resgistrados con este proposito, del primer examen
de practica propuesto por el Comite Organizador de la Olimpiada Mexicana de
Matematicas.
Del 26 al 28 de marzo en Guanajuato, Guanajuato
Curso de entrenadores.

60

Informacion Olmpica

Apendice
Teorema 1 (Factorizacion en primos) Todo entero n mayor que 1 puede expresarse
como un producto de primos (con, tal vez, solamente un factor).
Ver [5, 7].
Criterios 2 (Divisibilidad) Un numero entero es divisible
entre 2, si el dgito de las unidades es un numero par.
entre 3, si la suma de sus dgitos es divisible entre 3.
entre 4, si el numero formado por los dos u ltimos dgitos (el de las unidades y el
de las decenas) es divisible entre 4.
entre 5, si el dgito de las unidades es 5 o 0.
entre 6, si es divisible entre 2 y 3.
entre 7, si lo es tambien el numero de dos cifras que obtengamos con el siguiente
proceso: tomamos el dgito de las unidades y lo duplicamos; el resultado se lo
restamos al numero original sin el dgito de las unidades; repetimos el proceso
hasta obtener un numero de dos cifras.
entre 8, si el numero formado por sus tres u ltimos dgitos es divisible entre 8.
entre 9, si la suma de sus dgitos es divisible entre 9.
entre 10, si el dgito de las unidades es 0.
entre 11, si obtenemos 0 o un multiplo de 11 con el siguiente proceso: numeramos todos los dgitos del numero de izquierda a derecha. Sumamos todos los
dgitos que ocupan un lugar par en el numero y le restamos la suma de todos los
dgitos que ocupan una posicion impar en el numero.
Ver [7, 8].

62

Apendice

Definicion 3 (Modulos) Dados dos numeros enteros positivos n, m decimos que n


n
=
r(mod m) si el residuo al dividir n entre m es igual a r. Dicho de otra forma, m
r
d + m.
Ver [8].
teorema de Fermat) Si p es un numero primo, para cualquier
Teorema 4 (Pequeno
numero natural a tenemos que ap a (mod p).
Ver [5, 7].
Teorema 5 (Desigualdad media aritmetica - media geometrica) Si x1 , x2 , . . . , xn son
numeros reales positivos, entonces

x1 + x2 + + xn
n x1 x2 xn
n
y la igualdad se cumple si y solo si x1 = x2 = = xn .
Ver [3].
Teorema 6 (Formula de Pascal) Para dos enteros no negativos n, m con n > m tenemos que
  
 

n
n1
n1
=
+
.
m
m
m1
Ver [4, 6, 9].
Teorema 7 (Formulas de a rea)
El a rea de un rectangulo de lados a y b es a b.
El a rea de un triangulo es igual a 21 hl, donde l es la medida de un lado y h es la medida
de la altura sobre dicho lado.
El a rea de un crculo de radio r es igual a r2 .
Ver [1, 2].

Definicion 8 (Angulos
entre paralelas) Cuando una recta intersecta a otras dos rectas se forman ocho a ngulos que numeramos del 1 al 8, como se muestra en la figura.
l1

1
3

l2
2

l3

5
7

Si la recta l3 intersecta a las rectas l1 y l2 , decimos que es transversal a ellas. Los


a ngulos 2, 4, 5 y 7 estan entre las rectas l1 y l2 , los llamamos a ngulos internos, los
a ngulos restantes los llamamos a ngulos externos. Los a ngulos en lados opuestos por
la transversal l3 se llaman a ngulos alternos, como por ejemplo 3 y 5. A los a ngulos 4
y 5 les llamamos alternos internos y los a ngulos 3 y 6 son alternos externos.
A los a ngulos que estan en la posicion correspondiente respecto a la transversal, como
por ejemplo 3 y 7 los llamamos a ngulos correspondientes. Entonces, los pares de
a ngulos correspondientes en la figura anterior son 3 y 7, 1 y 5, 4 y 8, 2 y 6.

Apendice

63

Si l1 y l2 son paralelas los a ngulos alternos internos son iguales.


Ver [2].
Teorema 9 (Teorema de Thales) Consideremos tres rectas y dos rectas transversales
a e stas como se muestra en la figura. Tenemos que si AD, BE y CF son paralelas
AB
DE
AB
= DE
entonces BC
EF . Recprocamente, si BC = EF y dos de las rectas AD, BE o
CF son paralelas, entonces las tres rectas son paralelas.

A
B
C

D
E
F

Ver [1, 2].


Teorema 10 (Suma de los a ngulos internos de un triangulo) La suma de los a ngulos internos de un triangulo es 180.
Ver [1, 2].
Teorema 11 (Teorema de Pitagoras) En un triangulo rectangulo, el cuadrado de la
hipotenusa es igual a la suma de los cuadrados de los catetos.
Ver [1, 2] y el artculo de este numero.
Lema 12 (Segmento entre puntos medios) El segmento que une los puntos medios
de dos lados de un triangulo es paralelo al tercer lado y de longitud igual a la mitad
de tal tercer lado.
Ver [1, 2].
Definicion 13 (Congruencia de triangulos) Los triangulos ABC y A B C son congruentes si los a ngulos y los lados del triangulo ABC son iguales a los a ngulos y los
lados del triangulo A B C .
Ver [1, 2].
Criterio 14 (Criterio de congruencia ALA) Un criterio de congruencia de triangulos nos dice que si tenemos dos triangulos con un lado igual y dos a ngulos adyacentes
iguales, entonces son congruentes. A este criterio se le conoce como a ngulo-ladoa ngulo y lo denotamos como ALA.
Ver [1, 2].
Criterio 15 (Criterio de congruencia LLL) Un criterio de congruencia de triangulos nos dice que si tenemos dos triangulos con sus tres lados correspondientes iguales,
entonces son congruentes. A este criterio se le conoce como lado-lado-lado y lo denotamos como LLL.
Ver [1, 2].

64

Apendice

Definicion 16 (Semejanza de triangulos) Los triangulos ABC y A B C son semejantes, si sus a ngulos respectivos son iguales, es decir,
ABC = A B C
ACB = A C B
BAC = B A C
y sus lados homologos son proporcionales, esto es
BC
CA
AB
= = .

AB
BC
CA
Ver [1, 2].
Criterio 17 (Criterio de semejanza AA) Si dos pares de a ngulos correspondientes
de los triangulos ABC y A B C son iguales, entonces los triangulos son semejantes.
A esta relacion le llamamos a ngulo-angulo y la denotamos como AA.
Ver [1, 2].
Definicion 18 (Bisectriz) Dado un a ngulo ABC su bisectriz es la recta que lo divide
en dos a ngulos iguales. Equivalentemente, la bisectriz del a ngulo ABC es la recta
que equidista de AB y BC.
Ver [1, 2].
Teorema 19 (Bisectrices) Las bisectrices internas de un triangulo concurren en un
punto que es el centro de la circunferencia inscrita en el triangulo. El punto de concurrencia se llama el incentro.
Ver [1, 2].
Teorema 20 (Ley de los cosenos) En un triangulo de lados a, b y c, se cumple la relacion
a2 = b2 + c2 2bc cos ,
donde es el a ngulo opuesto al lado a.
Ver [2].

Teorema 21 (Medida del a ngulo inscrito) La medida de un a ngulo inscrito en una


circunferencia es igual a la mitad del arco comprendido entre sus lados, es decir, la
mitad del a ngulo central que subtiende el mismo arco.
Ver [1, 2].
Definicion 22 (Cuadrilatero cclico) Un cuadrilatero es cclico si existe un crculo
que pase por lo cuatro vertices.
Ver [2].
Teorema 23 (Cuadrilatero cclico) Un cuadrilatero ABCD es cclico si y solo si la
suma de los a ngulos opuestos es igual a 180 , es decir, si y solo si
DAB + BCD = ABC + CDA = 180 .
Ver [2].

Bibliografa
[1] A. Baldor. Geometra plana y del espacio. Publicaciones Cultural, Mexico, 1999.
[2] R. Bulajich Manfrino, J. A. Gomez Ortega. Geometra. Cuadernos de Olimpiadas
de Matematicas. Instituto de Matematicas de la UNAM, 2002.
[3] R. Bulajich Manfrino, J. A. Gomez Ortega, R. Valdez Delgado. Desigualdades. Cuadernos de Olimpiadas de Matematicas. Instituto de Matematicas de la
UNAM. Tercera edicion, 2007.
[4] R. Grimaldi, Matematicas Discreta y Combinatoria. Addison Wesley Longman,
Pearson. Tercera edicion, 1998.
[5] I. Niven, H. Zuckerman. Introduccion a la Teora de los Numeros. Limusa-Wiley,
Mexico 1972.
[6] M. L. Perez Segu. Combinatoria. Cuadernos de Olimpiadas de Matematicas.
Instituto de Matematicas de la UNAM, 2000.
[7] M. L. Perez Segu. Teora de Numeros. Cuadernos de Olimpiadas de Matematicas.
Instituto de Matematicas de la UNAM, 2003.
[8] A. Rechtman Bulajich, C.J. Rubio Barrios. Divisibilidad y congruencias. Revista
de la Olimpiada Mexicana de Matematicas, Tzaloa No. 2, 2009.
[9] N. Vilenkin. De cuantas formas? (Combinatoria). Editorial Mir, Moscu 1972.

66

Directorio

Directorio de los delegados estatales


AguascalientesLaura Soledad Casillas Serna
CECYTEA, Plantel Morelos

Area
de Matematicas y Fsica de Ingeniera
Chichen-Itza s/n, Cd. Satelite Morelos
Rincon 505, Colonia Guadalupe
C.P. 20059, Aguascalientes, Aguascalientes
Cel. (449) 414 13 85
lscasillass@yahoo.com.mx
www.ommags.com
Baja CaliforniaCarlos Yee Romero
Universidad Autonoma de Baja California
Facultad de Ciencias
Km. 103 Carretera de Tijuana-Ensenada
Unidad Universitaria
C.P. 22860, Ensenada, Baja California
Tel. (646) 1 74 59 25, ext. 116
Fax (646) 1 74 45 60
cyeer@uabc.mx
Baja California SurEdgar Netzahualcoyotl Soriano Arellano
Instituto Mar de Cortes
Desierto de Sonora esquina Prolongacion Francisco J. Mujica
Fracc. Villas de San Lorenzo
C.P. 23000, La Paz, Baja California Sur
Tel. y Fax (612) 123 22 02
netza soriano@hotmail.com
direccion@institutomardecortes.edu.mx

68

Directorio
CampecheJuan Jesus Moncada Bolon
Universidad Autonoma de Campeche
Facultad de Ingeniera
Av Agustn Melgar s/n entre Juan de la Barrera y Calle 20
Col. Lindavista, CP 24030
San Francisco de Campeche, Campeche
Tel. (981) 811 98 00 ext 70000
Cel. (981) 117 52 07
jjmb72@gmail.com

ChiapasMara del Rosario Soler Zapata


Universidad Autonoma de Chiapas
Centro de Estudios en Fsica y Matematicas Basicas y Aplicadas
Calle 4a Oriente 1428, entre 13a y 14a Norte
Tuxtla Gutierrez, Chiapas
Tel. (961) 6 18 30 21
msolerza@unach.mx

ChihuahuaDavid Cosso Ruiz


Universidad Autonoma de Ciudad Juarez
Instituto de Ingeniera y Tecnologa
Departamento de Fsica y Matematicas
Av. del Charro 450 Nte
C.P. 32310, Cd. Juarez, Chihuahua
Tel. (656) 6 88 48 87
Fax (656) 6 88 48 13
sirio11@gmail.com
http://ommch.blogspot.com

CoahuilaSilvia Carmen Morelos Escobar


Universidad Autonoma de Coahuila
Facultad de Ciencias Fsico Matematicas
Edif. D, Unidad Camporredondo
C.P. 25000, Saltillo, Coahuila
Tel. (844) 414 47 39, 414 88 69 y 411 82 57
Fax (844) 411 82 57
Tel. casa (844) 431 34 85 y Tel. cel. (844) 437 72 19
smorelos@mate.uadec.mx
silvia.morelos@gmail.com

Directorio
ColimaIng. Martn Eliseo Isaas Ramrez
Universidad de Colima, Facultad de Ciencias
Bernal Daz del Castillo 340
Col. Villa San Sebastian
C.P. 28045, Colima, Colima
Tel. (312) 31 610 00, ext. 47058
http://ommcolima.ucol.mx
ommcol@ucol.mx
martin isaias@ucol.mx
Distrito FederalAlejandro Bravo Mojica
Universidad Nacional Autonoma de Mexico
Facultad de Ciencias, Departamento de Matematicas, cubculo 230
Circuito Exterior, Ciudad Universitaria
C.P. 04510, Mexico D.F
Tel. (55) 56 22 38 99 # 45747
Fax (55) 56 22 54 10
abm@hp.fciencias.unam.mx
DurangoArmando Mata Romero
Universidad Juarez del Estado de Durango
Escuela de Matematicas
Av. Veterinaria 210, Col. Valle del Sur
C.P. 34120, Durango, Durango
Tel. y Fax (618) 1 30 11 39
angel hiram@hotmail.com
GuanajuatoDr. Manuel Cruz Lopez
Departamento de Matematicas
Universidad de Guanajuato
Callejon Jalisco s/n Valenciana Guanajuato, Guanajuato
Tel. (473) 1 02 61 02 ext. 1203
direc.demat@quijote.ugto.mx
GuerreroGonzalo Delgado Espinoza
Universidad Autonoma de Guerrero
Facultad de Matematicas
Calle Carlos E. Adame 54, Col. Garita
C.P. 39650, Acapulco, Guerrero
Tel. y Fax: (744) 4 87 25 00
Tel. cel. (744) 4 30 92 54
deg gonzalo@yahoo.com.mx

69

70

Directorio
HidalgoAnna Tarasenko
Universidad Autonoma del Estado de Hidalgo
Edif. Centro de Investigacion en Matematicas, Instituto de Ciencias Basicas
Carretera Pachuca Tulancingo km. 4.5
C.P. 42074, Mineral de la Reforma, Hidalgo
Tel. (771) 7 17 21 58
Fax (771) 7 17 21 58
anataras@uaeh.edu.mx
JaliscoMara Eugenia Guzman Flores
Universidad de Guadalajara
Centro Univ. de Ciencias Exactas e Ingeniera, Departamento de Matematicas
Av. Revolucion 1500, Edificio V, planta baja
C.P. 44420, Guadalajara, Jalisco
Tel. y Fax (33) 13 78 59 00 ext. 7753
floresguz55@yahoo.com.mx
Estado de MexicoOlga Rivera Bobadilla
Universidad Autonoma del Estado de Mexico
Facultad de Ciencias
Instituto Literario No. 100, Col. Centro
C.P. 50000, Toluca, Estado de Mexico
Tel. (722) 2 96 55 56
Fax (722) 2 96 55 54
orb@uaemex.mx
matematicas olimpiada@yahoo.com.mx
MichoacanArmando Sepulveda Lopez
Universidad Michoacana de San Nicolas de Hidalgo
Departamento de Matematica Educativa
Facultad de Ciencias Fsico Matematicas, Ciudad Universitaria
C.P. 58060, Morelia, Michoacan
Tel. (443) 3 26 21 46, ext. 130 y (443) 3 22 35 00, ext. 3063
asepulve@umich.mx
MorelosLarissa Sbitneva
Universidad Autonoma del Estado de Morelos, Facultad de Ciencias
Av. Universidad 1001, Col. Chamilpa
C.P. 62209, Cuernavaca, Morelos
Tel. (777) 3 29 70 20
Fax (777) 3 29 70 40
larissa@uaem.mx

Directorio
NayaritFrancisco Javier Jara Ulloa
Universidad Autonoma de Nayarit
Secretara de educacion media y superior
Cd. de la cultura, Amado Nervo
C.P. 63157, Tepic, Nayarit
Tel. (311) 2 11 88 00 ext. 8809
jaraulloa@gmail.com
Nuevo LeonAlfredo Alans Duran
Universidad Autonoma de Nuevo Leon
Facultad de Ciencias Fsico-Matematicas
Del Colegio 1077
Col. Valle de las Flores
C.P. 66450, San Nicolas, Nuevo Leon
Tel. (81) 83 29 40 30, ext. 6130 y (81) 83 13 16 26
Fax (81) 83 52 29 54
aalanis56@hotmail.com
OaxacaSara Carrillo Uribe
Academia de Matematicas, Escuela de Ciencias
Universidad Autonoma Benito Juarez de Oaxaca
Ciudad Universitaria, Av. Universidad s/n
Ex Hacienda de 5 Seores, CP 68120, Oaxaca, Oaxaca
Tel. y Fax (951) 1 44 80 56
mushe wini@hotmail.com
PueblaMara Araceli Juarez Ramrez
Benemerita Universidad Autonoma de Puebla
Facultad de Ciencias Fsico-Matematicas
San Claudio y Ro Verde, Ciudad Universitaria
C.P. 72570, Puebla, Puebla
Tel. (222) 2 29 55 00 ext. 7578
Fax (222) 2 29 56 36
arjuarez@fcfm.buap.mx
QueretaroTeresa de Jesus Valerio
Universidad Autonoma de Queretaro, Facultad de Ingeniera
Cerro de las Campanas s/n
Centro Universitario
C.P. 76010, Queretaro, Queretaro
Tel. (442) 1 92 12 00, ext. 6015
valeriotere@gmail.com
teresa.valerio@webtelmex.net.mx

71

72

Directorio
Quintana RooAlicia Ramon Barrios
Colegio de Bachilleres
Planteles Cancun 2 y Colegio Britanico
Region 236, Manzana 24, Lote 5
C.P. 77500, Cancun, Quintana Roo
Tel. (998) 1 74 01 56
Fax (998) 8 88 72 04 y (998) 8 84 12 95
olimpiadasquintanaroo@hotmail.com
tita1970@hotmail.com
San Luis PotosEugenio Daniel Flores Alatorre
Universidad Autonoma de San Luis Potos
Instituto de Fsica
Av. Salvador Nava 6, Zona Universitaria
C.P 78290, San Luis Potos, San Luis Potos
Tel. (444) 8 26 23 62 al 65
Fax (444) 8 13 38 74
floreseugenio@hotmail.com
SinaloaNicolas Pardo Viera
Universidad Autonoma de Sinaloa
Escuela de Ciencias Fsico-Matematicas
Ciudad Universitaria
C.P. 80010, Culiacan, Sinaloa
Tel. y Fax (667) 7 16 11 54
pardo@uas.uasnet.mx
SonoraMisael Avendao Camacho
Universidad de Sonora
Departamento de Matematicas
Av. Rosales y Boulevard Domnguez s/n, Col. Centro
C.P. 83000, Hermosillo, Sonora
Tel. (662) 2 59 21 55
Fax (662) 2 59 22 19
misaelave@mat.uson.mx
TabascoAntonio Guzman Martnez
Division Academica de Ciencias Basicas
Unidad Chontalpa
Universidad Juarez Autonoma de Tabasco
Km. 1 Carretera Cunduacan, Jalpa de Mendez
C.P. 86690, Cunduacan, Tabasco
Tel. y Fax (914) 3 36 09 28 y (914) 3 36 03 00
antonio.guzman@dacb.ujat.mx
antonioguzman@hotmail.com

Directorio
TamaulipasRamon Jardiel Llanos Portales
Universidad Autonoma de Tamaulipas
Unidad Academica Multidisciplinaria de Ciencias, Educacion y Humanidades
Centro Universitario Victoria
Cd. Victoria, Tamaulipas
Tel. (834) 3 18 17 23
rllanos@uat.edu.mx
rjardiel5@hotmail.com

TlaxcalaJose Erasmo Perez Vazquez


Universidad Autonoma de Tlaxcala
Facultad de Ciencias Basicas
Calzada a Apizaquito Km 1.5
Apartado Postal 140
C.P. 90300, Apizaco, Tlaxcala
Tel. (241) 4 17 25 44
Fax (241) 4 17 25 44 y (241) 4 17 58 44
joserasmo25@gmail.com

VeracruzRaquiel Rufino Lopez Martnez


Universidad Veracruzana, Facultad de Matematicas
Circuito Gonzalo Aguirre Beltran s/n, Lomas del Estadio
Zona Universitaria, Col. Centro
Apartado Postal 270
C.P. 91090, Xalapa, Veracruz
Tel. (228) 8 18 24 53 y (228) 8 42 17 45
Fax (228) 8 18 24 53
ralopez@uv.mx
raquiel1971@yahoo.com.mx

YucatanDidier Adan Sols Gamboa


Universidad Autonoma de Yucatan
Facultad de Matematicas
Periferico Norte Tablaje 13615
Parque industrial, junto al local del FUTV
C.P. 97110, Merida, Yucatan
Tel. (999) 9 42 31 47, ext 1102
Fax (999) 9 42 31 40
didier.solis@uady.mx
ommyuc@tunku.uady.mx

73

74

Directorio
ZacatecasNancy Janeth Calvillo Guevara
Universidad Autonoma de Zacatecas
Unidad Academica de Matematicas
Camino a la Bufa s/n, interseccion con Calzada Solidaridad
C.P. 98068, Zacatecas, Zacatecas
Tel. (492) 922 99 75 ext. 31 y (492) 923 94 07 ext. 1703
ncalvill@mate.reduaz.mx
matematicas.reduaz.mx

Directorio

75

Directorio del Comite Organizador de la OMM

Anne Alberro Semerena


Facultad de Ciencias, UAEM
Av. Universidad 1001
62210, Cuernavaca, Morelos
Tel. (777) 3 81 03 80
Fax (777) 3 29 70 40
aalberro@buzon.uaem.mx

Gerardo Arizmendi Echegaray


Centro de Investigacion en Matematicas
Callejon Jalisco s/n, Mineral de Valenciana
36240, Guanajuato, Guanajuato
Tel. (473) 7 32 71 55
gerardo@cimat.mx

Ignacio Barradas Bribiesca


Universidad de Guanajuato
L. de Retana #5, Centro
36000, Guanajuato, Guanajuato
Tel. (473) 7 32 00 06 ext 2006
barradas@quijote.ugto.mx

Radmila Bulajich Manfrino


Facultad de Ciencias, UAEM
Av. Universidad 1001
62210, Cuernavaca, Morelos
Tel. (777) 3 29 70 20
Fax (777) 3 29 70 40
bulajich@servm.fc.uaem.mx

Gabriela Campero Arena


Facultad de Ciencias, UNAM
Av. Universidad 3000
04510, Mexico, D.F.
Tel. (55) 56 22 48 67
Fax (55) 56 22 48 66
gabriela@matematicas.unam.mx

Fernando Campos Garca

1a de Angel
Rico 85
AU.H. Vicente Guerrero
09200, Iztapalapa, Distrito Federal
Tel. (55) 34 63 75 43
fermexico89@hotmail.com

Jose Antonio Climent Hernandez


Facultad de Ciencias, UNAM
Av. Universidad 3000
04510, Mexico, D.F.
Tel. (55) 56 24 59 22
Fax (55) 56 22 48 59
jach@fciencias.unam.mx

Jose Alfredo Cobian Campos


Facultad de Ciencias, UNAM
Av. Universidad 3000
04510, Mexico, D.F.
Tel. (55) 56 22 49 25
Fax (55) 56 22 48 59
cobian@matematicas.unam.mx

Luis Cruz Romo


UPIITA, IPN
Av. Instituto Politecnico Nacional 2580
Col. Barrio la Laguna Ticoman
07340, Mexico, D.F.
lucruz@ipn.mx

Marco Antonio Figueroa Ibarra


Facultad de Matematicas
Universidad de Guanajuato
Callejon Jalisco s/n, Mineral de Valencia
36240, Guanajuato, Guanajuato
Tel. (473) 7 32 01 40
marcant@cimat.mx

76

Directorio

Jesus
Jeronimo Castro
CIMAT
Apartado Postal 402
36000, Guanajuato, Guanajuato
Tel. (473) 7 32 71 55
Fax (473) 7 32 57 49
jeronimo@cimat.mx

Leonardo Ignacio Martnez Sandoval


Primera Cerrada de Alfalfares 41-2
Rinconada Coapa Primera Sec, Tlalpan
14330, Mexico, D.F.
Tel. (55) 26 52 23 29
ssbmplayer@gmail.com

Carlos Jacob Rubio Barrios


Universidad Autonoma de Yucatan
Periferico norte tablaje 13615
97119, Merida, Yucatan
Tel. (999) 942-3140 al 49
Fax (999) 942-31-40
carlos.rubio@uady.mx
jacob.rubio@gmail.com

Elena Ruiz Velazquez


Altair 12
Col. Lomas de Palmira
62550, Cuernavaca, Morelos
Tel. (777) 320 54 39
Cel. (777) 133 39 83
eleniux@gmail.com
A00375640@itesm.mx

Pablo Soberon Bravo


Circuito Interior no. 830
Fracc. La Herradura
62303, Cuernavaca, Morelos
Cel. (777) 134 55 49
bandrak@hotmail.com

Carmen Sosa Garza


Facultad de Ingeniera, UAQ
Cerro de las Campanas s/n
Queretaro, Queretaro
Tel. (442) 1 92 12 64 ext. 121 o 136
Fax (442) 1 92 12 646
carsg@uaq.mx

Rogelio Valdez Delgado


Facultad de Ciencias, UAEM
Av. Universidad 1001
62210, Cuernavaca, Morelos
Tel. (777) 3 29 70 20
Fax (777) 3 29 70 40
rogelio@matcuer.unam.mx

Eduardo Velasco Barreras


Universidad de Sonora
Calle Yucas 16, Vista Bella
83170, Hermosillo, Sonora
Tel. (662) 2 19 10 07
hamsteritokeweb@hotmail.com

Hugo Villanueva Mendez


Instituto de Matematicas, UNAM
Cub. 4 de Becarios,
Circuito Exterior, Ciudad Universitaria
Coyoacan 04510,
Mexico, D.F.
Tel (55) 56 22 45 32
vill hugo@hotmail.com
hvillan@matem.unam.mx

Directorio

77

Direccion Postal de la Olimpiada Mexicana de Matematicas:


Cubculo 201, Departamento de Matematicas.
Circuito Exterior, Facultad de Ciencias.
Universidad Nacional Autonoma de Mexico.
Ciudad Universitaria.
Colonia Copilco, C.P. 04510.
Delegacion Coyoacan.
Mexico, Distrito Federal.
Telefono: (55) 5622-4864.
Fax: (55) 5622-5410.
Email: omm@fciencias.unam.mx
Pagina oficial de la Olimpiada Mexicana de Matematicas:
http://www.omm.unam.mx/

TZALOA
Revista de la Olimpiada
Mexicana de Matematicas
2010, No. 2
Ano

Comite Editorial:
Anne Alberro Semerena
Ana Rechtman Bulajich
Carlos Jacob Rubio Barrios
Francisco Ruiz Benjumeda

Comite de la Olimpiada Mexicana de Matematicas


Cubculo 201
Departamento de Matematicas
Facultad de Ciencias, UNAM
Circuito Interior s/n
Ciudad Universitaria
Coyoacan C.P. 04510
Mexico D.F.
Telefono: (55) 56-22-48-64
www.omm.unam.mx

Diseno de Portada: Manuel Macas Beckmann


www.rayaenmedio.com

Impreso: Torre y de la Torre Impresos


Aragon no. 134

Col. Alamos,
03400
Mexico D.F.
Telefonos: (55) 55-30-14-82 y (55) 55-38-34-53

c
Queda
estrictamente prohibida la reproduccion parcial o total por cualquier sistema
o metodo, mecanico o electronico, sin autorizacion previa del autor.
Impreso y hecho en Mexico.
Abril de 2010.

Contenido

Presentacion

Artculos de matematicas: El Principio de las Casillas

Problemas de practica

Soluciones a los problemas de practica

11

Problemas propuestos
2010 No. 2
Problemas propuestos. Ano
2009 No. 4
Soluciones a los problemas propuestos. Ano

21
21
22

Problemas y Soluciones del Concurso Nacional 2009

29

Problemas y Soluciones de Olimpiadas Internacionales


XXIV Olimpiada Iberoamericana
XI Olimpiada Centroamericana y del Caribe

37
37
43

Informacion Olmpica

49

Apendice

51

Bibliografa

54

Directorio

57

IV

Contenido

Presentacion

Tzaloa es una publicacion periodica trimestral de la Olimpiada Mexicana de Matematicas


y su objetivo es fomentar el estudio de las matematicas como una disciplina dinamica y
creativa. El diseno de las secciones y la cuidadosa seleccion de sus contenidos buscan
apoyar de manera efectiva, con informacion y con materiales de calidad, a estudiantes
y profesores de nivel medio superior que cada ano se preparan para participar en los
diferentes concursos de la Olimpiada de Matematicas.
Esta revista, con orgullo, toma su nombre del nahuatl porque esta hecha por y para los
mexicanos. Tzaloa significa aprender y las paginas que la conforman buscan ayudar a
satisfacer la necesidad de contar con espacios adecuados para profesores, estudiantes
y, en general, para todas aquellas personas interesadas en desarrollar e incrementar sus
capacidades para el razonamiento logico matematico y la resolucion de problemas.

2010, Numero

Tzaloa, Ano
2
El contenido para este segundo numero del ano 2010 se selecciono pensando en estudiantes y profesores que actualmente se estan preparando para participar en las diferentes
etapas de los concursos estatales. De esta manera, las secciones Problemas de Practica
y Problemas Propuestos, estan integradas con material clasificado con niveles introductorio e intermedio. Esperamos que, en su conjunto, los problemas que las conforman
sean un apoyo efectivo para tu preparacion.
Por otro lado, para el artculo de matematicas de este numero, a sugerencia de varios
de nuestros lectores, hemos escogido tratar el Principio de Casillas, tambien conocido
como Principio del Palomar. La amplia experiencia que tiene Pablo Soberon Bravo
en concursos olmpicos se suma con su claridad para exponer, logrando as un material muy atractivo. El lector encontrara que detras de la sencillez de este principio, se
encierra un enorme poder para resolver problemas de gran complejidad. Los ejemplos
escogidos ilustran la enorme cantidad de contextos en que esta herramienta puede ser
aplicada. Asimismo, se debe destacar el tratamiento gradual del nivel de dificultad y

Presentacion

VI

la redaccion concisa, lo anterior permite comprender el principio de forma didactica y


desde un punto de vista muy practico.
Por u ltimo, cabe senalar que en este numero tambien aparecen los problemas y soluciones del Concurso Nacional de 2009, en la seccion correspondiente mencionamos
los nombres de los ganadores y ademas presentamos algunas de las soluciones dadas
por ellos. En el a mbito internacional hemos incluido los examenes con soluciones de
la XXIV Olimpiada Iberoamericana as como de la XI Olimpiada Centroamericana y
del Caribe, donde Mexico participo el ano pasado obteniendo el 5 y 1er lugar respectivamente.

Mexico y las Olimpiadas de Matematicas


Hace mas de 23 anos que la Sociedad Matematica Mexicana ha venido impulsando
vigorosamente los trabajos de la Olimpiada Mexicana de Matematicas (OMM). Desde
sus inicios, este programa se ha visto fortalecido gracias a la participacion de miles
de jovenes estudiantes y a la entusiasta colaboracion de muchos profesores quienes,
de manera espontanea y altruista, han dedicado sus esfuerzos a mejorar la ensenanza
y elevar la cultura matematica de nuestro pas. Motivados por el movimento olmpico,
en escuelas ubicadas a lo largo de todo el territorio nacional, se han desarrollado innumerables talleres de resolucion de problemas, donde estudiantes y profesores trabajan
con el u nico afan de incrementar sus capacidades para el razonamiento, el analisis y la
creatividad matematica.
En el a mbito internacional, mediante la destacada participacion de las delegaciones
mexicanas en diversos concursos, la Olimpiada Mexicana de Matematicas ha contribuido a elevar el prestigio de la matematica nacional. Pero, mas importante aun ha sido
la contribucion que el movimiento olmpico ha tenido para el desarrollo cientfico del
pas. En muchos casos, la deteccion temprana de jovenes con talento matematico excepcional ha permitido brindarles una formacion adecuada para desarrollar al maximo
todo su potencial. Asimismo, la participacion en los concursos olmpicos ha definido
las vocaciones de muchos otros estudiantes. Universidades de todo el pas se han visto beneficiadas con el ingreso de jovenes ex-olmpicos, mismos que cuentan con una
solida formacion matematica y muchos de los cuales han permanecido en ellas para
dedicar su vida profesional a la docencia y la investigacion.

24a Olimpiada Mexicana de Matematicas


El programa anual de la Olimpiada Mexicana de Matematicas se desarrolla en 3 etapas:
Concursos Estatales.
Concurso Nacional.

Presentacion

VII

Entrenamiento, seleccion y participacion de las delgaciones nacionales que representan a Mexico en concursos internacionales.
En la 24a Olimpiada Mexicana de Matematicas podran participar los estudiantes de
Mexico nacidos despues del 1 de agosto de 1991. Los concursantes deberan estar inscritos en una institucion preuniversitaria durante el primer semestre del ciclo escolar
2010-2011 y, para el 1 de julio de 2011, no deberan haber iniciado estudios universitarios. Para mayor informacion puedes consultar la pagina:
http://www.omm.unam.mx
Para la primera etapa, los participantes deberan inscribirse directamente con el Comite Estatal correspondiente.
El Concurso Nacional de la 24a Olimpiada Mexicana de Matematicas se realizara del
21 al 26 de noviembre de 2010 en Ensenada, Baja California. A los primeros lugares
de este certamen se les invitara a la etapa de entrenamiento y seleccion de las delegaciones que representaran a Mexico en las distintas Olimpiadas Internacionales del ano
2011: la XXIII Olimpiada Matematica de la Cuenca del Pacfico, que se llevara a cabo
en el mes de marzo; la XIII Olimpiada Matematica de Centroamerica y el Caribe, que
se celebrara en el mes de junio; la 52a Olimpiada Internacional de Matematicas, que se
llevara a cabo en julio en Amsterdam, Pases Bajos, y la XXVI Olimpiada Iberoamericana de Matematicas que se realizara en el mes de septiembre en Costa Rica.

VIII

Presentacion

El Principio de las Casillas


Por Pablo Soberon Bravo
Nivel Basico

El principio de las casillas es una de las ideas mas importantes a la hora de atacar un
problema de combinatoria. Lo que dice es realmente sencillo:
Siempre que se acomoden al menos n + 1 objetos en n lugares hay un lugar que tiene
al menos 2 objetos.
El primer uso del principio de las casillas tal cual se atribuye a Johann P. G. L. Dirichlet
(1805 - 1859) en 1834. Tambien es llamado el principio del Dirichlet o el principio del
palomar (se enuncia frecuentemente con palomas y palomares en vez de objetos y
lugares).
La demostracion no podra ser mas sencilla. Si hubiera a lo mas un objeto por lugar,
tendramos a lo mas n objetos, lo cual no sucede! A pesar de que este principio parece
completamente inocente, es sorprendente el numero de aplicaciones que tiene y la dificultad de los problemas que se pueden resolver usandolo. Hay una version un poco
mas fuerte de este principio, que dice lo siguiente:
Dados al menos nk + 1 objetos acomodados en n lugares, siempre hay un lugar con
al menos k + 1 objetos.
Iremos viendo algunas formas de utilizarlo a lo largo de este artculo. Comencemos
con el ejemplo mas facil:
Ejemplo 1 De cualesquiera 3 personas siempre hay al menos 2 del mismo sexo.
Aqu consideramos a las personas como los objetos y una casilla donde ponemos a los
hombres y una casilla donde ponemos a las mujeres. A pesar de que la explicacion

El Principio de las Casillas

parece exagerada para este ejemplo, hay que enfatizar que al resolver este tipo de problemas la estrategia siempre sera tratar de decidir cuales son los objetos y las casillas
para que se resuelva el problema. Veamos un ejemplo ligeramente mas complicado,
donde ya no es evidente.
Ejemplo 2 Dados n numeros enteros, demuestra que hay algunos de ellos cuya suma
es multiplo de n. (La suma puede ser de un solo elemento)
Para resolver este ejemplo consideremos a1 , a2 , . . . , an los n numeros y los numeros
b1

a1 ,

b2

=
..
.
=

a1 + a2 ,

bn

a1 + a2 + + an .

Si alguno de los bi es multiplo de n ya acabamos, por lo que podemos suponer que


cada uno de ellos deja algun residuo al dividirlo por n, dicho residuo esta entre 1 y
n 1. Si consideramos a los bi como los objetos y los acomodamos en n 1 lugares
segun su residuo al dividirlo por n, por el principio de las casillas hay dos de ellos que
dejan el mismo residuo. Digamos que son bi y bj con i < j. Como dejan el mismo
residuo al dividirlos por n, su diferencia debe ser multiplo de n. Como bj bi =
ai+1 + ai+2 + + aj hemos acabado.
Ejemplo 3 Demuestra que dados 13 puntos en el plano cartesiano con coordenadas
enteras siempre hay 4 cuyo gravicentro tiene coordenadas enteras.
El gravicentro de 4 puntos es el punto cuyas coordenadas son los promedios de las
coordenadas de los puntos. Por ejemplo, si tomas los puntos (1, 0), (3, 3), (2, 5) y
, 0+3+5+4
) = ( 32 , 3).
(0, 4) su gravicentro es el punto ( 1+3+2+0
4
4
Para resolver este problema hay que trabajar un poco mas antes de aplicar el principio
de las casillas. Vamos a ver primero que de 5 puntos con coordenadas enteras siempre
hay 2 cuyo punto medio tiene coordenadas enteras. Para ver esto, consideremos 4 casillas donde cada una representa alguna de las parejas (0, 0), (0, 1), (1, 0), (1, 1). Vamos
a colocar cada punto en la casilla cuya pareja tenga coordenadas con las mismas paridades que las coordenadas del punto (por ejemplo, el punto (5, 2) va a la casilla que
contiene la pareja (1, 0)). Como hay al menos 2 en la misma casilla, su punto medio
tambien tiene coordenadas enteras, podras explicar por que?
Ya que sabemos esto podemos atacar el problema. Como tenemos al menos 5 puntos
podemos sacar 2 cuyo punto medio tenga coordenadas enteras. Como nos quedan 11
puntos podemos repetir este proceso y seguir sacando parejas hasta que quedan 3 puntos nada mas. Entonces hemos sacado 5 parejas. Ahora veamos que como los 5 puntos
medios de estas parejas tienen coordenadas enteras (por el principio de las casillas)
hay 2 de ellos cuyo punto medio tiene coordenadas enteras. Es facil ver que este punto

El Principio de las Casillas

es realmente el gravicentro de los 4 puntos que generaban a estos u ltimos 2 puntos


medios.
En este ejemplo, ademas de haber necesitado usar el principio de las casillas mas de
una vez, se puede apreciar la fuerza de este tipo de conteos. Resulta que al cambiar el
numero 13 por 12 el teorema deja de ser cierto, puedes encontrar 12 puntos que no
cumplan con el problema?
Una de las a reas de las matematicas donde se encuentra casi siempre el principio de
las casillas es la teora de graficas. Una grafica es un conjunto de puntos en el plano
(llamados vertices) y algunas lneas que unen parejas de estos puntos (llamadas aristas).
Veamos un ejemplo de esto:
Ejemplo 4 Entre cualesquiera 6 personas siempre hay 3 que se conocen dos a dos o
hay 3 que dos a dos no se conocen. (Conocerse es una relacion mutua.)
Para resolver este ejemplo consideremos una grafica con 6 vertices que representan
a las personas y vamos a trazar una arista azul entre dos vertices si esas personas se
conocen o una arista verde si no se conocen. Queremos ver que hay 3 vertices que
forman un triangulo con los lados del mismo color.
Para hacer esto consideremos v0 un vertice cualquiera. Como de e l salen 5 aristas de
dos colores posibles (por el principio de las casillas!) deben salir al menos 3 del mismo
color (digamos que es azul). Llamemos v1 , v2 y v3 a los vertices que estan unidos a v0
por las tres aristas azules. Si dos de esos vertices estan unidos por una arista azul, con
v0 forman el triangulo que buscabamos. Si no, estan unidos por puras aristas verdes,
con lo que tambien tenemos el triangulo que buscabamos.
Resulta que el ejemplo anterior se puede generalizar mucho mas. De hecho para cualesquiera enteros positivos l y s hay un entero m tal que entre cualesquiera m personas
siempre hay l que se conocen todos o s donde no hay dos que se conocen. Para probar
esto se usa un argumento muy similar al que usamos para resolver el ejemplo. Resulta que si queremos encontrar el menor m que cumpla eso el problema ya se vuelve
enormemente difcil. De hecho si l y s son mayores que 5 no se conoce ninguno de
estos numeros m (pero se sabe que existen!).
Ahora ya estamos listos para un ejemplo bastante mas complicado.
Ejemplo 5 (Rusia 2000) En un tablero de 100 100 se colorean las casillas de 4
colores de tal manera que cada fila y cada columna tenga 25 casillas de cada color.
Demuestra que hay 2 filas y 2 columnas tales que sus 4 intersecciones estan pintadas
de colores distintos.
Para resolver este ejemplo primero vamos a contar el numero P de parejas de casillas
(a1 , a2 ) tales que a1 y a2 estan en la misma fila y tienen colores distintos. Como hay
4!
4 colores hay 42 = 2!2!
= 6 maneras de hacer parejas con dos colores distintos. En

cada fila hay 25 casillas de cada color, por lo que debe haber 42 25 25 = 6 25 25

parejas en cada fila. Entonces, P = 100 6 25 25. Sabemos que hay 100
parejas
2
de columnas, y cada pareja de P debe estar en alguna de esas parejas. Es decir, en este
problema los lugares que vamos a utilizar son las parejas de columnas y los objetos las

El Principio de las Casillas

parejas de casillas en la misma fila de distintos colores. Entonces, por el principio de


las casillas, hay una pareja de columnas que tiene al menos
P
100
2

100 6 25 25
10099
2

100 75
12 25 25
=
> 75,
99
99

parejas de P . A partir de ahora solo consideraremos las parejas de casillas en la misma


fila, con colores distintos y que usan estas dos columnas. Veamos que estas dos columnas son las que estamos buscando. Si no nos sirven, entonces cualesquiera dos de las
parejas que acabamos de contar deben compartir al menos un color. Consideremos una
de estas parejas, la cual debe tener dos colores distintos (digamos negro y azul). Como
hay mas de 50 de estas parejas, debe haber alguna que no tenga color negro. Si no
tuviera azul ya habramos acabado por lo que debe tener otro color y azul (digamos
verde y azul). Como hay mas de 50 de estas parejas, debe haber alguna que no tenga
color azul, por lo que debe ser negro y verde. Ya con estas 3 parejas, cualquier otra
debe ser azul y negra o azul y verde o negra y verde. Cada pareja usa 2 de esos colores,
por lo que estaramos usando en total mas de 150 veces estos colores (haba mas de 75
de estas parejas). Pero cada color aparece exactamente 25 veces en cada una de las 2
columnas, por lo que solo se pueden usar 150 veces los colores. Entonces debe haber
dos de las parejas que cumplan la condicion que buscamos.
Ademas de usarse en problemas de combinatoria, el principio de las casillas tambien
se usa en otras a reas. En teora de numeros se puede usar para probar resultados muy
fuertes, como que todo primo de la forma 4k + 1 se puede escribir como suma de
dos cuadrados, o que todo entero positivo se puede escribir como suma de 4 cuadrados. De otra manera para esto se necesita una prueba muy larga o saber mucha teora.
Normalmente las pruebas que salen usando el principio de las casillas suelen ser muy
elegantes.
Otra a rea en la que se puede usar el principio de las casillas es en geometra. Veamos
un par de ejemplos:
Ejemplo 6 Demuestra que no hay una recta que corte los 3 lados de un triangulo.
Para probar eso consideremos como las casillas las 2 partes en la que una recta divide
al plano. Dado un triangulo cualquiera y una recta, dos de sus vertices deben quedar en
la misma parte, por lo que el lado que forman no intersecta a la recta.
Ejemplo 7 Dentro de un cuadrado de lado 1 hay varios crculos cuyos permetros
suman 10. Demuestra que hay una recta paralela a un lado del cuadrado que intersecta
a al menos 4 de estos crculos.
Para ver esto proyectemos a los crculos sobre un lado del cuadrado. Cada crculo se
proyecta en un segmento de longitud igual a su diametro. Entonces se proyectan sobre
umero es mayor que
varios segmentos cuyas longitudes suman en total 10
. Como este n
3 y el lado del cuadrado es 1, hay un punto que recibio al menos 4 proyecciones distintas. Si trazamos por ese punto la recta perpendicular al lado, intersecta a al menos 4 de
los crculos.

El Principio de las Casillas

A continuacion proponemos una lista de ejercicios para que practiques usar este principio.
Ejercicio 1. Demuestra que en toda fiesta siempre hay dos personas que han dado el
mismo numero de saludos.
Ejercicio 2. En un zoologico hay animales de 3 especies distintas y hay 4 jaulas
disponibles. Demuestra que si hay al menos 25 animales entonces en al menos una
jaula hay al menos 2 animales de la misma especie y el mismo sexo.
Ejercicio 3. Demuestra que si se consideran n+1 numeros del conjunto {1, 2, . . . , 2n}
siempre hay dos que son primos relativos.
Ejercicio 4. Demuestra que de 5 enteros positivos siempre hay 3 de ellos cuya suma es
multiplo de 3.
Ejercicio 5. (Olimpiada Iberoamericana, 1998)
En una reunion hay representantes de n pases (n 2) sentados en una mesa redonda.
Se sabe que cualesquiera dos representantes del mismo pas sus vecinos a la derecha
son de pases distintos. Encuentra el mayor numero de representantes que puede haber.
Ejercicio 6. (Vietnam, 2007)
Dado un 2007-agono regular encuentra el menor k tal que entre cualesquiera k vertices
del polgono haya 4 tal que el cuadrilatero convexo que forman comparte 3 lados del
polgono.

Bibliografa
1.- Engel, A. Problem - solving strategies. Springer, 1998.
2.- Perez, M.L. Combinatoria. Cuadernos de Olimpiadas de Matematicas. Instituto
de Matematicas, UNAM. 2000.

El Principio de las Casillas

Problemas de practica

Para este numero hemos escogido 20 problemas cuya dificultad esta clasificada en los
niveles introductorio e intermedio, aunque es probable que algunos de ellos te resulten
difciles de resolver. Destacamos que, ademas de incrementar la dificultad de los problemas, otra diferencia con respecto del numero anterior, es que ahora abandonamos el
formato de opcion multiple, mismo que se acostumbra usar en la primera eliminatoria
de los concursos estatales, para adoptar el formato de pregunta abierta que caracteriza
a las etapas mas avanzadas de la olimpiada.
Te invitamos a poner en practica todas tus habilidades y usar todos tus conocimientos
para encontrar las soluciones de los 20 problemas de este numero. En la siguiente
seccion encontraras las respuestas de todos ellos, pero te recomendamos que no la
consultes sino hasta despues de que hayas llegado por ti mismo a tu propia solucion.
Por u ltimo, te invitamos a contribuir para que esta seccion de la revista se siga enriqueciendo con la participacion de todos. Estamos seguros que concoces y tienes
problemas interesantes que proponer, por eso ponemos a tu disposicion la direccion
revistaomm@gmail.com, donde con gusto recibiremos tus sugerencias.
Problema 1. Si n es un entero positivo divisible entre 7, de los numeros 6, 14, 21, 28 y
42, cual no es necesariamente un divisor de n3 n?
Problema 2. Un cuadrado grande es dividido en uno mas pequeno rodeado por cuatro
rectangulos congruentes como se muestra en la figura. Sabiendo que el permetro de
cada uno de los rectangulos congruentes mide 14 cm, determina el a rea del cuadrado
grande.

Problemas de practica

Problema 3. En un rectangulo de lados 8 cm y 9 cm se dibujan dos circunferencias


de igual radio tangentes entre s y de forma que una de ellas sea tangente a dos lados
consecutivos del rectangulo y la otra tangente a los otros dos. Cuanto mide el radio de
las circunferencias?
Problema 4. A una convencion asisten 50 polticos. Se sabe que:
Cada poltico es honesto o deshonesto (no hay otra posibilidad).
Al menos uno de los polticos es deshonesto.
Dado cualquier par de polticos, al menos uno de los dos es honesto.
Cuantos polticos son deshonestos y cuantos son honestos?
Problema 5. Se ha encuestado a un grupo de 132 alumnos preguntando que les gusta
jugar: basquet o futbol. A 16 alumnos les gustan ambos juegos; el numero de alumnos
a los que les gusta jugar futbol es el doble del numero de alumnos a los que les gusta
jugar basquet y el numero de alumnos a quienes no les gusta jugar ninguno de los dos
juegos es la mitad de quienes solo gustan de jugar futbol. A cuantos alumnos les gusta
jugar futbol?
Problema 6. Un cuadrado con lados de longitud 1 cm se divide en un pentagono y dos
trapecios iguales por medio de segmentos que parten del centro del cuadrado y van a
tres puntos en los lados del cuadrado como se muestra en la figura. Sabiendo que las
a reas de las tres figuras son iguales, determina el valor de x, el lado mas largo de cada
trapecio.
x

Problema 7. Sean a, b, c y d numeros enteros tales que a < 2b, b < 3c, c < 4d y
d < 40. Determina el mayor valor posible de a.
Problema 8. Una semicircunferencia de diametro AB se divide, mediante 29 puntos,
en treinta arcos de igual longitud. Los 29 puntos estan numerados en sentido horario
con los enteros del 1 al 29. Cual es la longitud de la proyeccion, sobre dicho diametro,
del arco
comprendido entre los puntos 5 y 10, sabiendo que la longitud de AB es
2 + 2 3 cm?
Problema 9. Luis tiene 5 pesas A, B, C, D, E que pesan 1 kg, 2 kg, 3 kg, 4 kg, 5 kg,
en algun orden. Utilizando una balanza Luis observo lo siguiente,

Problemas de practica

CDE
BC

AB

Cuanto pesa D?
Problema 10. El numero de cinco dgitos 36aa3 es multiplo de 7. Cual es la suma de
todos los valores posibles de a?
Problema 11. En un triangulo ABC, sea D un punto sobre el lado BC tal que DB =
14 cm, DA = 13 cm y DC = 4 cm. Si se sabe que el radio de la circunferencia
circunscrita al triangulo ADB es igual al radio de la circunferencia circunscrita al
triangulo ADC, determina el a rea del triangulo ABC.
Problema 12. En la piramide, el numero en cada casilla es igual al producto de las dos
casillas que estan abajo. De cuantas formas puedes llenar la piramide? (Dos formas se
consideran distintas si los numeros son distintos).
300

Problema 13. Carlos encontro una piramide de madera con base cuadrada, que estaba un poco maltratada en los vertices. Decidio cortar los vertices con un serrucho.
Cuantas aristas tiene el nuevo solido?
Problema 14. Tres cartas, con un numero entero positivo en cada una, se ponen boca
abajo en una mesa. Se les dice a Paco, Ana y Jacobo que los numeros de las tres cartas
son todos diferentes, en total suman 13 y estan acomodados en orden creciente de
izquierda a derecha. En primer lugar, Paco mira el numero de la carta situada en el
extremo izquierdo y dice, No tengo suficiente informacion para determinar los otros
dos numeros. Despues, Ana mira el numero de la carta del extremo derecho y dice,
No tengo suficiente informacion para determinar los otros dos numeros. Finalmente,
Jacobo mira el numero de la carta de enmedio y dice No tengo suficiente informacion
para determinar los otros dos numeros. Suponiendo que cada persona sabe que las
otras razonan prefectamente bien y que todos han escuchado los comentarios, tienes
suficiente informacion para determinar alguno de los tres numeros?
Problema 15. Un numero telefonico de 7 dgitos d1 d2 d3 d4 d5 d6 d7 se llama memorable si la sucesion d1 d2 d3 coincide exactamente con d4 d5 d6 o con d5 d6 d7 (o con
ambas). Suponiendo que cada di puede ser cualquiera de los dgitos 0, 1, . . . , 9, determina la cantidad de numeros telefonicos memorables.

10

Problemas de practica

Problema 16. Juan tiene muchos cubos blancos identicos. En cada cara de cada cubo
traza una diagonal. Cual es el mayor numero de cubos diferentes que puede obtener?
(Dos cubos son iguales si difieren por una rotacion.)
Problema 17. Determina todos los triangulos rectangulos que tienen lados de longitudes numeros enteros y tales que su a rea es igual a su permetro.
Problema 18. Calcula el valor de la suma
1
1
1
1
+
+
+ +
.
log2 100! log3 100! log4 100!
log100 100!
(Nota: n! = n(n 1) (2)(1)).
Problema 19. En un pizarron estan escritos los numeros 12 , 22 , 32 , . . . , 20102. Curro
y Jacob juegan un juego donde borran alternadamente un numero a la vez hasta que
queden solo dos numeros en el pizarron. Si la diferencia entre estos dos numeros es
un multiplo de 2011, Jacob gana. En caso contrario, gana Curro. Si Curro empieza el
juego, determina quien tiene una estrategia ganadora y explcala.
Problema 20. Cinco enteros positivos a, b, c, d y e mayores que 1 satisfacen las siguientes condiciones.
a(b + c + d + e) =
b(a + c + d + e) =

128,
155,

c(a + b + d + e) =

203,

d(a + b + c + e) =
e(a + b + c + d) =

243,
275.

Determina los valores de a, b, c, d y e.

Soluciones a los problemas de


practica

En esta seccion te presentamos las soluciones que hemos preparado para los 20 problemas de practica que figuran en este numero de tu revista. Date cuenta que para cada
solucion se incluye la explicacion que justifica su validez. Observa que, en todos los
casos, la argumentacion se basa en resultados conocidos y/o en razonamientos logicos
y que para ningun problema la solucion se presenta sin sustento.
Como siempre, las soluciones que presentamos no son u nicas y probabalemente tampoco son las mejores, por lo que es muy posible que tu hayas encontrado una solucion
distinta pero igualmente valida. Si este es el caso y no estas muy seguro de su validez
o simplemente la quieres compartir con nosotros te invitamos para que nos escribas a
revistaomm@gmail.com.

Solucion del problema 1. Podemos escribir n = 7k, para algun entero positivo k, y
tenemos que n3 n = 7k(49k 2 1). Si k = 1 es facil verificar que todos los numeros
6, 14, 21, 28 y 42 son divisores de n3 n. Si k = 2, tenemos que n3 n = 14 195 =
2 3 5 7 13 y el 28 no es divisor.
Solucion del problema 2. Sabemos que cada uno de los rectangulos congruentes tiene
permetro igual a 14 cm. Denotemos por a y b a la base y la altura de los rectangulos.
Como el permetro P = 2a + 2b = 2(a + b) = 14 cm, tenemos que a + b = 7 cm.
Ahora, como cada lado del cuadrado grande mide a + b = 7 cm, tenemos que el a rea
es A = (7)(7) = 49 cm2 .
Solucion del problema 3. Recordemos que las dos tangentes que podemos trazar a
una circunferencia desde un punto exterior a ella, tienen la misma longitud (ver el
teorema 15 del apendice). As, BL = BN = 9 r, CL = CM = 8 r y AM =
AN = r, donde r es el radio de las circunferencias.

12

Soluciones a los problemas de practica

A r M

8r C

r
N
L
9r
B
Por el teorema de Pitagoras (ver el teorema 8 del apendice) tenemos que,
(BL + LC)2

82 + 92

((9 r) + (8 r))2
(17 2r)2

=
=

145
145

4r2 68r + 144 =


r2 17r + 36 =

0
0

17 145
.
2

Como el radio no puede ser mayor que el lado del rectangulo, entonces r =

17 145
2

cm.

Solucion del problema 4. Sea D el poltico deshonesto (sabemos que hay al menos
uno). Para cada par formado por D y otro poltico, como al menos uno es honesto,
el otro poltico necesariamente tiene que ser honesto, es decir, todos los polticos son
honestos excepto D. Por lo tanto, hay un poltico deshonesto y 49 honestos.
Solucion del problema 5. Denotemos por x al numero de alumnos a los cuales les
gusta jugar basquet, entonces a 2x alumnos les gusta el futbol. Sea m el numero de
alumnos a los cuales no les gusta ninguno de los dos juegos, entonces a 2m alumnos
les gusta solo el futbol. Sabemos que a x alumnos les gusta el basquet y a 16 de ellos
les gusta tambien el futbol, luego a (x 16) alumnos les gusta solo el basquet. Con
estos datos, podemos completar el siguiente diagrama.

Basquet
x 16

Futbol
16

2m

Soluciones a los problemas de practica

13

Observemos que 2x = 2m + 16 es el numero de alumnos a los que les gusta el futbol


y que el total de alumnos es x + 3m = 132. Luego, resolviendo el sistema anterior de
dos ecuaciones con dos incognitas, tenemos que
x + 3m = (m + 8) + 3m = 4m + 8 = 132,
de donde m = 31 y x = 39. Por lo tanto, a 2x = 78 alumnos les gusta el futbol.
Solucion del problema 6. Recordemos que el a rea de un trapecio puede calcularse
mediante la formula A = (B+b)h
, donde B es la longitud de la base mayor, b la base
2
menor y h la altura. Observemos que B = x, b = 21 y h = 21 , por lo tanto
A=

(x + 12 ) 12
=
2

x
2

+
2

1
4

2x+1
4

2x + 1
cm2 .
8

Por otro lado, como el a rea del cuadrado es igual a 1 cm2 y como las tres figuras
(los dos trapecios y el pentagono) en que e ste se divide tienen a reas iguales, podemos
concluir que el a rea de cada trapecio es igual a 31 cm2 . Ahora es facil calcular el valor
= 13 , obtenemos que x = 65 cm.
de x, pues resolviendo la ecuacion 2x+1
8
Solucion del problema 7. Como a, b, c y d son enteros, tenemos que
a < 2b a 2b 1,
b < 3c b 3c 1,

c < 4d c 4d 1,
d < 40 d 39.
Luego,
a

2b 1 2(3c 1) 1 = 6c 3
6(4d 1) 3 = 24d 9
24(39) 9 = 927.

Tomando d = 39, c = 4(39)1 = 155, b = 3(155)1 = 464 y a = 2(464)1 = 927,


concluimos que el maximo valor de a es 927.
Solucion del problema 8. Denotemos por P1 , P2 , . . . , P29 a los puntos 1, 2, . . . , 29.
Como se ha dividido a la semicircuenferencia en 30 arcos de igual longitud, la medida

de cada uno de ellos es de 180


30 = 6 .
Sean O y R el centro de la circunferencia y la longitud de su radio, respectivamente.
Entonces, AOP5 = P5 OP10 = 30 , pues cada uno de estos a ngulos abarca 5 arcos
pequenos de 6 cada uno.
Sean M y N las proyecciones de los puntos P5 y P10 sobre el diametro AB, y R el
radio de la semicircunferencia.

14

Soluciones a los problemas de practica

P10
b

b
b

b
b

b
b

P5

30
30

b
b

AM

b
b

b
b

Los triangulos OP5 M y P10 ON son congruentes por el criterio ALA (ver el criterio 10
del apendice) y son la mitad de un triangulo equilatero de lado OP5 = P10 O = R.
Entonces, P5 M = ON = R2 , y aplicando el teorema de Pitagoras (ver el teorema 8
del apendice) tenemos que
OM 2 = R2
Entonces, OM =

3
2 R

R2
3R2
=
.
4
4

y en consecuencia,

1
R( 3 1)
3
R R=
,
M N = OM ON =
2
2
2

pero 2R = 2 + 2 3, luego R = 1 + 3 cm. Entonces,

R( 3 1)
( 3 + 1)( 3 1)
MN =
=
= 1 cm,
2
2
que es la longitud de la proyeccion del arco que va de P5 a P10 sobre el diametro AB.
Solucion del problema 9. Si escribimos las expresiones que corresponden tenemos
que
A+B
B+C

>
=

C+D+E
E.

Observemos que B + C 5, es decir B < 5. Ademas, como la suma de todos los


pesos es igual a 15 tenemos que 5 + 4 A + B > 7 lo que implica que A + B es igual
a 9 u 8. Si A + B = 9, tenemos que A = 5, B = 4 y B + C = 4 + 1 = 5 = E lo que
es una contradiccion. Entonces, A + B = 8. Como B < 5, tenemos que A = 5, B = 3
y B + C = 3 + 1 = 4, es decir C = 1 y E = 4. Por lo tanto, D pesa 2 kg.
Solucion del problema 10. Tenemos que 7 divide a 36aa3 si y solo si 7 divide a
(36003 + aa0). Como 36003 = 7(5143) + 2 y aa0 = 110 a = 7(15 a) + 5 a,
entonces 7 divide a (36003 + aa0) si y solo si 7 divide a 5 a + 2, si y solo si 7 divide
a 5 a + 2 + 28 = 5(a + 6). Pero 7 y 5 son primos relativos, entonces 7 divide a 36aa3

Soluciones a los problemas de practica

15

si y solo si 7 divide a a + 6. Luego, los u nicos valores posibles de a son 1 y 8. Por lo


tanto, la suma de todos los posibles valores de a es 1 + 8 = 9.
Solucion del problema 11. Como los radios de las circunferencias circunscritas son
iguales, se sigue del teorema del a ngulo inscrito (ver el teorema 16 del apendice) que
ACB = ABC, y en consecuencia AB = AC.

Sea AM la altura sobre el lado BC. Como el triangulo ABC es isosceles, tenemos que
AM tambien es mediana, es decir, M es punto medio de BC. Luego, M C = BC
2 =
DB+DC
14+4
18
=
=
=
9
cm
y
M
D
=
M
C

DC
=
9

4
=
5
cm.
Aplicando
el
2
2
2
teorema de Pitagoras (ver el teorema 8 del apendice) en el triangulo AM D, tenemos
que
p
p
AM = DA2 M D2 = 132 52 = 12 cm.
Por lo tanto, el a rea del triangulo ABC es 12 AM BC = 21 (12)(18) = 108 cm2 .

Solucion del problema 12. Los dos numeros del segundo renglon tienen que ser divisores de 300. Como 300 = 22 3 52, tenemos que este numero tiene 3 2 3 = 18
divisores positivos (ver el teorema 2 del apendice), los cuales acomodamos por parejas
como sigue:
(300, 1), (150, 2), (100, 3), (75, 4), (60, 5), (50, 6), (30, 10), (25, 12), (20, 15).
Para llenar el primer renglon de la piramide necesitamos analizar cada uno de los 9
casos anteriores. Si ponemos los numeros (300, 1) en el segundo renglon tenemos una
sola forma de llenar la piramide.
300
300
1
300
1
1
Si tomamos ahora los numeros (150, 2) tenemos dos formas de llenar el primer renglon,
ya sea con los numeros (150, 1, 2) o (75, 2, 1). Analogamente tenemos que para (100, 3)

16

Soluciones a los problemas de practica

hay una forma de llenar la piramide, para (75, 4) hay una, para (60, 5) hay dos formas,
para (50, 6) hay dos, para (30, 10) hay cuatro, para (25, 12) hay una y para (20, 15)
hay dos.
Por lo tanto, en total hay 1 + 2 + 1 + 1 + 2 + 2 + 4 + 1 + 2 = 16 formas de llenar la
piramide.
Solucion del problema 13. Como la piramide tiene base cuadrada, tiene una cara que
es un cuadrado y cuatro que son triangulos. En total tiene, 4 + 4 = 8 aristas.

Al cortar los vertices se crean cuatro caras triangulares y una cuadrada, entonces se
crean 4(3) + 4 = 16 aristas. Por lo tanto, el nuevo solido tiene 16 + 8 = 24 aristas.
Solucion del problema 14. La respuesta es s. Veamos que bajo las condiciones dadas,
el numero de la carta de enmedio forzosamente tiene que ser 4.
Considerando las tres condiciones inciales es facil ver que las u nicas combinaciones
de valores posibles para las cartas son:
(1, 2, 10), (1, 3, 9), (1, 4, 8), (1, 5, 7), (2, 3, 8), (2, 4, 7), (2, 5, 6) y (3, 4, 6).
Despues de que Paco mira la carta de la izquierda y declara que no tiene suficiente
informacion para determinar los valores de las otras dos, sabemos que Paco no vio el
numero 3 y podemos descartar la terna (3, 4, 6). La conclusion anterior se justifica con
base en que Paco razona perfectamente y conoce las tres condiciones inciales, por tanto
sabe que (3, 4, 6) es la u nica combinacion posible de valores que comienza con 3. De
haber visto el numero 3 hubiera podido determinar que los numeros de las otras cartas
eran 4 y 6.
En segundo lugar, despues de que Ana, quien concoce las condiciones iniciales y ha
escuchado el comentario de Paco, mira el numero de la derecha y declara que tampoco
puede determinar el valor de las otras cartas, podemos descartar las ternas (1, 2, 10),
(1, 3, 9) y (2, 5, 6). Sabemos que Ana no pudo ver los numeros 9 o 10, pues bajo las
condiciones inciales estas ternas son u nicas por lo que hubiera sido posible detreminar
los valores de los otros dos numeros. Ana tampoco vio el numero 6, pues aunque incialmente haba dos ternas posibles de la forma (a, b, 6), despues del comentario de Paco,
se ha descartado la posibilidad (3, 4, 6). Si Ana hubiera visto el numero 6, entonces
con facilidad hubiera determinado que los otros numeros eran 2 y 5.
Para cuando llega el turno de Jacobo, ya se han descartado 4 de las 8 posibles ternas
iniciales, por lo que antes de ver la carta de enmedio e l sabe que las u nicas posibles
combinaciones de valores son: (1, 4, 8), (1, 5, 7), (2, 3, 8) y (2, 4, 7).
A partir de aqu, la declaracion de Jacobo al ver el valor de la carta de enmedio, implica
que e l no vio los numeros 5 o 3, pues al ser ternas u nicas, en cualquiera de estos casos

Soluciones a los problemas de practica

17

hubiera podido determinar con seguridad los valores de los otros dos numeros.
Finalmente, considerando las condiciones iniciales y las declaraciones de Paco, Ana y
Jacobo, tenemos que las u nicas ternas posibles son (1, 4, 8) y (2, 4, 7). Es claro que la
carta de enmedio tiene al numero 4 y que no hay suficiente informacion para deteminar
el valor de las otras dos.
Solucion del problema 15. Comenzamos contando todos los valores posibles para
la sucesion d1 d2 d3 . Como cada di tiene 10 valores posibles, tenemos que hay 103
combinaciones distintas para el inicio de un numero memorable. Ahora contemos por
casos segun la terminacion del numero.
Caso 1.- La sucesion d1 d2 d3 coincide exactamente con d4 d5 d6 . En este caso,
como d7 puede tomar cualquier valor, tenemos un total de 103 10 = 104 numeros
memorables.
Caso 2.- La sucesion d1 d2 d3 coincide con d5 d6 d7 . Analogamente, dado que d4
puede tomar culaquier valor, nuevamente tenemos que la cantidad de combinaciones posibles es 104 .
Caso 3.- Numeros en los que d1 d2 d3 coincide con ambas sucesiones (d4 d5 d6
y d5 d6 d7 ). En este caso debe cumplirse que d1 = d4 = d5 , d2 = d5 = d6 y
d3 = d6 = d7 ; de donde se concluye que d1 = d2 = d3 = d4 = d5 = d6 = d7
y por lo tanto solo hay 10 de estos numeros.
Dado que los primeros dos casos contemplan el total de posibilidades y que, salvo
por los numeros del caso 3, los numeros considerados en ellos son todos distintos,
concluimos que la cantidad de numeros memorables es 104 + 104 10 = 19, 990.
Solucion del problema 16. Vamos a considerar el desarrollo de un cubo, y para cada
trazo de diagonales que haga Juan escogemos una cara de forma que el desarrollo quede
como en la figura.

Ahora es muy facil contar, pues en las otras 5 caras tenemos 2 diagonales posibles. Por
lo tanto, hay 25 = 32 cubos diferentes.
Solucion del problema 17. Supongamos que los catetos miden a, b y la hipotenusa
mide c. Como el a rea y el permetro son iguales, tenemos que 12 ab = a + b + c, de

18

Soluciones a los problemas de practica

donde c = 12 ab a b. Por otra parte, aplicando el teorema de Pitagoras (ver el


teorema 8 del apendice), tenemos que
a2 + b 2 =

1
ab a b
2

1
= a2 + b2 + 2ab a2 b b2 a + a2 b2 ,
4

es decir, 8ab 4a2b 4b2 a + a2 b2 = 0. Dividiendo esta ecuacion entre ab, obtenemos
(a 4)(b 4) = 8. Como a y b son enteros, se sigue que a 4 divide a 8. Luego,
los valores posibles de a son 2, 3, 5, 6, 8 y 12. Determinando los valores de b y c,
obtenemos los triangulos de lados a = 5, b = 12, c = 13, y a = 6, b = 8 y c = 10.
Solucion del problema 18. Usando la formula para cambio de base loga M =
comenzamos rescribiendo la expresion con logaritmos base 10.
1
log 100!
log 2

1
log 100!
log 3

1
log 100!
log 4

+ +

1
log 100!
log 100

logb M
logb a ,

Resolviendo los cocientes y sumando obtenemos,


log 3
log 4
log 100
log 2 + log 3 + log 4 + + log 100
log 2
+
+
+ +
=
.
log 100! log 100! log 100!
log 100!
log 100!
Finalmente, recordando que log A + log B = log(AB), concluimos que
100
X

k=2

1
logk 100!

log(2 3 4 100)
log 100!
=
= 1.
log 100!
log 100!

Solucion del problema 19. Observemos que (2011 x)2 x2 = 2011(2011 2x)
es un multiplo de 2011. Luego, siempre que Curro borre un numero, digamos x2 , basta
que Jacob borre el numero (2011 x)2 . De este modo, al final quedaran dos numeros
cuya diferencia es multiplo de 2011. Por lo tanto, Jacob gana.
Solucion del problema 20. Como a, b, c, d y e son enteros mayores o iguales que 2, la
suma de cualesquiera cuatro de ellos es por lo menos 8. Luego, ya que
b(a + c + d + e) = 155 = 5(31),
donde 5 y 31 son numeros primos, tenemos que b = 5 y a + c + d + e = 31. De manera
analoga, la igualdad
c(a + b + d + e) = 203 = 7(29)
implica que c = 7 y a + b + d + e = 29. Por lo tanto,
a+d+e =
a+b+c+d+e =

24,
36.

Soluciones a los problemas de practica

19

De la primera ecuacion tenemos que


a(b + c + d + e) =
a(36 a) =

128
27 ,

para que a y 36 a sean potencias de 2 las u nicas posbilidades son a = 4 o a = 32.


Si a = 32, tenemos que
36 = a + b + c + d + e 32 + 2 + 2 + 2 + 2 = 40,
lo cual no puede ser, de modo que
e(a + b + c + d)
e(16 + d)

= 275
= 275,

con d + e = 36 a b c = 20. Como 275 = 11(25) y 16 + d 18, tenemos


que e = 11 y d = 25 16 = 9. (Observemos que la factorizacion 275 = 5(55) dara
d = 39 y entonces 36 = a + b + c + d + e > 39, lo cual es un absurdo). Por lo tanto,
a = 4, b = 5, c = 7, d = 9 y e = 11.

20

Soluciones a los problemas de practica

Problemas propuestos

Problemas propuestos.
2010 No. 2.
Ano
Tzaloa se construye con la contribucion de todos y esta seccion esta especialmente
disenada para que sus lectores tengan un espacio de participacion. A continuacion, te
presentamos 5 problemas nuevos que te necesitan para encontrar su respuesta. En esta
ocasion queremos agradecer a Irving Daniel Calderon Camacho, del Estado de Mexico,
quien nos propone el problema 4.
Para dar tiempo a que nos puedas enviar tus soluciones, las respuestas de los problemas propuestos en cualquier numero de la revista, se publican con dos numeros de
diferencia. Es as, que en este numero (Tzaloa 2, ano 2010), aparecen las respuestas
de los problemas propuestos en Tzaloa 4, ano 2009 y las respuestas de los problemas
propuestos en esta ocasion, se publicaran en Tzaloa 4, ano 2010, por lo que aun tienes
tiempo para enviarnos tus contribuciones.
Ponemos a tu disposicion nuestra direccion electronica revistaomm@gmail.com
ya que a traves de ella estaremos recibiendo con gusto todas las soluciones que nos
lleguen desde cualquier rincon del pas.

3
xy
=
, calcula el valor de
Problema 1. (Introductorio) Si se sabe que 2
x + y2
6
4 4
x
x
+
.
y
y

Problema 2. (Introductorio) Los numeros 1, 2, 3, . . . , 24, 25, se han escrito en las casillas de un tablero cuadrado de 5 5, de tal forma que los numeros en cada renglon estan
ordenados en forma creciente de izquierda a derecha. Halla el maximo valor posible de
la suma de los numeros que estan en la tercera columna.

22

Problemas propuestos

Problema 3. (Intermedio) Si x es un numero real tal que x2 +


valores posibles de la expresion x5 + x15 .

1
x2

= 7, determina los

Problema 4. (Intermedio) Sean 1 y 2 dos circunferencias que no se intersectan,


tienen radios distintos y son tangentes interiormente a una circunferencia 3 en los
puntos A y B, respectivamente. Se traza la recta l tangente comun a 1 y 2 , tal como
se muestra en la figura. Demuestra que las rectas AB, l y la que pasa por los centros de
1 y 2 , son concurrentes. (Problema sugerido por Irving Daniel Calderon Camacho).
A

Problema 5. (Avanzado) Un entero n > 1 tiene la siguiente propiedad: para cada


divisor positivo d de n, d + 1 es un divisor de n + 1. Demuestra que n es un numero
primo.

Soluciones a los problemas propuestos.


2009 No. 4.
Ano
Como se menciono al principio de esta seccion, a continuacion publicamos las soluciones de los problemas propuestos en Tzaloa 4, ano 2009. Recuerda que esta revista
necesita de ti y ten la seguridad que en el proximo numero nos encantara poder publicar tus soluciones.
Problema 1. (Intermedio) Para cada entero positivo n, denotamos por a(n) al producto
de los dgitos de n.
(a) Demuestra que a(n) n.
(b) Determina todas las soluciones de la ecuacion n2 17n + 56 = a(n).
Solucion. (a) Supongamos que n tiene k dgitos bk , bk1 , . . . , b1 , con k 1, de modo
que n = bk bk1 b1 es la representacion decimal de n. Entonces
a(n)

= bk bk1 . . . b1
bk 9| .{z
. . 9} (ya que bi 9)
k1

= 9k1 bk .

Problemas propuestos

23

Sin embargo, n = b1 + 10b2 + + 10k1 bk 10k1 bk 9k1 bk a(n). Luego,


n a(n).
(b) Primero consideremos el caso en que n es un numero de un dgito. Entonces,
a(n) = n. Resolviendo la ecuacion n2 17n + 56 = n encontramos las soluciones
n = 4 o n = 14. Luego, n = 4 es la u nica solucion de un dgito.
Ahora, buscaremos soluciones con mas de un dgito. Aplicando la desigualdad del inciso anterior, tenemos que n2 17n + 56 n, es decir, (n 4)(n 14) 0.
Resolviendo esta desigualdad obtenemos que 4 n 14. Como n tiene mas de un
dgito, los posibles valores para n son 10, 11, 12, 13 y 14. Verificando cada uno de estos
valores, vemos que ninguno es solucion de la ecuacion.
Por lo tanto, n = 4 es la u nica solucion.
Solucion alternativa para (b). Del inciso (a) tenemos que a(n) n. Como todos los
dgitos de n son no negativos, tenemos que 0 a(n). Como n2 17n + 56 = a(n),
entonces
0 n2 17n + 56 n.
Consideremos primero la restriccion n2 17n + 56 0. Resolviendo la ecuacion
n2 17n + 56 = 0 encontramos
que la expresi
n2 17n + 56 se factoriza

on cuadratica
1
1
2
como (n 2 (17 65))(n 2 (17 + 65)). Como n 17n + 56 0, entonces

n 21 (17 + 65) > 12 (17 + 64) = 12 21 o n 21 (17 65) < 12 (17 64) = 4 21 .
Pero n es un entero, luego n 13 o n 4.
Consideremos ahora la restriccion n2 17n + 56 n. Entonces, (n 4)(n 14) 0
y de aqu se sigue que 4 n 14.
Combinando las dos restricciones sobre n, tenemos que los valores posibles de n son
n = 4, n = 13 o n = 14. Verificando cada uno de estos valores, vemos que solo n = 4
es solucion.
Problema 2. (Intermedio) Sea S un conjunto de 2010 puntos del plano tales que 3
cualesquiera de ellos no son colineales. Denotemos por L al conjunto de todas las
rectas (extendidas indefinidamente en ambas direcciones) que determinan dos puntos
de S. Demuestra que es posible colorear los puntos de S con a lo mas dos colores,
de modo que para cualesquiera dos puntos, p y q de S, el numero de rectas en L que
separan a p de q es impar si y solo si p y q tienen el mismo color.
Nota: Una recta l separa dos puntos p y q si p y q estan en lados opuestos de l pero ninguno de
los dos esta en l.

Solucion. Supongamos primero que el conjunto de 2010 puntos forman un 2010-agono


convexo P. Coloreamos sus vertices de manera alternada con rojo y azul. Si borramos
dos vertices u y v, el resto del polgono se divide en dos piezas con i y j vertices respectivamente, donde i + j = 2008, luego i y j son de la misma paridad (posiblemente
i o j es 0). El numero de rectas en L que separan a v de w es ij, que es par si v y w
tienen diferente color e impar si v y w tienen el mismo color. Por lo tanto tenemos una
buena coloracion, es decir, una que satisfaga las condiciones del problema.
Ahora consideremos los 2010 puntos con una configuracion arbitraria S y tales que
3 cualesquiera de ellos no sean colineales. Empezaremos con un polgono convexo
P y moveremos cada punto de P a un punto de S teniendo sumo cuidado en que el

24

Problemas propuestos

punto movido cruce rectas en L una a la vez. Despues de 2010 de estos movimientos,
tendremos una buena coloracion de S si se fue manteniendo la buena coloracion
durante los movimientos.
Para mantener la buena coloracion, cuando un punto A es movido y cruza una recta
definida por dos puntos B y C, invertimos los colores de A, B y C. Demostraremos que
esto mantiene la buena coloracion. Notemos que A termina del lado opuesto de la recta
BC en el que estaba, as que despues del movimiento BC separara a A de un punto
P (distinto de A, B o C) si y solo si BC no separaba a A de P antes del movimiento.
Dado que hemos cambiado el color de A pero no de P , A y P aun estan bien coloreados
o correctamente coloreados respecto a la recta BC. Lo mismo se cumple para el punto
B respecto a la recta AC, y el punto C respecto a la recta AB. Las posiciones relativas
de otros puntos o rectas no son afectadas por el movimiento del punto A. Por lo tanto,
la nueva coloracion sigue siendo buena.
Problema 3. (Intermedio) Si se ponen tres puntos en una circunferencia, cual es la
probabilidad de que esten en una misma semicircunferencia?
Solucion. Llamemos a los tres puntos A, B y C, y sea O el centro de la circunferencia. Podemos poner el primer punto A en el extremo derecho de la circunferencia.
Comencemos por poner el punto B en A y moverlo a lo largo de la circunferencia en
sentido inverso a la manecillas del reloj. Para cada posicion de B podemos definir la
zona donde colocar C para que los tres puntos esten en una misma semicircunferencia.

B
b

x
b

Cuando B esta encima de A, el punto C puede estar en cualquier punto de la circunferencia. Al mover B a lo largo de la circunferencia, los radios OA y OB forman un
a ngulo de x grados que va creciendo. Supongamos que x 180 . Entonces, para que
los tres puntos esten en la misma semicircunferencia tenemos que alguno de los a ngulos COA o COB tiene que ser menor o igual a 180 x, es decir, si medimos a partir
de A en sentido contrario a las manecillas del reloj, tenemos que 0 COA 180
o 180 + x COA 360 .
Observemos que cuando B esta diametralmente opuesto a A, no importa donde este el
punto C, los tres puntos estan en la misma semicircunferencia. Cuando x 180 , sea
y = x 180 entonces tenemos que para que C este en la misma semicircunferencia
necesitamos que 0 COA y o 180 COA 360 .
Podemos representar esta situacion en un cuadrado, donde en el lado horizontal ponemos
la medida del a ngulo BOA y en el vertical las medidas del a ngulo COA, y el a rea sombreada representa los valores del a ngulo COA para los cuales los tres puntos estan en

Problemas propuestos

25

la misma semicircunferencia.
360

180

180

360

Por lo tanto, la probabilidad de que los tres puntos esten en la misma semicircunferencia es igual a la porcion del a rea total que representa el a rea de la region sombreada, es
decir, es igual a 86 = 43 .
Problema 4. (Avanzado) En un triangulo acutangulo ABC, los puntos E y F estan en
AC y BC, respectivamente. Las rectas BE y AF se cortan en un punto T , de manera
BT
CE
que TAT
F = 4 y T E = 3. Encuentra el valor de EA .
Solucion. Observemos primero que para cualquier punto P en el lado BC de un
(ABP )
triangulo ABC, se cumple que BP
angulos ABP y AP C
P C = (AP C) , ya que los tri
tienen la misma altura desde A. (Los parentesis denotan a rea).
C
P

Usaremos esta propiedad varias veces en la solucion.


Sean x, y, numeros tales que (BT F ) = 3x y (T F C) = 3y.
Aplicando la propiedad en el triangulo ABF , obtenemos
4=

AT
(ABT )
(ABT )
=
=
,
TF
(BT F )
3x

de donde (ABT ) = 12x.


Aplicando ahora la propiedad en el triangulo ABE, tenemos
3=
de donde (AT E) = 4x.

(ABT )
12x
BT
=
=
,
TE
(AT E)
(AT E)

26

Problemas propuestos

Si ahora aplicamos la propiedad en el triangulo BEC, tenemos


3=

(BT C)
3(x + y)
BT
=
=
,
TE
(T EC)
(T EC)

de donde (T EC) = x + y.
C

E x+y
4x

3y

F
3x

T
12x

Aplicamos ahora la propiedad en el triangulo AF C y obtenemos


4=

(AT C)
(AT E) + (T EC)
5x + y
AT
=
=
=
,
TF
(T F C)
(T F C)
3y

de donde 12y = 5x + y. Luego, y = 5x


11 .
Finalmente, aplicando la propiedad en el triangulo AT C, obtenemos
16x
(T EC)
x+y
4
CE
=
=
= 11 =
.
EA
(AT E)
4x
4x
11

Problema 5. (Avanzado) Sea A = (a1 , a2 , . . . , a2010 ) una sucesion de enteros no necesariamente distintos, cada uno de ellos tomados del intervalo [1005, 1005]. Ademas,
supongamos que la suma de todos los terminos de A es igual a 1. Demuestra que existe
una subsucesion de A tal que la suma de sus terminos es igual a cero.
Solucion. En primer, lugar observemos que si algun termino de A es igual a cero (digamos ak ), entonces el resultado es trivial pues podemos tomar la subsucesion que contiene solo a ese termino: (ak ).
Supondremos entonces que para todo 1 k 2010, tenemos que ak 6= 0. Reordenemos A en una nueva sucesion B = (b1 , b2 , . . . , b2010 ) seleccionando los elementos de
A de uno en uno mediante el siguiente procedimiento: comencemos tomando b1 > 0.
Despues, para cada i {2, 3, . . . , 2010} escogemos bi como cualquiera de los elementos no seleccionados de A que tenga signo contrario al signo del resultado de la suma
parcial si1 = b1 + b2 + + bi1 . Notese que si en algun paso llegara a suceder que
escogieramos bi = si1 , entonces el resultado es trivial, por lo que a partir de este
momento supondremos que si1 6= 0.
Notese que para cada paso del proceso de seleccion, la existencia de un candidato
apropiado para bi esta garantizada, toda vez que la condicion a1 + a2 + + a2010 = 1
implica que la suma de los terminos todava no seleccionados de A tiene que ser cero

Problemas propuestos

27

o tiene que ser de signo contario que si1 .


Por la manera en que hemos ido seleccionando a los terminos de B, cada una de las
sumas parciales s1 , s2 , , s2010 es alguno de los 2009 enteros distintos de cero del
intervalo [1004, 1005]. Por el principio de las casillas, existen enteros m y n tales que
sm = sn , donde 1 m n 2010. Entonces es claro que bm+1 + bm+2 + + bn =
0.

28

Problemas propuestos

Problemas y Soluciones del


Concurso Nacional 2009

Del 8 al 14 de noviembre de 2009 se llevo a cabo en Campeche, Campeche, el Concurso


Nacional de la 23a Olimpiada Mexicana de Matematicas, con la participacion de 31
estados de la Republica. El estado de Tabasco no participo. Los 17 alumnos ganadores
del primer lugar fueron:
Hernandez Gonzalez Flavio (Aguascalientes)
Arreola Gutierrez Fernando Ignacio (Aguascalientes)
Zhou Tan David (Baja California)
Dosal Bustillos Manuel Enrique (Chihuahua)
Embarcadero Ruiz Daniel (Distrito Federal)
Calderon Camacho Irving Daniel (Estado de Mexico)
Leal Camacho Manuel Alejandro (Jalisco)
Miranda Olvera Jose Luis (Jalisco)
Ortiz Rhoton Juan Carlos (Jalisco)
Belanger Albarran Georges (Morelos)
Perales Anaya Daniel (Morelos)
Anorve Lopez Fernando Josafath (Nuevo Leon)
Roque Montoya Diego Alonso (Nuevo Leon)
Jimenez Reichow Tilman (Oaxaca)
Guardiola Espinosa Jose Ramon (San Luis Potos)
Jimenez Bentez Jose Manuel (San Luis Potos)
Ucan Ake Raul Eugenio (Yucatan)
Los 8 alumnos preseleccionados para la Olimpiada Matematica de Centroamerica y el
Caribe fueron:
Garca Gonzalez Hector Benjamn (Colima)

30

Problemas y Soluciones, Concurso Nacional 2009

Ortiz Rhoton Juan Carlos (Jalisco)


Gonzalez Cazares Jorge Ignacio (Jalisco)
Arancibia Alberro Mara Natalie (Morelos)
Roque Montoya Diego Alonso (Nuevo Leon)
Anorve Lopez Fernando Josafath (Nuevo Leon)
Daz Calderon Julio Cesar (Oaxaca)

Cervantes Perez Angel


Gustavo (Yucatan)

Aunque la participacion en el Concurso Nacional es individual, es importante destacar


la labor que han llevado a cabo los estados de la Republica apoyando a sus concursantes. Con el proposito de reconocer este trabajo, presentamos el registro de los estados que ocuparon los primeros 10 lugares en el Concurso Nacional de la 23a Olimpiada
Mexicana de Matematicas.
1. Jalisco
2. Morelos
3. San Luis Potos
4. Nuevo Leon
5. Distrito Federal
6. Yucatan
7. Chihuahua
8. Baja California
9. Aguascalientes
10. Oaxaca
En esta ocasion, el premio a la Superacion Academica se llamo Copa San Francisco
de Campeche y fue ganado por San Luis Potos. El segundo y tercer lugar de este
premio lo ocuparon, Distrito Federal y Nuevo Leon, respectivamente.
A continuacion presentamos los problemas y las soluciones del Concurso Nacional
2009. Los alumnos tuvieron dos sesiones de cuatro horas y media cada una para resolverlos.
Solucion del Examen del Concurso Nacional 2009
Problema 1. Sean ABC un triangulo y AD la altura sobre el lado BC. Tomando a D
como centro y a AD como radio, se traza una circunferencia que corta a la recta AB en
P , y corta a la recta AC en Q. Muestra que el triangulo AQP es semejante al triangulo
ABC.
Solucion. (Georges Belanger Albarran.) Tenemos que D es el centro del crculo que
pasa por A, P y Q. Por lo tanto, D es el circuncentro del triangulo AP Q. Llamemos
x al a ngulo P AD. Como DA = DP por ser radios, el triangulo DAP es isosceles y
DAP = AP D = x.

Problemas y Soluciones, Concurso Nacional 2009

31

A
xy

B
x

Como en un triangulo los a ngulos internos suman 180 , entonces ADP = 180 2x.
El a ngulo central ADP abre el mismo arco que el a ngulo inscrito AQP , entonces
ADP = 2AQP , luego AQP = 90 x. Ahora bien, en el triangulo ADB
tenemos que ADB = 90 y DAB = x, entonces ABD = 90 x = AQP .
As, los triangulos ABC y AQP comparten el a ngulo en A y ABC = AQD, luego
por el criterio AA los dos triangulos son semejantes, que es lo que queramos probar.
(Analogamente, si DAC = y podemos probar que ACB = AP Q = 90 y.
Entonces, los tres a ngulos de los triangulos ABC y AQP son iguales y por lo tanto los
triangulos son semejantes.)
Problema 2. En cajas marcadas con los numeros 0, 1, 2, 3, ... se van a colocar todos los
enteros positivos de acuerdo con las siguientes reglas:
si p es un numero primo e ste se coloca en la caja con el numero 1;
si el numero a se coloca en la caja con el numero ma y b se coloca en la caja con
el numero mb , entonces el producto de a y b, es decir ab, se coloca en la caja con
el numero amb + bma .
Encuentra todos los enteros positivos n que cuando se coloquen queden en la caja con
el numero n.
Solucion. (Jose Luis Miranda Olvera.) Los numeros n que se colocan en la caja con el
numero n son tales que n = ab = amb + bma para algun par de enteros a y b.
Si n se puede escribir como el producto de 2 numeros enteros positivos x y y distintos
de 1 y tales que (x, y) = 1 y xy = n, entonces n = xy 6= xmy + ymx . Esto se debe a
que si xy = xmy + ymx entonces x | xmy + ymx , luego x | ymx , de donde x | mx ,
entonces mx x, y de aqu que
xmy + ymx > ymx xy = n,
lo que no es posible. Por lo tanto, n no se puede expresar como el producto de dos
numeros primos relativos distintos de 1. Luego, n = pr donde p es un numero primo
y r 2. Ahora bien, demostremos que n = mn si y solo si n = pp . Supongamos

32

Problemas y Soluciones, Concurso Nacional 2009

que n = mn = pr , para algun entero r 2. Como los numeros primos van en la caja
numero 1, tenemos que
mpr = pr = pr1 mp + pmpr1 ,
lo cual implica que mpr1 = pr1 pr2 . Ademas, para todo 1 y < r, tenemos
que
mpry = pr(y+1) + pmpr(y+1) .
Luego,
pr2 + pmpr2
mpr2

= mpr1 = pr1 pr2


= pr2 2pr3 .

Recursivamente llegamos a que


mpry = pry ypr(y+1),
para todo 1 y < r. En particular,
mpr(r2)

mp2

pr(r2) (r 2)pr(r1)

p2 (r 2)p

p2 rp + 2p
r,

2p =
p =

ya que mp2 = pmp + pmp = 2p.


Por lo tanto, n = mn si y solo si n = pp , donde p es primo.
Problema 3. Sean a, b, c numeros reales positivos tales que abc = 1. Muestra que
b3
c3
a3
+
+
1
a3 + 2 b 3 + 2 c3 + 2

1
1
1
+
+
1.
a3 + 2 b 3 + 2 c3 + 2

y que

Solucion. (Manuel Enrique Dosal Bustillos.) Tenemos que


a3
b3
c3
+
+
1
a3 + 2 b 3 + 2 c3 + 2

a2
a3 +2

b2
b3 +2

c2
c3 +2

a2
a2 +

2
a

b2
b2 +

c
2
b

c2
c2 +

2
c

1.

Aplicando una desigualdad u til1 , tenemos que


a2
1 Si

a2
+

2
a

b2

b2
+

2
b

c2

c2
+

2
c

a2

(a + b + c)2
+ + c2 + a2 +
b2

2
b

2
c

a1 , a2 , . . . , an , x1 , x2 , . . . , xn son numeros reales y x1 , x2 , . . . , xn son positivos, entonces


a21
x1

a22
x2

+ +

a2n
(a1 + a2 + + an )2

xn
x1 + x2 + + xn

Problemas y Soluciones, Concurso Nacional 2009

33

Entonces, la primera desigualdad quedara demostrada si probamos que


(a + b + c)2
a2 + b2 + c2 + a2 +

2
b

2
c

1.

Tenemos que
(a + b + c)2
a2 + b2 + c2 + a2 +

1 1 1
1 (a + b + c) a + b + c + 2
+ +
a
b
c

1 1 1
+ +
2(ab + bc + ac) 2
a b
c
2(ab + bc + ac) 2(bc + ac + ab),
2

2
b

2
c

ya que la condicion abc = 1 implica que a1 = bc, 1b = ac y 1c = ab. Como la u ltima


desigualdad es cierta, hemos demostrado as la primera desigualdad.
1
1
1
+ 3
+ 3
1 por (a3 + 2)(b3 + 2)(c3 +
+2 b +2 c +2
2), obtenemos la desigualdad equivalente
Multiplicando la desigualdad

a3

(b3 + 2)(c3 + 2) + (a3 + 2)(c3 + 2) + (a3 + 2)(b3 + 2) (a3 + 2)(b3 + 2)(c3 + 2),
la cual se simplifica a
a3 b3 c3 + b3 c3 + a3 c3 + a3 b3 4.
Como abc = 1, tenemos que a3 b3 c3 = 1, y la desigualdad anterior se reduce a la
desigualdad
b3 c3 + a3 c3 + a3 b3 3.
Pero esta desigualdad es verdadera, pues si aplicamos la desigualdad media aritmeticamedia geometrica a los numeros reales positivos b3 c3 , a3 c3 y a3 b3 , tenemos que

3
b3 c3 + a3 c3 + a3 b3 3 3 (b3 c3 )(a3 c3 )(a3 b3 ) = 3 a6 b6 c6 = 3(abc)2 = 3.
Por lo tanto, hemos demostrado as la segunda desigualdad.
Problema 4. Sea n > 1 un entero impar y sean a1 , a2 , . . . , an numeros reales distintos.
Sea M el mayor de estos numeros y sea m el menor de ellos. Muestra que es posible
escoger los signos en la expresion s = a1 a2 an de manera que
m < s < M.
Solucion. (Flavio Hernandez Gonzalez.) Renombrando los numeros podemos suponer
que
a1 < a2 < < an .
Ahora escogemos
s = a1 a2 + a3 an1 + an .

34

Problemas y Soluciones, Concurso Nacional 2009

Entonces
s = a1 + (a2 + a3 ) + (a4 + a5 ) + + (an1 + an ) > a1
pues (ak + ak+1 ) > 0 para toda k.
Por otra parte
s = (a1 a2 ) + (a3 a4 ) + + (an2 an1 ) + an < an
pues (ak ak+1 ) < 0 para toda k.
Por lo tanto
m < s < M.
Problema 5. Considera un triangulo ABC y un punto M sobre el lado BC. Sea P la
interseccion de las perpendiculares a AB por M y a BC por B, y sea Q la interseccion
de las perpendiculares a AC por M y a BC por C. Muestra que P Q es perpendicular
a AM si y solo si M es punto medio de BC.
Solucion. (Daniel Perales Anaya.) Sean E el punto de interseccion de AB y P M , D
el punto de interseccion de AC y M Q, y R el punto de interseccion de AM y P Q.
A
Q
R

D
E

Supongamos que AM es perpendicular a P Q.


Como M CQ = M RQ = M RP = M BP = 90 , entonces P BM R y
QCM R son cuadrilateros cclicos. Entonces
M AD = 90 RM Q = RQM = RCM.
Analogamente,
BAR = 90 RM P = M P R = M BR.
Por lo tanto, el circuncrculo del triangulo ARC es tangente a BC en C y el circuncrculo del triangulo ARB es tangente a BC en B. Luego, por potencia a estos
crculos desde M tenemos que M C 2 = M A M R = M B 2 , de donde M C = M B.
Por lo tanto, M es punto medio de BC.
Supongamos que M es punto medio de BC, es decir, M B = M C.

Problemas y Soluciones, Concurso Nacional 2009

35

Como M CQ = M DC = 90 y QM C = CM D, entonces el triangulo QM C


es semejante al triangulo CM D. Analogamente, el triangulo BEM es semejante al
triangulo P BM . Luego, los lados correspondientes son proporcionales, por lo que
MC
PM
BM
2
tenemos que QM
MC = MD y BM = ME , de donde QM M D = M C y P M M E =
2
2
2
BM . Entonces, P M M E = BM = M C = QM M D, luego por potencia desde
M tenemos que P EDQ es cclico. Como AEM = ADM = 90 , tenemos que
AEM D es cclico. Entonces,
P QD = 180 P ED = DEM = DAM = 90 AM D,
por lo que P QD + AM D = 90 . Luego,
M RQ =
=

180 (RQM + RM Q) = 180 (P QD + AM D)


180 90 = 90 .

Por lo tanto, P Q es perpendicular a AM .


Problema 6. En una fiesta con n personas, se sabe que de entre cualesquiera 4 personas,
hay 3 de las 4 que se conocen entre s o hay 3 que no se conocen entre s. Muestra que
las n personas se pueden separar en 2 salones de manera que en un salon todos se
conocen entre s y en el otro salon no hay dos personas que se conozcan entre s.
Nota: conocerse se considera una relacion mutua.
Solucion. (Diego Alonso Roque Montoya.) Resolveremos el problema por induccion
sobre el numero de personas en la fiesta.
Claramente los casos para n = 1, 2, 3, 4 cumplen la condicion del problema. Supongamos entonces que la condicion se cumple para n = k. Digamos que llega otra persona
P a la fiesta y que antes de que llegara estaban separados de forma que el numero
de personas en el cuarto donde todos se conocen sea el mayor posible. Llamaremos a
este cuarto el primer cuarto. Si la nueva persona conoce a todos los del cuarto donde
todos se conocen o a ninguno de los del otro cuarto, entonces puede entrar al cuarto
correspondiente y se cumple la condicion.
De lo contrario, la persona P no conoce al menos a alguien del primer cuarto (llamemos
A a esta persona) y conoce al menos a alguien del segundo cuarto (llamemos B a esta
persona).
Consideramos a una persona C del primer cuarto. Si C no conoce a B, entonces no
puede haber tres personas en el conjunto {A, B, C, P } que no se conozcan, ni tres que
se conozcan. Luego, B conoce a C. Como C es cualquier persona del primer cuarto
podemos deducir que B conoce a todos los del primer cuarto con la posible excepcion
de A.
Usando este mismo argumento demostramos que A no conoce a nadie del segundo
cuarto con la posible excepcion de B.
Si B conoce a A, entonces B podra mandarse al primer cuarto y tendra mas personas
lo cual no es posible por la suposicion de que el numero de personas era el mayor
posible. Luego, B no conoce a A.
Si P no conoce a C, no se cumple la condicion del problema, luego P conoce a C y por
lo tanto a todos los del primer cuarto excepto a A. Si pasamos a A al segundo cuarto, P

36

Problemas y Soluciones, Concurso Nacional 2009

conoce a todos los del primer cuarto. Entonces podemos poner a P en el primer cuarto
y se cumple la condicion.
Por lo anterior podemos separar a n = k + 1 personas en dos cuartos cumpliendo con
las condiciones del problema, lo que termina la induccion.

Problemas y Soluciones de
Olimpiadas Internacionales

XXIV Olimpiada Iberoamericana


El ano pasado, Mexico tuvo el privilegio de organizar la XXIV Olimpiada Iberoamer
icana de Matematicas. Esta
se llevo a cabo en la ciudad de Queretaro, del 17 al 27 de
septiembre de 2009. Mexico ocupo el 5 lugar de entre los 21 pases que participaron.
La delegacion mexicana estuvo integrada por los alumnos: Manuel Guillermo Lopez
Buenfil (Chihuahua), Erik Alejandro Gallegos Banos (Oaxaca), Daniel Perales Anaya
(Morelos), y Cesar Bibiano Velasco (Morelos).
Manuel Guillermo obtuvo medalla de oro, Erick Alejandro y Daniel obtuvieron medalla de plata y Cesar mencion honorfica.
A continuacion presentamos los problemas con sus soluciones de la XXIV Olimpiada
Iberoamericana. Los alumnos tuvieron dos sesiones de cuatro horas y media cada una
para resolverlos.
Problema 1. Sea n un natural mayor que 2. Supongamos que n islas estan ubicadas en
un crculo y que entre cada dos islas vecinas hay dos puentes como en la figura.
x1

x2

xn

x3

xn1

xj

Comenzando en la isla x1 , de cuantas maneras se pueden recorrer los 2n puentes

38

XXIV Olimpiada Iberoamericana

pasando por cada puente exactamente una vez?


Solucion. Jorge Alberto Olarte (Colombia). Al principio se tienen dos opciones: ir a
x2 o a xn . Entonces sin perdida de generalidad supongamos que empieza yendo hacia
x2 y luego multiplicamos por 2.
Si sigue avanzando hasta xk
x1 x2 xk ,
y en xk en vez de ir a xk+1 va a xk1 se tienen 2k1 posibilidades para llegar a xk
ya que se cruzaron k 1 puentes y cada vez se tenan 2 posibles puentes. Cuando se
devuelve esta obligado a tomar los otros puentes hasta x1 .
Posteriormente sale de x1 hacia xn y tiene que ir hasta xk , ya que si se devuelve antes
cuando llega nuevamente a x1 habra puentes que no podra recorrer. Luego de x1 a xk
cruza n + 1 k puentes, es decir tuvo 2n+1k posibilidades, y de regreso a x1 toma el
u nico camino restante.
Multiplicando se tiene 2n+1k 2k1 = 2n . Como se puede devolver en cualquier xk
(si se devolvio en x1 dio toda la vuelta antes) hay n2n posibilidades. A eso le sumamos
el caso de que nunca se haya devuelto, lo cual da otras 2n formas (2 posibilidades por
cada uno de los puentes en la primera vuelta), y una vez que regresa por primera vez
a x1 vuelve a dar una vuelta por los puentes que quedan. Entonces hay n 2n + 2n
caminos, y multiplicando por 2 se tienen (n + 1)2n+1 caminos en total.
Problema 2. Para cada entero positivo n se define an = n + m donde m es el mayor
m
entero tal que 22 n2n . Determinar que enteros positivos no aparecen en la sucesion
an .
Solucion. Reynaldo Gil Pons (Cuba).
Lema 8 Dado m un entero positivo, el mayor entero n tal que n2n < 22m es 2m m.
Prueba del lema.
m

Si n 2m m + 1 y 22m n2n (2m m + 1)22


m
por 22 m+1 obtenemos que
2m m + 1
2m1

<

2m1

<

m 1,

m+1

entonces al dividir

lo cual es imposible.
m

Si n = 2m m, entonces n2n = (2m m)22

= 22 m22

< 22 .
m1

Entonces, con este lema, si 2m1 (m 1) < J 2m m tenemos que 22

m
J2J < 22 . Por lo tanto si J recorre los numeros entre 2m1 (m 1) + 1 y 2m m
(inclusive), aJ = J + m 1 recorrera los numeros desde 2m1 + 1 hasta 2m 1. Con
esto tenemos que los numeros que no aparecen en la sucesion son las potencias de dos,
2 con 1.

XXIV Olimpiada Iberoamericana

39

Problema 3. Sean C1 y C2 dos circunferencias de centros O1 y O2 con el mismo radio,


que se cortan en A y en B. Sea P un punto sobre el arco AB de C2 que esta dentro de
C1 . La recta AP corta a C1 en C, la recta CB corta a C2 en D y la bisectriz de CAD
intersecta a C1 en E y a C2 en L. Sea F el punto simetrico a D con respecto al punto
medio de P E. Demostrar que existe un punto X que satisface XF L = XDC =
30 y CX = O1 O2 .

Solucion. Percy Guerra Ros (Peru).


Los a ngulos ACB y ADB son iguales, ya que
sostienen el mismo arco AB en circunferencias congruentes, as que el triangulo ACD
es isosceles y AL es perpendicular a CD.
C2
A
C1
b

F
b

b
b

O1

E
b

O2

B
C
L
Sea = CAL = LAD, entonces ACB = 90 , y su a ngulo central es
AO1 B = 180 2. Por simetra se tiene que AO1 O2 = 90 . Analogamente,
O1 O2 A = 90 . Ahora nos fijamos en el triangulo O1 O2 A, donde tenemos que
O1 AB = BAO2 = y O1 O2 = 2r sin , donde r es el radio comun de las
circunferencias.
Por ley de senos en los triangulos ACE, ALD y AP L tenemos CE = 2r sin ,
LD = 2r sin y P L = 2r sin , por lo tanto CE = LD = P L = O1 O2 .
La recta AL es un eje de simetra respecto a C y D, entonces DE = EC = LD = CL.
Y como F es simetrico de D con respecto al punto medio de EP tengo F P = ED =
CL y F P ||ED y por el rombo CEDL tengo que ED||CL, por lo tanto F P y CL son
paralelas y de la misma longitud, por lo que el cuadrilatero F CLP es un paralelogramo
y F C = P L.
Sea X el punto tal que el triangulo CXL sea equilatero. Como CL = O1 O2 , se tiene
que CX = O1 O2 .

40

XXIV Olimpiada Iberoamericana


C2
C1
F

O1
b

O2
b

X
Como C es el circuncentro del triangulo F LX, entonces XF L = 12 XCL = 30
y como L es el circuncentro del triangulo CXD tenemos que XDC = 21 XLC =
30 .
Por lo tanto, dicho punto X cumple con las condiciones pedidas.
Problema 4. Sea ABC un triangulo con AB 6= AC. Sean I el incentro de ABC y P
el otro punto de interseccion de la bisectriz exterior del a ngulo A con el circuncrculo
de ABC. La recta P I intersecta por segunda vez al circuncrculo de ABC en el punto
J. Demostrar que los circuncrculos de los triangulos JIB y JIC son tangentes a IC
y a IB, respectivamente.
Solucion. Ricardo Jesus
Ramos Castillo (Peru).
Sean M , N y R puntos en las prolongaciones de BA, CI y BI, respectivamente. Por ser AP bisectriz exterior, tenemos
que
M AP = P AC =

1
B + C
M AC =
.
2
2

Sabemos que M AP = BJP , pues el cuadrilatero BAJP es inscriptible, y tambien


P AC = P JC, porque el cuadrilatero P AJC es inscriptible. Ademas

N IB
RIC

B
C
B + C
+
=
,
2
2
2
C
B + C
B
+
=
.
= IBC + ICB =
2
2
2

= IBC + ICB =

Lo anterior implica que


B + C
= M AP = BJP = BJI,
2
B + C
= P AC = P JC = IJC.
RIC =
2

N IB =

XXIV Olimpiada Iberoamericana

41

A
R
N
b

I
B
C
J

Por el teorema del a ngulo seminscrito, de N IB = BJI se sigue que la recta N I es


tangente al circuncrculo del triangulo BIJ, y de RIC = IJC se sigue que la recta
RI es tangente al circuncrculo del triangulo CIJ, que son justamente las tangencias
que queramos demostrar.
Problema 5. La sucesion an esta definida por
a1 = 1, a2k = 1 + ak y a2k+1 =

1
, para todo entero k 1.
a2k

Demostrar que todo numero racional positivo aparece exactamente una vez en esta
sucesion.
Solucion. Reynaldo Gil Pons (Cuba). Es claro que a2r q = r + aq , pues
a2r q = 1 + a2r1 q = 1 + 1 + a2r2 q = = r + aq ,
tambien, a2k > 1 y a2k+1 < 1 para toda k > 0. Vamos a probar que no existe i 6= j
tal que ai = aj . Para esto, asumimos que i = 2 (2r + 1) y j = 2 (2m + 1) para
algunos numeros , , r y m mayores o iguales que 0. Supongamos que ai = aj . Por
lo anterior,
ai = a2 (2r+1) = + a2r+1
y
aj = a2 (2m+1) = + a2m+1 .
1
Luego, = y a2r+1 = a2m+1 , de aqu tenemos a12r = a2m
, lo que implica ar = am .
Este procedimiento se puede repetir un numero finito de veces hasta que llegamos a una
ecuacion de la forma
1 = a1 = a2l+1

42

XXIV Olimpiada Iberoamericana

para alguna l. Pero esto implica que l = 0 por lo que vimos al principio. Por lo tanto
i = j.
Ahora vamos a demostrar que para toda pareja (n, m) de numeros naturales tales que
n
. La demostracion es por induccion en la
mcd(n, m) = 1, existe j tal que aj = m
cantidad de pasos k del algoritmo de la division de Euclides,
n

= q1 m + r1

m
r1

= q2 r1 + r2
= q3 r2 + r3
..
.
= qk rk1 + rk

rk2

donde k es el primer numero tal que rk = 0.


Si k = 1, entonces n es un multiplo de m, de donde m = 1 y a2n1 = n 1 + a1 = n.
n
Supongamos que todos los numeros m
aparecen en la sucesion, siempre y cuando, el
numero de pasos en el algoritmo de Euclides es menor que cierto N . Sean x y y dos
numeros naturales tales que la cantidad de pasos en el algoritmo de Euclides es N + 1,
x

= q1 y + r1

y
r1

= q2 r1 + r2
= q3 r2 + r3
..
.

rN 1

= qN +1 rN .

Note que la cantidad de pasos en el algoritmo de Euclides para la pareja (y, r1 ) es N .


Por induccion, existe s tal que as = ry1 . Si s es par as+1 = ry1 , si s es impar as1 = ry1 ,
en ambos casos podemos decir que existe un t tal que at = ry1 , luego
a2q1 t = q1 + at = q1 +

r1
q1 y + r1
y
=
=
y
y
x

lo que completa el paso de induccion.


Problema 6. Alrededor de una circunferencia se marcan 6000 puntos y cada uno se
colorea con uno de 10 colores dados, de manera tal que entre cualesquiera 100 puntos
consecutivos siempre figuran los 10 colores. Hallar el menor valor k con la siguiente
propiedad: Para toda coloracion de este tipo existen k puntos consecutivos entre los
cuales figuran los 10 colores.
Solucion. Manuel Guillermo Lopez Buenfil (Mexico). Sean c1 , c2 , . . . , c10 los colores y consideremos los puntos numerados del 1 al 6000. Dividimos a los puntos en
60 grupos de 100 puntos y los coloreamos de la siguiente manera: Para r = 1, 2, . . . , 9
pintamos los puntos de la forma 100n + 11r con el color cr y el resto de los puntos los coloreamos del color c10 . Veamos que cumple la propiedad: cada 100 puntos
consecutivos contienen a un grupo completo y contiene los diez colores, en este caso,

XI Olimpiada Centroamericana y del Caribe

43

para obtener un conjunto de puntos con los diez colores debemos cruzar al menos 8
intervalos de puntos de color c10 para que esten los otros 9 colores, el intervalo entre
99 y 11 (mod 100) es el mas largo con 11 puntos c10 mientras que los demas tienen
10, entonces lo mejor es no tomar ese intervalo dando k = 9 + 8 10 = 89, de donde
k 89, por necesitarse 89 en este caso.
Supongamos que en cierta coloracion no hay 89 puntos consecutivos en los que aparezcan los 10 colores. Tomemos 11 puntos cualesquiera y los 89 siguientes, en los u ltimos
89 no aparecen los 10 colores pero en los 100 s, entonces hay al menos un color en los
primeros 11 puntos que no aparece en los siguientes 89. Ahora para 0 s 7 hacemos el siguiente razonamiento: tomemos los puntos del 11s + 1 al 11s + 100, en esos
puntos aparecen los 10 colores pero en los u ltimos 89 no, entonces hay al menos un
color entre los puntos 11s + 1 y 11s + 11 que no aparece en los u ltimos 89 puntos, pero
por lo demostrado anteriormente, este color debe ser distinto al color que obtuvimos
en el intervalo con extremos 11i + 1 y 11i + 11, 0 i < s, lo que nos prohbe s 1
colores, al acabar tendremos que los puntos del 89 al 100 solo pueden contener dos colores. Repitiendo este argumento se concluye que cualesquiera 12 puntos consecutivos
tienen a lo mas dos colores.
Tomemos dos puntos de colores distintos, sin perdida de generalidad c1 y c2 , y volvamos a numerar de tal forma que sean los puntos 1 y 2, respectivamente. Del punto 3
al 12 solo puede haber puntos de color c1 y c2 as que el punto 13 sera el primero en
tener la posibilidad de ser de otro color, digamos c3 . Como e ste es el primer punto de
color c3 , el anterior es distinto, por lo que los 11 puntos anteriores al c3 tienen que ser
del color c2 ya que del punto 2 al 13 solo hay puntos de colores c2 y c3 . Repitiendo
este argumento los siguientes 11 puntos seran de color c3 , luego 11 puntos de color c4
y as sucesivamente hasta 11 puntos de color c9 , obteniendo as 99 puntos y habiendo
usado 9 colores, por lo que el u ltimo punto es de color c10 . Si un intervalo hubiera contenido mas de 11 puntos entonces habramos construido un grupo de 100 puntos que
no contiene los 10 colores, lo cual sera una contradiccion. Continuando el argumento
vemos que cada grupo de 11 puntos consecutivos tienen que ser del mismo color, sin
embargo 11 no divide a 6000 por lo cual la coloracion es imposible, es decir, k 89.
Por las dos desigualdades para k, tenemos que k = 89.

XI Olimpiada Centroamericana y del Caribe


Del 4 al 10 de octubre de 2009, se celebro en Girardot, Colombia, la XI Olimpiada
Matematica de Centroamerica y el Caribe. La delegacion mexicana estuvo integrada
por los alumnos: Manuel Enrique Dosal Bustillos (Chihuahua), Jorge Vargas Garza
(Distrito Federal), y Diego Alonso Roque Montoya (Nuevo Leon).
Jorge y Manuel Enrique obtuvieron medalla de oro, y Diego Alonso medalla de plata.
Mexico ocupo el primer lugar de 12 pases participantes.
A continuacion presentamos los problemas con sus soluciones de la XI Olimpiada
Centroamericana y del Caribe. Los alumnos tuvieron dos sesiones de cuatro horas y
media cada una para resolverlos.

44

XI Olimpiada Centroamericana y del Caribe

Problema 1. Sea P (n) el producto de los dgitos no nulos del entero positivo n. Por
ejemplo, P (4) = 4, P (50) = 5, P (123) = 6, P (2009) = 18. Halle el valor de la suma
P (1) + P (2) + + P (2008) + P (2009).

Solucion. (Manuel Enrique Dosal Bustillos). Tenemos que


P (1) + P (2) + + P (9) =
P (10) + P (11) + + P (19) =

P (20) + P (21) + + P (29) =


P (30) + P (31) + + P (39) =
..
.
P (90) + P (91) + + P (99) =

1 + 2 + + 9 = 45
1 + 45 = 1(46)
2(1 + 45) = 2(46)
3(1 + 45) = 3(46)

9(1 + 45) = 9(46).

Luego,
P (1) + P (2) + + P (98) + P (99) = 45 + 46(1 + 2 + + 9) = 45(47).
Ahora bien, si fijamos a 6= 0 tenemos que P (abc) = aP (bc), luego
X
X
X
P (abc) =
aP (bc) = a
P (bc),
donde b y c varan de 0 a 9.
Pero ya tenemos la suma de P (1) hasta P (99), luego tenemos que
P (101) + P (102) + + P (199) =

P (201) + P (202) + + P (299) =


P (301) + P (302) + + P (399) =
..
.
P (901) + P (902) + + P (999) =

1(45)(47)
2(45)(47)
3(45)(47)

9(45)(47).

Observemos que nos falta sumar P (100) = 1, P (200) = 2 hasta P (900) = 9, luego
P (100) + P (101) + + P (998) + P (999) =
=

(1 + 2 + + 9) +
+ (45)(47)(1 + 2 + + 9)
45(1 + 47(45)).

Por lo tanto, la suma desde P (1) hasta P (999) es


P (1) + P (2) + + P (999) = (45)(47) + 45(1 + 47(45)) = 45(1 + 47(46)).
Observemos que P (1000 + k) = P (k) si 1 k 999. Luego,
P (1000) + P (1001) + + P (1999) = 1 + P (1) + P (2) + + P (999)
= 1 + 45(1 + 47(46)),

XI Olimpiada Centroamericana y del Caribe

45

de donde
P (1) + P (2) + + P (1999) = 2[45(1 + 47(46))] + 1.
Finalmente, solo nos resta calcular la suma desde P (2000) hasta P (2009). Tenemos
que
P (2000) + P (2001) + + P (2009) =
=
=

2 + 2 + 4 + 6 + + 18
2(1 + 1 + 2 + 3 + + 9)
2(46).

Por lo tanto, la suma final es


P (1) + P (2) + + P (2009) = 2[45(1 + 47(46))] + 1 + 2(46) = 194, 763.
Segunda solucion. Sea R(n) el producto de todos los dgitos de n incluyendo los ceros
y tomando los numeros de 1 y 2 dgitos como 00x y 0xy, respectivamente. Entonces,
R(1) + R(2) + + R(999) =
=

0 0 0 + 0 0 1 + + 9 9 9 0 0 0

(0 + 1 + 2 + + 9)3 0 = 453 .

Si en lugar de los ceros colocamos unos, el producto de los dgitos distintos de 0 se


mantiene, es decir, calcular P (1) + P (2) + + P (999) es equivalente a sustituir en
la expresion R(1) + R(2) + + R(999) los ceros por unos, por lo que
P (1) + P (2) + + P (999) = (1 + 1 + 2 + 3 + + 9)3 1
= 463 1 = 97, 335.
Luego, P (1000) = 1,
P (1001) + P (1002) + + P (1999) = P (1) + P (2) + + P (999) = 97, 335
y
P (2000)+P (2001)+ +P (2009) = 2+2+4+6+8+10+12+14+16+18 = 92.
Finalmente,
P (1) + P (2) + + P (2009) = 97, 335 + 1 + 97, 335 + 92 = 194, 763.
Problema 2. Dos circunferencias 1 y 2 se intersectan en los puntos A y B. Considere
una circunferencia contenida en 1 y 2 , tangente a ellas respectivamente en D y E.
Sean C uno de los puntos de interseccion de la recta AB con , F la interseccion de
la recta EC con 2 y G la interseccion de la recta DC con 1 . Sean H e I los puntos
de interseccion de la recta ED con 1 y 2 , respectivamente. Demuestre que F , G, H
e I estan sobre una misma circunferencia.
Solucion. (Jorge Garza Vargas). Como esta contenida en 1 y es tangente a esta,
entonces es tangente internamente a 1 . Sabemos que el centro de homotecia de

46

XI Olimpiada Centroamericana y del Caribe

y 1 es el punto D (por ser el punto de tangencia de e stas). Luego, los triangulos


ECD y HGD son homoteticos con centro en D, entonces EC es paralelo a GH.
Analogamente, es tangente internamente a 2 , E es el centro de homotecia de y
2 y CD es paralelo a F I.
F
G

1
b

H
E

C
b

D
I

Por los paralelismos anteriores tenemos que CED = GHD, GDE = F IE y


ECD = HGD = EF I. Como C esta en el eje radical de las circunferencias 1
y 2 , entonces EC CF = DC GC, luego el cuadrilatero GEDF es cclico, entonces
GDE = GF E y DEF = DGF .
Por lo tanto, GHI + GF I = CED + GF E + EF I = CED + CDE +
ECD = 180 ya que estos a ngulos son los a ngulos internos del triangulo ECD.
As que el cuadrilatero HGF I es cclico porque sus a ngulos opuestos suman 180 .
Por lo tanto, F , G, H e I estan sobre una misma circunferencia.
Problema 3. Se tienen 2009 cajas numeradas del 1 al 2009, algunas de las cuales
contienen piedras. Dos jugadores A y B juegan alternadamente, comenzando por A.
Una jugada consiste en seleccionar una caja i que no este vaca, tomar una o mas
piedras de esa caja y ponerlas en la caja i + 1. Si i = 2009, las piedras que se tomen se
desechan. El jugador que retire la u ltima piedra (dejando todas las cajas vacas) gana.
1. Suponiendo que inicialmente en la caja 2 hay 2009 piedras y todas las demas
cajas (1, 3, 4, 5, . . . , 2009) estan vacas, halle una estrategia ganadora para uno
de los dos jugadores y justifquela.
2. Suponiendo que inicialmente cada caja contiene exactamente una piedra, halle
una estrategia ganadora para uno de los dos jugadores y justifquela.
Solucion.
1. El jugador B tiene una estrategia ganadora, que consiste en lo siguiente: cada vez
que A mueva k > 0 piedras de la caja i a la caja i + 1, B responde moviendo k
piedras de la caja i + 1. Como inicialmente todas las piedras estan en cajas pares,
la estrategia de B hace que se mantenga esta situacion cada vez que le toque

XI Olimpiada Centroamericana y del Caribe

47

jugar a A, quien nunca podra retirar piedras de la caja 2009. Como eventualmente
todas las piedras tendran que salir de la caja 2009, B sera quien saque la u ltima
piedra.
2. Ahora es A quien tiene una estrategia ganadora. Como primera jugada mueve
una piedra de cualquier caja impar. De este modo quedaran 1004 cajas impares
no vacas. En lo sucesivo, si B mueve una piedra de una caja impar, A debe
responder moviendo una piedra de otra caja impar (siempre podra hacerlo por la
paridad del numero de cajas impares no vacas). Si en cambio B mueve k > 0
piedras de una caja par 2i a la caja 2i + 1, entonces A responde moviendo k
piedras de la caja 2i + 1.
Problema 4. Se desea colocar numeros naturales alrededor de una circunferencia cumpliendo la siguiente propiedad: Las diferencias entre cada par de numeros vecinos, en valor
absoluto, son todas diferentes.
1. Sera posible colocar los numeros del 1 al 2009 satisfaciendo la propiedad?
2. Sera posible suprimir alguno de los numeros del 1 al 2009, de tal manera que
los 2008 numeros restantes se puedan colocar satisfaciendo la propiedad?
Solucion.
1. No, pues debera haber 2009 diferencias, y como la menor diferencia posible es
1 y la mayor posible es |2009 1| = 2008, por el principio de las casillas alguna
diferencia debera aparecer mas de una vez.
2. S. Supongamos que se retira a. Si se colocan los numeros restantes en el orden
1, 2009, 2, 2008, . . . , a1, 2011a, a+1, 2010a, . . ., 1005, 1006, se obtienen
las diferencias 2008, 2007, . . . , 2012 2a, 2010 2a, 2009 2a, . . . , 2, 1 y la
diferencia entre el primero y el u ltimo es 1005. Para que sean todas diferentes
basta que sea 2011 2a = 1005, es decir a = 503, y nos queda el orden
1, 2009, 2, 2008, . . ., 502, 1508, 504, 1507, 505, 1506, . . ., 1005, 1006.

Problema 5. Dado ABC un triangulo acutangulo y escaleno, sea H su ortocentro, O


su circuncentro, E y F los pies de las alturas trazadas desde B y C, respectivamente.
La recta AO corta nuevamente al circuncrculo del triangulo en un punto G y a los
segmentos F E y BC en los puntos X y Y , respectivamente. La recta AH corta a la
tangente al circuncrculo trazada por G en un punto Z. Demuestre que HX es paralelo
a Y Z.
Solucion. (Diego Alonso Roque Montoya). Denotemos por N al pie de la altura trazada
desde A, entonces AN es perpendicular a BC. Como HN B = 90 = HF B,
entonces el cuadrilatero HN BF es cclico.

48

XI Olimpiada Centroamericana y del Caribe


A
F

E
b

O
B

Como F ECB es cclico, entonces BF E + BCE = 180, pero BCE y BGA


abren el mismo arco, entonces 180 = BF E + BCE = BF E + BGA =
BF X + BGX. Luego, F XGB es cclico.
Por potencia de punto en F XGB y HN BF tenemos que
AX AG = AF AB = AH AN,
entonces, XHN G es cclico.
Ademas, AGZ = OGZ = Y N Z = 90 , entonces Y N ZG es cclico. Entonces,
XHA = XGN = Y GN = Y ZN = Y ZA.
Por lo tanto, XH es paralelo a Y Z.
Problema 6. Encuentre todos los numeros primos p y q tales que p3 q 5 = (p + q)2 .
Solucion. La u nica solucion es p = 7, q = 3.
Primero supongamos que p y q son distintos de 3. Entonces p 1 (mod 3) y q
1 (mod 3). Checando todas las posibilidades, obtenemos que el lado izquierdo de la
ecuacion es divisible entre 3 mientras que el lado derecho no, o bien el lado derecho es
multiplo de 3 mientras que el lado izquierdo no. Luego, no hay solucion y por lo tanto
p = 3 o q
= 3. Si p = 3, entonces q 5 = 33 (3 + q)2 = 27 (3 + q)2 < 27 de
donde q < 5 27 < 2, lo cual no es posible. Entonces, q = 3 y p3 243 = (p + 3)2 . La
ecuacion p3 243 = (p + 3)2 es equivalente a la ecuacion (p 7)(p2 + 6p + 36) = 0.
Luego, p = 7 o p2 + 6p + 36 = 0. Es facil verificar que la ecuacion p2 + 6p + 36 = 0
no tiene soluciones reales. Por lo tanto, q = 3 y p = 7 es la u nica solucion que cumple
el problema.

Errata. En el numero anterior de Tzaloa, el Problema 2 de la XI Olimpiada Centroamericana y del Caribe aparece con un error en su redaccion. Sin embargo, en esta
seccion hemos enunciado de manera correcta dicho problema.

Informacion Olmpica
A continuacion presentamos las actividades programadas por el comite organizador de
la Olimpiada Mexicana de Matematicas, de abril a julio de 2010.
Del 29 de abril al 9 de mayo, Cuernavaca, Morelos
Entrenamientos para los seleccionados nacionales y aplicacion de tres examenes
selectivos para determinar la delegacion que representara a Mexico en la 51a
Olimpiada Internacional (un maximo de 6 alumnos), la delegacion que representara a Mexico en la XII Olimpiada Centroamericana y del Caribe (un maximo de
3 alumnos) y la preseleccion para la XXV Olimpiada Iberoamericana.
Primera quincena de junio
Lmite para registro de delegados que quieran aplicar el examen propuesto por el
Comite Organizador de la OMM como semifinal de su Concurso Estatal y envo
de este examen semifinal.
Junio
Entrenamientos para los seleccionados nacionales que asistiran a la XII Olimpiada Centroamericana y del Caribe.
Junio, Puerto Rico
XII Olimpiada Centroamericana y del Caribe.
Del 19 al 27 de junio, Morelia, Michoacan
Entrenamientos para los seleccionados nacionales para ir a la 51a Olimpiada
Internacional.
18 y 19 de junio
Aplicacion de los examenes semifinales en los estados registrados con este proposito.
Del 2 al 15 de julio, Astana, Kasajstan
51a Olimpiada Internacional.

50

Informacion Olmpica

Apendice
Teorema 1 (Factorizacion en primos) Todo entero n mayor que 1 puede expresarse
como un producto de primos (con, tal vez, solamente un factor).
Ver [5, 7].
Teorema 2 (Numero

de divisores) Si la factorizacion en primos del entero n es n =


2
r
1
p

p
donde
p1 , p2 , . . . , pr son primos distintos, entonces el numero de dir
1
2
visores positivos de n es igual a (1 + 1)(2 + 1) (r + 1).
Ver [5, 7].
Definicion 3 (Congruencias) Dados dos numeros enteros a, b, y un entero positivo m,
decimos que a es congruente con b modulo m, si a b es multiplo de m. En este caso
escribimos a b (mod m).
Ver [9].
Teorema 4 (Desigualdad media aritmetica - media geometrica) Si x1 , x2 , . . . , xn son
numeros reales positivos, entonces

x1 + x2 + + xn
n x1 x2 xn
n
y la igualdad se cumple si y solo si x1 = x2 = = xn .
Ver [3].
Teorema 5 (Formulas de a rea)
1. El a rea de un rectangulo de lados a y b es a b.
2. El a rea de un triangulo es igual a 12 hl, donde l es la medida de un lado y h es la
medida de la altura sobre dicho lado.
3. El a rea de un crculo de radio r es igual a r2 .
Ver [1, 2].

52

Apendice

Definicion 6 (Angulos
entre paralelas) Cuando una recta intersecta a otras dos rectas se forman ocho a ngulos que numeramos del 1 al 8, como se muestra en la figura.
l1

1
3

l2
2

l3

5
7

Si la recta l3 intersecta a las rectas l1 y l2 , decimos que es transversal a ellas. Los


a ngulos 2, 4, 5 y 7 estan entre las rectas l1 y l2 , los llamamos a ngulos internos, los
a ngulos restantes los llamamos a ngulos externos. Los a ngulos en lados opuestos por
la transversal l3 se llaman a ngulos alternos, como por ejemplo 3 y 5. A los a ngulos 4
y 5 les llamamos alternos internos y los a ngulos 3 y 6 son alternos externos.
A los a ngulos que estan en la posicion correspondiente respecto a la transversal, como
por ejemplo 3 y 7 los llamamos a ngulos correspondientes. Entonces, los pares de
a ngulos correspondientes en la figura anterior son 3 y 7, 1 y 5, 4 y 8, 2 y 6.
Si l1 y l2 son paralelas los a ngulos alternos internos son iguales.
Ver [2].
Teorema 7 (Suma de los a ngulos internos de un triangulo) La suma de los a ngulos
internos de un triangulo es 180 .
Ver [1, 2].
Teorema 8 (Teorema de Pitagoras) En un triangulo rectangulo, el cuadrado de la
hipotenusa es igual a la suma de los cuadrados de los catetos.
Ver [1, 2, 8].
Definicion 9 (Congruencia de triangulos) Los triangulos ABC y A B C son congruentes si los a ngulos y los lados del triangulo ABC son iguales a los a ngulos y los
lados del triangulo A B C .
Ver [1, 2].
Criterio 10 (Criterio de congruencia ALA) Un criterio de congruencia de triangulos nos dice que si tenemos dos triangulos con un lado igual y dos a ngulos adyacentes
iguales, entonces son congruentes. A este criterio se le conoce como a ngulo-ladoa ngulo y lo denotamos como ALA.
Ver [1, 2].
Criterio 11 (Criterio de congruencia LLL) Un criterio de congruencia de triangulos nos dice que si tenemos dos triangulos con sus tres lados correspondientes iguales,
entonces son congruentes. A este criterio se le conoce como lado-lado-lado y lo denotamos como LLL.
Ver [1, 2].

Apendice

53

Definicion 12 (Semejanza de triangulos) Los triangulos ABC y A B C son semejantes, si sus a ngulos respectivos son iguales, es decir,
ABC = A B C
ACB = A C B
BAC = B A C
y sus lados homologos son proporcionales, esto es
BC
CA
AB
= = .

AB
BC
CA
Ver [1, 2].
Criterio 13 (Criterio de semejanza AA) Si dos pares de a ngulos correspondientes
de los triangulos ABC y A B C son iguales, entonces los triangulos son semejantes.
A esta relacion le llamamos a ngulo-angulo y la denotamos como AA.
Ver [1, 2].
Teorema 14 (Ley de los cosenos) En un triangulo de lados a, b y c, se cumple la
relacion
a2 = b2 + c2 2bc cos ,
donde es el a ngulo opuesto al lado a.
Ver [2].
Teorema 15 Si trazamos dos rectas tangentes a una circunferencia desde un mismo
punto P , entonces los segmentos de recta desde P a los puntos de tangencia son iguales
y el centro de la circunferencia yace en la bisectriz del a ngulo entre las rectas.
Ver [2].
Teorema 16 (Medida del a ngulo inscrito) La medida de un a ngulo inscrito en una
circunferencia es igual a la mitad del arco comprendido entre sus lados, es decir, la
mitad del a ngulo central que subtiende el mismo arco.
Ver [1, 2].
Definicion 17 (Cuadrilatero cclico) Un cuadrilatero es cclico si sus cuatro vertices
estan sobre una misma circunferencia.
Ver [2].
Teorema 18 (Cuadrilatero cclico) Un cuadrilatero convexo ABCD es cclico si y
solo si la suma de los a ngulos opuestos es igual a 180, es decir, si y solo si
DAB + BCD = ABC + CDA = 180 .
Ver [2].

54

Apendice

Bibliografa
[1] A. Baldor. Geometra plana y del espacio. Publicaciones Cultural, Mexico, 1999.
[2] R. Bulajich Manfrino, J. A. Gomez Ortega. Geometra. Cuadernos de Olimpiadas
de Matematicas. Instituto de Matematicas de la UNAM, 2002.
[3] R. Bulajich Manfrino, J. A. Gomez Ortega, R. Valdez Delgado. Desigualdades.
Cuadernos de Olimpiadas de Matematicas. Instituto de Matematicas de la UNAM. Tercera edicion, 2007.
[4] R. Grimaldi, Matematicas Discreta y Combinatoria. Addison Wesley Longman,
Pearson. Tercera edicion, 1998.
[5] I. Niven, H. Zuckerman. Introduccion a la Teora de los Numeros. Limusa-Wiley,
Mexico 1972.
[6] M. L. Perez Segu. Combinatoria. Cuadernos de Olimpiadas de Matematicas.
Instituto de Matematicas de la UNAM, 2000.
[7] M. L. Perez Segu. Teora de Numeros. Cuadernos de Olimpiadas de Matematicas.
Instituto de Matematicas de la UNAM, 2003.
[8] A. Rechtman Bulajich. Algunas demostraciones del teorema de Pitagoras. Revista de la Olimpiada Mexicana de Matematicas, Tzaloa No. 1, 2010.
[9] A. Rechtman Bulajich, C.J. Rubio Barrios. Divisibilidad y congruencias. Revista
de la Olimpiada Mexicana de Matematicas, Tzaloa No. 2, 2009.
[10] N. Vilenkin. De cuantas formas? (Combinatoria). Editorial Mir, Moscu 1972.

56

Directorio

Directorio del Comite Organizador de la OMM


Anne Alberro Semerena
Facultad de Ciencias, UAEM
Av. Universidad 1001
62210, Cuernavaca, Morelos
Tel. (777) 3 81 03 80
Fax (777) 3 29 70 40
aalberro@uaem.mx

Gerardo Arizmendi Echegaray


Centro de Investigacion en Matematicas
Callejon Jalisco s/n, Mineral de Valenciana
36240, Guanajuato, Guanajuato
Tel. (473) 7 32 71 55
gerardo@cimat.mx

Ignacio Barradas Bribiesca


Universidad de Guanajuato
L. de Retana #5, Centro
36000, Guanajuato, Guanajuato
Tel. (473) 7 32 00 06 ext 2006
barradas@quijote.ugto.mx

Radmila Bulajich Manfrino


Facultad de Ciencias, UAEM
Av. Universidad 1001
62210, Cuernavaca, Morelos
Tel. (777) 3 29 70 20
Fax (777) 3 29 70 40
bulajich@uaem.mx

Gabriela Campero Arena


Facultad de Ciencias, UNAM
Av. Universidad 3000
04510, Mexico, D.F.
Tel. (55) 56 22 48 67
Fax (55) 56 22 48 66
gabriela@matematicas.unam.mx

Fernando Campos Garca

1a de Angel
Rico 85
AU.H. Vicente Guerrero
09200, Iztapalapa, Distrito Federal
Tel. (55) 34 63 75 43
fermexico89@hotmail.com

Jose Antonio Climent Hernandez


Facultad de Ciencias, UNAM
Av. Universidad 3000
04510, Mexico, D.F.
Tel. (55) 56 24 59 22
Fax (55) 56 22 48 59
jach@fciencias.unam.mx

Jose Alfredo Cobian Campos


Facultad de Ciencias, UNAM
Av. Universidad 3000
04510, Mexico, D.F.
Tel. (55) 56 22 49 25
Fax (55) 56 22 48 59
cobian@matematicas.unam.mx

58

Directorio

Luis Cruz Romo


UPIITA, IPN
Av. Instituto Politecnico Nacional 2580
Col. Barrio la Laguna Ticoman
07340, Mexico, D.F.
lucruz@ipn.mx

Marco Antonio Figueroa Ibarra


Facultad de Matematicas
Universidad de Guanajuato
Callejon Jalisco s/n, Mineral de Valencia
36240, Guanajuato, Guanajuato
Tel. (473) 7 32 01 40
marcant@cimat.mx

Jesus
Jeronimo Castro
CIMAT
Apartado Postal 402
36000, Guanajuato, Guanajuato
Tel. (473) 7 32 71 55
Fax (473) 7 32 57 49
jeronimo@cimat.mx

Leonardo Ignacio Martnez Sandoval


Primera Cerrada de Alfalfares 41-2
Rinconada Coapa Primera Sec, Tlalpan
14330, Mexico, D.F.
Tel. (55) 26 52 23 29
ssbmplayer@gmail.com

Carlos Jacob Rubio Barrios


Universidad Autonoma de Yucatan
Periferico norte tablaje 13615
97119, Merida, Yucatan
Tel. (999) 942-3140 al 49
Fax (999) 942-31-40
carlos.rubio@uady.mx
jacob.rubio@gmail.com

Elena Ruiz Velazquez


Altair 12
Col. Lomas de Palmira
62550, Cuernavaca, Morelos
Tel. (777) 320 54 39
Cel. (777) 133 39 83
eleniux@gmail.com
A00375640@itesm.mx

Pablo Soberon Bravo


Circuito Interior no. 830
Fracc. La Herradura
62303, Cuernavaca, Morelos
Cel. (777) 134 55 49
bandrak@hotmail.com

Carmen Sosa Garza


Facultad de Ingeniera, UAQ
Cerro de las Campanas s/n
Queretaro, Queretaro
Tel. (442) 1 92 12 64 ext. 121 o 136
Fax (442) 1 92 12 646
carsg@uaq.mx

Rogelio Valdez Delgado


Facultad de Ciencias, UAEM
Av. Universidad 1001
62210, Cuernavaca, Morelos
Tel. (777) 3 29 70 20
Fax (777) 3 29 70 40
rogelio@matcuer.unam.mx

Eduardo Velasco Barreras


Universidad de Sonora
Calle Yucas 16, Vista Bella
83170, Hermosillo, Sonora
Tel. (662) 2 19 10 07
hamsteritokeweb@hotmail.com

Hugo Villanueva Mendez


Instituto de Matematicas, UNAM
Cub. 4 de Becarios,
Circuito Exterior, Ciudad Universitaria
Coyoacan 04510,
Mexico, D.F.
Tel (55) 56 22 45 32
vill hugo@hotmail.com
hvillan@matem.unam.mx

Directorio

59

Direccion Postal de la Olimpiada Mexicana de Matematicas:


Cubculo 201, Departamento de Matematicas.
Circuito Exterior, Facultad de Ciencias.
Universidad Nacional Autonoma de Mexico.
Ciudad Universitaria.
Colonia Copilco, C.P. 04510.
Delegacion Coyoacan.
Mexico, Distrito Federal.
Telefono: (55) 5622-4864.
Fax: (55) 5622-5410.
Email: omm@ciencias.unam.mx
Pagina oficial de la Olimpiada Mexicana de Matematicas:
http://www.omm.unam.mx/

TZALOA
Revista de la Olimpiada
Mexicana de Matematicas
2010, No. 3
Ano

Comite Editorial:
Anne Alberro Semerena
Ana Rechtman Bulajich
Carlos Jacob Rubio Barrios
Francisco Ruiz Benjumeda

Comite de la Olimpiada Mexicana de Matematicas


Cubculo 201
Departamento de Matematicas
Facultad de Ciencias, UNAM
Circuito Interior s/n
Ciudad Universitaria
Coyoacan C.P. 04510
Mexico D.F.
Telefono: (55) 56-22-48-64
www.omm.unam.mx

Diseno de Portada: Manuel Macas Beckmann


www.rayaenmedio.com

Impreso: Torre y de la Torre Impresos


Aragon no. 134

Col. Alamos,
03400
Mexico D.F.
Telefonos: (55) 55-30-14-82 y (55) 55-38-34-53

c
Queda
estrictamente prohibida la reproduccion parcial o total por cualquier sistema
o metodo, mecanico o electronico, sin autorizacion previa del autor.
Impreso y hecho en Mexico.
Julio de 2010.

Contenido

Presentacion

Artculos de matematicas: Solucion de problemas y temas iniciales para la


olimpiada de matematicas

Problemas de practica

11

Soluciones a los problemas de practica

15

Problemas propuestos
2010 No. 3
Problemas propuestos. Ano
2010 No. 1
Soluciones a los problemas propuestos. Ano

25
25
26

Olimpiadas Internacionales
XXII Olimpiada de la Cuenca del Pacfico
American Mathematics Competition (AMC)
XII Olimpiada Centroamericana y del Caribe

31
31
32
41

Informacion Olmpica

43

Apendice

45

Bibliografa

49

Directorio

51

IV

Contenido

Presentacion

Tzaloa es una publicacion periodica trimestral de la Olimpiada Mexicana de Matematicas y su objetivo es fomentar el estudio de las matematicas como una disciplina dinamica y creativa. El diseno de las secciones y la cuidadosa seleccion de sus contenidos
buscan apoyar de manera efectiva, con informacion y con materiales de calidad, a estudiantes y profesores de nivel medio superior que cada ano se preparan para participar
en los diferentes concursos de la Olimpiada de Matematicas.
Esta revista, con orgullo, toma su nombre del nahuatl porque esta hecha por y para los
mexicanos. Tzaloa significa aprender y las paginas que la conforman buscan ayudar a
satisfacer la necesidad de contar con espacios adecuados para profesores, estudiantes
y, en general, para todas aquellas personas interesadas en desarrollar e incrementar sus
capacidades para el razonamiento logico matematico y la resolucion de problemas.

2010, Numero

Tzaloa, Ano
3
El material que se presenta en este tercer numero del ano 2010 se escogio pensando
en estudiantes y profesores que se preparan para las u ltimas etapas de los concursos
estatales, asimismo busca apoyar a las delegaciones estatales que participaran en el
concurso nacional. Es por eso que, en esta ocasion, las secciones de Problemas de
Practica y Problemas Propuestos no contienen problemas introductorios y el material
escogido se ubica en las categoras intermedia y avanzada.
A pesar de lo anterior, no olvidamos que muchos de nuestros lectores ya comienzan a
preparase para los concursos del proximo ano y por ello incluimos un excelente artculo para principiantes. A traves de las paginas que lo conforman, Hector Flores nos
comparte su amplia experiencia y nos ofrece numerosos consejos de gran utilidad para
estudiantes y profesores que se inician en la aventura olmpica. Al final del artculo
se incluye una lista con 20 problemas introductorios, de los cuales no se presentan las
soluciones. Invitamos a nuestros lectores para que los resuelvan y nos enven cuanto antes sus soluciones a la direccion electronica revistaomm@gmail.com, para

Presentacion

VI

que podamos publicar las mas ingeniosas en alguno de los proximos numeros de esta
revista.
Como siempre, en la seccion de Olimpiadas Internacionales presentamos los resultados
y examenes de los u ltimos concursos en los que Mexico participo: la XXII Olimpiada de
la Cuenca del Pacfico y la XII Olimpiada Centroamericana y del Caribe. Ademas, en
esta ocasion, tambien incluimos los examenes de la American Mathematicas Competition (AMC), mismos que sirvieron como preparacion para las delegaciones mexicanas
que este ano participan en los concursos internacionales.

Mexico y las Olimpiadas de Matematicas


Hace mas de 23 anos que la Sociedad Matematica Mexicana ha venido impulsando
vigorosamente los trabajos de la Olimpiada Mexicana de Matematicas (OMM). Desde
sus inicios, este programa se ha visto fortalecido gracias a la participacion de miles
de jovenes estudiantes y a la entusiasta colaboracion de muchos profesores quienes,
de manera espontanea y altruista, han dedicado sus esfuerzos a mejorar la ensenanza
y elevar la cultura matematica de nuestro pas. Motivados por el movimento olmpico,
en escuelas ubicadas a lo largo de todo el territorio nacional, se han desarrollado innumerables talleres de resolucion de problemas, donde estudiantes y profesores trabajan
con el u nico afan de incrementar sus capacidades para el razonamiento, el analisis y la
creatividad matematica.
En el a mbito internacional, mediante la destacada participacion de las delegaciones
mexicanas en diversos concursos, la Olimpiada Mexicana de Matematicas ha contribuido a elevar el prestigio de la matematica nacional. Pero, mas importante aun ha sido
la contribucion que el movimiento olmpico ha tenido para el desarrollo cientfico del
pas. En muchos casos, la deteccion temprana de jovenes con talento matematico excepcional ha permitido brindarles una formacion adecuada para desarrollar al maximo
todo su potencial. Asimismo, la participacion en los concursos olmpicos ha definido
las vocaciones de muchos otros estudiantes. Universidades de todo el pas se han visto beneficiadas con el ingreso de jovenes ex-olmpicos, mismos que cuentan con una
solida formacion matematica y muchos de los cuales han permanecido en ellas para
dedicar su vida profesional a la docencia y la investigacion.

24a Olimpiada Mexicana de Matematicas


El programa anual de la Olimpiada Mexicana de Matematicas se desarrolla en 3 etapas:
Concursos Estatales.
Concurso Nacional.
Entrenamiento, seleccion y participacion de las delgaciones nacionales que representan a Mexico en concursos internacionales.

Presentacion

VII

En la 24a Olimpiada Mexicana de Matematicas podran participar los estudiantes de


Mexico nacidos despues del 1 de agosto de 1991. Los concursantes deberan estar inscritos en una institucion preuniversitaria durante el primer semestre del ciclo escolar
2010-2011 y, para el 1 de julio de 2011, no deberan haber iniciado estudios universitarios. Para mayor informacion puedes consultar la pagina:
http://www.omm.unam.mx
Para la primera etapa, los participantes deberan inscribirse directamente con el Comite Estatal correspondiente.
El Concurso Nacional de la 24a Olimpiada Mexicana de Matematicas se realizara del
21 al 26 de noviembre de 2010 en Ensenada, Baja California. A los primeros lugares
de este certamen se les invitara a la etapa de entrenamiento y seleccion de las delegaciones que representaran a Mexico en las distintas Olimpiadas Internacionales del ano
2011: la XXIII Olimpiada Matematica de la Cuenca del Pacfico, que se llevara a cabo
en el mes de marzo; la XIII Olimpiada Matematica de Centroamerica y el Caribe, que
se celebrara en el mes de junio; la 52a Olimpiada Internacional de Matematicas, que se
llevara a cabo en julio en Amsterdam, Pases Bajos, y la XXVI Olimpiada Iberoamericana de Matematicas que se realizara en el mes de septiembre en Costa Rica.

VIII

Presentacion

Solucion de problemas y temas


iniciales para la olimpiada de
matematicas
Por Hector Raymundo Flores Cantu

El objetivo de este escrito es proporcionar un primer acercamiento al tipo de conocimientos y razonamientos que son necesarios para los participantes en la olimpiada de
matematicas en Mexico. Aunque hay excelente material publicado, tanto colecciones
de problemas como textos con teora, muchos profesores y participantes han manifestado su dificultad para extraer el material necesario para iniciar la preparacion en la
competencia. Por otra parte tambien espero que los temas y problemas que aqu son
presentados sean de interes para cualquier otra persona interesada en la solucion de
problemas y en matematicas. As como que sirva para dar una vision distinta de lo que
representa el verdadero quehacer dentro de la matematica. Me refiero a la solucion de
problemas.

Solucion de problemas
(Matematica es la ciencia de resolver problemas)

La solucion de problemas es considerada la mas compleja de las funciones intelectuales y ha sido definida como un proceso que requiere el control y manejo de otras
habilidades fundamentales como el analisis, la deduccion, la creatividad y la memoria.
El proceso ocurre cuando una persona (grupo, ente, ...) desea llegar a un objetivo, pero
no sabe exactamente como proceder. Llamamos solucion del problema justamente a la
forma de llegar al objetivo deseado. Cuando la persona sabe como proceder para llegar
al objetivo, la situacion no es considerada como problema, sino como un ejercicio y lo
que procede es simplemente seguir los pasos para llegar a este objetivo. Aunque los

Solucion de problemas y temas iniciales para la olimpiada de matematicas

ejercicios forman parte fundamental en el proceso de aprendizaje de la matematica, no


sera e ste el e nfasis aqu.
Problema 1. Encuentra la suma de todos los numeros impares menores a 1, 000, 000.
Obtener esa suma usando lapiz y papel no es una solucion satisfactoria. Aun usando
una calculadora el tiempo requerido para hacer todas las sumas hace que esta opcion
tampoco lo sea. Mas adelante propondremos el problema de estimar el tiempo que e sto
tardara. Por otra parte muchas personas con conocimiento de computacion podran
hacer un programa o usar una hoja de calculo para obtener esa suma. Pero asumiendo
que no admitimos el uso de una computadora, encontrar el valor de la suma del problema anterior, es un buen ejemplo de una situacion que sera considerada como un
problema para muchas personas. Sin embargo cualquier participante intermedio de la
olimpiada de matematicas dara la solucion a este tipo de problema en unos segundos
en media hoja de papel. Esto no porque sean genios sino porque la estrategia para
este problema es uno de los primeros topicos de la olimpiada y obtener la suma ya no
es un problema, sino un ejercicio para ellos.
De esta forma, una situacion puede representar un problema para algunos y no serlo
para otros, dependiendo de la experiencia de cada uno. Podemos decir entonces que un
problema se resuelve una sola vez, despues se convierte en ejercicio, as como cualquier
otra situacion parecida. Una consecuencia importante de esto es que cada problema debe intentarse durante un tiempo suficiente antes de revisar la solucion. Las personas
que buscan la solucion al final del libro antes de hacer un intento serio por resolver
el problema, estan desperdiciando una oportunidad valiosa de ejercitarse en sus habilidades que nunca podran recuperar.
Los problemas, tal como los definimos, aparecen constantemente en todas las a reas
del conocimiento y en cualquier sector, como la industria, los negocios, la economa,
la poltica, la administracion, as como en todas las ciencias naturales e ingenieras.
Es practicamente imposible encontrar algun a rea donde no haya problemas. En todas
partes nos enfrentaremos en algun momento con querer llegar a algo, pero no saber
como proceder.

Olimpiada de Matematicas
Uno de los objetivos centrales de la olimpiada de matematicas, es identificar y fomentar
la capacidad de los participantes para resolver problemas. Ya sean problemas de matematicas o problemas no matematicos que requieren el uso de metodos matematicos
para obtener su solucion. La olimpiada de matematicas representa un marco excelente
para ejercitar la habilidad para resolver problemas debido a que estos solo requieren
de conocimientos relativamente elementales en matematicas. La mayora se resuelve
con lo que aparece en el programa de estudios de nivel secundaria. En particular, no
requieren de trigonometra, calculo ni geometra analtica. Pero la dificultad de los problemas no esta en el conocimiento de los temas matematicos sino en la habilidad del

Solucion de problemas y temas iniciales para la olimpiada de matematicas

alumno para organizar, controlar y usar adecuadamente esos conocimientos para hallar
la solucion del problema. Los problemas iniciales de olimpiada estan mas relacionados
con el tipo de problemas que aparecen en las pruebas de rendimiento de estudiantes como PISA, ENLACE, TIMSS, etc. De hecho la olimpiada de matematicas es una forma
excelente de preparar a los alumnos para estas pruebas de rendimiento.
En los entrenamientos para la olimpiada de matematicas, cada vez que propongo un
problema a un nuevo alumno, el primer comentario que hace es: - No se que hacer.- A
menudo no tardan mas de un minuto en hacerlo. Mi respuesta es: -EXACTO! Por eso
es un problema. Si supieras que hacer, sera un ejercicio.- Hacer ejercicios es importante en el proceso de reforzar los conocimientos aprendidos y es tambien importante
en la olimpiada de matematicas, pero es algo que los alumnos pueden hacer generalmente por su cuenta o con tareas. La parte mas importante de los entrenamientos es la
solucion de problemas, no de ejercicios. El proposito es desarrollar la capacidad para
resolver problemas y resolver cada vez problemas mas difciles. Aunque existen muchas estrategias para resolver problemas, lo que es indudable es que para mejorar es
necesario practicar. Enfrentarse a ese no se que hacer.

Temas de Matematicas para la Olimpiada


Antes de resumir los temas de matematicas mas utilizados en la olimpiada de matematicas, es necesario mencionar que lo mas importante, ademas de la concentracion
y la paciencia, es el sentido comun y la logica. De hecho podemos pensar en la logica
como sentido comun bien estructurado y no por nada es la parte central de la matematica. Los razonamientos deben ser, antes que nada, correctos. Esto representa tener bien
claras las relaciones entre causas y consecuencias y hacer deducciones correctas basado
en la informacion que esta a disposicion.
Es necesario ser precavido. Reconocer lo falso de lo correcto no siempre es tan facil.
No todos los razonamientos que parecen correctos, son correctos. En el lenguaje de la
logica se llama falacia a un razonamiento falso que parece correcto o razonable.
Problema 2. Si 3 gatos cazan 3 ratones en 3 minutos, cuantos minutos tarda un gato
en cazar un raton?
Una gran cantidad de alumnos responden que un gato caza un raton en un minuto. Esto
suena razonable, pero es falso. En este caso, basta pensar un poco mas el problema
para encontrar la respuesta correcta. Se espera que los alumnos participantes sean cuidadosos y capaces de resolver problemas de razonamiento logico.
Problema 3. Andres, Benito, Carlos y Daniel tienen sus oficinas en el mismo edificio.
Uno de ellos es abogado, otro es banquero, otro es contador y otro es dentista. Si tenemos la siguiente informacion: - Daniel es cliente del abogado. - El contador es amigo
de Benito, pero ninguno es cliente del otro. - El dentista tiene como cliente a Daniel. Ni Andres ni el dentista conocen a Carlos. Como se llama el abogado?

Solucion de problemas y temas iniciales para la olimpiada de matematicas

Este problema requiere extraer la informacion, representarla ordenadamente y hacer


deducciones correctas. La dificultad de resolver el problema 3 radica muchas veces en
no poder extraer toda la informacion que ofrece el enunciado del problema.
Los temas de matematicas que mas son necesarios en la olimpiada de matematicas se
dividen en cuatro a reas principales que son: geometra, matematicas discretas, teora
de numeros y a lgebra. Muchos de los temas estan incluidos en los programas de educacion basica y secundaria. A pesar de esto, en los entrenamientos siempre es necesario
dar un repaso a esos temas enfocandose, no en aprender las cosas de memoria, sino en
entender los conceptos de forma tal que puedan ser utilizados en la solucion de problemas no triviales. Veremos ahora de forma breve los contenidos principales de cada una
de estas a reas.

Geometra
Los problemas de geometra son (por mucho) los mas comunes en la olimpiada de matematicas y los que menos son tratados en las clases en la escuela. Uno de cada tres
problemas en los examenes de olimpiada son de geometra. Geometra se refiere a la
geometra del plano y del espacio, no a la geometra analtica ni a la trigonometra. Se
requiere de los resultados de a ngulos entre paralelas, a ngulos opuestos por el vertice,
suma de a ngulos en un triangulo, definicion de tipos de triangulos (equilatero, isosceles, triangulo rectangulo, etc.), postulados de triangulos congruentes, a rea de figuras
elementales y el teorema de Pitagoras. En entrenamientos posteriores se cubren mas
temas de geometra, pero el e nfasis esta en el razonamiento geometrico y no en aprenderse formulas.
Problema 4. En dos caras no opuestas de un cubo se trazan diagonales a partir de uno
de los vertices. Encuentra la medida del a ngulo formado entre ellas. (Nota: Un cubo es
una figura solida formada por seis caras cuadradas, se puede pensar en un dado.)

Matematicas discretas
Este tema se refiere a las situaciones donde se trata con conjuntos finitos o conjuntos
que pueden enumerarse (1, 2, 3, . . .). Los problemas de este tipo incluyen temas de
combinatoria, permutaciones, principios de conteo, conjuntos y otros que se cubren en
etapas posteriores. Algunos de los problemas mas interesantes son de conteo. Aunque

Solucion de problemas y temas iniciales para la olimpiada de matematicas

los ninos aprenden a contar desde los 4 anos, contar puede no ser tan facil algunas veces.
Problema 5. Cuantos caminos distintos se pueden seguir para llegar del punto A al
punto B en la figura de la torre petrolera si solo esta permitido moverse hacia abajo
y hacia los lados, pero no hacia arriba?
A

Teora de Numeros
En algunos libros se le llama tambien aritmetica, pero no se refiere solo a saber sumar,
restar, multiplicar y dividir. En teora de numeros se trata sobre problemas que se refieren a las propiedades de los numeros enteros {. . . , 3, 2, 1, 0, 1, 2, 3, . . .}. Son
importantes los conceptos de multiplo, divisor, maximo comun divisor, mnimo comun
multiplo, numero primo, factorizacion en primos, algoritmo de la division, residuo y
sistema decimal.
Problema 6. Es facil ver que 1, 000 1, 000 = 1, 000, 000. Existen dos numeros
enteros que no tengan ceros como dgitos y que al multiplicarlos den como resultado
1, 000, 000?

Algebra
Aunque el a lgebra esta bien cubierta en el programa de las escuelas, los problemas de
a lgebra suelen ser los mas complicados para los participantes, esto a pesar de que no
se utilizan temas avanzados. Se requiere manipular expresiones algebraicas, conocer
los identidades o productos notables (diferencia de cuadrados, trinomio cuadrado perfecto, suma y diferencia de cubos), factorizacion, leyes de los exponentes y radicales,
solucion de ecuaciones y sistemas de ecuaciones lineales, soluciones de la ecuacion
cuadratica, razones y proporciones (regla de tres), progresiones aritmeticas, progresiones geometricas y desigualdades. El problema 1, por ejemplo, representa una progre-

Solucion de problemas y temas iniciales para la olimpiada de matematicas

sion aritmetica, pero hay otros tipos de problemas.


Problema 7. Si x, y son dos numeros que cumplen
x+y
2

x +y

1,

2,

encuentra el valor de x3 + y 3 .

Estrategias elementales para resolver problemas


Ya en 1945 Polya escribio un libro sobre el tema de solucion de problemas. Se mencionan 4 pasos en el proceso de solucion. Entender el problema, hacer un plan, desarrollar
el plan e interpretar los resultados. Pocas personas estan acostumbradas e seguir estos
pasos. La mayora de las personas espera que la idea brillante para resolver el problema
se le ocurra de forma milagrosa y no esta dispuesta a perder el tiempo con ideas o
planes que no llevan a la solucion. Pero cuando tenemos un problema, no es posible
predecir si una idea sera u til o inutil sin haberla desarrollado.
Las buenas ideas provienen de la experiencia,
la experiencia proviene de las malas ideas.
Proverbio de los nativos americanos.
En ocasiones la idea brillante se nos ocurre, pero e sto pasa solo cuando el problema
es suficientemente sencillo comparado con nuestra experiencia, con problemas mas
complicados es necesario ayudarnos. La generacion de ideas y planes es un proceso
creativo. Que hacer cuando no tenemos ideas? Algunas de las estrategias mas comunes
y u tiles son muy conocidas y consisten en concentrarse en alguno de los siguientes
puntos.
a) Reconoce correctamente los datos, las variables y objetivos del problema.
b) Concentrarse solo en una parte del problema.
c) Intentar dar valores a las cantidades desconocidas.
d) Hallar un problema mas simple parecido o relacionado.
e) Suponer que ya se llego a la solucion, que se puede deducir de eso?
f) Tratar de presentar la informacion de forma distinta (hacer un diagrama, grafica
o tabla).
Hay una gran cantidad de estrategias adicionales. Se recomienda consultar especialmente [1] y [4] de la bibliografa.
En los problemas 1, 5 y 6 evidentemente una de las cosas que dificulta los problemas es la magnitud. En todos los problemas donde la dificultad es la magnitud de una

Solucion de problemas y temas iniciales para la olimpiada de matematicas

cantidad, una estrategia es imaginar que esa cantidad es menor y tratar de resolver el
problema que resulta. En el problema 2 vale la pena concentrarse solo en una parte del
problema, digamos el numero de gatos. Si imaginamos que ese numero es fijo y analizamos que pasa con las otras cantidades, no tardaremos en hallar la solucion correcta.
En el problema 3 conviene representar la informacion de alguna manera visual, porque
es difcil mantener todos los datos en la mente como para hacer los razonamientos.
En el problema 4 podra ser suficiente construir un pequeno modelo para sospechar
cual es la respuesta. La justificacion es un poco mas delicada, ya que es una demostracion geometrica. Pero es conveniente meditar en la pregunta: Como justifico que un
a ngulo mide x grados? En el problema 7 la respuesta no es 3, y aunque es posible resolver el sistema de ecuaciones, hay una solucion mas simple. Conviene observar bien
los elementos que aparecen (cuadrados, suma de cubos). La suma de cubos se puede
factorizar, pero que te recuerda la expresion x2 + y 2 ? Conoces algo en a lgebra que
contenga esa expresion?

Lista de problemas
Como lo hemos mencionado, lo mas importante al enfrentarse a un problema es concentrarse y ser paciente. Los problemas presentados aqu fueron creados o elegidos
de forma que pueden ser resueltos con un mnimo de conocimientos teoricos para las
primeras etapas de la olimpiada de matematicas en Nuevo Leon, pero no por esto son
faciles. No estan ordenados necesariamente por grado de dificultad. Esto en parte por
lo complicado de determinar la dificultad de un problema, pero tambien porque normalmente no es conveniente para el alumno saber de antemano si el problema es facil
o difcil. Recomendamos dedicar no menos de 20 o 30 minutos a cada problema y en
caso de no poder encontrar la solucion, generar muchas ideas.
Problema 8. El primer examen selectivo para la olimpiada de matematicas en Nuevo
Leon tiene 20 preguntas. Cada pregunta tiene un valor de 3 puntos si es contestada
correctamente, 0 puntos si es contestada equivocadamente y 1 punto si se deja sin contestar. La calificacion de los participantes es la suma de los puntos obtenidos en los
20 problemas. Cuantas calificaciones distintas se pueden obtener en el examen? Si el
examen es de opcion multiple y cada problema tiene 4 opciones, que conviene mas,
adivinar las respuestas o dejar el examen sin contestar?
Problema 9. En la figura se muestra un triangulo y un hexagono regular cuya interseccion es una figura de 3 lados (un triangulo). Dibuja un hexagono regular y un triangulo
(no tiene que ser equilatero) de modo que la figura formada por la interseccion de ambos tenga el maximo numero posible de lados. Justifica que no es posible obtener otra
figura con mas lados.

Solucion de problemas y temas iniciales para la olimpiada de matematicas

Problema 10. Supongamos que desea resolverse el problema 1 (la suma de todos los
impares menores a 1, 000, 000) haciendo toda la operacion:
1 + 3 + 5 + 7 + 9 + 11 + + 999, 997 + 999, 999 =
en una calculadora. Si tardamos 1 minuto en presionar 100 teclas de la calculadora,
cuanto tiempo tardaremos en obtener el resultado de la suma?
Problema 11. En una pizzera las pizzas medianas miden 8 pulgadas de diametro y
cuestan $55. La pizza familiar mide 16 pulgadas de diametro y cuesta $165. Un grupo de amigos quiere comprar varias pizzas, no saben si comprar seis pizzas medianas
o comprar dos familiares. Cual de estas dos opciones les da mas pizza por su dinero?
Problema 12. El precio de los dulces en una tienda es menor a $2.00, pero mayor a
$1.03. En la tienda se vendieron todos los dulces a un total de $31.45. Si todos los
dulces valen lo mismo, cuantos dulces se vendieron?
Problema 13. El lado del cuadrado grande mide 10 metros. Si se unen los puntos
medios de los lados con los vertices, cual es el a rea del cuadro central? Nota: La
respuesta no es 25 m2 .

Problema 14. Encuentra todos los valores de x que sean solucion de la ecuacion:
8x + 2 = 4x + 2x+1 .
Problema 15. Considera una piramide triangular formada por cuatro triangulos equilateros (tambien se llama tetraedro regular). Llamamos a los vertices A, B, C, D. Si E es
el punto medio de AB, F es el punto medio de AC y los cuatro triangulos equilateros
tienen lados de longitudes 2 cm, encuentra el a rea del triangulo DEF .

Solucion de problemas y temas iniciales para la olimpiada de matematicas

A
F
E
C
B
D
Problema 16. Encuentra (sin usar calculadora y sin aproximar con decimales) el valor
numerico exacto de la siguiente expresion:
r

3
4+
7+
2

3
7.
2

Problema 17. Los numeros de dos dgitos 96 y 46 tienen la curiosa propiedad de que al
multiplicarlos el resultado es igual al obtenido si cambiamos la posicion de los dgitos
de cada uno. Es decir 96 46 = 69 64. Determina si que existe otro numero de dos
dgitos; distinto de 46, que tiene la misma propiedad al multiplicarlo por 96. Cuantos
numeros diferentes hay con esta propiedad?
Problema 18. En un baile haba 28 personas, N de ellas eran mujeres. La mujer 1
bailo con 5 hombres, la mujer 2 bailo con 6 hombres, la mujer 3 bailo con 7 hombres
y as sucesivamente hasta la mujer N que bailo con todos los hombres. Cuantos hombres y cuantas mujeres haba en el baile?
Problema 19. Un logico (L) y un matematico (M) son amigos y cumplen anos el mismo da. En una de sus fiestas de cumpleanos platican sobre sus respectivas edades.
Aqu esta el dialogo:
L: Estoy pensando en tres numeros enteros que multiplicados dan 2450 y sumados dan
tu edad.
M: Despues de pensarlo mucho, no puedo saber con seguridad en cuales numeros estas
pensando.
L: Cada uno de los numeros es menor a mi edad.
M: Ahora ya se cuales son los numeros en los que estas pensando.
Encuentra las edades de ambos amigos.
Problema 20. Un grupo de pintores debe pintar 279 puertas de las cuales 186 puertas
deben ser pintadas de blanco y el resto deben ser pintadas de negro. Durante la primera
mitad del da todos los pintores se dedican a pintar puertas blancas. En la segunda mitad
del da, la mitad de los pintores pintan puertas blancas y la otra mitad pintan puertas
negras. Las puertas blancas quedan terminadas justo al terminar el primer da, pero no
todas las negras. El siguiente da es dedicado por un solo pintor para terminar de pintar
el resto de las puertas negras. Cuantos pintores haba?

10

Solucion de problemas y temas iniciales para la olimpiada de matematicas

Consideraciones finales
Como sugerencia para los profesores es necesario mencionar que los problemas deben
ser, antes que nada, interesantes y estimulantes para los alumnos. Pocos estan dispuestos a dedicar 30 minutos a pensar una situacion que parece tediosa o aburrida. Tambien
es importante no enfrentar a los alumnos con problemas demasiado difciles (ni demasiado faciles). Los problemas faciles aburren y los difciles frustran. Tampoco es
conveniente dar la solucion de los problemas. Es preferible dar sugerencias, especialmente tratar de imaginar, como se le puede a alguien haber ocurrido esta idea? Lo
mejor es impulsar al alumno para que tenga sus propias ideas.
Tambien recomendamos no descartar las ideas de los alumnos si son diferentes de las
soluciones conocidas. No debemos olvidar que el proposito no es solamente hallar la
solucion, sino aprender y desarrollar las habilidades para resolver problemas. De esta
forma, sin importar si al final se resuelve el problema o no, siempre sera de beneficio
para el alumno (y el profesor).
Esperamos que este material pueda servir como una introduccion amigable a los problemas y temas propios de la olimpiada de matematicas. Todos los involucrados en
estas competencias sabemos que la primera impresion al acercarse a estos problemas
por parte de alumnos y profesores, no siempre es facil. Existe una gran cantidad de
material publicado en la Web pero no aparece ordenado por grado de dificultad y suele
ser complicado encontrar el material adecuado. Para aquellos lectores que desean continuar conociendo problemas interesantes y mejorando sus habilidades para resolver
problemas usando material escrito, recomendamos los siguientes libros.

Bibliografa
1. E. Bono. Lateral Thinking. Penguin Books 1978.
2. D. Fomin, S. Genkin, I. Itenberg. Mathematical Circles: Russian Experience.
American Mathematical Society 1996.
3. http://sites.google.com/site/eommnl/
4. G. Polya. Como plantear y resolver problemas. Trillas 2005.
5. A. Posamentier, C. Salkind. Challenging Problems in Geometry. Dover 1996.
6. P. Zeitz. The Art and Craft of Problem Solving. Wiley 2006.

Problemas de practica
En esta seccion encontraras 20 interesantes problemas de nivel intermedio y avanzado
con los que podras poner a prueba tus habilidades y mismos que esperamos sean interesantes y u tiles para tu preparacion. En la siguiente seccion encontraras sus soluciones
pero, como nos menciona Hector Flores en su artculo, no debes consultar la solucion
de un problema sino hasta despues de que tengas tu propia solucion o, por lo menos,
hasta que le hayas dedicado bastante tiempo. Ten en cuenta que al consultar la solucion
de un problema sin haber hecho un verdadero esfuerzo para resolverlo, desperdicias la
oportunidad de incrementar tus habilidades para enfrentar situaciones difciles.
Por u ltimo, te invitamos a contribuir para que esta seccion de la revista se siga enriqueciendo con la participacion de todos. Estamos seguros que concoces y tienes
problemas interesantes que proponer, por eso ponemos a tu disposicion la direccion
revistaomm@gmail.com, donde con gusto recibiremos tus sugerencias.
Problema 1. Sea ABC un triangulo rectangulo cuyo a ngulo recto esta en B y AB <
BC. En la bisectriz del a ngulo ABC tomamos el punto P tal que AP es perpendicular
a dicha bisectriz. Sea M el punto medio de AC, y E la interseccion de M P con el
cateto AB. Si EM = 15 cm, cuanto mide BC?
Problema 2. Si a, b, c y d son las races de la ecuacion x4 2x3 7x2 2x + 1 = 0,
encuentra el valor de
1 1 1 1
+ + + .
a b
c d

Problema 3. La isla de Nuncajamas tiene 2010 habitantes, divididos en tres clanes:


caballeros que siempre dicen la verdad, plebeyos, que siempre mienten, y espas que
alternadamente dicen mentiras un da y dicen la verdad el siguiente da. Un reportero
visita la isla por dos das. El primer da, e l se encuentra con todos los habitantes. El
primero dice: hay exactamente un plebeyo en la isla; el segundo dice: hay exactamente dos plebeyos en la isla; y as sucesivamente hasta llegar al habitante numero

12

Problemas de practica

2010 que dice: hay exactamente 2010 plebeyos en la isla. El segundo da, el reportero habla con todos los habitantes otra vez, en el mismo orden. El primero dice: hay
exactamente un caballero en la isla; el segundo dice: hay exactamente dos caballeros
en la isla; hasta llegar al habitante 2010 que dice: hay 2010 caballeros en la isla.
Determina el numero de espas que viven en la isla.
Problema 4. Sean a, b y c enteros positivos menores que 10 tales que
(100a + 10b + c)2 = (a + b + c)5 .
Determina el valor de a2 + b2 + c2 .
Problema 5. Sobre la base de un triangulo isosceles se traza una perpendicular desde
un punto cualquiera P . Esta recta corta a los lados iguales en dos puntos M y N .
Demuestra que P M + P N es constante y determina dicha constante.
Problema 6. Existen enteros de la forma 444 . . . 443 (todos sus dgitos 4 excepto el
u ltimo que es 3) tal que sean divisibles entre 13? En caso afirmativo encuentra uno
de estos numeros, en caso contrario demuestra que ningun numero de esta forma es
divisible entre 13.
Problema 7. Sean w, x, y, z numeros reales mayores o iguales que cero tales que w +
x + y + z = 100. Determina el mayor valor posible de la suma wx + xy + yz.
Problema 8. Sea N un numero de 21 dgitos, donde todos son iguales a 1 excepto el
dgito central. Si N es multiplo de 7, cual es el dgito central?
Problema 9. Desde un punto P , exterior a una circunferencia C, se traza la tangente
P T y la secante P AB, con A y B puntos de la circunferencia y estando A entre P y
B. Si T A es bisectriz del a ngulo BT P , BT = 15 cm y AT = 10 cm, cuanto mide
PT?
Problema 10. Sea T un conjunto formado por enteros positivos que tienen la siguiente
propiedad: si x y y son elementos distintos de T , con x > y, entonces x y tiene todos
sus dgitos en el conjunto {2, 3, 6, 9}. Determina la mayor cantidad de elementos que
puede tener T .
Problema 11. Cuantas parejas (x, y) de numeros enteros satisfacen la ecuacion
x2 y 3 = 612 ?

Problema 12. Demuestra que no existe ningun entero a tal que a2 3a 19 sea
divisible por 289.
Problema 13. Sea ABC un triangulo tal que AB = AC y sea I su incentro. Si BC =
AB + AI, determina la medida del a ngulo BAC.

Problemas de practica

13

Problema 14. Si se sabe que P (x) es un polinomio de grado 2008 tal que P (k) =
para k {1, 2, . . . , 2009}, calcula el valor de P (2010).

1
k

Problema 15. Determina el numero de enteros a, con 1 a 100, tales que aa es un


cuadrado perfecto.
Problema 16. Sean ABC un triangulo y P un punto en su interior. Definimos D, E y
F como los puntos medios de los segmentos AP , BP y CP respectivamente. Ademas
definimos R como la interseccion de AE y BD; S como la interseccion de BF y CE;
y T como la interseccion de CD y AF . Demuestra que la medida del a rea del hexagono
DRESF T es independiente de la eleccion de P .
Problema 17. Sean a, b y c enteros tales que b 6= c. Si
ax2 + bx + c y (c b)x2 + (c a)x + (a + b)
tienen una raz en comun, demuestra que a + b + 2c es multiplo de 3.
Problema 18. La bisectriz del a ngulo BAD de un paralelogramo (no rombo) ABCD
intersecta a las rectas CD y BC en los puntos K y L, respectivamente. Demuestra
que el centro O de la circunferencia que pasa por los puntos C, K y L esta sobre la
circunferencia que pasa por B, C y D.
Problema 19. Sea A un conjunto con 8 elementos. Determina el maximo numero de
subconjuntos de A, de 3 elementos cada uno, tales que la interseccion de cualesquiera
dos subconjuntos no contenga 2 elementos.
Problema 20. Para n 2, sean a1 , a2 , . . . , an numeros reales positivos tales que
(a1 + a2 + + an )

1
1
1
+
+ +
a1
a2
an

2

1
.
n+
2

Demuestra que max{a1 , a2 , . . . , an } 4 mn{a1 , a2 , . . . , an }.

14

Problemas de practica

Soluciones a los problemas de


practica

A continuacion encontraras las soluciones de los 20 problemas de practica de la seccion


anterior. Como siempre, las soluciones que presentamos no son u nicas y probablemente tampoco son las mejores, por lo que es muy posible que tu hayas encontrado una
solucion distinta pero igualmente valida. Si este es el caso y no estas muy seguro de
su validez o simplemente la quieres compartir con nosotros te invitamos para que nos
escribas a revistaomm@gmail.com.
Solucion del problema 1. Prolonguemos la recta AP hasta su interseccion con BC y
llamemos a la interseccion F .
A

M
P

Los triangulos ABP y BF P son isoceles ya que tienen un a ngulo recto y otro de 45 .
Por lo tanto, AP = BP = F P , es decir P es punto medio de AF . Utilizando el
teorema de Thales (ver en el apendice el teorema 6), tenemos que M P es paralela a
F C, es decir, EM es paralela a BC y como M es punto medio, E es punto medio de
AB. Por lo tanto,
BC = 2EM = 30 cm.

Solucion del problema 2. Sea p(x) = x4 2x3 7x2 2x + 1. Como a, b, c, d son

16

Soluciones a los problemas de practica

las races de p(x) tenemos que,


p(x) =
=

(x a)(x b)(x c)(x d)

x4 (a + b + c + d)x3 + (ab + ac + ad + bc + bd + cd)x2


(abc + abd + acd + bcd)x + abcd.

Entonces, abc + abd + acd + bcd = 2 y abcd = 1, y por lo tanto,


abc + abd + acd + bcd
2
1 1 1 1
+ + + =
= = 2.
a b
c d
abcd
1
Solucion del problema 3. Es claro que dos habitantes no pueden estar diciendo la
verdad al mismo tiempo ni el primer da ni el segundo da, ya que de otro modo se
tendran un numero diferente de plebeyos o de caballeros.
Ahora, supongamos que todos los habitantes mienten el primer da. Luego, en la isla
solo viven plebeyos y espas. Sean k el numero de plebeyos y s el numero de espas, es
decir, k + s = 2010. Si k > 0, entonces cuando el reportero le pregunta el primer da
a la k-esima persona, e sta responde que hay exactamente k plebeyos en la isla, luego
esta diciendo la verdad, lo cual es una contradiccion, por lo tanto k = 0. Entonces
s = 2010, es decir, todos en la isla son espas; pero el primer da todos mintieron,
luego el segundo da, todos diran la verdad, lo cual es una contradiccion pues no hay
caballeros en la isla.
De este modo, sabemos que solo hay una persona que dice la verdad el primer da y
digamos que es la n-esima persona a la que le pregunta el reportero. Si esta persona es
un caballero, entonces el segundo da tambien dira la verdad, es decir, viven n plebeyos
en la isla y viven n caballeros en la isla tambien. Pero entonces, todas las personas
restantes estan mintiendo ambos das, de donde hay 2009 plebeyos, y en consecuencia,
2009 caballeros, lo que es nuevamente una contradiccion.
Finalmente, podemos concluir que la u nica persona que dice la verdad el primer da
es un espa. Como todos los demas habitantes mienten el primer da, no pueden ser
caballeros, luego no hay caballeros en la isla, de donde todos mienten el segundo da,
asegurando que solo hay un espa en la isla.
Solucion del problema 4. Sea N = (100a + 10b + c)2 = (a + b + c)5 . Entonces, N
es un cuadrado perfecto y es la quinta potencia de un entero, lo que implica que N sea
la decima potencia de un entero. Ahora, como N es el cuadrado de un numero de tres
dgitos y
210 = (25 )2 = 322 , 310 = (35 )2 = 2432 y 410 = (45 )2 = 10242,
deducimos que N = 2432 . Por lo tanto, a = 2, b = 4 y c = 3 son los u nicos valores
posibles para a, b y c. Es facil verificar que 2432 = (2 + 4 + 3)5 . Por lo tanto, a2 +
b2 + c2 = 4 + 16 + 9 = 29.
Solucion del problema 5. Sea ABC un triangulo isosceles con AB = AC y supongamos que P esta mas cerca de C que de B. Observemos que si P M + P N es una

Soluciones a los problemas de practica

17

constante tiene que ser dos veces la altura desde A en el triangulo ABC, ya que P
puede ser el pie de dicha altura y en ese caso M = N .
N

A
M
b

Denotemos por (XY Z) al a rea del triangulo XY Z. Sea H el pie de la altura desde A
sobre el lado BC. Queremos demostrar que P M + P N = 2AH, lo cual equivale a
demostrar que
(M BC) + (N BC) = 2(ABC).
Tenemos que,
(M BC) + (N BC)

= [(ABC) (ABM )] + [(ABC) + (N AC)]


= 2(ABC) (ABM ) + (N AC)

Luego, basta demostrar que (ABM ) = (N AC). Pero,


(ABM ) (N AC)

=
=

(N BM ) (N AM ) (N AM ) (N M C)
NM
(P B 2P H P C),
2

ya que P B, P H, P C son las alturas de los triangulos N BM , N AM y N M C, respectivamente. Como ABC es un triangulo isoceles el punto H es el punto medio de BC,
luego P H + P C = HB = P B P H. Por lo tanto, (M BC) + (N BC) = 2(ABC)
y P M + P N = 2AH.
Solucion del problema 6. Supongamos que existe un entero de la forma
444
. . 4443} donde k 3,
| . {z
k

que es divisible entre 13. Ahora, es claro que si restamos 13 a cualquier multiplo
de 13, obtenemos un nuevo numero que tambien es divisible entre 13. De lo anterior se sigue que 444 . . . 4430 = 444 . . . 4443 13 es divisible entre 13. Como
444 . . . 4430 = 444 . . . 443 10 y como 10 no es divisible entre 13, concluimos que
444
. . 443}
| .{z
k1

18

Soluciones a los problemas de practica

debe ser divisible entre 13. Podemos repetir este argumento (k 2 veces) hasta concluir que 43 es divisible entre 13, lo cual es evidentemente falso. Por lo tanto la hipotesis con que inciamos es falsa quedando demostrado que ningun numero de la forma
444 . . . 4443 es divisible entre 13. En los casos donde el numero de dgitos 4 es 1 o 0
(numeros 43 o 3), el resultado es trivial.
Solucion del problema 7. Ya que (w+y)+(x+z) = 100, tenemos que w+y = 50+t
y x + z = 50 t para algun numero real t. Luego,
wx + xy + yz (w + y)(x + z) = (50 + t)(50 t) = 2500 t2 2, 500.

Finalmente, es facil ver que este valor maximo se obtiene, por ejemplo, cuando w =
x = 50 y y = z = 0. Por lo tanto, la respuesta es 2, 500.
Solucion del problema 8. Observemos que 1001 = 7(143) 0 (mod 7), luego
111111 = 1001(111) 0 (mod 7).
Sea N = |111{z
. . . 1} a 111
. . . 1}, donde a es el dgito central de N . Luego,
| {z
10

10

1 + 10 + 102 + + 109 + 1010 a + 1011 + + 1020


1010 (a 1) + |111{z
. . . 1}
21

(98 + 2)5 (a 1) + 1000(111


. . . 1}) + 111
| {z
18

25 (a 1) + 1000(111111)(1000001000001) + 111 (mod 7)

(28 + 4)(a 1) + 111 (mod 7)


4(a 1) + (105 + 6) 4a 4 + 6 4a + 2 (mod 7).

Como N es divisible por 7, entonces (ver en el apendice la definicion 5 y el artculo


de Tzaloa No. 2 de 2009) 4a + 2 0 (mod 7), es decir, 4a 12 (mod 7), de donde
a 3 (mod 7). Como a es un dgito, la u nica posibilidad es a = 3.
Solucion del problema 9. Como P T es tangente a la circunferencia, entonces P T A =
T BA (ver en el apendice el teorema 21). Luego, el triangulo T BA es isosceles con
AB = AT = 10 cm.
T
x

Soluciones a los problemas de practica

19

Por el teorema de la bisectriz (ver en el apendice el teorema 7), tenemos que,


PT
BT
15
3
=
=
= .
PA
BA
10
2
Sea P T = 3k, luego por la relacion anterior tenemos que P A = 2k. Aplicando la
potencia del punto P (ver en el apendice el teorema 20 y el artculo de Tzaloa No. 4 de
2009), tenemos que P T 2 = P A P B. Luego,
(3k)2 = (2k)(2k + 10),
de donde k = 4. Por lo tanto, P T = 12 cm.
Solucion del problema 10. Veamos que la maxima cantidad es 5.
Primero daremos un conjunto T con 5 elementos que cumplen la propiedad. Sea
T = {1, 4, 7, 10, 33}.
Las posibles diferencias son:
41=3
74=3
10 7 = 3
33 10 = 23
71=6

10 4 = 6
33 7 = 26
33 4 = 29
33 1 = 32
10 1 = 9

Observemos que todas las diferencias tienen sus dgitos en el conjunto {2, 3, 6, 9}.
Ahora demostraremos que si un conjunto T tiene mas de 5 elementos entonces no tiene
la propiedad mencionada.
Sea T un conjunto con mas de 5 elementos, entonces por el principio de las casillas
(ver en el apendice el teorema 1 y el artculo de Tzaloa No. 2 de 2010), habra dos de
ellos que dejan el mismo residuo al dividirlos entre 5. Denotemos por n y m a estos
numeros, y supongamos sin perdida de generalidad que m > n. Entonces (m n) es
multiplo de 5, luego el u ltimo dgito de (m n) es 0 o 5, lo cual no es posible pues
ninguno de estos dos numeros pertenecen al conjunto {2, 3, 6, 9}.
Solucion del problema 11. Es claro que x 6= 0 y y 6= 0. Observemos que si (x, y)
es una solucion, entonces (x, y) tambien lo es. Ademas, y es un entero positivo ya
que x2 lo es. Luego, la cantidad de parejas (x, y) de enteros que cumplen la ecuacion
x2 y 3 = 612 es igual al doble de la cantidad de parejas (x, y) de enteros positivos que
la cumplen.
Sea (x, y) una pareja de enteros positivos que cumplen x2 y 3 = 612 , entonces ambos
numeros x, y, solo pueden tener factores primos 2 y 3 en su descomposicion en primos
(ver en el apendice el teorema 3), pues ningun otro primo divide a 612 . Sean x = 2a 3b
y y = 2c 3d , entonces,
x2 y 3 = (2a 3b )2 (2c 3d )3 = 22a+3c 32b+3d = 212 312 = 612 .

20

Soluciones a los problemas de practica

Luego, tenemos que 2a + 3c = 12 y 2b + 3d = 12. Ahora, 2a + 3c = 12, con a y c


enteros no negativos implica que
(a, c) {(0, 4), (3, 2), (6, 0)}.
Analogamente, 2b + 3d = 12, con b y d enteros no negativos implica que
(b, d) {(0, 4), (3, 2), (6, 0)}.
Luego, como cada eleccion distinta de los numeros a, b, c y d genera una pareja distinta
(x, y), tenemos que hay 3 3 = 9 parejas de enteros positivos. Finalmente, por la
observacion que se hizo al inicio, hay exactamente 2 9 = 18 parejas (x, y) de enteros
que cumplen la ecuacion x2 y 3 = 612 .
Solucion del problema 12. Supongamos que a2 3a 19 es divisible entre 289 para
algun entero a. Ya que 289 = 172 , tenemos que 17 divide a
a2 3a 19 = (a 10)(a + 7) + 51.
Como 51 = 3(17), se sigue que 17 divide a (a10)(a+7). Ya que 17 es primo, resulta
que 17 divide a a 10 o a a + 7. Pero a 10 a + 7 (mod 17). Luego, 17 divide a
ambos factores y por lo tanto 172 = 289 divide a (a 10)(a + 7). Esto implica que
289 debe dividir a a2 3a 19 (a 10)(a + 7) = 51 lo cual no es posible. Por lo
tanto, no existe ningun entero a tal que a2 3a 19 sea divisible por 289.
Solucion del problema 13. Sea D un punto en la prolongacion del segmento AB tal
que AD = AI.
D

B
Entonces, BC = AB + AI = AB + AD = BD, de modo que el triangulo CBD es
isosceles. Ademas BI es la bisectriz del a ngulo DBC, luego I esta en la mediatriz de
CD y como BI = CI, concluimos que I es el circuncentro del triangulo CBD.
Luego, el triangulo DIB es isosceles y IDB = x = IBD. Como el triangulo

Soluciones a los problemas de practica

21

IAD es isosceles por construccion (AI = AD), tenemos que BAI = 2IDA = 2x
y BAC = 4x. Observemos que el triangulo CIB es congruente al triangulo DIB,
luego, ICB = x = IBC y ABC = 2x. Por lo tanto, en el triangulo isosceles
ABC con ABC = ACB, tenemos que
ABC = ACB = 2x =

1
BAC,
2

de donde 180 = 2ABC + BAC = 2BAC y de aqu BAC =

180
2

= 90 .

Solucion del problema 14. A partir de P definimos un nuevo polinomio Q, de forma


que Q(x) = xP (x)1. Este nuevo polinomio es de grado 2009 y Q(k) = k k1 1 = 0
para k = 1, 2, . . . , 2009. Por lo tanto,
Q(x) = C(x 1)(x 2) (x 2008)(x 2009)
donde C es una constante. Ahora, el valor de C puede calcularse facilmente, pues
Q(0) = 0 P (0) 1 = 1 y Q(0) = C(1)(2) (2009) = (C)(2009!). De
1
y entonces,
lo anterior se sigue que C = 2009!
Q(x) =

1
2009! (x

1)(x 2) (x 2009) = xP (x) 1.

Por u ltimo, tomando x = 2010 en la ecuacion anterior obtenemos que,


1=
de donde P (2010) =

2
2010

2009!
2009!

= 2010 P (2010) 1,

1
1005 .

Solucion del problema 15. Sabemos que en la factorizacion en primos (ver en el


apendice el teorema 3) de un cuadrado perfecto, cada factor primo debe aparecer un
numero par de veces. Ademas, tambien se cumple que si en la factorizacion en primos
de un numero, cada factor aparece un numero par de veces, entonces el numero es un
cuadrado perfecto. Procedamos haciendo un analisis por casos:
Caso 1. Suponemos que a es par. Entonces, a = 2c, de donde aa = (2c)2c =
(2c cc )2 . Luego, en este caso, aa siempre es un cuadrado perfecto, y por lo tanto
hay 50 de estos numeros.
Caso 2. Suponemos que a es impar. Entonces a = 2c + 1, de donde,
2

aa = (2c + 1)2c+1 = (2c + 1)2c (2c + 1) = ((2c + 1)c ) (2c + 1).


2

Ahora, como ((2c + 1)c ) es un cuadrado perfecto, entonces aa es un cuadrado


perfecto si y solo si cada primo de la factorizacion en primos de (2c + 1) ocurre
un numero par de veces, es decir, si y solo si (2c + 1) = a es en s mismo un
cuadrado perfecto. Como los u nicos cuadrados perfectos impares menores que
100 son: 1, 9, 25, 49 y 81, concluimos que hay 5 de estos numeros.

22

Soluciones a los problemas de practica

Juntando los casos 1 y 2, tenemos que a {2, 4, 6, . . . , 98, 100} {1, 9, 25, 49, 81},
por lo que hay 50 + 5 = 55 de estos numeros.
Solucion del problema 16. En lo que sigue denotaremos por (ABC) al a rea de un
triangulo ABC. Analogamente, usaremos (ABCD) para representar al a rea de un
cuadrilatero ABCD.
En primer lugar, consideremos al triangulo ABP . Notese que los segmentos AE y
BD son dos medianas de dicho triangulo, por lo que R es su baricentro. Si trazamos
desde P a R la tercera mediana del triangulo, es facil ver que el triangulo ABP queda
dividido en seis triangulos que tienen a reas iguales (ver en el apendice el teorema 17).
Por lo tanto (DREP ) = 26 (ABP ).
C

F
S

P
D

A traves de razonamientos analogos sobre los triangulos BCP y CAP , obtenemos que
(ESF P ) = 13 (BCP ) y (F T DP ) = 13 (CAP ). Por lo tanto,
(DRESF T ) =

1
3

[(ABP ) + (BCP ) + (CAP )] = 13 (ABC).

De donde es claro que el a rea del hexagono solo depende del a rea del triangulo y no
depende de la ubicacion del punto inerior P .
Solucion del problema 17. Sea r la raz comun de los dos polinomios. Entonces,
ar2 + br + c = 0 y (c b)r2 + (c a)r + (a + b) = 0.
Restando estas igualdades obtenemos (a + b c)r2 + (a + b c)r (a + b c) = 0.
Si a + b c = 0, entonces a + b + 2c = c + 2c = 3c es multiplo de 3.
Supongamos que a + b c 6= 0. Entonces, la igualdad
(a + b c)r2 + (a + b c)r (a + b c) = 0
es equivalente a la igualdad r2 + r 1 = 0, de donde r2 = 1 r. Luego,
0 = ar2 + br + c = a(1 r) + br + c = (b a)r + a + c,

Soluciones a los problemas de practica

23

y de aqu, r(a b) = a + c. Si a = b, entonces a + c = 0, es decir, c = a, y por


a+c
lo tanto a + b + 2c = a + a 2a = 0 es multiplo de 3. Si a 6= b, entonces r = ab
2
es un numeroracional. Por otro lado, las soluciones de la ecuacion r + r 1 = 0
5
son r = 1
, es decir, son irracionales, lo que es una contradiccion. Por lo tanto, el
2
caso a 6= b no puede suceder, y en los demas casos tenemos que a + b + 2c es multiplo
de 3.
Solucion del problema 18. Como el paralelogramo ABCD no es un rombo, tenemos
que AB 6= AD. Sin perdida de generalidad, supongamos, que AB > AD. En este caso
K esta entre C y D; y C esta entre B y L. Tomando en cuenta que AK es bisectriz de
BAD, que AB es paralela con DC y que AD es paralela con BC (ver en el apendice
la definicion 9), es facil ver que: BAK = DAK = DKA = CKL = CLK.
A

B
K

L
O

Por lo tanto, DK = DA = BC y BL = BA = DC. Sean O el centro y r el radio


de la circunferencia que pasa por C, K y L. Tomando la potencia del punto D con
respecto a esta circunferencia (ver en el apendice el teorema 20 y el artculo de Tzaloa
No. 4 de 2009) tenemos que
(DO r)(DO + r) = DO2 r2 = DK DC.
De forma analoga, tomando la potencia del punto B obtenemos
(BO r)(BO + r) = BO2 r2 = BC BL.
Por lo que concluimos que DO = BO. Ahora sabemos que los triangulos DKO y
BCO son congruentes por el criterio LLL (ver en el apendice el criterio 14), por lo que
OBC = ODK = ODC, quedando demostrado que OCBD es un cuadrilatero
cclico.
Solucion del problema 19. Si A = {a1 , a2 , a3 , a4 , a5 , a6 , a7 , a8 }, los siguientes subconjuntos de A satisfacen las condiciones del problema.
A1 = {a1 , a2 , a3 }, A2 = {a1 , a4 , a5 }, A3 = {a1 , a6 , a7 }, A4 = {a8 , a3 , a4 },
A5 = {a8 , a2 , a6 }, A6 = {a8 , a5 , a7 }, A7 = {a3 , a5 , a6 }, A8 = {a2 , a4 , a7 }.

24

Soluciones a los problemas de practica

Demostraremos que el maximo numero de subconjuntos de A que cumplen las condiciones del problema es 8. Supongamos, por contradiccion, que podemos hallar 9 subconjuntos. Como cada uno tiene 3 elementos, tenemos 27 elementos entre los 9 subconjuntos. Como A tiene 8 elementos, por el principio de las casillas (ver en el apendice
el teorema 1 y el artculo de Tzaloa No. 2 de 2010), hay un elemento x que aparece
en al menos 4 de los subconjuntos. En estos 4 subconjuntos tenemos 3(4) 4 = 8
elementos distintos de x, y como hay 7 elementos distintos de x, por el principio de las
casillas hay un elemento y que aparece en 2 subconjuntos que contienen a x. Esos dos
subconjuntos contienen a x y y en su interseccion, lo que contradice que la interseccion
de cualesquiera dos subconjuntos no tiene 2 elementos. Por lo tanto, la respuesta es 8.
Solucion del problema 20. Reordenando, si es necesario, podemos suponer que a1
a2 an . Debemos demostrar que a1 4an . Aplicando la desigualdad de
Cauchy-Schwarz (ver en el apendice el teorema 2), tenemos que
(an + a2 + a3 + + an1 + a1 )

1
1
1
+
+ +
a1
a2
an

an
+n2+
a1

a1
an

Luego,

2
1
n+
2
1
n+
2
5
2
17
4
0

r

r 2
an
a1
+ (n 2) +
a1
an
r
r
an
a1
+
n2+
a1
an
r
r
an
a1
+
a1
an
a1
an
+
a1
an

an 
.
(a1 4an ) a1
4

Ya que a1 an , tenemos que a1


a1 4an , como queramos.

an
4

> 0, de modo que a1 4an 0, es decir,

Problemas propuestos

Problemas propuestos.
2010 No. 3.
Ano
Tzaloa se construye con la contribucion de todos y esta seccion esta especialmente
disenada para que sus lectores tengan un espacio de participacion. A continuacion,
presentamos 5 problemas nuevos que necesitan de ti para encontrar su solucion.
Para dar tiempo a que nos puedas enviar tus soluciones, las respuestas de los problemas propuestos en cualquier numero de la revista, se publican con dos numeros de
diferencia. Es as, que en este numero (Tzaloa 3, ano 2010), aparecen las respuestas
de los problemas propuestos en Tzaloa 1, ano 2010; y las respuestas de los problemas
propuestos de este numero, se publicaran en Tzaloa 1, ano 2011, por lo que aun tienes
tiempo para enviarnos tu contribucion.
Ponemos a tu disposicion nuestra direccion electronica revistaomm@gmail.com
ya que a traves de ella estaremos recibiendo con gusto todas las soluciones que nos
lleguen desde cualquier rincon del pas.
Problema 1. (Intermedio) Considera la suma, la diferencia positiva, el producto y el
cociente mayor que 1 de dos enteros positivos distintos. Si al sumar los cuatro resultados obtienes 450, determina los dos numeros.
Problema 2. (Intermedio) En un tablero de 123 renglones y 123 columnas, cada casilla
es pintada de rojo o azul de acuerdo con las siguientes condiciones:
1. Cada casilla pintada de rojo que no este en el borde del tablero tiene exactamente
5 casillas azules entre sus 8 casillas vecinas.
2. Cada casilla pintada de azul que no este en el borde del tablero tiene exactamente
4 casillas rojas entre sus 8 casillas vecinas.
Determina el numero de casillas pintadas de rojo en el tablero.

26

Problemas propuestos

Problema 3. (Intermedio) Demuestra que si en un triangulo de a rea S el producto de


las longitudes de dos de sus medianas es igual a 32 S, entonces dichas medianas son
perpendiculares.
Problema 4. (Intermedio) Sea x un numero real que satisface la ecuacion x2 + x12 =
p2 2, donde p es un numero primo. Demuestra que para todo entero n, el valor de la
expresion xn + x1n es un numero entero y calcula su valor en funcion de p.
Problema 5. (Avanzado) Determina todos los enteros positivos a y b tales que
b2 a
a+b
sea un numero primo.

Soluciones a los problemas propuestos.


2010 No. 1.
Ano
Como se menciono al principio de esta seccion, a continuacion publicamos las soluciones de los problemas propuestos en Tzaloa 1, ano 2010. Recuerda que esta revista
necesita de ti y ten la seguridad que en el proximo numero nos encantara poder publicar tus soluciones. En esta ocasion nos da mucho gusto felicitar a Francisco Gomez
Hernandez, del estado de Hidalgo, quien nos envio su solucion del problema 5.
Problema 1. (Introductorio) Encuentra todas las parejas de numeros enteros (p, q) tales
que la diferencia entre las dos soluciones de la ecuacion x2 + px + q = 0 sea 2010.

p p2 4q
, entonces la diferencia entre las dos solucioSolucion. Tenemos que x =
2
nes es
p
p
p + p2 4q p p2 4q p 2

= p 4q = 2010.
2
2

Luego, queremos que 4q = p2 20102 = (p + 2010)(p 2010). Esto implica que


p tiene que ser un numero par, y ademas para cada numero par hay un valor de q que
satisface las condiciones del problema. Por lo tanto hay una infinidad de soluciones,
basta que p = 2k para cualquier entero k y q = (k + 1005)(k 1005).
Problema 2. (Introductorio) Si n es un entero divisible entre 7 que es igual al producto
de tres numeros consecutivos, cual (o cuales) de los enteros 6, 14, 21, 28 y 42 no es
necesariamente un divisor de n?
Solucion. Tenemos que n = (a 1)a(a + 1) = a(a2 1) y es tambien igual a 7k, para
algunos enteros a y k. Entonces, a(a2 1) = 7k. Observemos primero que alguno de
los numeros a o a2 1 es par, por lo que k es un numero par y n es siempre divisible
entre 14. Entonces k = 2l y a(a2 1) = 14l para algun entero l.

Problemas propuestos

27

Como a(a2 1) es el producto de tres numeros consecutivos, uno de ellos tiene que ser
divisible entre 3. Por lo tanto, l es divisible entre 3, es decir, l = 3m para algun entero
m. Lo que implica que n = 42m y que 6, 21 y 42 son siempre divisores de n.
Finalmente veamos que 28 no es necesariamente divisor de n. Por lo anterior, el menor
valor posible de n es cuando a + 1 = 7, entonces n = 5 6 7 = 210 que no es
divisible entre 28.
Problema 3. (Introductorio) Sea A1 A2 A3 A4 A5 A6 A7 A8 A9 un polgono regular de
nueve lados. Cuantos triangulos equilateros se pueden formar tales que al menos dos
de sus vertices esten en el conjunto {A1 , A2 , A3 , A4 , A5 , A6 , A7 , A8 , A9 }?
Solucion. Podemos suponer que los lados del polgono miden 1 cm, llamemos O al
centro del polgono. Antes de empezar a contar observemos que la distancia de un
vertice Ai a O es mayor que 1 cm ya que el a ngulo en O del triangulo A1 OA2 mide
360

9 = 40 .
Vamos a contar los triangulo equilateros por casos dependiendo de los dos vertices que
estan en el conjunto V = {A1 , A2 , A3 , A4 , A5 , A6 , A7 , A8 , A9 }.
Si Ai Ai+1 es una arista del triangulo equilatero (observemos que estamos contando la arista A9 A1 ya que contamos los subndices modulo 9, es decir, 9 + 1 =
10 = 1 modulo 9). Si construimos un triangulo equilatero con un lado igual
Ai Ai+1 su tercer vertice no pertenece al conjunto V . Como con cada arista podemos construir dos triangulo equilateros y hay 9 aristas, tenemos en este caso
18 triangulos equilateros.
Observemos que este caso y el caso Ai Ai+8 son el mismo.
Si Ai Ai+2 es una arista. La mediatriz del segmento Ai Ai+2 contiene al vertice
Ai+1 y no contiene ningun otro vertice del conjunto V . Por lo tanto, si construimos un triangulo equilatero con lado Ai Ai+2 el tercer vertice no esta en V .
Como en el caso precendente, en este caso podemos formar 29 = 18 triangulos
equilateros.
Observemos que este caso es el mismo que considerar Ai Ai+7 .
Si Ai Ai+3 es una arista. En la mediatriz de este segmento esta el vertice Ai+6
y el triangulo Ai Ai+3 Ai+6 es equilatero. Triangulos como este hay 3 (A1 A4 A7 ,
A2 A5 A8 y A3 A6 A9 ). Con cada una de estas 9 aristas podemos formar otro
triangulo equilatero, por lo que en este caso hay 3 + 9 = 12 triangulos.
Observemos que este caso es el mismo que considerar Ai Ai+6 .
Si Ai Ai+4 es una arista. Este caso es analogo a los primeros dos, por lo que hay
18 triangulos equilateros.
Observemos que este caso es el mismo que considerar Ai Ai+5 .
Por lo tanto, en total podemos formar 18 3 + 12 = 66 triangulos equilateros.
Problema 4. (Intermedio) Sea ABC un triangulo acutangulo e isosceles con AC =
AB. Sean O su circuncentro e I su incentro. Si D es el punto de interseccion de AC
con la perpendicular a CI que pasa por O, demuestra que ID y AB son paralelas.

28

Problemas propuestos

Solucion. Sea M el punto medio de BC. Como el triangulo ABC es isosceles, tenemos
que los puntos A, I, O y M son colineales y AM es perpendicular a BC. Sea Q el
punto de interseccion de OD con IC.
A

O
Q

Como OQC = OM C = 90 , el cuadrilatero OQM C es cclico, de modo que


QOM = QCM . Como IC es bisectriz del a ngulo ACB (ya que I es incentro),
tenemos que QOI = ICM = DCI. Luego,
180 = DOI + QOI = DOI + DCI,
y por lo tanto, el cuadrilatero DOIC es cclico. De aqu que DCO = DIO. Ahora,
como O es el circuncentro del triangulo ABC, tenemos que AO = OC y OAC =
DCO. Luego, como AI es bisectriz del a ngulo BAC, resulta que OAC = IAB
y por lo tanto IAB = AID. Es decir, ID y AB son paralelas.
Problema 5. (Avanzado) Sea A = {1, 2, 3, . . . , n}. A cada subconjunto B de A se
le asocia su suma alternada SB , definida como sigue: si B = {a1 , a2 , . . . , ak } con
a1 < a2 < < ak , entonces SB = ak ak1 + ak2 a1 . Por ejemplo, si
n = 10 y B = {2, 4, 5, 7, 8} entonces SB = 8 7 + 5 4 + 2 = 4.
Si n es un numero fijo, determina el valor de la suma
X
SB ,
BA

es decir, la suma de todos los numeros SB con B subconjunto de A.


Solucion de Francisco Gomez Hernandez. Sea A = {1, 2, . . . , n}. Supongamos que
X
f (n) =
SB .
BA

Vamos a demostrar que f (n) = n 2n1 .


Si n = 1, tenemos que A = {1}, y por lo tanto solo hay 2 subconjuntos y {1}, de
modo que f (1) = 1 = 1 211 .

Problemas propuestos

29

Si n > 1, tenemos dos casos, que el subconjunto B contenga a n o que no lo contenga.


Sabemos que hay 2n1 subconjuntos que tienen a n, y 2n1 subconjuntos que no lo
tienen.
Si consideramos los subconjuntos que no contienen a n, entonces solo vamos a calcular
la suma de las sumas alternadas de los subconjuntos del conjunto {1, 2, . . . , n1}. Pero
esta suma es igual a f (n 1).
Si ahora consideramos los subconjuntos que contienen a n, podemos suponer que son
de la forma B = {a1 , a2 , . . . , ak , n} con a1 < a2 < < ak < n, de modo que SB =
n (ak a2 a1 ). Pero al considerar los subconjuntos B = {a1 , a2 , . . . , ak },
entonces la suma de las sumas alternadas de los subconjuntos de B sera igual a la
suma de las sumas alternadas de los subconjuntos del conjunto {1, 2, . . . , n 1} que
es igual a f (n 1), y como hay 2n1 subconjuntos de estos, n se repite esta cantidad
de veces. Por lo tanto, la suma de estas sumas alternadas es igual a n 2n1 f (n 1).
De todo lo anterior, concluimos que f (n) = f (n1)+(n2n1 f (n1)) = n2n1 ,
como queramos demostrar.
Segunda solucion. Para cada k = 1, 2, . . . , n, sean Ak = {1, 2, . . . , k} y
X
SB .
Sk =
BAk

Observemos que
Sk+1 =

SB =

BAk+1

SC +

CAk+1
k+16C

SD .

DAk+1
k+1D

Como k + 1 6 C, tenemos que C Ak . Luego,


X
X
SC =
SB = Sk .
CAk+1
k+16C

Por otra parte,

SD =

DAk+1
k+1D

BAk

BAk

[(k + 1) SB ]

ya que si D = {a1 , a2 , . . . , ak , k + 1}, entonces SD = (k + 1) ak + ak1


a1 , y como {a1 , a2 , . . . , ak } Ak se sigue que SD = (k + 1) SB con
B = {a1 , a2 , . . . , ak }.
Luego,
X
X
X
[(k + 1) SB ] =
(k + 1)
SD = (k + 1)2k Sk ,
BAk

BAk

BAk

ya que el numero de subconjuntos de Ak es 2k .


Por lo tanto,
X
Sk+1 =
SB = Sk + (k + 1)2k Sk = (k + 1)2k
BAk+1

para todo k = 1, 2, . . . , n 1.
En particular, Sn = n 2n1 .

30

Problemas propuestos

Olimpiadas Internacionales

A continuacion presentamos los resultados y los examenes de las distintas olimpiadas


internacionales en las que Mexico participo en la primera mitad de este ano 2010.

XXII Olimpiada de la Cuenca del Pacfico


Desde 1991, los ganadores del Concurso Nacional participan anualmente en la Olimpiada Matematica de la Cuenca del Pacfico, APMO, por sus siglas en ingles. En Mexico, el 8 de marzo de este ano, se aplico el examen de la XXII Olimpiada Matematica
de la Cuenca del Pacfico a los alumnos que en ese momento formaban parte de la
preseleccion nacional. Dicho examen se califico en Mexico y los 10 mejores examenes se enviaron, para ser evaluados, al pas organizador que en esta ocasion fue Japon.
Los alumnos que obtuvieron medalla fueron: Daniel Perales Anaya (Morelos) y Flavio
Hernandez Gonzalez (Aguascalientes) obtuvieron medalla de plata; Fernando Josafath
Anorve Lopez (Nuevo Leon), Jose Luis Miranda Olvera (Jalisco), Irving Daniel Calderon Camacho (Estado de Mexico), Jose Ramon Guardiola Espinosa (San Luis Potos) y Julio Cesar Daz Calderon (Oaxaca), obtuvieron medalla de bronce; Fernando
Ignacio Arreola Gutierrez (Aguascalientes), obtuvo una mencion honorfica. Mexico
ocupo el lugar numero 15 de los 33 pases participantes.
A continuacion presentamos el examen de la XXII Olimpiada de la Cuenca del Pacfico. Los alumnos tuvieron 4 horas para resolverlo.
Problema 1. Sea ABC un triangulo con BAC 6= 90 . Sea O el circuncentro del
triangulo ABC y sea el circuncrculo del triangulo BOC. Suponga que intersecta
a los segmentos AB y AC en los puntos P (diferente de B) y Q (diferente de C), respectivamente. Sea ON el diametro del crculo . Muestra que el cuadrilatero AP N Q
es un paralelogramo.
Problema 2. Dado un entero positivo k, decimos que un entero es una potencia ke sima pura si puede ser representado como mk para algun entero m. Muestra que para

32

Olimpiadas internacionales

cada entero positivo n existen n enteros positivos distintos tales que su suma es una
potencia 2009-esima pura, y su producto es una potencia 2010-esima pura.

Problema 3. Sea n un entero positivo. En cierta fiesta asisten n personas. Para cualquier par de participantes, o los dos se conocen entre ellos o los dos no se conocen entre
ellos. Encuentra el maximo numero posible de parejas tal que en cada pareja, las dos
personas no se conocen entre s pero existe un amigo en comun entre los participantes
de la fiesta.

Problema 4. Sea ABC un triangulo acutangulo que satisface que AB > BC y que
AC > BC. Sean O y H el circuncentro y el ortocentro del triangulo ABC, respectivamente. Suponga que el circuncrculo del triangulo AHC intersecta a la recta AB
en M (diferente de A), y suponga que el circuncrculo del triangulo AHB intersecta a
la recta AC en N (diferente de A). Muestra que el circuncrculo del triangulo M N H
esta sobre la recta OH.

Problema 5. Encuentra todas las funciones f del conjunto R de los numeros reales a
R que satisfacen para x, y, z R la identidad
f (f (x) + f (y) + f (z)) = f (f (x) f (y)) + f (2xy + f (z)) + 2f (xz yz).

American Mathematics Competition (AMC)


En el mes de marzo se pidio al comite de la olimpiada de Estados Unidos, el examen
de la primera fase que aplican a nivel nacional. Dicho examen consta de dos niveles,
el AMC 10 y el AMC 12, y en cada nivel los concursantes tienen 75 minutos para
resolverlo. Los estudiantes mexicanos que en ese momento eran parte de la preseleccion nacional para la Olimpiada Centroamericana y del Caribe, presentaron el examen
AMC 10, y los estudiantes que en ese momento eran parte de la preseleccion nacional para las Olimpiadas Iberoamericana e Internacional, presentaron el examen AMC
12. Los ganadores del equipo mexicano fueron: Diego Alonso Roque Montoya (Nuevo
Leon) quien obtuvo el primer lugar con 109.5 puntos en el AMC 10; Flavio Hernandez
Gonzalez (Aguascalientes), quien obtuvo segundo lugar con 105 puntos en el AMC 12;
Juan Carlos Ortiz Rhoton (Jalisco), quien obtuvo el tercer lugar con 103.5 puntos en el
AMC 12. Diego y Juan Carlos obtuvieron un reconocimiento especial por ser menores
de 15 anos y obtener mas de 90 puntos en el examen.
A continuacion presentamos los examenes del concurso AMC (American Mathematics
Competition) de este ano.

Olimpiadas internacionales

33

AMC 10A
Problema 1. La parte superior de la biblioteca de Mara tiene cinco libros con los siguientes anchos, en centmetros: 6, 21 , 1, 2.5 y 10. Cual es el ancho promedio de los
libros, en centmetros?
(a) 1

(b) 2

(c) 3

(d) 4

(e) 5

Ancho

Problema 2. Cuatro cuadrados identicos y un rectangulo son organizados para formar


un cuadrado mas grande como se muestra. Cual es la proporcion entre el largo y el
ancho del rectangulo?

Largo

(a)

5
4

(b)

4
3

3
2

(c)

(d) 2

(e) 3

Problema 3. Tyrone tena 97 canicas y Eric tena 11 canicas. Tyrone le dio algunas de
sus canicas a Eric de tal manera que Tyrone termino con el doble de canicas que Eric.
Cuantas canicas le dio Tyrone a Eric?
(a) 3

(b) 13

(c) 18

(d) 25

(e) 29

Problema 4. La lectura de un libro que se va a grabar en discos compactos dura 412


minutos. Cada disco puede tener hasta 56 minutos de lectura. Asuma que se usan el
menor numero posible de discos y que cada disco contiene la misma cantidad de lectura. Cuantos minutos de lectura contendra cada disco?
(a) 50.2

(b) 51.5

(c) 52.4

(d) 53.8

(e) 55.2

Problema 5. La longitud de una circunferencia es 24 y su a rea es k. Cual es el


valor de k?
(a) 6

(b) 12

(c) 24

(d) 36

(e) 144

Problema 6. Se define la operacion (x, y) para numeros positivos x y y como


1
(x, y) = x .
y
Cual es el resultado de (2, (2, 2))?
(a)

2
3

(b) 1

(c)

4
3

(d)

5
3

(e) 2

34

Olimpiadas internacionales

Problema 7. Crystal trota diariamente el mismo recorrido. Ella comienza su trote dirigiendose hacia el norte una milla. Luego trota al noreste por una milla, luego al sureste
por una milla. El u ltimo tramo de su trote la lleva en lnea recta de regreso a donde ella
comenzo. Cuan larga, en millas, es esta u ltima porcion de su trote?
(a) 1

(b)

(c)

(e) 2 2

(d) 2

Problema 8. Tony trabaja 2 horas al da y se le paga $0.50 por hora por cada ano completo de su edad. Durante un periodo de seis meses Tony trabajo 50 das y gano $630.
Que edad tena Tony al final del periodo de seis meses?
(a) 9

(b) 11

(c) 12

(d) 13

(e) 14

Problema 9. Un palndrome, tal como 83438, es un numero que permanece igual cuando sus dgitos son puestos en orden inverso. Los numeros x y x + 32 son palndromes
de tres y cuatro dgitos, respectivamente. Cual es la suma de los dgitos de x?
(a) 20

(b) 21

(c) 22

(d) 23

(e) 24

Problema 10. Marvin cumplio anos el martes 27 de mayo en el ano bisiesto 2008. En
que ano sera la proxima vez que caiga su cumpleanos en un da sabado?
(a) 2011

(b) 2010

(c) 2013

(d) 2015

(e) 2017

Problema 11. La longitud del intervalo de soluciones de la desigualdad a 2x+3 b


es 10. Cual es el valor de b a?
(a) 6

(b) 10

(c) 15

(d) 20

(e) 30

Problema 12. Logan esta construyendo un modelo a escala de su pueblo. La torre del
tanque de agua tiene 40 metros de alto, y la parte superior es una esfera que contiene
100, 000 litros de agua. La torre miniatura de Logan contiene 0.1 litros. Cuan alta, en
metros, debera hacer Logan su torre?
(a) 0.04

(b)

0.4

(c) 0.4

(d)

(e) 4

Problema 13. Angelina manejo con una velocidad promedio de 80 kph y luego hizo
una parada de 20 minutos por gasolina. Despues de la parada, ella manejo con una
velocidad promedio de 100 kph. En total ella recorrio 250 km con un tiempo total de
viaje de 3 horas incluyendo la parada. Que ecuacion podra ser usada para hallar el
tiempo t en horas en el que ella manejo antes de su parada?

(a) 80t + 100 38 t = 250
(e) 80 83 t + 100t = 250

(b) 80t = 250

(c) 100t = 250

(d) 90t = 250

Problema 14. En el triangulo ABC se tiene que AB = 2 AC. Sean D y E puntos


sobre AB y BC, respectivamente tales que BAE = ACD. Sea F la interseccion

Olimpiadas internacionales

35

de los segmentos AE y CD, y suponga que el triangulo CF E es equilatero. Cuanto


mide el a ngulo ACB?
(a) 60

(b)75

(c) 90

(d) 105

(e) 120

Problema 15. En un pantano magico hay dos especies de anfibios parlantes: sapos,
que siempre dicen la verdad, y ranas, quienes siempre mienten. Cuatro anfibios, Brian,
Chris, LeRoy y Mike viven juntos en este pantano y cada uno dice lo siguiente:
Brian:
Chris:
LeRoy:
Mike:

Mike y yo somos de especies diferentes.


LeRoy es una rana.
Chris es una rana.
De los cuatro de nosotros, al menos dos son sapos.

Cuantos de los anfibios son ranas?


(a) 0

(b) 1

(c) 2

(d) 3

(e) 4

Problema 16. Las longitudes de los lados del triangulo ABC son numeros enteros y
ninguno de sus a ngulos mide 0 . Sea D un punto sobre el lado AC tal que BD es
bisectriz del a ngulo ABC, AD = 3, y DC = 8. Cual es el menor valor que el
permetro de dicho triangulo puede tener?
(a) 30

(b) 33

(c) 35

(d) 36

(e) 37

Problema 17. Las aristas de un cubo solido tienen 3 pulgadas de longitud. Se hace
un agujero cuadrado de 2 pulgadas por 2 pulgadas en el centro de cada cara del cubo.
Las aristas de cada corte son paralelas a las aristas del cubo, y cada agujero atraviesa
totalmente el cubo. Cual es el volumen del solido resultante?
(a) 7

(b) 8

(c) 10

(d) 12

(e) 15

Problema 18. Bernardo elige aleatoriamente 3 numeros distintos del conjunto


{1, 2, 3, 4, 5, 6, 7, 8, 9}
y los ordena en orden descendente para formar un numero de 3 dgitos. Silvia elige
aleatoriamente 3 numeros distintos del conjunto
{1, 2, 3, 4, 5, 6, 7, 8}
y tambien los ordena en orden descendente para formar un numero de 3 dgitos. Cual
es la probabilidad que el numero de Bernardo sea mayor que el numero de Silvia?
(a)

47
12

(b)

37
56

(c)

2
3

(d)

49
72

(e)

39
56

Problema 19. Los lados del hexagono equiangular ABCDEF tienen losgitudes AB =
CD = EF = 1 y BC = DE = F A = r. El a rea del triangulo ACE es el 70 % del

36

Olimpiadas internacionales

a rea del hexagono. Cual es la suma de todos los valores que puede tener r?
(a)

4 3
3

(b)

10
3

(c) 4

(d)

17
4

(e) 6

Problema 20. Una mosca atrapada dentro de una caja cubica con arista de longitud 1
metro decide aliviar su aburrimiento visitando cada esquina de la caja. Comenzara y
terminara en la misma esquina y visitara cada una de las otras esquinas exactamente
una vez. Para ir de una esquina a cualquier otra esquina, lo hara volando o caminando
en el interior del cubo siempre en lnea recta. Cual es la longitud maxima posible, en
metros, de su recorrido?

(a) 4 + 4 2 (b) 2 + 4 2 + 2 3 (c) 2 + 3 2 + 3 3 (d) 4 2 + 4 3(e) 3 2 + 5 3


Problema 21. El polinomio x3 ax2 + bx 2010 tiene tres races enteras positivas.
Cual es el menor valor que a puede tener?
(a) 78

(b) 88

(c) 98

(d) 108

(e) 118

Problema 22. Se eligen ocho puntos en una circunferencia y se trazan cuerdas conectando cada par de puntos. No hay tres cuerdas que se intersecten en un mismo punto en
el interior de la circunferencia. Cuantos triangulos con todos sus vertices en el interior
de la circunferencia son formados?
(a) 28

(b) 56

(c) 70

(d) 84

(e) 140

Problema 23. Cada una de 2010 cajas alineadas contiene una sola canica roja, y para
1 k 2010, la caja en la posicion k-esima contiene tambien k canicas blancas.
Isabella comienza en la primera caja y extrae sucesivamente en orden una sola canica
aleatoriamente de cada caja. Ella se detiene cuando extrae por primera vez una canica
roja. Sea P (n) la probabilidad de que Isabella se detenga despues de extraer exacta1
?
mente n canicas. Cual es el menor valor de n para el cual P (n) < 2010
(a) 45

(b) 63

(c) 64

(d) 201

(e) 1005

Problema 24. Sea n el numero formado por los dos u ltimos dgitos diferentes de cero
de 90!. A que es igual n?
(a) 12

(b) 32

(c) 48

(d) 52

(e) 68

Problema 25. Jim comienza con un entero positivo n y crea una sucesion de numeros.
Cada termino sucesivo es obtenido sustrayendo el mayor numero entero cuadrado perfecto que es menor o igual que el termino anterior, hasta que obtenga cero. Por ejemplo,
si Jim comienza con n = 55, entonces su sucesion contiene 5 numeros:
55, 55 72 = 6, 6 22 = 2, 2 12 = 1, 1 12 = 0.

Olimpiadas internacionales

37

Sea N el menor numero entero para el cual la sucesion de Jim tiene 8 numeros. Cual
es el dgito en las unidades de N ?
(a) 1

(b) 3

(c) 5

(d) 7

(e) 9

AMC 12A
Problema 1. Cual es el valor de (20 (2010 201)) + (2010 (201 20))?
(a) 4020

(b) 0

(c) 40

(d) 401

(e) 4020

Problema 2. Un transbordador lleva turistas a una isla cada hora comenzando a las 10
AM hasta su u ltimo viaje, que comienza a las 3 PM. Un da el capitan de la embarcacion nota que en el viaje de las 10 AM haba 100 turistas en el transbordador, y que
en cada viaje sucesivo, el numero de turistas fue uno menor que en el viaje anterior.
Cuantos turistas llevo el transbordador a la isla ese da?
(a) 585

(b) 594

(c) 672

(d) 679

(e) 694

Problema 3. El rectangulo ABCD, mostrado a continuacion, comparte el 50 % de su


a rea con el cuadrado EF GH. El cuadrado EF GH comparte el 20 % de su a rea con el
AB
?
rectangulo ABCD. Cual es el valor de AD
E

(a) 4

(b) 5

(c) 6

(d) 8

(e) 10

Problema 4. Si x < 0, cual de las siguientes opciones debe ser positiva?


(a)

x
|x|

(b) x2

(c) 2x

(d) x1

(e)

Problema 5. Despues de cumplirse la mitad de un torneo de arquera de 100 tiros,


Chelsea lo va liderando por 50 puntos. Para cada tiro en el centro del blanco se obtienen 10 puntos, con otros puntajes posibles de 8, 4, 2 y 0. Chelsea obtiene al menos 4
puntos en cada tiro. Si en los siguientes n tiros Chelsea da en el centro del blanco, ella
garantizara su victoria. Cual es el menor valor que puede tener n?
(a) 38

(b) 40

(c) 42

(d) 44

(e) 46

Problema 6. Un palndrome, tal como 83438, es un numero que permanece igual cuando sus dgitos son puestos en orden inverso. Los numeros x y x + 32 son palndromes

38

Olimpiadas internacionales

de tres y cuatro dgitos, respectivamente. Cual es la suma de los dgitos de x?


(a) 20

(b) 21

(c) 22

(d) 23

(e) 24

Problema 7. Logan esta construyendo un modelo a escala de su pueblo. La torre del


tanque de agua tiene 40 metros de alto, y la parte superior es una esfera que contiene
100, 000 litros de agua. La torre miniatura de Logan contiene 0.1 litros. Cuan alta, en
metros, debera hacer Logan su torre?
(a) 0.04

(b)

0.4

(c) 0.4

(d)

(e) 4

Problema 8. En el triangulo ABC se tiene que AB = 2 AC. Sean D y E puntos


sobre AB y BC, respectivamente, tales que BAE = ACD. Sea F la interseccion
de los segmentos AE y CD, y suponga que el triangulo CF E es equilatero. Cuanto
mide el a ngulo ACB?
(a) 60

(b) 75

(c) 90

(d) 105

(e) 120

Problema 9. Las aristas de un cubo solido tienen 3 pulgadas de longitud. Se hace un


agujero cuadrado de 2 pulgadas por 2 pulgadas en el centro de cada cara del cubo. Las
aristas de cada corte son paralelas a las aristas del cubo, y cada agujero atraviesa totalmente el cubo. Cual es el volumen del solido resultante?
(a) 7

(b) 8

(c) 10

(d) 12

(e) 15

Problema 10. Los primeros cuatro terminos de una sucesion aritmetica son p, 9, 3p q
y 3p + q. Cual es el termino 2010-esimo de esta sucesion?
(a) 8041

(b) 8043

(c) 8045

(d) 8047

(e) 8049

Problema 11. La solucion de la ecuacion 7x+7 = 8x puede ser expresada en la forma


x = logb 77 . A que es igual b?
(a)

7
15

(b)

7
8

(c)

8
7

(d)

15
8

(e)

15
7

Problema 12. En un pantano magico hay dos especies de anfibios parlantes: sapos,
que siempre dicen la verdad, y ranas, quienes siempre mienten. Cuatro anfibios, Brian,
Chris, LeRoy y Mike viven juntos en este pantano y cada uno dice lo siguiente:
Brian:
Chris:
LeRoy:
Mike:

Mike y yo somos de especies diferentes.


LeRoy es una rana.
Chris es una rana.
De los cuatro de nosotros, al menos dos son sapos.

Cuantos de estos anfibios son ranas?


(a) 0

(b) 1

(c) 2

(d) 3

(e) 4

Olimpiadas internacionales

39

Problema 13. Para cuantos valores enteros de k resulta que los graficos de x2 + y 2 =
k 2 y xy = k no se intersectan?
(a) 0

(b) 1

(c) 2

(d) 4

(e) 8

Problema 14. Las longitudes de los lados del triangulo ABC son numeros enteros y
ninguno de sus a ngulos mide 0 . Sea D un punto sobre el lado AC tal que BD es
bisectriz del a ngulo ABC, AD = 3, y DC = 8. Cual es el menor valor que el
permetro de dicho triangulo puede tener?
(a) 30

(b) 33

(c) 35

(d) 36

(e) 37

Problema 15. Se altera una moneda de tal manera que la probabilidad de que caiga en
cara es menor que 12 y cuando se arroja la moneda cuatro veces, la probabilidad de que
se obtenga un numero igual de caras y sellos es 61 . Cual es la probabilidad de que la
moneda caiga en cara?

6 6 6+2
153
(b)
(c) 21
(d) 36 3
(e) 31
(a) 6
12
2
2
Problema 16. Bernardo elige aleatoriamente 3 numeros distintos del conjunto
{1, 2, 3, 4, 5, 6, 7, 8, 9}
y los ordena en orden descendente para formar un numero de 3 dgitos. Silvia elige
aleatoriamente 3 numeros distintos del conjunto
{1, 2, 3, 4, 5, 6, 7, 8}
y tambien los ordena en orden descendente para formar un numero de 3 dgitos. Cual
es la probabilidad que el numero de Bernardo sea mayor que el numero de Silvia?
(a)

47
72

(b)

37
56

(c)

2
3

(d)

49
72

(e)

39
56

Problema 17. Los lados del hexagono equiangular ABCDEF tienen longitudes AB =
CD = EF = 1 y BC = DE = F A = r. El a rea del triangulo ACE es el 70 % del
a rea del hexagono. Cual es la suma de todos los valores que r puede tener?
(a)

4 3
3

(b)

10
3

(c) 4

(d)

17
4

(e) 6

Problema 18. Un camino que consta de 16 pasos va de (4, 4) a (4, 4) de tal forma
que con cada paso se incrementa en 1 o bien la coordenada en x o bien la coordenada
en y. Cuantos de estos caminos son tales que permanecen en el exterior o en el borde
del cuadrado 2 x 2, 2 y 2 en todos los pasos?
(a) 92

(b) 144

(c) 1, 568

(d) 1, 698

(e) 12, 800

Problema 19. Cada una de 2010 cajas alineadas contiene una sola canica roja, y para
1 k 2010, la caja en la posicion k-esima contiene tambien k canicas blancas.

40

Olimpiadas internacionales

Isabella comienza en la primera caja y extrae sucesivamente en orden una sola canica
aleatoriamente de cada caja. Ella se detiene cuando ella extrae por primera vez una
canica roja. Sea P (n) la probabilidad de que Isabella se detenga despues de extraer
1
?
exactamente n canicas. Cual es el menor valor de n para el cual P (n) < 2010
(a) 45

(b) 63

(c) 64

(d) 201

(e) 1005

Problema 20. Las progresiones aritmeticas (an ) y (bn ) tienen terminos enteros con
a1 = b1 = 1 < a2 b2 y an bn = 2010 para algun n. Cual es el mayor valor que n
puede tener?
(a) 2

(b) 3

(c) 8

(d) 288

(e) 2009

Problema 21. El grafico de y = x6 10x5 + 29x4 4x3 + ax2 yace en la parte del
plano por encima de la lnea recta y = bx + c excepto en tres valores de x, donde el
grafico y la lnea recta se intersectan. Cual es el mayor de estos tres valores?
(a) 4

(b) 5

(c) 6

(d) 7

(e) 8

Problema 22. Cual es el valor mnimo de f (x) = |x 1| + |2x 1| + |3x 1| +


+ |199x 1|?
(a) 49

(b) 50

(c) 51

(d) 52

(e) 53

Problema 23. Sea n el numero formado por los dos u ltimos dgitos diferentes de cero
de 90!. A que es igual n?
(a) 12

(b) 32

(c) 48

(d) 52

(e) 68

Problema 24. Sea f (x) = log10 (sen(x) sen(2x) sen(3x) sen(8x)). La


interseccion del dominio de f (x) con el intervalo [0, 1] es la union de n intervalos
abiertos disjuntos. Cual es el valor de n?
(a) 2

(b) 12

(c) 18

(d) 22

(e) 36

Problema 25. Se considera que dos cuadrilateros son iguales si uno puede ser obtenido
del otro por una rotacion y/o una traslacion. Cuantos cuadrilateros convexos cclicos
diferentes hay con lados enteros y permetro igual a 32?
(a) 560

(b) 564

(c) 568

(d) 1498

(e) 2255

Olimpiadas internacionales

41

XII Olimpiada Centroamericana y del Caribe


Del 24 de mayo al 1 de junio de 2010, se celebro en Mayaguez, Puerto Rico, la XII
Olimpiada Matematica de Centroamerica y el Caribe. Mexico ocupo el primer lugar,
con 108 puntos, de entre los 14 pases que participaron. Barbados y Cuba no pudieron
asistir, y Jamaica, Trinidad y Tobago e Islas Vrgenes Americanas participaron por primera vez. En total participaron 41 estudiantes, ya que Trinidad y Tobago solo llevo 2
alumnos. La delegacion mexicana estuvo integrada por los alumnos: Diego Alonso Roque Montoya (Nuevo Leon), Julio Cesar Daz Calderon (Oaxaca) y Fernando Josafath
Anorve Lopez (Nuevo Leon).
De las 3 medallas de oro, 7 de plata y 11 de bronce que se entregaron en el certamen,
Diego obtuvo medalla de oro, y Julio Cesar y Fernando medalla de plata. Ademas
se otorgo el reconocimiento de Solucion Creativaa Diego Alonso por su solucion
del problema 5, reconocimiento que solo se ha otorgado 2 veces, hasta ahora, en una
Olimpiada Matematica de Centroamerica y el Caribe.
A continuacion presentamos el examen de la XII Olimpiada Centroamericana y del
Caribe. Los alumnos tuvieron dos sesiones de cuatro horas y media cada una para
resolverlo.
Problema 1. Si S(n) denota la suma de los dgitos de un numero natural n, encuentre
todas las soluciones de
n(S(n) 1) = 2010
mostrando que son las u nicas.
Problema 2. Dado el ABC, sean L, M y N los puntos medios de BC, CA y AB,
respectivamente. Se traza una tangente al circuncrculo del ABC en A, siendo P y Q
las intersecciones respectivas de las rectas LM y LN con dicha tangente. Demuestre
que CP es paralela a BQ.
Problema 3. Un jugador coloca una ficha en una casilla de un tablero de mn, dividido
en casillas de tamano 11. El jugador mueve la ficha de acuerdo a las siguientes reglas:
En cada movimiento, el jugador cambia la ficha de la casilla en la que e sta se
encuentra a una de las casillas que tienen un lado en comun con ella.
El jugador no puede ubicar la ficha en una casilla que e sta ha ocupado previamente.
Dos movimientos consecutivos no pueden tener la misma direccion.
El juego termina cuando el jugador no puede mover la ficha. Determine todos los valores de m y n para los cuales el jugador puede colocar la ficha en alguna casilla tal que
e sta haya ocupado todas las casillas al terminar el juego.
Problema 4. Se desea embaldosar un patio cuadrado de lado N entero positivo. Se dispone de dos tipos de baldosas: cuadradas de 5 5 y rectangulares de 1 3. Determine

42

Olimpiadas internacionales

los valores de N para los cuales es posible hacerlo.


Nota: El patio debe quedar completamente cubierto sin que las baldosas se sobrepongan.
Problema 5. Sean p, q y r numeros racionales distintos de cero tales que
p
p
p
3
pq 2 + 3 qr2 + 3 rp2
es un numero racional distintos de cero. Pruebe que

1
1
1
p
+ p
+ p
3
3
3
2
2
pq
qr
rp2

tambien es un numero racional.

Problema 6. Sean y 1 dos circunferencias tangentes internamente en A, de centros


O y O1 , y radios r y r1 (r > r1 ), respectivamente. Sea B el punto diametralmente
opuesto a A en la circunferencia , y C un punto en tal que BC es tangente a 1 en
P . Sea A el punto medio de BC. Si se cumple que O1 A es paralela a AP , determine
la razon rr1 .

Informacion Olmpica
A continuacion presentamos las actividades programadas por el comite organizador de
la Olimpiada Mexicana de Matematicas, de julio a octubre de 2010.
Del 2 al 15 de julio, Astana, Kasajstan
51a Olimpiada Internacional de Matematicas.
Del 19 al 29 de agosto, Cuernavaca
Entrenamientos para los seleccionados nacionales y aplicacion de tres examenes
selectivos para determinar la delegacion para la XXV Olimpiada Iberoamericana
(un maximo de 4 alumnos).
Primera semana de septiembre
Lmite para registro de delegados que quieran aplicar el examen propuesto por el
Comite Organizador de la OMM como final de su Concurso Estatal y envo del
examen a los delegados.
Septiembre, Paraguay
XXV Olimpiada Iberoamericana de Matematicas.
24 y 25 de septiembre
Aplicacion de los examenes finales en los estados registrados con este proposito.
Primera quincena de octubre
Envo del cuarto numero de la revista Tzaloa.

44

Informacion Olmpica

Apendice
Teorema 1 (Principio de las Casillas) Dados al menos nk + 1 objetos acomodados
en n lugares, siempre hay un lugar con al menos k + 1 objetos.
Ver [5, 11].
Teorema 2 (Desigualdad de Cauchy-Schwarz) Si x1 , . . . , xn , y1 , . . . , yn son numeros reales, entonces,
!
! n
!2
n
n
X
X
X
2
2
yi .
xi

xi yi
i=1

i=1

i=1

La igualdad ocurre si y solo si existe un numero real c tal que xi = cyi para i =
1, 2, . . . , n.
Ver [3].
Teorema 3 (Factorizacion en primos) Todo entero n mayor que 1 puede expresarse
como un producto de primos (con, tal vez, solamente un factor).
Ver [6, 8].
Teorema 4 (Numero

de divisores) Si la factorizacion en primos del entero n es n =


2
r
1
p
umero de di1 p2 pr donde p1 , p2 , . . . , pr son primos distintos, entonces el n
visores positivos de n es igual a (1 + 1)(2 + 1) (r + 1).
Ver [6, 8].
Definicion 5 (Congruencias) Dados dos numeros enteros a, b, y un entero positivo m,
decimos que a es congruente con b modulo m, si a b es multiplo de m. En este caso
escribimos a b (mod m).
Ver [10].
Teorema 6 (Teorema de Thales) Consideremos dos rectas transversales a tres rectas
como se muestra en la figura. Tenemos que si AD, BE y CF son paralelas entonces
AB
DE
AB
DE
BC = EF . Recprocamente, si BC = EF y dos de las rectas AD, BE o CF son
paralelas, entonces las tres rectas son paralelas.

46

Apendice

Ver [1, 2].


Teorema 7 (Teorema de la bisectriz) La bisectriz interna AL (bisectriz externa AL )
del a ngulo en A de un triangulo ABC, divide internamente (externamente) al lado
opuesto BC en razon AB
CA , esto es:


AB
AB
BL
BL
=
=
respectivamente
LC
CA
CL
CA
donde L es el punto de interseccion de la bisectriz interna con el lado BC (L es el
punto de interseccion de la bisectriz externa con la prolongacion del lado BC).
Ver [2].
Teorema 8 (Formulas de a rea)
1. El a rea de un rectangulo de lados a y b es a b.
2. El a rea de un triangulo es igual a 21 hl, donde l es la medida de un lado y h es la
medida de la altura sobre dicho lado.
3. El a rea de un crculo de radio r es igual a r2 .
Ver [1, 2].

Definicion 9 (Angulos
entre paralelas) Cuando una recta intersecta a otras dos rectas se forman ocho a ngulos que numeramos del 1 al 8, como se muestra en la figura.
l1

1
3

l2

l3

5
7

Si la recta l3 intersecta a las rectas l1 y l2 , decimos que es transversal a ellas. Los


a ngulos 2, 4, 5 y 7 estan entre las rectas l1 y l2 , los llamamos a ngulos internos, los
a ngulos restantes los llamamos a ngulos externos. Los a ngulos en lados opuestos por

Apendice

47

la transversal l3 se llaman a ngulos alternos, como por ejemplo 3 y 5. A los a ngulos 4


y 5 les llamamos alternos internos y los a ngulos 3 y 6 son alternos externos.
A los a ngulos que estan en la posicion correspondiente respecto a la transversal, como
por ejemplo 3 y 7 los llamamos a ngulos correspondientes. Entonces, los pares de
a ngulos correspondientes en la figura anterior son 3 y 7, 1 y 5, 4 y 8, 2 y 6.
Si l1 y l2 son paralelas los a ngulos alternos internos son iguales.
Ver [2].
Teorema 10 (Suma de los a ngulos internos de un triangulo) La suma de los a ngulos internos de un triangulo es 180.
Ver [1, 2].
Teorema 11 (Teorema de Pitagoras) En un triangulo rectangulo, el cuadrado de la
hipotenusa es igual a la suma de los cuadrados de los catetos.
Ver [1, 2, 9].
Definicion 12 (Congruencia de triangulos) Los triangulos ABC y A B C son congruentes si los a ngulos y los lados del triangulo ABC son iguales a los a ngulos y los
lados del triangulo A B C .
Ver [1, 2].
Criterio 13 (Criterio de congruencia ALA) Un criterio de congruencia de triangulos nos dice que si tenemos dos triangulos con un lado igual y dos a ngulos adyacentes
iguales, entonces son congruentes. A este criterio se le conoce como a ngulo-ladoa ngulo y lo denotamos como ALA.
Ver [1, 2].
Criterio 14 (Criterio de congruencia LLL) Un criterio de congruencia de triangulos nos dice que si tenemos dos triangulos con sus tres lados correspondientes iguales,
entonces son congruentes. A este criterio se le conoce como lado-lado-lado y lo denotamos como LLL.
Ver [1, 2].
Definicion 15 (Semejanza de triangulos) Los triangulos ABC y A B C son semejantes, si sus a ngulos respectivos son iguales, es decir,
ABC = A B C
ACB = A C B
BAC = B A C
y sus lados homologos son proporcionales, esto es
BC
CA
AB
= = .
A B
BC
CA
Ver [1, 2].

48

Apendice

Criterio 16 (Criterio de semejanza AA) Si dos pares de a ngulos correspondientes


de los triangulos ABC y A B C son iguales, entonces los triangulos son semejantes.
A esta relacion le llamamos a ngulo-angulo y la denotamos como AA.
Ver [1, 2].
Teorema 17 Sea ABC un triangulo cualquiera, sean D, E y F los puntos medios de
AB, BC y CA, respectivamente y sea G el baricentro del triangulo ABC. Entonces
los seis triangulos: AGD, BGD, BGE, CGE, CGF y AGF tienen a reas iguales
entre s e iguales a 61 del a rea del triangulo ABC.
Ver [2].
Teorema 18 (Ley de los cosenos) En un triangulo de lados a, b y c, se cumple la relacion
a2 = b2 + c2 2bc cos ,
donde es el a ngulo opuesto al lado a.
Ver [2].
Teorema 19 Si trazamos dos rectas tangentes a una circunferencia desde un mismo
punto P , entonces los segmentos de recta desde P a los puntos de tangencia son iguales
y el centro de la circunferencia yace en la bisectriz del a ngulo entre las rectas.
Ver [2].
Teorema 20 (Potencia de un punto)
1. Si dos cuerdas AB y CD de una circunferencia se intersectan en un punto P , entonces P A P B = P C P D.
2. Si A, B y T son puntos sobre una circunferencia y la tangente en T intersecta
en un punto P a la prolongacion de la cuerda AB, entonces P T 2 = P A P B.
Ver [2, 4]
Teorema 21 (Medida del a ngulo inscrito) La medida de un a ngulo inscrito en una
circunferencia es igual a la mitad del arco comprendido entre sus lados, es decir, la
mitad del a ngulo central que subtiende el mismo arco.
Ver [1, 2].
Definicion 22 (Cuadrilatero cclico) Un cuadrilatero es cclico si sus cuatro vertices
estan sobre una misma circunferencia.
Ver [2].
Teorema 23 (Cuadrilatero cclico) Un cuadrilatero convexo ABCD es cclico si y
solo si la suma de los a ngulos opuestos es igual a 180 , es decir, si y solo si
DAB + BCD = ABC + CDA = 180 .
Ver [2].

Bibliografa

[1] A. Baldor. Geometra plana y del espacio. Publicaciones Cultural, Mexico, 1999.
[2] R. Bulajich Manfrino, J. A. Gomez Ortega. Geometra. Cuadernos de Olimpiadas
de Matematicas. Instituto de Matematicas de la UNAM, 2002.
[3] R. Bulajich Manfrino, J. A. Gomez Ortega, R. Valdez Delgado. Desigualdades. Cuadernos de Olimpiadas de Matematicas. Instituto de Matematicas de la
UNAM. Tercera edicion, 2007.
[4] J. A. Gomez Ortega. Algunas maneras de usar la potencia. Revista de la Olimpiada Mexicana de Matematicas, Tzaloa No. 4, 2009.
[5] R. Grimaldi, Matematicas Discreta y Combinatoria. Addison Wesley Longman,
Pearson. Tercera edicion, 1998.
[6] I. Niven, H. Zuckerman. Introduccion a la Teora de los Numeros. Limusa-Wiley,
Mexico 1972.
[7] M. L. Perez Segu. Combinatoria. Cuadernos de Olimpiadas de Matematicas.
Instituto de Matematicas de la UNAM, 2000.
[8] M. L. Perez Segu. Teora de Numeros. Cuadernos de Olimpiadas de Matematicas.
Instituto de Matematicas de la UNAM, 2003.
[9] A. Rechtman Bulajich. Algunas demostraciones del teorema de Pitagoras. Revista de la Olimpiada Mexicana de Matematicas, Tzaloa No. 1, 2010.
[10] A. Rechtman Bulajich, C.J. Rubio Barrios. Divisibilidad y congruencias. Revista
de la Olimpiada Mexicana de Matematicas, Tzaloa No. 2, 2009.

50

Bibliografa

[11] P. Soberon Bravo. El Principio de las Casillas. Revista de la Olimpiada Mexicana


de Matematicas, Tzaloa No. 2, 2010.
[12] N. Vilenkin. De cuantas formas? (Combinatoria). Editorial Mir, Moscu 1972.

Directorio

Directorio del Comite Organizador de la OMM


Anne Alberro Semerena
Facultad de Ciencias, UAEM
Av. Universidad 1001
62210, Cuernavaca, Morelos
Tel. (777) 3 81 03 80
Fax (777) 3 29 70 40
aalberro@uaem.mx

Gerardo Arizmendi Echegaray


Centro de Investigacion en Matematicas
Callejon Jalisco s/n, Mineral de Valenciana
36240, Guanajuato, Guanajuato
Tel. (473) 7 32 71 55
gerardo@cimat.mx

Ignacio Barradas Bribiesca


Universidad de Guanajuato
L. de Retana #5, Centro
36000, Guanajuato, Guanajuato
Tel. (473) 7 32 00 06 ext 2006
barradas@quijote.ugto.mx

Radmila Bulajich Manfrino


Facultad de Ciencias, UAEM
Av. Universidad 1001
62210, Cuernavaca, Morelos
Tel. (777) 3 29 70 20
Fax (777) 3 29 70 40
bulajich@uaem.mx

Gabriela Campero Arena


Facultad de Ciencias, UNAM
Av. Universidad 3000
04510, Mexico, D.F.
Tel. (55) 56 22 48 67
Fax (55) 56 22 48 66
gabriela@matematicas.unam.mx

Fernando Campos Garca

1a de Angel
Rico 85
AU.H. Vicente Guerrero
09200, Iztapalapa, Distrito Federal
Tel. (55) 34 63 75 43
fermexico89@hotmail.com

Jose Antonio Climent Hernandez


Facultad de Ciencias, UNAM
Av. Universidad 3000
04510, Mexico, D.F.
Tel. (55) 56 24 59 22
Fax (55) 56 22 48 59
jach@fciencias.unam.mx

Jose Alfredo Cobian Campos


Facultad de Ciencias, UNAM
Av. Universidad 3000
04510, Mexico, D.F.
Tel. (55) 56 22 49 25
Fax. (55) 56 22 48 59
cobian@matematicas.unam.mx

52
David Cosso Ruiz
Universidad Autonoma de Ciudad Juarez
Instituto de Ingeniera y Tecnologa
Departamento de Fsica y Matematicas
Av. del Charro 450 Nte.
CP 32310, Cd. Juarez, Chihuahua
Tel. (656) 688 48 87
Fax. (656) 688 48 13
sirio11@gmail.com

Directorio

Luis Cruz Romo


UPIITA, IPN
Av. Instituto Politecnico Nacional 2580
Col. Barrio la Laguna Ticoman
07340, Mexico, D.F.
lucruz@ipn.mx

Marco Antonio Figueroa Ibarra


Facultad de Matematicas
Universidad de Guanajuato
Callejon Jalisco s/n, Mineral de Valencia
36240, Guanajuato, Guanajuato
Tel. (473) 7 32 01 40
marcant@cimat.mx

Jesus
Jeronimo Castro
CIMAT
Apartado Postal 402
36000, Guanajuato, Guanajuato
Tel. (473) 7 32 71 55
Fax (473) 7 32 57 49
jeronimo@cimat.mx

Leonardo Ignacio Martnez Sandoval


Primera Cerrada de Alfalfares 41-2
Rinconada Coapa Primera Sec, Tlalpan
14330, Mexico, D.F.
Tel. (55) 26 52 23 29
ssbmplayer@gmail.com

Carlos Jacob Rubio Barrios


Universidad Autonoma de Yucatan
Periferico norte tablaje 13615
97119, Merida, Yucatan
Tel. (999) 942-3140 al 49
Fax (999) 942-31-40
carlos.rubio@uady.mx
jacob.rubio@gmail.com

Elena Ruiz Velazquez


Altair 12
Col. Lomas de Palmira
62550, Cuernavaca, Morelos
Tel. (777) 320 54 39
Cel. (777) 133 39 83
eleniux@gmail.com
A00375640@itesm.mx
Carmen Sosa Garza
Facultad de Ingeniera, UAQ
Cerro de las Campanas s/n
Queretaro, Queretaro
Tel. (442) 1 92 12 64 ext. 121 o 136
Fax (442) 1 92 12 646
carsg@uaq.mx

Pablo Soberon Bravo


Circuito Interior no. 830
Fracc. La Herradura
62303, Cuernavaca, Morelos
Cel. (777) 134 55 49
bandrak@hotmail.com

Rogelio Valdez Delgado


Facultad de Ciencias, UAEM
Av. Universidad 1001
62210, Cuernavaca, Morelos
Tel. (777) 3 29 70 20
Fax (777) 3 29 70 40
rogelio@matcuer.unam.mx

Directorio

Eduardo Velasco Barreras


Universidad de Sonora
Calle Yucas 16, Vista Bella
83170, Hermosillo, Sonora
Tel. (662) 2 19 10 07
hamsteritokeweb@hotmail.com

53
Hugo Villanueva Mendez
Instituto de Matematicas, UNAM
Cub. 4 de Becarios,
Circuito Exterior, Ciudad Universitaria
Coyoacan 04510,
Mexico, D.F.
Tel (55) 56 22 45 32
vill hugo@hotmail.com
hvillan@matem.unam.mx

54

Directorio
Direccion Postal de la Olimpiada Mexicana de Matematicas:
Cubculo 201, Departamento de Matematicas.
Circuito Exterior, Facultad de Ciencias.
Universidad Nacional Autonoma de Mexico.
Ciudad Universitaria.
Colonia Copilco, C.P. 04510.
Delegacion Coyoacan.
Mexico, Distrito Federal.
Telefono: (55) 5622-4864.
Fax: (55) 5622-5410.
Email: omm@fciencias.unam.mx
Pagina oficial de la Olimpiada Mexicana de Matematicas:
http://www.omm.unam.mx/

TZALOA
Revista de la Olimpiada
Mexicana de Matematicas
2010, No. 4
Ano

Comite Editorial:
Anne Alberro Semerena
Ana Rechtman Bulajich
Carlos Jacob Rubio Barrios
Francisco Ruiz Benjumeda

Comite de la Olimpiada Mexicana de Matematicas


Cubculo 201
Departamento de Matematicas
Facultad de Ciencias, UNAM
Circuito Interior s/n
Ciudad Universitaria
Coyoacan C.P. 04510
Mexico D.F.
Telefono: (55) 56-22-48-64
www.omm.unam.mx

Diseno de Portada: Manuel Macas Beckmann


www.rayaenmedio.com

Impreso: Torre y de la Torre Impresos


Aragon no. 134

Col. Alamos,
03400
Mexico D.F.
Telefonos: (55) 55-30-14-82 y (55) 55-38-34-53

c
Queda
estrictamente prohibida la reproduccion parcial o total por cualquier sistema
o metodo, mecanico o electronico, sin autorizacion previa del autor.
Impreso y hecho en Mexico.
Octubre de 2010.

Contenido

Presentacion

Artculos de matematicas: Circuncrculos

Problemas de practica

Soluciones a los problemas de practica

13

Problemas propuestos
2010 No. 4
Problemas propuestos. Ano
2010 No. 2
Soluciones a los problemas propuestos. Ano

25
25
26

Olimpiadas Internacionales
51a Olimpiada Internacional de Matematicas

31
31

Problemas y Soluciones de Olimpiadas Internacionales


XXII Olimpiada de la Cuenca del Pacfico
American Mathematics Competition (AMC)
XII Olimpiada Centroamericana y del Caribe

33
33
40
59

Una vida, un matematico: Una conversacion con Emilio Lluis Riera

67

Informacion Olmpica

75

Apendice

77

Bibliografa

80

Directorio

83

IV

Contenido

Presentacion
Tzaloa es una publicacion periodica trimestral de la Olimpiada Mexicana de Matematicas y su objetivo es fomentar el estudio de las matematicas como una disciplina dinamica y creativa. El diseno de las secciones y la cuidadosa seleccion de sus contenidos
buscan apoyar de manera efectiva, con informacion y con materiales de calidad, a estudiantes y profesores de nivel medio superior que cada ano se preparan para participar
en los diferentes concursos de la Olimpiada de Matematicas.
Esta revista, con orgullo, toma su nombre del nahuatl porque esta hecha por y para los
mexicanos. Tzaloa significa aprender y las paginas que la conforman buscan ayudar a
satisfacer la necesidad de contar con espacios adecuados para profesores, estudiantes
y, en general, para todas aquellas personas interesadas en desarrollar e incrementar sus
capacidades para el razonamiento logico matematico y la resolucion de problemas.

2010, Numero

Tzaloa, Ano
4
Para despedir al ano 2010, en este numero de tu revista, hemos incluido dos interesantes artculos, los examenes de los u ltimos concursos internacionales en los que
participo Mexico, la informacion Olmpica mas actualizada y, desde luego, una rica
variedad de problemas para que pongas a prueba toda tu capacidad.
Como es el u ltimo numero del ano, para la seleccion del material que conforma las
secciones Problemas de Practica y Problemas Propuestos, hemos contemplando problemas cuya solucion no es trivial y muchos de los cuales se consideran de nivel avanzado. Sin embargo, lo anterior no debe desanimar a todos aquellos lectores que apenas
se inician, hay que tener en cuenta que la valoracion sobre la dificultad de un problema
siempre es subjetiva y que muchas veces, una idea original y creativa, puede hacer que
un problema considerado como difcil encuentre una solucion sencilla.
El artculo de temas matematicos que seleccionamos para este numero trata sobre Circuncrculos y es una colaboracion de Eduardo Velasco Barreras. Con su amplia experiencia olmpica, Eduardo nos presenta un excelente material que estamos seguros

VI

Presentacion

sera de gran valor para todos aquellos que trabajan en su preparacion para participar
en algun concurso. Es un hecho que, a lo largo de todo el mundo, los problemas de
geometra son los que aparecen con mayor frecuencia en los examenes olmpicos de
todos los niveles y que, de entre los problemas geometricos, los que involucran al circuncentro de un triangulo (ya sea en su planteamiento o en su solucion) tiene un lugar
destacado. Aunado a lo anterior, la seleccion de los ejemplos escogidos para este artculo es excepcional, a traves de ellos es posible observar como los circuncrculos junto
con resultados clasicos sobre a ngulos inscritos, semi-inscritos, potencia de un punto
con respecto a un crculo, etc. permiten resolver, de manera elegante, una gran variedad de problemas geometricos.
Desde el ano pasado decidimos que en el cuarto numero de cada ano, se presentara
un artculo adicional con la vision de un matematico famoso. Esta seccion sobre el
quehacer de los matematicos busca acercar a nuestros lectores con quienes han dedicado su vida profesional a explorar la belleza del mundo matematico. Es as, que en esta
ocasion y como regalo de fin de ano, presentamos la entrevista que Emilio Lluis Riera
dio en exclusiva para los lectores de Tzaloa. La pasion de Lluis por las matematicas ha
dejado una imborrable huella en muchas generaciones de matematicos mexicanos. Su
nombre es conocido en todo el mundo y sus importantes contribuciones al desarrollo
de esta ciencia han puesto en alto el nombre de Mexico. Hoy, Emilio Lluis Riera, con la
sencillez y carisma que le caracterizan, comparte con nuestros lectores una agradable
charla que estamos seguros disfrutaran tanto como nosotros.
Como siempre, en la seccion de Olimpiadas Internacionales presentamos los resultados y examenes de los u ltimos concursos en los que Mexico participo. Presentamos el
examen de la 51a Olimpiada Internacional de Matematicas, en la que Mexico tuvo una
participacion destacada. Ademas incluimos los examenes con soluciones de la XXII
Olimpiada Matematica de la Cuenca del Pacfico (APMO, por sus siglas en ingles) y de
la XII Olimpiada Centroamericana y del Caribe; eventos en los que Mexico alcanzo el
15 y 1er lugar respectivamente. Ademas, en esta ocasion, tambien incluimos las soluciones de los examenes de la American Mathematics Competition (AMC), mismos que
sirvieron como preparacion para las delegaciones mexicanas que este ano participaron
en concursos internacionales.

Mexico y las Olimpiadas de Matematicas


Hace mas de 23 anos que la Sociedad Matematica Mexicana ha venido impulsando
vigorosamente los trabajos de la Olimpiada Mexicana de Matematicas (OMM). Desde
sus inicios, este programa se ha visto fortalecido gracias a la participacion de miles
de jovenes estudiantes y a la entusiasta colaboracion de muchos profesores quienes,
de manera espontanea y altruista, han dedicado sus esfuerzos a mejorar la ensenanza
y elevar la cultura matematica de nuestro pas. Motivados por el movimento olmpico,
en escuelas ubicadas a lo largo de todo el territorio nacional, se han desarrollado innu-

Presentacion

VII

merables talleres de resolucion de problemas, donde estudiantes y profesores trabajan


con el u nico afan de incrementar sus capacidades para el razonamiento, el analisis y la
creatividad matematica.
En el a mbito internacional, mediante la destacada participacion de las delegaciones
mexicanas en diversos concursos, la Olimpiada Mexicana de Matematicas ha contribuido a elevar el prestigio de la matematica nacional. Pero, mas importante aun ha sido
la contribucion que el movimiento olmpico ha tenido para el desarrollo cientfico del
pas. En muchos casos, la deteccion temprana de jovenes con talento matematico excepcional ha permitido brindarles una formacion adecuada para desarrollar al maximo
todo su potencial. Asimismo, la participacion en los concursos olmpicos ha definido
las vocaciones de muchos otros estudiantes. Universidades de todo el pas se han visto beneficiadas con el ingreso de jovenes ex-olmpicos, mismos que cuentan con una
solida formacion matematica y muchos de los cuales han permanecido en ellas para
dedicar su vida profesional a la docencia y la investigacion.

24a Olimpiada Mexicana de Matematicas


El programa anual de la Olimpiada Mexicana de Matematicas se desarrolla en 3 etapas:
Concursos Estatales.
Concurso Nacional.
Entrenamiento, seleccion y participacion de las delgaciones nacionales que representan a Mexico en concursos internacionales.
En la 24a Olimpiada Mexicana de Matematicas podran participar los estudiantes de
Mexico nacidos despues del 1 de agosto de 1991. Los concursantes deberan estar inscritos en una institucion preuniversitaria durante el primer semestre del ciclo escolar
2010-2011 y, para el 1 de julio de 2011, no deberan haber iniciado estudios universitarios. Para mayor informacion puedes consultar la pagina:
http://www.omm.unam.mx
Para la primera etapa, los participantes deberan inscribirse directamente con el Comite Estatal correspondiente.
El Concurso Nacional de la 24a Olimpiada Mexicana de Matematicas se realizara del
21 al 26 de noviembre de 2010 en Ensenada, Baja California. A los primeros lugares
de este certamen se les invitara a la etapa de entrenamiento y seleccion de las delegaciones que representaran a Mexico en las distintas Olimpiadas Internacionales del ano
2011: la XXIII Olimpiada Matematica de la Cuenca del Pacfico, que se llevara a cabo
en el mes de marzo; la XIII Olimpiada Matematica de Centroamerica y el Caribe, que
se celebrara en el mes de junio; la 52a Olimpiada Internacional de Matematicas, que se
llevara a cabo en julio en Amsterdam, Pases Bajos, y la XXVI Olimpiada Iberoamericana de Matematicas que se realizara en el mes de septiembre en Costa Rica.

VIII

Presentacion

Circuncrculos
Por Eduardo Velasco Barreras
Nivel Avanzado

Entre los primeros puntos importantes de los triangulos que estudiamos en la olimpia
da esta el circuncentro. Este
es el punto de concurrencia de las mediatrices que tiene
la propiedad de que es el u nico punto que se encuentra a la misma distancia de los
vertices del triangulo, en otras palabras, podemos trazar un crculo con centro en el
circuncentro que pasa por todos los vertices del triangulo. Recordemos que la mediatriz de un segmento AB es la recta perpendicular a AB que pasa por su punto medio.
Empecemos con un ejemplo.
Ejemplo 1. Sea ABC un triangulo equilatero. Sean: A el punto en la recta BC (distinto de C) tal que BC = BA , B el punto en la recta CA (distinto de A) tal que
CA = CB y C el punto en la recta AB (distinto de B) tal que AB = AC . Pruebe
que los triangulos ABC y A B C tienen el mismo circuncentro.
C

A
b

Circuncrculos

Solucion. Sea O el circuncentro de ABC. Demostraremos que O es tambien el circuncentro de A B C , probando que OA = OB = OC , y de esta manera quedara demostrado que los triangulos tienen el mismo circuncentro.
Como O es circuncentro de ABC se tiene que OA = OB. Ademas, por ser ABC
equilatero,

OBA = 180 OBC = 180 30 = 180 OAB = OAC .


Finalmente, por construccion, y por ser ABC un triangulo equilatero, se tiene que
BA = BC = BA = AC .
De las tres igualdades anteriores, OA = OB, OBA = OAC , BA = AC , se
concluye que los tringulos OBA y OAC son congruentes y entonces OA = OC . De
manera analoga podemos demostrar que los triangulos OAC y OCB son congruentes, de lo cual se concluye que OC = OB , de donde se sigue que OA = OB = OC
y O es circuncentro de A B C .
Una propiedad muy utilizada en la olimpiada es la relacion entre a ngulos inscritos en
una circunferencia. Como utilizaremos mas adelante este resultado, vamos a enunciarlo:

Teorema del Angulo


Inscrito La medida del a ngulo inscrito es igual a la mitad de la
medida del a ngulo central.

En la figura se ilustra este teorema: El a ngulo inscrito mide y el a ngulo central mide
2.
Una relacion importante que existe entre el circuncentro y las alturas de un triangulo
es la siguiente:
Propiedad 1. Sea ABC un triangulo cuyo circuncentro es O y sea AD la altura desde
A. Entonces BAD = CAO.

Circuncrculos

D
A1

Solucion. Consideremos el circuncrculo del triangulo ABC y prolonguemos la recta


AO hasta que corte al circuncrculo en un punto A1 . Como AA1 pasa por el centro O entonces es un diametro de la circunferencia de donde se sigue que el a ngulo
ACA1 = 90 (por el Teorema del a ngulo inscrito). Como los a ngulos AA1 C y
del circuncrculo, entonces ABD = AA1 C. Por otro laABC abren el arco AC

do, BDA = 90 = A1 CA. Estas dos u ltimas igualdades implican que los triangulos BDA y A1 CA son semejantes, de donde se concluye que BAD = CAO.
Propiedad 2. Sea ABC un triangulo con circuncentro O. Entonces CBA+CAO =
90 .
A

Solucion. Como AD es altura, tenemos que DBA + BAD = 90 . Sin embargo,


DBA = CBA y por la Propiedad 1 BAD = CAO. Sustituyendo las igualdades en la suma, obtenemos el resultado.
Esta propiedad esta enunciada aqu para los a ngulos CBA y CAO. Sin embargo,
se puede demostrar de la misma manera que
ABC + ACO

90

ACB + ABO
BCA + BAO

=
=

90
90

CAB + CBO
BAC + BCO

=
=

90
90 .

Circuncrculos
A

+ + = 90
Cuando tenemos problemas en los que se involucra al circuncentro, casi siempre resulta u til trazar el circuncrculo del triangulo y trazar alguno de los diametros. Sin
embargo, tambien se pueden utilizar propiedades como las que acabamos de demostrar
para resolver problemas. Para ilustrar las propiedades, resolvamos el siguiente ejemplo:
Ejemplo 2. Sea ABC un triangulo de circuncentro O y sea AD la bisectriz de BAC
con D sobre BC. La perpendicular a BC por D corta a AO en P . Demuestre que
P A = P D.
A
P
b

O
D

Solucion. Demostraremos que P DA = P AD, lo que implica que el triangulo


ADP es isosceles con P A = P D. Tenemos que P DA = 90 ADB, por ser
P D perpendicular a BC. Tambien se tiene que ADB = DAC + ACD, por ser
ADB exterior en el triangulo DAC. Entonces
P DA = 90 DAC ACD.
Por otro lado, tenemos que P AD = P AB DAB = P AB DAC ya que
AD es bisectriz. Queremos demostrar que P DA = P AD, lo cual es cierto ya que
utilizando la Propiedad 2 tenemos que
P AB

= OAB
= 90 ACB
= 90 ACD,

Circuncrculos

lo que finaliza la demostracion.


Ahora, vamos a combinar algunos de los resultados ya demostrados para dar una caracterizacion del circuncentro:
Propiedad 3. Sea ABC un triangulo tal que BAC es un a ngulo a gudo. Si O es un
punto del mismo lado de la recta BC que A tal que BOC = 2BAC, y OB = OC,
entonces O es el circuncentro del triangulo ABC.

O
2

Solucion. Notemos primero que el circuncentro cumple con ambas propiedades, es


decir, si trazamos el circuncentro de ABC, por ser BAC un a ngulo inscrito y BOC
el a ngulo central tendremos que BOC = 2BAC, ademas de que OB = OC
porque estas longitudes son radios.
Para ver que solo el circuncentro cumple con esta propiedad, veamos lo siguiente: El
hecho de que un punto O satisfaga que OB = OC es equivalente a que O se encuentre
en la mediatriz del segmento BC (y de hecho el circuncentro esta en esa recta), y el
hecho de que O satisfaga la relacion de a ngulos y que este del mismo lado de BC
que pasa tambien por el
que A significa que O pertenece al arco de circunferencia BC
circuncentro.

El punto O, debera estar por lo tanto en la interseccion del arco con la mediatriz. Como
la mediatriz y el arco se intersectan en un solo punto, dicho punto debe ser u nico, por
lo que solo el circuncentro cumple con esas condiciones.
Esta u ltima propiedad es bastante u til para demostrar problemas que no son sencillos
sin esta herramienta.

Ejemplo 3. Sea ABC un triangulo de incentro I. El incrculo del triangulo ABC toca a
los lados BC, CA, AB en A , B , C , respectivamente. Sea K el punto diametralmente
opuesto a C , y sea D la interseccion de las lineas B C y A K. Pruebe que CD =
CB .

Circuncrculos

D
A

B
C

I
K
2

Solucion. Como CB y CA son tangentes al incrculo desde C, entonces tienen la


misma longitud1. Demostrar que CD = CB es equivalente entonces a demostrar que
CD = CB = CA , lo cual nos dice que el problema es equivalente a demostrar que
C es el circuncentro de A B D.
Observemos que C y D se encuentran del mismo lado de la recta A B . Para demostrar
que C es el circuncentro de A B D utilizaremos la Propiedad 3. Para poder aplicarla
necesitamos verificar tres cosas:
B DA

<

90

=
=

2B DA
CB .

B CA
CA

1. Como C K es diametro del incrculo, entonces C A D = 90 , por lo que el


triangulo C A D es rectangulo en A y el a ngulo B DA es agudo.
2. Como B es punto de tangencia del incrculo con el lado AC entonces IB C =
90 . De la misma manera, IA C = 90 . Como IB C + IA C = 90 +
90 = 180 , entonces el cuadrilatero B IA C es cclico2 y A IB +A CB =
180 . Esta u ltima ecuacion es equivalente a A CB = 180 2A C B ,
pues A CB es inscrito y A IB es el a ngulo central correspondiente. Como
A C B + A DB = 90 , se tiene que B CA = 2B DA .
3. CA = CB ya lo habamos demostrado.
Como se cumplen las tres condiciones, C es circuncentro del triangulo B A D, y entonces CD = CB , como queramos.
Ejemplo 4. Sea ABCD un cuadrilatero convexo inscrito en una circunferencia tal que
AD es diametro, y sean P el punto de interseccion de AB y CD, y Q el punto de
interseccion de AC y BD. Sea O el punto de interseccion de las tangentes por B y C
a la circunferencia. Demuestre que O, P, Q son colineales.
1 Ver
2 Ver

en el apendice el teorema POTENCIAS.


en el apendice la definicion ... y el teorema ... CUADRILATERO CICLICO.

Circuncrculos

P
O

B
Q
A

Solucion. Para demostrar que estos puntos son colineales, demostraremos que
AP O = AP Q.
Primero notemos que ACD = ABD = 90 ya que AD es diametro. Sea =
BAC. Observemos que, como el triangulo AP C es rectangulo en C, entonces
BP C = 90 .
Ahora, como OB y OC son tangentes a la circunferencia, entonces, por el Teorema

del Angulo
Seminscrito 3 , se tiene que OBC = OCB = BAC = , por lo que
OB = OC y ademas
BOC = 180 2.

Observemos que el triangulo BP C es acutangulo, que el punto O esta en el interior


de e ste y satisface OB = OC ya que OB y OC son tangentes. Ademas tenemos que
BOC = 2BP C, por lo que concluimos por la Propiedad 3 que O es el circuncentro
del triangulo BP C. Como O es circuncentro, aplicando la Propiedad 2 tenemos que
BP O = 90 P CB. Utilizando que el cuadrilatero ABCD es cclico, P AD =
180 BCD = P CB. Entonces
AP O = BP O = 90 P CB = 90 P AD.
Ahora, como ACD = ABD = 90 , tenemos que AC y DB son alturas del
triangulo AP D. Luego, Q es ortocentro de este triangulo y entonces P Q es altura
tambien, de donde se sigue que
AP Q = 90 P AD.
Por lo tanto, AP Q = AP O, y entonces P, O, Q son colineales, como queramos.

Ejercicios
1. Sea ABCD un cuadrilatero cclico tal que sus diagonales AC y BD son perpendiculares. Sea M el punto medio de AB y sea P el punto de interseccion de
las diagonales. Demuestra que M P es perpendicular a CD.
3 Ver

en el apendice el teorema ANGULO SEMINSCRITO

Circuncrculos
2. Sea ABCD un cuadrilatero cclico y sea P el punto de interseccion de las diagonales. Sea O el circuncentro de ABP y H el ortocentro de CDP . Demuestra
que O, H, P son colineales.
3. Sea ABC un triangulo acutangulo. Sea AD la bisectriz del a ngulo en A, con D
sobre BC y sea O su circuncentro. La perpendicular a AO por D corta a AC en
el punto B . Demuestra que AB = AB.
4. Sea ABC un triangulo y AD la altura sobre el lado BC. Tomando a D como
centro y a AD como radio, se traza una circunferencia que corta a la recta AB
en P y corta a la recta AC en Q. Demuestra que el triangulo AQP es semejante
al triangulo ABC.
5. Sea ABC un triangulo de ortocentro H y sean OA , OB , OC los circuncentros de
los triangulos HBC, HCA, HAB, respectivamente. Demuestra que el triangulo
OA OB OC es congruente al triangulo ABC y que su circuncentro es H.
6. Sea ABC un triangulo acutangulo de circuncentro O. Sea P un punto sobre
el lado BC y sean Q el punto de interseccion del circuncculo de OP B con
AB (Q 6= B) y R el punto de interseccion del circuncrculo de OP C con AC
(R 6= C). Demuestra que el triangulo P QR es semejante al triangulo ABC y
que el ortocentro de P QR es O.
7. Sea ABC un triangulo. Sean A un punto en la recta BC tal que B este entre
A y C y A B = AB; B un punto en CA tal que C este entre B y A y
B C = BC; C un punto en AB tal que A este entre C y B y AC = AC.
Si los triangulos ABC y A B C tienen el mismo circuncentro, demuestra que
ABC es equilatero.

Bibliografa
T. Andrescu, Z. Feng, Mathematical Olympiads: Problems and Solutions from Around the World,
1999-2000, The Mathematical Association of America.

Problemas de practica
A continuacion te presentamos los 20 problemas de nivel intermedio y avanzado que
hemos seleccionado para este numero y mismos con los que podras poner a prueba tus
conocimientos y habilidades. Aunque en la siguiente seccion encontraras las soluciones
de todos ellos, no te recomendamos consultarla sino hasta despues de que le hayas
dedicado bastante tiempo a cada problema. Ten en cuenta que al consultar la solucion
de un problema sin haber hecho un verdadero esfuerzo para resolverlo, desperdicias la
oportunidad de incrementar tu capacidad para enfrentar situaciones difciles.
Por u ltimo, te invitamos a contribuir para que esta seccion de la revista se siga enriqueciendo con la participacion de todos. Estamos seguros que conoces y tienes problemas
interesantes que proponer, por eso ponemos a tu disposicion la direccion revistaomm@
gmail.com, donde con gusto recibiremos tus sugerencias.
Problema 1. Si el radio del crculo mide 5 cm, cuanto vale la suma de las longitudes

de los arcos P
Q y RS?
S
P

20
R

Problema 2. Si x es un numero real que satisface la ecuacion


x

22 + 42 = 42,
determina el valor de

222 . (Nota: ab denota el valor de a(b ) .)

Problema 3. En un triangulo rectangulo, las medidas de las longitudes de sus lados


son 8, x + 5 y x + 7. Encuentra el valor del seno del mayor de los a ngulos agudos del
triangulo, si se sabe que x > 3.

10

Problemas de practica

Problema 4. Dos alumnos colocan alternativamente numeros enteros en los lugares


vacos de la ecuacion
x3 + ( )x2 + ( )x + ( ) = 0.
Dispone el alumno que escribe el primer numero de alguna estrategia que le permita
asegurar que al finalizar el juego siempre se obtendra una ecuacion con tres soluciones
enteras?
Problema 5. Un tablero de 66 esta cubierto por fichas de domino de 21. Demuestra
que existe una lnea recta que separa las piezas del tablero sin cortar ningun domino.
Problema 6. Demuestra que el numero 9, 999, 999 + 1, 999, 000 no es primo.
Problema 7. Sea ABCD un cuadrilatero convexo tal que sus diagonales AC y BD
son perpendiculares. Sean P la interseccion de AC con BD, y M el punto medio de
AD. Demuestra que el cuadrilatero ABCD es cclico si y solo si las rectas M P y BC
son perpendiculares.
Problema 8. Coloca varios palitos sobre una mesa de manera que se forme un rectangulo de m n como se muestra en la figura. Pinta cada palito de azul, rojo o negro de
manera que cada uno de los cuadritos de 1 1 de la figura quede delimitado por exactamente dos palitos de un color y dos de otro color. De cuantas maneras puedes hacerlo?

..
.

..
.

..
.

..
.

n
Problema 9. Sean A, B, C y D vertices consecutivos de un heptagono regular; AL
y AM las tangentes desde A a la circunferencia de centro C y radio CB; y N la
interseccion de AC y BD. Demuestra que los puntos L, M y N son colineales.
Problema 10. La desigualdad media
aritmetica - media geometrica establece que si

ab,
y
la igualdad se cumple si y solo si a = b.
a 0 y b 0, entonces a+b

Suponiendo que x > y > 0 y xy = 2, determina el menor valor posible de

x2 +y 2
xy .

Problema 11. En un triangulo acutangulo ABC, se construye un semicrculo con centro sobre BC y tangente a los otros dos lados. Sea ra el radio de dicho semicrculo y
mediante construcciones analogas definimos a rb y rc . Demuestra que si r es el radio
del incrculo de ABC, entonces r2 = r1a + r1b + r1c .

Problemas de practica

11

Problema 12. De cuantas formas distintas se puede escribir el numero 1, 000, 000
como producto de tres enteros mayores que 1? (Se considera que abc es la misma
factorizacion que bac, etc.)
Problema 13. Determina todas las ternas de numeros naturales (a, b, c) tales que a, b
y c esten en progresion geometrica y que cumplan con la propiedad a + b + c = 111.
Problema 14. Demuestra que es posible elegir 17 segmentos de longitudes enteras
menores o iguales que 2010, de manera que con ninguna terna de ellos sea posible
construir un triangulo. Podra tambien elegirse una coleccion de 18 segmentos con las
mismas caractersticas?
Problema 15. Para que valores de n la escritura en base 10 de 11n tiene sus dos
u ltimos dgitos iguales?, y sus tres u ltimos dgitos iguales?
Problema 16. Determina todos los enteros positivos a y b, con a < b, tales que exac1
de los enteros consecutivos a2 , a2 + 1, a2 + 2, . . . , b2 , sean cuadrados
tamente 100
perfectos.
Problema 17. Sea ABC un triangulo con centroide G y sean A1 , B1 , C1 los puntos
medios de los lados BC, CA y AB, respectivamente. Una recta paralela a BB1 por
A1 intersecta el segmento B1 C1 en el punto F . Demuestra que los triangulos ABC y
F A1 A son semejantes si y solo si el cuadrilatero AB1 GC1 es cclico.
Problema 18. Demuestra que hay una infinidad de enteros positivos k tales que el
numero k k se puede expresar como suma de dos cubos de enteros positivos.
Problema 19. Sea K un punto en el interior de un paralelogramo ABCD tal que el
punto medio de AD equidista de K y de C, y el punto medio de CD equidista de K y
de A. Sea N el punto medio de BK. Demuestra que N AK = N CK.
Problema 20. Sean a y b enteros distintos tales que 2 a 100 y 2 b 100.
Demuestra que el numero
n
n
a2 + b 2
no es primo para algun entero positivo n.

12

Problemas de practica

Soluciones a los problemas de


practica

A continuacion encontraras las soluciones de los 20 problemas de practica de la seccion


anterior. Como siempre, las soluciones que presentamos no son u nicas y probablemente tampoco son las mejores, por lo que es muy posible que tu hayas encontrado una
solucion distinta pero igualmente valida. Si este es el caso y no estas muy seguro de
su validez o simplemente la quieres compartir con nosotros te invitamos para que nos
escribas a revistaomm@gmail.com
Solucion del problema 1. Llamemos O al centro del crculo. Entonces, los a ngulos
inscritos (ver en el apendice el teorema 18) P OQ y SOR miden 40 cada uno.
S
P
Q

T
O

1

Como 360
40 = 9, los arcos P Q y RS miden 9 de la longitud de la circunferencia. Es
10
25
decir, miden 9 = 9 cm cada uno, por lo que la suma de sus longitudes es igual a
20
9 cm.

Solucion del problema 2. Notemos que 42 = (22 )2 = 22(2

= (22 )2 . Luego, si

14

Soluciones a los problemas de practica


x

hacemos y = 22 , la ecuacion se convierte en


y 2 + y 42 = 0
(y + 7)(y 6) = 0.
x

de
y = 6 o y = 7. Como y > 0, tenemos que y = 22 = 6 y por lo tanto
p donde

x
2
22 = 26 = 23 = 8.

Solucion del problema 3. Como x > 3, entonces x+ 7 > 10 y en particular x+ 7 > 8.


Por otro lado, es claro que x + 7 > x + 5. Con lo anterior deducimos que x + 7 es la
longitud del lado mayor del triangulo rectangulo, por lo tanto x + 7 es la longitud de
la hipotenusa, y ademas, 8 y x + 5 son las longitudes de los catetos. Por el teorema de
Pitagoras (ver en el apendice el teorema 10) tenemos que
82 + (x + 5)2

64 + x2 + 10x + 25 =
4x =
x =

17

(x + 7)2
x2 + 14x + 49
40
10.

15

8
Luego, los lados del triangulo miden 8, 15 y 17. Por lo tanto, el seno del mayor de sus
15
a ngulos agudos es 17
.
Solucion del problema 4. El alumno que empieza tiene una estrategia ganadora. En el
primer movimiento coloca un 0 en el lugar correspondiente al termino de grado cero.
Con esto, la ecuacion se convierte en x3 + ( )x2 + ( )x = x[x2 + ( )x + ( )], as que
x = 0 es una solucion. El segundo alumno va a colocar un numero en cualquiera de
los dos lugares disponibles. Digamos que dicho numero es a, en cualquiera de los dos
lugares. Entonces, el primero puede colocar (a + 1) en el lugar que quede vaco.
Obtienen la ecuacion
x(x2 + ax a 1) = 0

x(x2 (a + 1)x + a) = 0.

Como al final la suma de los coeficientes es igual a 1, tenemos que x = 1 es otra


solucion de la ecuacion. En la primera ecuacion, la tercera solucion es x = (a + 1) y
en la segunda ecuacion es x = a.

Soluciones a los problemas de practica

15

Solucion del problema 5. Como cada ficha de domino esta formada por dos cuadrados,
con 18 fichas de domino el tablero queda totalmente cubierto. Imaginemos ahora una
recta horizontal que separe el tablero en dos partes. Si tal recta no corta ninguna ficha
de domino, el problema esta resuelto. Supongamos entonces que la recta corta por en
medio solo a una ficha de domino. Entonces, arriba de esta recta tenemos n dominos
enteros mas medio domino, es decir, 2n + 1 cuadrados, que es un numero impar. Pero
esto es imposible porque si el tablero tiene 6 unidades de longitud, cualquier recta lo
dividira en partes que contienen un numero par de cuadrados arriba y abajo de ella.
As, si una recta corta a una ficha de domino, debera cortar a otra. Para la division
del tablero existen 10 rectas posibles (5 rectas horizontales y 5 verticales) y si cada
una de ellas corta dos dominos deberamos tener 20 dominos en el tablero, ya que un
domino puede ser cortado solo por una recta. Como el tablero se cubre con 18 fichas,
entonces existe por lo menos una recta que no corta ninguna ficha de domino.
Solucion del problema 6. Tenemos que,
9, 999, 999 + 1, 999, 000 = (107 1) + (2 103 1) 103

= (9 106 + 106 1) + (2 106 103 )


= 3 103 (3 103 1) + 3 106 103 + 3 103 1

= 3 103 (3 103 1) + 3 103 (103 + 1) (103 + 1)

= 3 103 (3 103 1) + (103 + 1)(3 103 1)


= (3 103 1)(3 103 + 103 + 1)
= 2999 4001,

de donde se sigue que el numero 9, 999, 999 + 1, 999, 000 no es primo.


Solucion del problema 7. Primero supongamos que las rectas M P y BC son perpendiculares. Llamemos K al punto de interseccion de M P y BC, y x = ADB. Como
el triangulo AP D es rectangulo y M es punto medio de AD, entonces P M = AM =
M D. Luego, M P D = x, de donde AM P = 2x.
A

D
x

90

P
x

B
Entonces, M P A = 90 x, de donde CP K = M P A = 90 x. Pero
P KC = 90 , entonces P CB = x, luego, ADB = ACB = x. Por lo tanto, el
cuadrilatero ABCD es cclico (ver en el apendice la definicion 19 y el teorema 20).

16

Soluciones a los problemas de practica

Ahora supongamos que el cuadrilatero ABCD es cclico.

90

P
y

K
B

Como M es punto medio de la hipotenusa del triangulo rectangulo AP D, entonces


P M = M A = M D. Sea x = ADB, entonces DAP = M P A = 90 x, luego
CP K = 90 x. Como el cuadrilatero ABCD es cclico, entonces ACB =
ADB = x, de donde P KC = 180 (90 x + x) = 90 . Por lo tanto, M P es
perpendicular a BC.
Solucion del problema 8. Observemos que hay 3n maneras de pintar la hilera horizontal superior de palitos. El palito vertical a la izquierda de la primera lnea, tambien
se puede pintar de 3 maneras.
n

Una vez definidos los colores de los palitos superiores y del vertical que esta en el
extremo izquierdo, hay dos maneras de pintar los otros dos palitos que forman el primer
cuadrito segun las condiciones del problema.
1. Si los dos palitos que pintamos son del mismo color, entonces hay dos colores
para pintar los dos palitos restantes.
2. Si los palitos que pintamos son de distinto color, entonces hay dos maneras de
pintar los dos palitos restantes con estos colores.
Esta situacion se repite con cada uno de los n cuadritos de la primera lnea. Luego, para
completar la primera lnea de cuadrados hay 3n 3 2n maneras.
Analogamente, hay 3 maneras de pintar el palito que esta en el extremo izquierdo en la
segunda lnea, y 2n maneras de pintar los demas palitos de esa lnea. As, para m = 2
hay 3n 3 2n 3 2n = 3n (3 2n )2 coloraciones posibles. Por lo tanto, en el
caso de un arreglo de m n cuadritos de 1 1 tenemos 3n (3 2n)m = 3m+n 2mn
coloraciones distintas.

Soluciones a los problemas de practica

17

Solucion del problema 9. Consideremos la circunferencia que circunscribe al hepta y CD


son iguales, entonces DAC = BDC, luego, los
gono. Como los arcos BC
AC
CD
triangulos ACD y DCN son semejantes. De aqu que CD
= N
C , pero CD = CL ya
CL
que son radios de la circunferencia con centro en C, de modo que AC
CL = N C .

L
b

C
N
B
b

Luego, como el a ngulo ACL es comun para los triangulos ACL y LCN , tenemos
que los triangulos ACL y LCN son semejantes por el criterio LAL (ver en el apendice
el criterio 16). Pero ACL es un triangulo rectangulo, entonces LCN tambien lo es,
con a ngulo recto en N . Por lo tanto, LN es perpendicular a AC. Por otra parte, como
AC
CD = CM , entonces CM
= CM
aloga llegamos a que M N es
N C y de manera an
perpendicular a AC. Por lo tanto, L, M y N son colineales.
Solucion del problema 10. Sean
a=xy

b=

2xy
.
xy

4
. Ademas, x > y > 0 implica que a > 0 y b > 0.
Como xy = 2, tenemos que b = xy
Luego, aplicando la desigualdad MA-MG (ver en el apendice el teorema 4), tenemos
que

x2 + y 2
2xy
=xy+
= a + b 2 ab = 2
xy
xy

(x y)

4
xy

= 2 4 = 4,

y la igualdad se cumple si y solo si a = b. Observemos que el menor valor posible


2
+y 2
se da cuando se cumple la igualdad, es decir, si existen numeros x, y, con
de xxy
x > y > 0, tales que,
xy = 2

De la igualdad xy = 2, tenemos que y =

xy =
2
x.

4
.
xy

Sustituyendo esta u ltima igualdad en

18
xy =

Soluciones a los problemas de practica


2xy
xy

y simplificando, tenemos que


4
=
x2
(x2 )2 8x2 + 4 =
x2 8 +

x
Tomando la solucion x =
que x > y > 0, xy = 2 y

0
0
8

64 16
= 4 2 3.
2

4 + 2 3 = 3+1 y y =
x2 +y 2
xy

2
x

2
3+1

3 1, tenemos

= 4. Por lo tanto, el valor mnimo buscado es 4.

Solucion del problema 11. Sea I el incentro de ABC y D el centro del semicrculo
sobre BC. Si desde I y D trazamos las alturas sobre AB y AC, entonces sus longitudes son r y ra , respectivamente. Sean a, b y c las longitudes de BC, AC y AB,
respectivamente, y usemos la notacion (ABC) para representar al a rea del triangulo
ABC.
A

Es facil ver que


2(ABD) = cra ,

2(ACD) = bra ,

entonces 2(ABC) = (b + c)ra . Ademas,


2(IAB) = cr,

2(IAC) = br.

Razonando de manera analoga a partir de los semicrculos sobre AB y AC obtenemos,


2(ABC) = (c + a)rb = (a + b)rc

2(IBC) = ar.

Como (IAB) + (IAC) + (IBC) = (ABC), tenemos que 2(ABC) = (a + b + c)r,


de donde se sigue
(b + c)ra = (c + a)rb = (a + b)rc = (a + b + c)r.
Usando lo anterior, tenemos que

(a + b + c) rra + rrb + rrc = (b + c) + (c + a) + (a + b) = 2(a + b + c).

Soluciones a los problemas de practica

19

Por u ltimo, dividiendo la ecuacion anterior entre r(a + b + c) obtenemos el resultado


deseado.
Solucion del problema 12. En un primer acercamiento contaremos todas las formas
de factorizar 1, 000, 000 = abc, considerando que (a, b, c) es distinto de (a, c, b) etc.,
asimismo consideraremos los casos donde uno o mas factores son iguales a 1. A este
primer acercamiento le llamaremos la cuenta burda.
Usando la factorizacion en primos del numero 1, 000, 000 = 26 56 y sabiendo que

n+k1
nos permite calcular el numero de formas de acomodar n objetos identicos
k1

en k casilleros4 , tenemos que hay 82 = 28 formas de distibuir los seis factores 2 en
tres casilleros o grupos (a, b y c). Lo mismo ocurre para los factores 5. Entonces la
cuenta burda es igual a 282 = 784. Ahora, procederemos a refinar nuestro calculo con
la ayuda de un analisis por casos:
Caso 1 (a = b = c). Este caso es u nico.
Caso 2 (a = b 6= c). Como a = b, tenemos que a debe ser de la forma 2m 5n ,
donde 0 m, n 3. Hay (4)(4) = 16 casos disintos de este tipo, siendo uno
de ellos el caso a = b = c. Observemos que cada uno de los otros 15 casos fue
contado 3 veces en la cuenta burda.
Caso 3 (a 6= b 6= c 6= a). Si quitamos de la cuenta burda las ternas de los casos
1 y 2, nos quedan 784 1 (15)(3) = 738 ternas de este tipo, mismas que
corresponden a un numero menor de factorizaciones escencialmente distintas,
pero que fueron contadas 6 veces cada una. Por lo tanto, hay 738
6 = 123 casos
distintos de este tipo.
Considerando el analisis anterior, solo hay 1 + 15 + 123 = 139 factorizaciones distintas, pero aun debemos eliminar los casos donde uno o dos de los factores son 1.
Para facilitar la cuenta, supondremos, si perdida de generalidad, que a = 1; es decir
que bc = 26 56 . Por la observacion hecha en el primer parrafo, es facil ver que hay
7
1 = 7 formas de distribuir los factores 2 y lo mismo sucede para los factores 5, lo
que nos da un total de 49 casos. De ellos, uno es el caso b = c y los otros 48 estan
formados por 24 factorizaciones escencialmente distintas, pero que han sido contadas
doble. Por lo tanto el numero de casos distintos que tienen factores 1 es 24 + 1 = 25.
Finalmente concluimos que la solucion es 139 25 = 114.
Solucion del problema 13. Sean a, ar y ar2 los numeros de la terna. Como son numeros enteros tenemos que a y r son numeros racionales. Sea r = xy la razon geometrica,
2
donde x, y son primos relativos. Dado que a xy es un entero y mcd(x, y) = 1, entonces existe un entero n tal que a = ny 2 . Como a + ar + ar2 = 111 tenemos que
n y 2 + xy + x2 = 111 = 3 37. De lo anterior se concluye que y 2 + xy + x2 es
3, 37 o 111.
4 Para ver esto, considere que los n objetos son puestos en una hilera junto con (k 1) objetos distintos
intercalados entre ellos y que sirven como separadores. El resultado anterior se obtiene facilmente al contar
el numero de formas de colocar los separadores.

20

Soluciones a los problemas de practica


Consideremos la siguiente tabla donde se muestran todos los valores para y 2 + xy + x2 ,
donde x toma el valor de la columna y y el del renglon, con y x.

1
2
3
4
5
6

1
3

2
7
12

3
13
19
27

4
21
28
37
48

5
31
39
49
61
75

6
43
52
63
76
91
108

7
57
67
79
93

8
73
84
97

9
91
103

10
111

De aqu obtenemos las soluciones: x = y = 1, n = 37; x = 10, y = 1, n = 1; x = 4,


y = 3, n = 3. Para obtener todas las posibilidades basta completar con las soluciones
para el caso y > x. De lo anterior concluimos que las ternas buscadas son: (37, 37, 37),
(1, 10, 100), (100, 10, 1), (27, 36, 48) y (48, 36, 27).
Solucion del problema 14. Comencemos por la segunda parte y veamos que cualquier
coleccion de 18 segmentos con longitudes menores o iguales que 2010 siempre contendra una terna con la que es posible construir un triangulo.
Procedamos por contradiccion y supongamos que existe una coleccion de 18 segmentos m1 m2 m18 2010 tal que con ninguna terna es posible construir un
triangulo. Entonces, por la desigualdad del triangulo (ver en el apendice el teorema 9),
tenemos que
m1 1, m2 1, m3 m1 +m2 2, m4 m2 +m3 3, m5 m3 +m4 5, . . .
Si observamos la sucesion formada por los terminos derechos de estas desigualdades,
descubriremos que se trata de la sucesion de Fibonacci (ver en el apendice la definicion 5). Deducimos entonces que, para toda i {1, 2, . . . , 18}, mi Fi . En particular
tenemos que m18 F18 = 2584, lo que es una contradiccion.
Lo anterior, tambien nos ayuda a resolver la primera parte del problema, pues si tomamos la coleccion de segmentos con longitudes F1 , F2 , . . . , F17 , dada cualquier terna (Fi , Fj , Fk ) (sin perdida de generalidad supondremos que i < j < k), tenemos
que Fk = Fk1 + Fk2 Fj + Fi , y por lo tanto no es posible construir un
triangulo con ellos. Notese que esta coleccion de segmentos cumple con todos los
requerimientos del problema ya que para toda s {1, 2, . . . , 17}, se cumple que
Fs F17 = 1597 < 2010.
Solucion del problema 15. Como 11n 1n 1 (mod 10), cualquier potencia de 11
termina en 1. Es as, que nos interesan aquellas potencias que terminan en 11, o sea,
debemos caracterizar los numeros naturales n que satisfacen 11n 11 (mod 100).
Aplicando el teorema del binomio (ver en el apendice el teorema 6), tenemos que
11n = (10 + 1)n =

n  
X
n

k=0

10k 1 + 10n (mod 100),

pues a partir de k = 2 todos los terminos de la suma son multiplos de 100. Luego,

Soluciones a los problemas de practica

21

11n 11 (mod 100) 10n 10 (mod 100) n 1 (mod 10).


Con lo que hemos probado que 11n tiene sus u ltimos dgitos iguales si y solo si n
termina en 1.
Supongamos ahora que los tres u ltimos dgitos de 11n son iguales. Por lo anterior, n es
de la forma 10k + 1 y 11n 111 (mod 1000). Empleando nuevamente el teorema del
binomio tenemos


102 + 10n + 1 (mod 1000),
111 n2 102 + n1 10 + 1 = n(n1)
2
por lo que

111 5k(10k + 1)102 + 10(10k + 1) + 1 600k + 11 (mod 1000),


de donde se sigue que
100 600k (mod 1000) y 1 6k (mod 10),
lo que, por paridad, es imposible. En consecuencia ninguna potencia de 11 puede tener
sus tres u ltimos dgitos iguales.
Solucion del problema 16. Sea d = b a. Entonces hay (a + d)2 a2 + 1 =
2ad + d2 + 1 enteros en consideracion, de los cuales d + 1 son cuadrados perfectos.
Luego, necesitamos que 100(d + 1) = 2ad + d2 + 1, de donde
a=

100(d + 1) d2 1
100 d 99
=
+ .
2d
2
2d

Si d es par, el primer termino es un entero y el segundo termino no lo es. Luego, d debe


ser impar. Entonces, el primer termino es una fraccion con denominador igual a 2, de
modo que el segundo termino debe ser tambien una fraccion con denominador igual a
2. Esto significa que d debe ser un divisor de 99, es decir, d = 1, 3, 9, 11, 33 o 99.
Si d = 1, entonces a =

99
2

99
2

= 99 y b = 99 + 1 = 100.

Si d = 3, entonces a =

97
2

99
6

= 65 y b = 65 + 3 = 68.

Si d = 9, entonces a =

91
2

99
18

= 51 y b = 51 + 9 = 60.

Si d = 11, entonces a =

89
2

99
22

= 49 y b = 49 + 11 = 60.

Si d = 33, entonces a =

67
2

99
66

= 35 y b = 35 + 33 = 68.

Si d = 99, entonces a =

1
2

99
198

= 1 y b = 1 + 99 = 100.

Solucion del problema 17. Dado que BB1 k A1 F y F C1 k BC, tenemos que
AC1 B1 = ABC y AGB1 = AA1 F . Luego, los a ngulos ABC y AA1 F son
iguales si y solo si el cuadrilatero AB1 GC1 es cclico.

22

Soluciones a los problemas de practica


A

B1

C1
G
D

A1

Ademas, como B1 CA1 C1 es un paralelogramo, los a ngulos ACB y F C1 A1 son iguales. Entonces ACB = F AA1 si y solo si el cuadrilatero AF A1 C1 cclico, si y solo
si AB1 GC1 es cclico.
Otra manera de terminar. Ya que F A1 BB1 es un paralelogramo, tenemos que B1 F =
BA1 = B1 C1 . De lo anterior se tiene que AF CC1 es un paralelogramo ya que sus
diagonales se bisecan mutuamente. Sea D el punto de interseccion de A1 F y CC1 .
Luego, AF A1 = F DC = B1 GC, y de aqu AF A1 = BAC si y solo si
AB1 GC1 es cclico.
Solucion del problema 18. Tenemos que
(a + 1)a+1 = (a + 1)a (a + 1) = a(a + 1)a + (a + 1)a
para todo numero a. Hagamos a = 33t para un entero positivo arbitrario t. Entonces,
3t1

a = (3t )3 y (a + 1)a = ((33t + 1)3

)3

son ambos cubos de enteros.


3t1
3t1
Si hacemos k = 33t + 1, m = 3t (33t + 1)3
y n = (33t + 1)3
, entonces
k k = m3 + n3 . Como t es un entero positivo arbitrario, las posibilidades para k son
infinitas.
Solucion del problema 19. Sean E, F , E , F , L y M los puntos medios de los segmentos AD, DC, CB, BA, CK y AK respectivamente. Sean tambien, P el punto de
interseccion de las rectas CK y EE , y Q el punto de interseccion de las rectas AK y
F F . Tenemos que los segmentos M F y N B, as como F F y BC son iguales y paralelos. Entonces, los triangulos BCN y F F M son congruentes y los a ngulos BCN
y F F M son iguales. Ademas AF F = ABC.
F

A
M

P
L

B
N

Soluciones a los problemas de practica

23

Puesto que AQF es un a ngulo exterior del triangulo AF Q, se tiene que


AQF = AF Q + QAF = ABC + BAN + N AK
Por otro lado como F esta a la misma distancia de A y de K, F M es la mediatriz de
AK. Luego,
QF M + M QF = BCN + ABC + BAN + N AK = 90

(1)

Analogamente, para el triangulo EE L


P EL + EP L = BAN + ABC + BCN + N CK = 90

(2)

Finalmente, de (1) y (2), se sigue que N AK = N CK.


Solucion del problema 20. Sean 2 a 100 y 2 b 100 enteros distintos y
n
n
supongamos que el numero a2 + b2 es primo para todo entero positivo n. Consideremos el numero primo 257 = 28 + 1. Como 257 es primo relativo con a y con b,
tenemos por el pequeno teorema de Fermat (ver en el apendice el teorema 3),
8

a2 = a2571 1 (mod 257) y b2 = b2571 1 (mod 257).


8

Luego, a2 b2 (mod 257) y por lo tanto


8

a2 b2 = (a b)(a + b)(a2 + b2 ) (a2 + b2 )


es multiplo de 257. Como 2 a 100 y 2 b 100, es claro que a b y a + b no
k
k
son multiplos de 257. Luego, 257 divide a a2 + b2 para algun entero 1 k 7. Ya
k
k
k
k
que a2 + b2 es primo por hipotesis, se sigue que a2 + b2 = 257. En particular, 257
es suma de dos cuadrados. Es facil ver que las u nicas formas de escribir a 257 como
suma de dos cuadrados de enteros positivos son 12 + 162 = 257 y 162 + 12 = 257,
k
k
de modo que alguno de los numeros a2 o b2 es igual a 1. Esto es una contradiccion
ya que a y b son enteros mayores que 1. Por lo tanto, hay un entero positivo n tal que
n
n
a2 + b2 no es primo.

24

Soluciones a los problemas de practica

Problemas propuestos

Problemas propuestos.
2010 No. 4.
Ano
Tzaloa se construye con la contribucion de todos y esta seccion esta especialmente
disenada para que sus lectores tengan un espacio de participacion. A continuacion,
presentamos 5 problemas nuevos que necesitan de ti para encontrar su respuesta.
A partir de este numero y con el fin de dar mas tiempo a que nuestros lectores puedan
enviar sus soluciones, las respuestas de los problemas propuestos en cualquier numero
de la revista, se publicaran con tres numeros de diferencia. Es as, que los problemas
propuestos en esta ocasion, se publicaran en Tzaloa No. 3, ano 2011, por lo que aprovecha ahora que tienes mas tiempo y envanos tus contribuciones cuanto antes.
Ponemos a tu disposicion nuestra direccion electronica revistaomm@gmail.com
ya que a traves de ella estaremos recibiendo con gusto todas las soluciones que nos
lleguen desde cualquier rincon del pas.
Problema 1. (Introductorio) Un entero positivo n se llama completo si satisface la
siguiente propiedad: Si a es un dgito de n, entonces el numero 9 a tambien es un
dgito de n. Por ejemplo, el numero 465, 930 es completo, mientras que el numero
3, 671 no lo es. Cuantos numeros completos hay entre 1 y 106 ?
(Nota: El numero 9 se considera completo ya que se puede escribir como 09).
Problema 2. (Intermedio) Sean a1 , a2 , . . . , a8 ocho enteros distintos cualesquiera escogidos del conjunto A = {1, 2, . . . , 16, 17}. Demuestra que existe un entero k > 0 tal
que la ecuacion ai aj = k tiene al menos 3 soluciones diferentes. Ademas encuentra
un subconjunto de A con 7 elementos tal que la ecuacion ai aj = k no tenga tres
soluciones distintas para ningun valor de k > 0.
Problema 3. (Intermedio) En un triangulo ABC la mediana y la altura desde el vertice
A dividen al a ngulo BAC en tres a ngulos de a reas iguales. Determina las medidas
de los a ngulos del triangulo ABC.

26

Problemas propuestos

Problema 4. (Avanzado) Sean a, b, c y d numeros reales tales que a2 +b2 +c2 +d2 1.
Determina el valor maximo de la suma
(a + b)4 + (a + c)4 + (a + d)4 + (b + c)4 + (b + d)4 + (c + d)4 .

Problema 5. (Avanzado) Sea n un entero positivo. Demuestra que


S(2n)
S(n) 5 S(2n)
2
donde S(n) denota la suma de los dgitos de n.
Demuestra tambien que existe un entero positivo n tal que
S(n) = 2010 S(3n).

Soluciones a los problemas propuestos.


2010 No. 2.
Ano
A continuacion publicamos las soluciones de los problemas propuestos que aparecieron
en Tzaloa 2, ano 2010. Recuerda que esta revista necesita de ti y ten la seguridad que en
el proximo numero nos encantara poder publicar tus soluciones. En esta ocasion nos
da mucho gusto felicitar a Daniel Antonio Martnez Munoz, del estado de Chihuahua,
quien nos envio la solucion del problema 1; y a Francisco Gomez Hernandez, del estado
de Hidalgo, quien nos envio la solucion del problema 3.

3
xy
=
, calcula el valor de
Problema 1. (Introductorio) Si se sabe que 2
x + y2
6
4 4
x
x
+
.
y
y

Solucion de Daniel Antonio Martnez Munoz.


Buscamos el valor de ( xy )4 + ( xy )4 .
2
la expresion anterior por xy,
Sabemos que 6xy = 3(x + y 2 ), luego si dividimos

obtenemos, 6 = 3( xy ) + 3( xy ) = 3 xy + xy , entonces tenemos que, xy + xy =

2 3. Llamemos a = xy y b = xy , entonces buscamos a4 + b4 .

Tenemos que, (a + b)4 = (24 ) ( 3)4 = 16 9 = 144.


Ademas, (a + b)4 = a4 + 4a3 b + 6a2 b2 + 4ab3 + b4 . Pero ab = 1, entonces 6a2 b2 = 6,
luego (a+b)4 = a4
+b4 +4ab(a2 +b2 )+6 = 144, de donde a4 +b4 = 1384(a2 +b2 ).
Pero (a + b)2 = (2 3)2 = a2 + b2 + 2ab, entonces a2 + b2 = 10.

Problemas propuestos

27

Luego, a4 + b4 = 138 4(a2 + b2 ) = 138 4(10) = 138 40 = 98. Por lo tanto,


a4 + b4 = 98, que es el valor que buscabamos.
Problema 2. (Introductorio) Los numeros 1, 2, 3, . . . , 24, 25 se han escrito en las casillas de un tablero cuadrado de 55, de tal forma que los numeros en cada renglon estan
ordenados en forma creciente de izquierda a derecha. Halla el maximo valor posible de
la suma de los numeros que estan en la tercera columna.
Solucion. Observemos que dado que los numeros en cada fila deben estar ordenados en
forma creciente de izquierda a derecha, tenemos que hacer que la suma de los numeros
de la tercera y cuarta columna difieran lo menos posible, as como los de la cuarta y
quinta columna. Un ejemplo es:
1
3
5
7
9

2
4
6
8
10

23
20
17
14
11

24
21
18
15
12

25
22
19
16
13

donde los numeros de la tercera columna suman 85.


Demostraremos que la suma maxima para los numeros de la tercera columna es 85.
Sea Ai la suma de los numeros de la i-esima columna, con 1 i 5. Debemos
demostrar que A3 85. Si a y b son dos numeros en el mismo renglon, con a en la
tercera columna y b en la cuarta, entonces a < b, o de manera equivalente, a + 1
b. Aplicando esta desigualdad a todos los numeros de esas columnas y sumando los
resultados obtenidos tenemos que A3 + 5 A4 y A4 + 5 A5 . Luego, A3 + 10 A5 ,
y por lo tanto,
(A3 + 10) + (A3 + 5) + A3 = 3A3 + 15 A3 + A4 + A5 .
Por otro lado, A3 + A4 + A5 representa la suma de los 15 numeros de las tres u ltimas
columnas, pero dicha suma es menor o igual que la suma de los 15 numeros mayores
que estan escritos en el tablero, es decir,
A3 + A4 + A5 25 + 24 + 23 + + 12 + 11 = 270.
Luego,
3A3 + 15 A3 + A4 + A5 270,
de donde A3 85, que es lo que queramos demostrar.
Problema 3. (Intermedio) Si x es un numero real tal que x2 +
valores posibles de la expresion x5 + x15 .

1
x2

= 7, determina los

Solucion de Francisco Gomez Hernandez. Recordemos la siguiente factorizacion:


a5 + b5 = (a + b)(a4 a3 b + a2 b2 ab3 + b4 ).

28

Problemas propuestos

Ahora, ocupando esta factorizacion con a = x y b = x1 , tenemos que,


3 4


2
1
1
1
1
5
4
3 1
2 1
x + 5 =
x
+
x x
x+
+x
x
x
x
x
x
x

1
1
1
4
2
=
x+
x + 4 x + 2 +1 .
x
x
x
Pero x2 +
x4 +

1
x2
1
x4

= 7, y podemos calcular el valor de x4 +


= x4 + 2 +

1
x4

Por lo tanto,
x5 +

1
x5

2 = x2 +


1 2
x2

1
x4 ,

2 = 72 2 = 49 2 = 47.

1
1
1
x4 + 4 x2 + 2 + 1
x
x
x

1
((47) (7) + 1)
=
x+
x

1
.
= 41 x +
x

x+

De aqu, si conocemos todos los valores posibles de x +


valores posibles de x5 + x15 . Ahora recordemos que,
x2 +

entonces

1
x

obtendremos todos los

1
1
1
1 2
2
=
9

x
+
=
7

x
+
2
+
=
x
+
= 3.
x2
x2
x
x

Entonces todos los valores posibles de la expresion son 3(41) = 123.


Problema 4. (Intermedio) Sean 1 y 2 dos circunferencias que no se intersectan,
tienen radios distintos y son tangentes interiormente a una circunferencia 3 en los
puntos A y B, respectivamente. Se traza la recta l tangente comun a 1 y 2 , tal como
se muestra en la figura. Demuestra que las rectas AB, l y la que pasa por los centros de
1 y 2 , son concurrentes. (Problema sugerido por Irving Daniel Calderon Camacho).
A

Solucion. Sea Oi el centro de i para i = 1, 2, 3. Sean P 6= A el punto de interseccion


de AB con 1 , y C, D los puntos de tangencia de l con 1 y 2 , respectivamente.

Problemas propuestos

29

O1

O2
b

D
C

O3

Veamos que los triangulos O1 CP y O2 DB son isosceles y semejantes. Son isosceles


ya que dos de sus lados son radios de 1 y 2 , respectivamente. Para ver que son
semejantes basta probar que P O1 C = BO2 D.
El triangulo AO3 B es isosceles, por lo que BAO3 = O3 BA, y como el triangulo
AO1 P tambien es isosceles, concluimos que O1 P A = BAO3 = O3 BA, por
lo que O1 P y O2 B son paralelas. Por otra parte, O1 C es paralela a O2 D ya que son
perpendiculares a l. Luego, tenemos que P O1 C = BO2 D y los triangulos O1 CP
y O2 DB son semejantes. Ademas, P C es paralela a BD.
Finalmente, como los triangulos O1 CP y O2 DB tienen sus respectivos lados paralelos y son de tamano distinto, por el teorema de Desargues (ver en el apendice la
definicion 21 y el teorema 22) sabemos que los triangulos estan en perspectiva desde
un punto. Entonces, CD, O1 O2 y AB concurren en dicho punto.
Problema 5. (Avanzado) Un entero n > 1 tiene la siguiente propiedad: para cada
divisor positivo d de n, d + 1 es un divisor de n + 1. Demuestra que n es un numero
primo.
Solucion. Sea p el menor divisor primo de n, y sea d =
np+p
n+p

p(n+1)
p(d+1)

n+1
d+1

n
p.

Entonces, por hipotesis,

tenemos que
=
=
es un entero. Como n + p tambien divide a
2
np + p = p(n + p), entonces n + p debe dividir a la diferencia (np + p2 ) (np + p) =
p2 p. Luego, n + p p2 p, ya que p2 p es positivo. Por lo tanto, n < p2 y de
2
aqu d = np < pp = p. Ahora, si d tuviera un divisor primo q, entonces q d < p.
Como q tambien divide a n, entonces por la minimalidad de p tenemos que q p, lo
que es una contradiccion. Luego, d no tiene divisores primos y por lo tanto d = 1. As,
n = p es primo.

30

Problemas propuestos

Olimpiadas Internacionales
A continuacion presentamos los resultados y los examenes de las distintas olimpiadas
internacionales en las que Mexico participo en este ano 2010.

51a Olimpiada Internacional de Matematicas


Del 2 al 14 de julio de 2010, se llevo a cabo la 51a Olimpiada Internacional de Matematicas en Astana, Kazajstan, con la participacion de 517 competidores provenientes
de 96 pases.
El desempeno de Mexico en esta olimpiada internacional fue muy bueno ya que logro colocarse en el primer tercio de la lista de pases participantes, segundo lugar de los pases
iberoamericanos, por encima de Argentina, Espana y Brasil. Ademas, por primera ocasion, cinco de los seis integrantes de la delegacion mexicana obtuvieron medalla.
La delegacion mexicana estuvo integrada por Daniel Perales Anaya (Morelos), Flavio
Hernandez Gonzalez (Aguascalientes), Jose Luis Miranda Olvera (Jalisco), Irving Daniel Calderon Camacho (Estado de Mexico), Diego Alonso Roque Montoya (Nuevo
Leon), y Manuel Enrique Dosal Bustillos (Chihuahua). Todos ellos alumnos menores
de 18 anos y Diego, el mas joven de los participantes de Mexico, con tan solo 14 anos
de edad. Daniel se vio galardonado con una medalla de plata; Flavio, Jose Luis, Irving
Daniel y Diego obtuvieron, cada uno, una medalla de bronce; y Manuel recibio una
mencion honorfica.
A continuacion presentamos el examen de la 51a Olimpiada Internacional de Matematicas. Los alumnos tuvieron dos sesiones de 4.5 horas cada una para resolverlo.
Problema 1. Determine todas las funciones f : R R tales que
f (xy) = f (x)f (y)
para todos los numeros x, y R. (z denota el mayor entero que es menor o igual
que z.)

32

51a Olimpiada Internacional de Matematicas

Problema 2. Sea ABC un triangulo, I su incentro y su circunferencia circunscrita.


y F un punto
La recta AI corta de nuevo a en D. Sean E un punto en el arco BDC
en el lado BC tales que
BAF = CAE <

1
BAC.
2

Sea G el punto medio del segmento IF . Demuestre que las rectas DG y EI se cortan
sobre .
Problema 3. Sea N el conjunto de los enteros positivos. Determine todas las funciones
g : N N tales que
(g(m) + n)(m + g(n))
es un cuadrado perfecto para todo m, n N.
Problema 4. Sea la circunferencia circunscrita al triangulo ABC y P un punto en
el interior del triangulo. Las rectas AP , BP y CP cortan de nuevo a en los puntos
K, L y M , respectivamente. La recta tangente a en C corta a la recta AB en S. Si se
tiene que SC = SP , demuestre que M K = M L.
Problema 5. En cada una de las seis cajas B1 , B2 , B3 , B4 , B5 , B6 hay inicialmente
solo una moneda. Se permiten dos tipos de operaciones:
Elegir una caja no vaca Bj , con 1 j 5. Retirar una moneda de Bj
y anadir dos monedas a Bj+1 .
Tipo 2: Elegir una caja no vaca Bk , con 1 k 4. Retirar una moneda de Bk
e intercambiar los contenidos de las cajas (posiblemente vacas) Bk+1
y Bk+2 .
Determine si existe una sucesion finita de estas operaciones que deja a las cajas B1 , B2 ,
2010
B3 , B4 , B5 vacas y a la caja B6 con exactamente 20102010
monedas. (Observe que
c
c
ab = a(b ) .)
Tipo 1:

Problema 6. Sea a1 , a2 , a3 , . . . una sucesion de numeros reales positivos. Se tiene que


para algun entero positivo s,
an = max{ak + ank tal que 1 k n 1}
para todo n > s. Demuestre que existen enteros positivos l y N , con l s, tales que
an = al + anl para todo n N .

Problemas y Soluciones de
Olimpiadas Internacionales

XXII Olimpiada de la Cuenca del Pacfico


Desde 1991, los ganadores del Concurso Nacional participan anualmente en la Olimpiada Matematica de la Cuenca del Pacfico, APMO, por sus siglas en ingles. En Mexico, el 8 de marzo de este ano, se aplico el examen de la XXII Olimpiada Matematica
de la Cuenca del Pacfico a los alumnos que en ese momento formaban parte de la
preseleccion nacional. Dicho examen se califico en Mexico y los 10 mejores examenes se enviaron, para ser evaluados, al pas organizador que en esta ocasion fue Japon.
Los alumnos que obtuvieron medalla fueron: Daniel Perales Anaya (Morelos) y Flavio
Hernandez Gonzalez (Aguascalientes) obtuvieron medalla de plata; Fernando Josafath
Anorve Lopez (Nuevo Leon), Jose Luis Miranda Olvera (Jalisco), Irving Daniel Calderon Camacho (Estado de Mexico), Jose Ramon Guardiola Espinosa (San Luis Potos) y Julio Cesar Daz Calderon (Oaxaca), obtuvieron medalla de bronce; Fernando
Ignacio Arreola Gutierrez (Aguascalientes), obtuvo una mencion honorfica. Mexico
ocupo el lugar numero 15 de los 33 pases participantes.
A continuacion presentamos el examen con sus soluciones de la XXII Olimpiada de la
Cuenca del Pacfico. Los alumnos tuvieron 4 horas para resolverlo.
Problema 1. Sea ABC un triangulo con BAC 6= 90 . Sea O el circuncentro del
triangulo ABC y sea el circuncrculo del triangulo BOC. Suponga que intersecta
a los segmentos AB y AC en los puntos P (diferente de B) y Q (diferente de C), respectivamente. Sea ON el diametro del crculo . Muestra que el cuadrilatero AP N Q
es un paralelogramo.
Solucion de Julio Cesar Daz Calderon. Como CAB 6= 90 , entonces C, O y B
no son colineales, luego el triangulo OBC no es degenerado.

34

XXII Olimpiada de la Cuenca del Pacfico


A

O
Q

O
b

Llamemos A al punto medio del lado BC, O al centro de , BAC = , ABO =


y ACO = . Como BOC es un a ngulo central y es un a ngulo inscrito, tenemos
que BOC = 2. Como el triangulo OBC es isosceles (pues OB = OC al ser radios
del circuncrculo) tenemos que la mediatriz OA es tambien altura del triangulo BOC
y bisectriz del a ngulo BOC, entonces BOO = O OC = BOC
= 2
2
2 = .
Llamemos Criterio 1, al hecho de que la suma de los a ngulos interiores de un triangulo
es 180 . Entonces, por el Criterio 1 aplicado a los triangulos OBA y OA C, y porque
OA B = OA C = 90 y BOA = A OC = , tenemos que OBA =
OCA = 90 .
Ahora, P BO = P N O = y ON Q = OCQ = por abrir el mismo arco en
, entonces P N Q = P N O + ON Q = + .
Tenemos que,
BAC + ACB + CBA

180

+ ( + 90 ) + (90 + )
+ 180 +

=
=

180
180

entonces, P N Q = .
Finalmente, N OB = N P B, por abrir el mismo arco, y N OB = , entonces
N P B = . Luego, = QN P = N P B, entonces AP es paralela a QN .
Tambien, como = N P B = QAP , tenemos que QA es paralela a P N . Por lo
tanto, AP N Q es un paralelogramo.
Problema 2. Dado un entero positivo k, decimos que un entero es una potencia ke sima pura si puede ser representado como mk para algun entero m. Muestra que para
cada entero positivo n existen n enteros positivos distintos tales que su suma es una
potencia 2009-esima pura, y su producto es una potencia 2010-esima pura.

XXII Olimpiada de la Cuenca del Pacfico

35

Solucion. Por simplicidad, tomemos k = 2009. En primer lugar escojamos, convenientemente, n enteros positivos distintos b1 , b2 , . . . , bn de manera que su producto sea una (k + 1)-esima potencia pura. Por ejemplo, si hacemos bi = ik+1 para
i = 1, 2, . . . , n, tenemos que b1 b2 bn = tk+1 , para algun entero positivo
2
t. Sea b1 + b2 + + bn = s. Ahora, tomemos ai = bi sk 1 para i = 1, 2, . . . , n.
Probaremos que a1 , a2 , . . . , an satisfacen las condiciones del problema.
Como b1 , b2 , . . . , bn son enteros positivos distintos, entonces es claro que a1 , a2 , . . . , an
tambien lo son. Ademas:
a1 + a2 + + an = sk

(b1 + b2 + + bn ) = sk = (sk )2009 ,

y
a1 a2 an

=
=

sk

sk

1
1

b1 b2 bn
tk+1
n

s(k+1)(k1) tk+1
2010
s2008n t
.

=
=

Problema 3. Sea n un entero positivo. En cierta fiesta asisten n personas. Para cualquier par de participantes, o los dos se conocen entre ellos o los dos no se conocen entre
ellos. Encuentra el maximo numero posible de parejas tal que en cada pareja, las dos
personas no se conocen entre s pero existe un amigo en comun entre los participantes
de la fiesta.

Solucion de Fernando Josafath Anorve


Lopez. Diremos que las parejas que no se
conocen pero que tienen un amigo en comun son buenas. Tomemos a una persona
de la fiesta y supongamos que conoce a todos, pero que los demas no se conocen

entre s, entonces con estas n 1 personas, podemos formar n1
= (n1)(n2)
2
2
parejas buenas. Con n personas se pueden formar en total n(n1)
parejas,
por
lo
tanto
2
n(n1)
(n1)(n2)
si demostramos que hay al menos

= n 1 parejas no buenas,
2
2
habremos demostrado que el maximo numero de parejas buenas es (n1)(n2)
.
2
Ahora, supongamos que hay una persona que conoce a x personas y desconoce a las
demas, entonces hay x parejas no buenas.

s conoce
no conoce
b

36

XXII Olimpiada de la Cuenca del Pacfico

Por cada persona que no conoce, existe la posibilidad de que esa pareja sea buena, o
que no lo sea.
Si la pareja es buena, entonces deben tener un amigo en comun, luego ya tenemos otra
pareja no buena.

Si la pareja no es buena, no hay nada que hacer. Luego, por cada persona a la cual
no conoce, hay una pareja no buena, y ademas con las personas que s conoce forma
parejas no buenas. Por lo tanto, hay mnimo n 1 parejas no buenas.
Problema 4. Sea ABC un triangulo acutangulo que satisface que AB > BC y que
AC > BC. Sean O y H el circuncentro y el ortocentro del triangulo ABC, respectivamente. Suponga que el circuncrculo del triangulo AHC intersecta a la recta AB
en M (diferente de A), y suponga que el circuncrculo del triangulo AHB intersecta a
la recta AC en N (diferente de A). Muestra que el circuncrculo del triangulo M N H
esta sobre la recta OH.

Solucion de Daniel Perales Anaya. Sean O1 , O2 , O3 , O4 , O5 y O6 los circuncentros


de los triangulos AHB, AHC, BHC, BM H, CHN y M HN , repectivamente.
Notemos que ABH = 90 BAC = ACH = . Como el cuadrilatero ABHN
es cclico, tenemos que HN C = , y como el cuadrilatero AM HC es cclico tenemos que BM H = . Por lo tanto, los triangulos BHM y HN C son isosceles, y
como CH es perpendicular a M B y BH es perpendicular a N C, entonces O4 esta en
la lnea CH y O5 esta en la lnea BH.
entonces el circunradio del
Notemos que como ABH = ACH y abren el arco AH,
triangulo AHC es igual al circunradio del triangulo AHB. Analogamente, podemos
probar que el circunradio del triangulo BHC es igual a los dos anteriores.
Notemos que O1 , O4 y O3 son colineales por estar en la mediatriz de BH. Ademas,
como el circunradio del triangulo AHB es igual al del triangulo BHC, entonces
O1 B = BO3 = O3 H = HO1 , es decir, O1 BO3 H es un rombo. Entonces, BH
es la mediatriz de O1 O3 . Analogamente, O2 , O3 y O5 son colineales (mediatriz de
CH) y como O2 HO3 C es un rombo, entonces CH es la mediatriz de O2 O3 . Luego,
en el triangulo O1 O2 O3 , H es el circuncentro, ya que esta en la mediatriz de O1 O3 y
de O2 O3 .
Ademas, O2 , O4 y O6 son colineales por estar en la mediatriz de M H, y O1 , O5 y O6
son colineales por estar en la mediatriz de N H.
Como AB es perpendicular a CH y O2 O3 es perpendicular a CH, tenemos que AB

XXII Olimpiada de la Cuenca del Pacfico

37

es paralela a O2 O3 . Analogamente, como AC es perpendicular a BH y O1 O3 es perpendicular a BH, tenemos que AC es paralela a O1 O3 .


A

M
b

O2

O1
b

O6

O4

O
b

G
b

O5

H
Q

P
b

O3

Como O1 O es perpendicular a AB, entonces O1 O es perpendicular a O2 O3 , de donde O1 O es la altura del triangulo O1 O2 O3 . Analogamente, O2 O es perpendicular a
O1 O3 , y O2 O es altura del triangulo O1 O2 O3 . Por lo tanto, O es el ortocentro de
dicho triangulo.
Denotemos por P a la interseccion de O2 O3 y CH y por Q a la interseccion de O1 O3
y BH. Sabemos que, O3 P = P O2 y O3 Q = QO1 . Consideremos las rectas O1 O4 Q y
O5 O2 P , luego por Papus (tomando en orden los puntos O1 , O5 , Q, O2 , O4 , P ) tenemos

38

XXII Olimpiada de la Cuenca del Pacfico

que O6 = O1 O5 O2 O4 , H = O5 Q O4 P y G = QO2 P O1 son colineales.


Ahora bien, en el triangulo O1 O2 O3 tenemos que G es el gravicentro (pues es la interseccion de dos medianas), H es el circuncentro y O es el ortocentro, luego O6 esta en
la recta de Euler del triangulo O1 O2 O3 . Por lo tanto, O6 , que es el circuncentro del
triangulo M HN , esta en la recta OH.
Problema 5. Encuentra todas las funciones f del conjunto R de los numeros reales a
R que satisfacen para x, y, z R la identidad
f (f (x) + f (y) + f (z)) = f (f (x) f (y)) + f (2xy + f (z)) + 2f (xz yz).

Solucion. Es claro que si f es una funcion constante que satisface la ecuacion dada,
entonces la constante debe ser 0. Recprocamente, f (x) = 0 claramente satisface la
ecuacion dada, de modo que, la funcion identicamente 0 es una solucion. En el resto
de la solucion, consideraremos el caso donde f no es una funcion constante.
Sea t R y sustituyamos (x, y, z) = (t, 0, 0) y (x, y, z) = (0, t, 0) en la ecuacion
funcional dada. Entonces, obtenemos respectivamente,
f (f (t) + 2f (0)) = f (f (t) f (0)) + f (f (0)) + 2f (0),
f (f (t) + 2f (0)) = f (f (0) f (t)) + f (f (0)) + 2f (0),

de donde se sigue que f (f (t) f (0)) = f (f (0) f (t)) se satisface para todo t R.
Supongamos ahora que para algun par u1 , u2 , se cumple f (u1 ) = f (u2 ). Entonces,
sustituyendo (x, y, z) = (s, 0, u1 ) y (x, y, z) = (s, 0, u2 ) en la ecuacion funcional
dada y comparando las identidades resultantes, podemos concluir facilmente que
f (su1 ) = f (su2 )

(3)

para todo s R. Ya que f no es constante, existe s0 tal que f (s0 ) f (0) 6= 0. Si


hacemos u1 = f (s0 ) f (0) y u2 = u1 , entonces f (u1 ) = f (u2 ). Luego, segun (3)
tenemos que
f (su1 ) = f (su2 ) = f (su1 )
para todo s R. Como u1 6= 0, concluimos que f (x) = f (x) para todo x R.
Ahora, si f (u) = f (0) para algun u 6= 0, entonces de (3) tenemos que
f (su) = f (s0) = f (0)
para todo s, lo cual implica que f es una funcion constante, que es contrario a nuestra
suposicion. Por lo tanto, f (s) 6= f (0) si s 6= 0.
Demostraremos que si f (x) = f (y), entonces x = y o x = y. Supongamos por el
contrario que f (x0 ) = f (y0 ) para algun par de numeros distintos de cero x0 , y0 con
x0 6= y0 y x0 6= y0 . Puesto que f (y0 ) = f (y0 ), podemos suponer, reemplazando
y0 por y0 si es necesario, que x0 y y0 tienen el mismo signo. En vista de (3), tenemos
que f (sx0 ) = f (sy0 ) para todo s, y por lo tanto, existe r > 0, r 6= 1, tal que
f (x) = f (rx)

XXII Olimpiada de la Cuenca del Pacfico

39

para todo x (ya que existen numeros reales a y b tales que ax0 = x = by0 y r = ab).
Reemplazando x por rx y y por ry en la ecuacion funcional dada, obtenemos,
f (f (rx) + f (ry) + f (z)) = f (f (rx) f (ry)) + f (2r 2 xy + f (z)) + 2f (r(x y)z),

(4)

y reemplazando x por r x en la ecuacion funcional, obtenemos,


f (f (r 2 x) + f (y) + f (z)) = f (f (r 2 x) f (y)) + f (2r 2 xy + f (z)) + 2f ((r 2 x y)z).

(5)

Ya que f (rx) = f (x) para todo x R, tenemos que todos los terminos correspondientes excepto el u ltimo, en el lado derecho de las identidades (4) y (5) son iguales, y
por lo tanto la igualdad
f (r(x y)z) = f ((r2 x y)z)

(6)

se debe satisfacer para cualesquiera x, y, z R. Para un par arbitrario y fijo u, v R,


vr 2 u
sustituyamos (x, y, z) = ( rvu
2 1 , r 2 1 , 1) en la identidad (6). Entonces, obtenemos
f (v) = f (ru) = f (u), ya que x y = u y r2 x y = v. Pero esto implica que la
funcion f es constante, lo que contradice nuestra suposicion. Concluimos entonces que
si f (x) = f (y), entonces x = y o x = y.
Sustituyendo z = 0 en la ecuacion funcional dada, obtenemos
f (f (x) + f (y) + f (0)) =

f (f (x) f (y)) + f (2xy + f (0)) + 2f (0).

Intercambiando y por y en esta u ltima identidad, y usando el hecho de que f (y) =


f (y), tenemos que todos los terminos excepto el segundo en el lado derecho de la
identidad anterior son los mismos. Luego, concluimos que f (2xy +f (0)) = f (2xy +
f (0)), de donde se sigue que,
2xy + f (0) = 2xy + f (0) o bien 2xy + f (0) = 2xy f (0)
para todo x, y R. En el primer caso, se sigue que 4xy = 0, lo cual es imposible si
xy 6= 0. Por lo tanto, el segundo caso debe satisfacerse y obtenemos que f (0) = 0.
Por u ltimo, demostraremos que si f satisface la ecuacion funcional dada y f no es
constante, entonces f (x) = x2 . Hagamos x = y en la ecuacion funcional. Entonces,
ya que f (0) = 0, tenemos que
f (2f (x) + f (z)) = f (2x2 + f (z)),
de donde se sigue que alguna de las igualdades
2f (x) + f (z) = 2x2 + f (z) o 2f (x) + f (z) = 2x2 f (z)
se debe satisfacer para todos los valores de x y z. Supongamos que existe x0 tal que
f (x0 ) 6= x20 . Entonces, de la segunda alternativa se sigue que f (z) = f (x0 )x20 para
todo z, lo que significa que f debe ser constante, que es contrario a nuestra suposicion.
Por lo tanto, la primera alternativa anterior se debe satisfacer, y as tenemos que f (x) =
x2 para todo x, como queramos demostrar.
Es facil verificar que f (x) = x2 satisface la ecuacion funcional dada. Por lo tanto,
concluimos que f (x) = 0 y f (x) = x2 son las u nicas funciones que satisfacen el
problema.

40

Olimpiadas internacionales

American Mathematics Competition (AMC)


En el mes de marzo se pidio al comite de la olimpiada de Estados Unidos, el examen
de la primera fase que aplican a nivel nacional. Dicho examen consta de dos niveles,
el AMC 10 y el AMC 12, y en cada nivel los concursantes tienen 75 minutos para resolverlo. Los estudiantes mexicanos que en ese momento eran parte de la preseleccion
nacional para la Olimpiada Centroamericana y del Caribe, presentaron el examen AMC
10, y los estudiantes que en ese momento eran parte de la preseleccion nacional para
las Olimpiadas Iberoamericana e Internacional, presentaron el AMC 12. Los ganadores del equipo mexicano fueron: Diego Alonso Roque Montoya (Nuevo Leon) quien
obtuvo el primer lugar con 109.5 puntos en el AMC 10; Flavio Hernandez Gonzalez
(Aguascalientes), quien obtuvo segundo lugar con 105 puntos en el AMC 12; Juan Carlos Ortiz Rhoton (Jalisco), quien obtuvo el tercer lugar con 103.5 puntos en el AMC
12. Diego y Juan Carlos obtuvieron un reconocimiento especial por ser menores de 15
anos y obtener mas de 90 puntos en el examen.
A continuacion presentamos los examenes con sus soluciones del concurso AMC de
este ano.

AMC 10A
Problema 1. La parte superior de la biblioteca de Mara tiene cinco libros con los siguientes anchos, en centmetros: 6, 12 , 1, 2.5 y 10. Cual es el ancho promedio de los
libros, en centmetros?
(a) 1

(b) 2

(c) 3

(d) 4

(e) 5

Solucion. La respuesta es (d).


El promedio de los cinco valores es,
20
6 + 0.5 + 1 + 2.5 + 10
=
= 4.
5
5

Ancho

Problema 2. Cuatro cuadrados identicos y un rectangulo son organizados para formar


un cuadrado mas grande como se muestra. Cual es la proporcion entre el largo y el
ancho del rectangulo?

Largo

(a)

5
4

(b)

4
3

(c)

3
2

(d) 2

(e) 3

Olimpiadas internacionales

41

Solucion. La respuesta es (b).


Denotemos por s la medida del lado del cuadrado pequeno. Entonces el largo del
rectangulo mide 4s y el ancho mide 4s s = 3s. Luego, la proporcion entre el largo y
4
el ancho del rectangulo es 4s
3s = 3 .
Problema 3. Tyrone tena 97 canicas y Eric tena 11 canicas. Tyrone le dio algunas de
sus canicas a Eric de tal manera que Tyrone termino con el doble de canicas que Eric.
Cuantas canicas le dio Tyrone a Eric?
(a) 3

(b) 13

(c) 18

(d) 25

(e) 29

Solucion. La respuesta es (d).


Denotemos por x al numero de canicas que Tyrone le dio a Eric. Entonces 97 x =
2(11 + x), de donde x = 25. Por lo tanto, Tyrone le dio a Eric 25 canicas.
Problema 4. La lectura de un libro que se va a grabar en discos compactos dura 412
minutos. Cada disco puede tener hasta 56 minutos de lectura. Asuma que se usan el
menor numero posible de discos y que cada disco contiene la misma cantidad de lectura. Cuantos minutos de lectura contendra cada disco?
(a) 50.2

(b) 51.5

(c) 52.4

(d) 53.8

(e) 55.2

Solucion. La respuesta es (b).


a 8 discos. Por lo tanto, cada disco conComo 7 < 412
56 < 8, la lectura necesitar
tendra 412
=
51.5
minutos
de
lectura.
8
Problema 5. La longitud de una circunferencia es 24 y su a rea es k. Cual es el
valor de k?
(a) 6

(b) 12

(c) 24

(d) 36

(e) 144

Solucion. La respuesta es (e).


Como la longitud de la circunferencia de radio r es 2r = 24, entonces r = 12.
Luego, el a rea es r2 = 144, y por lo tanto k = 144.
Problema 6. Se define la operacion (x, y) para numeros positivos x y y como
1
(x, y) = x .
y
Cual es el resultado de (2, (2, 2))?
(a)

2
3

(b) 1

Solucion. La respuesta es (c).


Observemos que (2, 2) = 2

(c)
1
2

4
3

(d)

= 32 . Luego,

3
4
2
(2, (2, 2)) = 2,
=2 = .
2
3
3

5
3

(e) 2

42

Olimpiadas internacionales

Problema 7. Crystal trota diariamente el mismo recorrido. Ella comienza su trote dirigiendose hacia el norte una milla. Luego trota al noreste por una milla, luego al sureste
por una milla. El u ltimo tramo de su trote la lleva en lnea recta de regreso a donde ella
comenzo. Cuan larga, en millas, es esta u ltima porcion de su trote?
(a) 1

(b)

(c)

(d) 2

(e) 2 2

Solucion. La respuesta es (c).


Cuando Crystal trota una milla al noreste y una milla al sureste, la distancia que recorrio al norte y al sur es la
misma, por lo que, utilizando el teorema de Pitagoras,
sabemos que en total recorrio 2 millas al este.

2
2

2
2

Luego, justo antes deiniciar el u ltimo tramo de su recorrido, esta a una milla al norte
del
punto inicial
y a 2 millas al este. Por lo tanto, la u ltima porcion de su recorrido es
de 1 + 2 = 3 millas.
Problema 8. Tony trabaja 2 horas al da y se le paga $0.50 por hora por cada ano completo de su edad. Durante un periodo de seis meses Tony trabajo 50 das y gano $630.
Que edad tena Tony al final del periodo de seis meses?
(a) 9

(b) 11

(c) 12

(d) 13

(e) 14

Solucion. La respuesta es (d).


Tony trabajo por 2 50 = 100 horas. Su ganancia promedio por hora durante este
periodo es de $630
100 = $6.30. Por lo tanto, su promedio de edad durante este periodo fue
$6.30
de $0.50 = 12.6 anos. Luego, al final del periodo de seis meses tena 13 anos.
Problema 9. Un palndrome, tal como 83438, es un numero que permanece igual cuando sus dgitos son puestos en orden inverso. Los numeros x y x + 32 son palndromes
de tres y cuatro dgitos, respectivamente. Cual es la suma de los dgitos de x?
(a) 20

(b) 21

(c) 22

(d) 23

(e) 24

Solucion. La respuesta es (e).


Escribamos a x + 32 de la forma CDDC. Como x tiene tres dgitos, entonces 1000 <

Olimpiadas internacionales

43

x + 32 < 1032, de donde C = 1 y D = 0. Por lo tanto, x = 1001 32 = 969 y la


suma de los dgitos de x es 9 + 6 + 9 = 24.
Problema 10. Marvin cumplio anos el martes 27 de mayo en el ano bisiesto 2008. En
que ano sera la proxima vez que caiga su cumpleanos en un da sabado?
(a) 2011

(b) 2010

(c) 2013

(d) 2015

(e) 2017

Solucion. La respuesta es (e).


Un ano no bisiesto tiene 365 das. Como 365 = 52 7 + 1, tenemos que en un ano no
bisiesto hay 52 semanas y 1 da. Como el 2008 fue un ano bisiesto y el 27 de mayo fue
martes, que es despues del 29 de febrero, tenemos que en el 2009 Marvin cumplio anos
un miercoles, y cumplira anos un jueves en el 2010 y un viernes en el 2011. Ahora
bien, como el 2012 es bisiesto, ese ano su cumpleanos sera en domingo, en el 2013
sera en lunes, en el 2014 sera en martes, en el 2015 sera en miercoles, en el 2016
sera en viernes (pues es otro ano bisiesto) y en el 2017 sera en sabado.
Problema 11. La longitud del intervalo de soluciones de la desigualdad a 2x+3 b
es 10. Cual es el valor de b a?
(a) 6

(b) 10

(c) 15

(d) 20

(e) 30

Solucion. La respuesta es (d).


Resolviendo la desigualdad tenemos que,
b3
a3
x
.
2
2
Luego, si

b3
2

a3
2

= 10, entonces b a = 20.

Problema 12. Logan esta construyendo un modelo a escala de su pueblo. La torre del
tanque de agua tiene 40 metros de alto, y la parte superior es una esfera que contiene
100, 000 litros de agua. La torre miniatura de Logan contiene 0.1 litros. Cuan alta, en
metros, debera hacer Logan su torre?
(a) 0.04

(b)

0.4

(c) 0.4

(d)

(e) 4

Solucion. La respuesta es (c).


La escala del volumen del modelo
de Logan es 0.1 : 100, 000 = 1 : 1, 000, 000.
Entonces, la escala lineal es 1 : 3 1, 000, 000 = 1 : 100. Luego, la torre miniatura debe
40
medir 100
= 0.4 metros de alto.
Problema 13. Angelina manejo con una velocidad promedio de 80 km/h y luego hizo
una parada de 20 minutos para cargar gasolina. Despues de la parada, manejo con una
velocidad promedio de 100 km/h. En total recorrio 250 km con un tiempo total de
viaje de 3 horas incluyendo la parada. Cual ecuacion podra ser usada para hallar el
tiempo t en horas en el que Angelina manejo antes de su parada?

44

Olimpiadas internacionales


(a) 80t + 100 38 t = 250
(e) 80 83 t + 100t = 250

(b) 80t = 250

(c) 100t = 250

(d) 90t = 250

Solucion. La respuesta es (a).


Angelina manejo 80t kilometros antes de detenerse. Despues de detenerse, manejo durante (3 31 t) horas a una velocidad promedio de 100 km/h, de modo que en ese
tiempo recorrio 100( 38 t) kilometros. Por lo tanto, 80t + 100( 38 t) = 250 y t = 65 .

Problema 14. En el triangulo ABC se tiene que AB = 2 AC. Sean D y E puntos


sobre AB y BC, respectivamente tales que BAE = ACD. Sea F la interseccion
de los segmentos AE y CD, y suponga que el triangulo CF E es equilatero. Cuanto
mide el a ngulo ACB?
(a) 60

(b)75

(c) 90

(d) 105

(e) 120

Solucion. La respuesta es (c).


Sea x = BAE = ACD = ACF . Como el triangulo CF E es equilatero, tenemos que CF A = 120 , de donde,
F AC = 180 120 ACF = 60 x.
Entonces,
BAC = BAE + F AC = x + (60 x) = 60 .
A
D
F

Como AB = 2 AC, entonces el triangulo BAC tiene a ngulos de 30 , 60 y 90 . Por


lo tanto, ACB = 90 .
Problema 15. En un pantano magico hay dos especies de anfibios parlantes: sapos,
que siempre dicen la verdad, y ranas, quienes siempre mienten. Cuatro anfibios, Brian,
Chris, LeRoy y Mike viven juntos en este pantano y cada uno dice lo siguiente:
Brian:
Chris:
LeRoy:
Mike:

Mike y yo somos de especies diferentes.


LeRoy es una rana.
Chris es una rana.
De los cuatro de nosotros, al menos dos son sapos.

Cuantos de los anfibios son ranas?


(a) 0

(b) 1

(c) 2

(d) 3

(e) 4

Olimpiadas internacionales

45

Solucion. La respuesta es (d).


LeRoy y Chris no pueden ser ambos ranas, pues los dos diran la verdad y las ranas
mienten. Tampoco pueden ser ambos sapos, pues estaran diciendo mentiras y los sapos
dicen la verdad. Entonces, uno de ellos es sapo y el otro rana.
Si Brian fuera sapo, entonces Mike sera rana, luego habra al menos 2 sapos y la
afirmacion de Mike sera verdadera, pero como e l es rana, miente. Luego, Brian es
rana y lo que dice es mentira, entonces Mike tambien es rana. Por lo tanto, hay 3 ranas:
Brian, Mike, y LeRoy o Chris.
Problema 16. Las longitudes de los lados del triangulo ABC son numeros enteros y
ninguno de sus a ngulos mide 0 . Sea D un punto sobre el lado AC tal que BD es
bisectriz del a ngulo ABC, AD = 3, y DC = 8. Cual es el menor valor que el
permetro de dicho triangulo puede tener?
(a) 30

(b) 33

(c) 35

(d) 36

(e) 37

Solucion. La respuesta es (b).


Por el teorema de la bisectriz, tenemos que 8 BA = 3 BC. Entonces, la longitud de
BA debe ser multiplo de 3.
A

D
8

16

Si BA = 3, el triangulo es degenerado, ya que sus lados mediran 3, 8 y 11. Si BA = 6,


entonces BC = 16 y el permetro del triangulo ABC es 6 + 16 + 11 = 33.
Problema 17. Las aristas de un cubo solido tienen 3 pulgadas de longitud. Se hace
un agujero cuadrado de 2 pulgadas por 2 pulgadas en el centro de cada cara del cubo.
Las aristas de cada corte son paralelas a las aristas del cubo, y cada agujero atraviesa
totalmente el cubo. Cual es el volumen, en pulgadas cubicas, del solido resultante?
(a) 7

(b) 8

(c) 10

(d) 12

(e) 15

Solucion. La respuesta es (a).


El volumen del cubo solido es de 27 pulgadas cubicas. El primer agujero le quita al
volumen total 2 2 3 = 12 pulgadas cubicas. Cada uno de los 4 agujeros restantes,
le quita al volumen total 2 2 0.5 = 2 pulgadas cubicas. Por lo tanto, el volumen
del solido resultante es 27 12 4(2) = 7 pulgadas cubicas.
Problema 18. Bernardo elige aleatoriamente 3 numeros distintos del conjunto
{1, 2, 3, 4, 5, 6, 7, 8, 9}

46

Olimpiadas internacionales

y los ordena en orden descendente para formar un numero de 3 dgitos. Silvia elige
aleatoriamente 3 numeros distintos del conjunto
{1, 2, 3, 4, 5, 6, 7, 8}
y tambien los ordena en orden descendente para formar un numero de 3 dgitos. Cual
es la probabilidad que el numero de Bernardo sea mayor que el numero de Silvia?
(a)

47
12

(b)

37
56

(c)

2
3

(d)

49
72

(e)

39
56

Solucion. La respuesta es (b).


La probabilidad de que Bernardo elija un 9 es 93 = 31 . En ese caso, su numero de tres
dgitos iniciara con 9 y sera mayor que el numero de tres dgitos de Silvia.
Si Bernardo no elige un 9, entonces Bernardo y Silvia formaran el mismo numero con
1
.
probabilidad 18 = 56
(3)
Si ellos no forman el mismo numero, entonces el numero de Bernardo sera mayor la
mitad de las veces. Luego, la probabilidad es

1 2 1
37
1
111
+
=
.
1
=
3 3 2
56
168
56
Problema 19. Los lados del hexagono equiangular ABCDEF tienen longitudes AB =
CD = EF = 1 y BC = DE = F A = r. El a rea del triangulo ACE es el 70 % del
a rea del hexagono. Cual es la suma de todos los valores que puede tener r?
(a)

4 3
3

(b)

10
3

(c) 4

(d)

17
4

(e) 6

Solucion. La respuesta es (e).


Observemos que los triangulos ABC, CDE y EF A son congruentes. Entonces, el
triangulo ACE es equilatero. Sea X la interseccion de las rectas AB y EF , y definamos Y y Z de manera similar, como se muestra en la figura.
Z
E

Como ABCDEF es equiangular, tenemos que XAF = AF X = 60 , luego


el triangulo XAF es equilatero. Sea H el punto medio de XF . Por el teorema de

Olimpiadas internacionales

47

Pitagoras, tenemos que


2 
2
r
3
+
r
+ 1 = r2 + r + 1.
AE = AH + HE =
2
2
2

Entonces, el a rea del triangulo ACE es

3
3 2
2
AE =
(r + r + 1).
4
4

Ahora bien, el a rea del hexagono ABCDEF es igual a,

3
3 2
2
2
(XY Z) (XAF ) (Y CB) (ZED) =
((2r + 1) 3r ) =
(r + 4r + 1),
4
4
donde (XY Z) denota el a rea del triangulo XY Z. Como (ACE) =
se sigue que
r2 + r + 1
3
3 2 18
r r+
10
10
10
r2 6r + 1
r

7
10 (ABCDEF ),

7 2
(r + 4r + 1)
10

= 0
= 0

= 3 2 2.

Por lo tanto, la suma de todos los valores posibles de r es 6.


Problema 20. Una mosca atrapada dentro de una caja cubica con arista de longitud 1
metro decide aliviar su aburrimiento visitando cada esquina de la caja. Comenzara y
terminara en la misma esquina y visitara cada una de las otras esquinas exactamente
una vez. Para ir de una esquina a cualquier otra esquina, lo hara volando o caminando
en el interior del cubo siempre en lnea recta. Cual es la longitud maxima posible, en
metros, de su recorrido?

(a) 4 + 4 2 (b) 2 + 4 2 + 2 3 (c) 2 + 3 2 + 3 3 (d) 4 2 + 4 3(e) 3 2 + 5 3

Solucion. La respuesta es (d).


Cada uno de los 8 segmentos de recta de la ruta de la mosca es una arista, ladiagonal

de una cara o una diagonal interior del cubo. Estos tres segmentos miden 1, 2 y 3,
respectivamente. Como cada vertice del cubo es visitado
una sola
vez, la longitud de
los dos segmentos que se unen en un vertice es a
lo mas 2 + 3. Entonces la suma
de las longitudes de los 8 segmentos es a lo mas 4 2 + 4 3. El maximo se obtiene si
la mosca hace el recorrido: A G B H C E D F A.

48

Olimpiadas internacionales

F
D

Problema 21. El polinomio x3 ax2 + bx 2010 tiene tres races enteras positivas.
Cual es el menor valor que puede tener a?
(a) 78

(b) 88

(c) 98

(d) 108

(e) 118

Solucion. La respuesta es (a).


Sean r, s y t las races del polinomio con 0 < r s t. Entonces, x3 ax2 + bx
2010 = (x r)(x s)(x t). Luego, rst = 2010 = 2 3 5 67 y r + s + t = a.
Si t = 67, entonces rs = 30 y r + s es mnimo con r = 5 y s = 6. En este caso,
a = 67 + 5 + 6 = 78. Si t 6= 67, entonces a > t 2 67 = 134. Por lo tanto, el menor
valor para a es 78.
Problema 22. Se eligen ocho puntos en una circunferencia y se trazan cuerdas conectando cada par de puntos. No hay tres cuerdas que se intersecten en un mismo punto en
el interior de la circunferencia. Cuantos triangulos con todos sus vertices en el interior
de la circunferencia son formados?
(a) 28

(b) 56

(c) 70

(d) 84

(e) 140

Solucion. La respuesta es (a).


Tres cuerdas forman un triangulo si y solo si se intersectan dos a dos en el interior
de la circunferencia. Dos cuerdas se intersectan en el interior de la circunferencia si y
solo si los puntos finales de las cuerdas tienen un orden alternado en la circunferencia.
Por lo tanto, si los puntos A, B, C, D, E y F estan ordenados en la circunferencia,
entonces solo las cuerdas AD, BE y CF se intersectan dos a dos en el interior de la
circunferencia.
Luego, cualquier conjunto de 6 puntos determinan un u nico triangulo,

y hay 86 = 28 de tales triangulos.
Problema 23. Cada una de 2010 cajas alineadas contiene una sola canica roja, y para
1 k 2010, la caja en la posicion k-esima contiene tambien k canicas blancas.
Isabella comienza en la primera caja y extrae sucesivamente en orden una sola canica
aleatoriamente de cada caja. Se detiene cuando extrae por primera vez una canica roja.
Sea P (n) la probabilidad de que Isabella se detenga despues de extraer exactamente n

Olimpiadas internacionales

49

canicas. Cual es el menor valor de n para el cual P (n) <


(a) 45

(b) 63

(c) 64

1
2010 ?

(d) 201

(e) 1005

Solucion. La respuesta es (a).


Si Isabella alcanza la k-esima caja, la probabilidad de sacar una canica blanca de
k
ah sera k+1
. Para n 2 la probabilidad de extraer una canica blanca de cada una
de las primeras n 1 cajas es,
n1
1
1 2 3

= .
2 3 4
n
n
Entonces, la probabilidad de que saque la primera canica roja de la n-esima caja es
1
1
. La condicion P (n) < 2010
es equivalente a la desigualdad
P (n) = n(n+1)

n2 + n 2010 > 0,

de donde se sigue que n > 12 (1 + 8041) y (2n + 1)2 > 8041. El menor entero
positivo impar cuyo cuadrado es mayor que 8041 es 91, y el correspondiente valor de
n es 45.
Problema 24. Sea n el numero formado por los dos u ltimos dgitos diferentes de cero
de 90!. A que es igual n?
(a) 12

(b) 32

(c) 48

(d) 52

(e) 68

Solucion. La respuesta es (a).


En 90! hay 18 multiplos de 5, 3 multiplos de 25 = 52 y ningun multiplo de 125 = 53 .
Tambien hay 45 multiplos de 2 en 90!. Entonces, 90! = 1021 N donde N es un entero
que no es divisible entre 10. Tenemos que N n (mod 100) con 0 < n 99, y n es
un multiplo de 4.
Sea 90! = AB donde A tiene a los factores que son primos relativos con 5 y B tiene a
los factores divisibles entre 5. Observando que
4
Y

j=1

(5k + j) 5k(1 + 2 + 3 + 4) + 1 2 3 4 24 (mod 25),

tenemos que
A

= (1 2 3 4) (6 7 8 9) (86 87 88 89)

2418 (1)18 1 (mod 25).


Analogamente,

B = (5 10 15 20) (30 35 40 45) (55 60 65 70) (80 85 90) (25 50 75)


y de aqu,
B
521

(1 2 3 4) (6 7 8 9) (11 12 13 14) (16 17 18) (1 2 3)

243 (9) (8) (7) 6 (1)3 1 1 (mod 25).

50

Olimpiadas internacionales

Finalmente, 221 = 2 (210 )2 = 2 (1024)2 2 (1)2 2 (mod 25), de modo que


13 221 13 2 1 (mod 25). Entonces,
N

90!
B
= 13 A 21 13 1 (1) (mod 25)
521
5
13 12 (mod 25).

(13 221 )N = 13

Por lo tanto, n = 12, 37, 62 o 87, y como n es multiplo de 4, se sigue que n = 12.
Problema 25. Jim comienza con un entero positivo n y crea una sucesion de numeros.
Cada termino sucesivo es obtenido sustrayendo el mayor numero entero cuadrado perfecto que es menor o igual que el termino anterior, hasta que obtenga cero. Por ejemplo,
si Jim comienza con n = 55, entonces su sucesion contiene 5 numeros:
55, 55 72 = 6, 6 22 = 2, 2 12 = 1, 1 12 = 0.
Sea N el menor numero entero para el cual la sucesion de Jim tiene 8 numeros. Cual
es el dgito en las unidades de N ?
(a) 1

(b) 3

(c) 5

(d) 7

(e) 9

Solucion. La respuesta es (b).


Sea (a1 , a2 , . . . , a8 ) la sucesion. Para j > 1, aj1 = aj + m2 para algun m tal
que aj < (m + 1)2 m2 = 2m + 1. Para minimizar el valor de a1 , construyamos
la sucesion al reves y escojamos el menor valor posible de m para cada j tal que
2 j 8. Los terminos en sentido contrario son: a8 = 0, a7 = 1, a6 = 1 + 12 = 2,
a5 = 2 + 12 = 3, a4 = 3 + 22 = 7, a3 = 7 + 42 = 23, a2 = 23 + 122 = 167, y
N = a1 = 167 + 842 = 7223 cuyo dgito de las unidades es 3.

AMC 12A
Problema 1. Cual es el valor de (20 (2010 201)) + (2010 (201 20))?
(a) 4020

(b) 0

(c) 40

(d) 401

(e) 4020

Solucion. La respuesta es (c).


Distribuyendo los signos negativos tenemos que

=
=

(20 (2010 201)) + (2010 (201 20))


20 2010 + 201 + 2010 201 + 20

40.

Problema 2. Un transbordador lleva turistas a una isla cada hora comenzando a las 10
AM hasta su u ltimo viaje, que comienza a las 3 PM. Un da el capitan de la embarcacion nota que en el viaje de las 10 AM haba 100 turistas en el transbordador, y que
en cada viaje sucesivo, el numero de turistas fue uno menor que en el viaje anterior.

Olimpiadas internacionales

51

Cuantos turistas llevo el transbordador a la isla ese da?


(a) 585

(b) 594

(c) 672

(d) 679

(e) 694

Solucion. La respuesta es (a).


El transbordador hace 6 viajes a la isla. El numero de turistas que llevo fue

=
=
=

100 + (100 1) + (100 2) + (100 3) + (100 4) + (100 5)


6(100) (1 + 2 + 3 + 4 + 5)
600 15
585.

Problema 3. El rectangulo ABCD, mostrado a continuacion, comparte el 50 % de su


a rea con el cuadrado EF GH. El cuadrado EF GH comparte el 20 % de su a rea con el
AB
?
rectangulo ABCD. Cual es el valor de AD
E

(a) 4

(b) 5

(c) 6

(d) 8

(e) 10

Solucion. La respuesta es (e).


Sea s la longitud del lado del cuadrado. Ya que la mitad del a rea del rectangulo es del
cuadrado, tenemos que 12 AB = s. Tambien, 51 del a rea del cuadrado es de la region
AB
sombreada, de modo que s = 5 AD. Por lo tanto, 21 AB = 5 AD, de donde AD
= 10.
Problema 4. Si x < 0, cual de las siguientes opciones debe ser positiva?
(a)

(b) x2

x
|x|

(c) 2x

(d) x1

(e)

Solucion. La respuesta es (d).


Observemos que x1 = x1 . Si x es negativo, el valor en (d) es positivo. Para ver que
las otras opciones no necesariamente son positivas, consideremos el numero x = 1.
Luego,
(a)

1
|1|

= 1,

(b) (1)2 = 1,

(c) 21 = 21 ,

(e)

3
1 = 1.

Problema 5. Despues de cumplirse la mitad de un torneo de arquera de 100 tiros,


Chelsea lo va liderando por 50 puntos. Para cada tiro en el centro del blanco se obtienen 10 puntos, con otros puntajes posibles de 8, 4, 2 y 0. Chelsea obtiene al menos 4
puntos en cada tiro. Si en los siguientes n tiros Chelsea da en el centro del blanco, ella

52

Olimpiadas internacionales

garantizara su victoria. Cual es el menor valor que puede tener n?


(a) 38

(b) 40

(c) 42

(d) 44

(e) 46

Solucion. La respuesta es (c).


El arquero en segundo lugar podra obtener un maximo de 50 10 = 500 puntos con
los u ltimos tiros. Por lo tanto, Chelsea necesita mas de 500 50 = 450 puntos para
garantizar su victoria. Si en los siguientes n tiros Chelsea obtiene 10n puntos, en sus
u ltimos 50 n tiros obtendra al menos 4(50 n) puntos. Para garantizar la victoria se
debe tener que
10n + 4(50 n) >
6n + 200 >
n

>

450,
450,
2
41 .
3

Por lo tanto, Chelsea necesita la menos 42 tiros al centro del blanco para garantizar su
victoria.
Solucion alternativa. Si Chelsea no le da al centro del blanco, la maxima diferencia
de puntos que sus oponentes podran obtener por tiro sera 10 4 = 6. Chelsea debe
hacer suficientes tiros al centro del blanco para evitar que sus oponentes obtengan una
diferencia de 50 puntos. Como 8 6 < 50 < 9 6, lo mas que Chelsea puede lograr en
tiros que no dan al centro del blanco es 8 puntos, dejando 50 8 = 42 tiros al centro
del blanco necesarios para garantizar su victoria.
Problema 6. Un palndrome, tal como 83438, es un numero que permanece igual cuando sus dgitos son puestos en orden inverso. Los numeros x y x + 32 son palndromes
de tres y cuatro dgitos, respectivamente. Cual es la suma de los dgitos de x?
(a) 20

(b) 21

(c) 22

(d) 23

(e) 24

Solucion. La respuesta es (e).


Vease la solucion del problema 9 del AMC 10.
Problema 7. Logan esta construyendo un modelo a escala de su pueblo. La torre del
tanque de agua tiene 40 metros de alto, y la parte superior es una esfera que contiene
100, 000 litros de agua. La torre miniatura de Logan contiene 0.1 litros. Cuan alta, en
metros, debera hacer Logan su torre?
(a) 0.04

(b)

0.4

(c) 0.4

(d)

(e) 4

Solucion. La respuesta es (c).


Vease la solucion del problema 12 del AMC 10.
Problema 8. En el triangulo ABC se tiene que AB = 2 AC. Sean D y E puntos
sobre AB y BC, respectivamente, tales que BAE = ACD. Sea F la interseccion

Olimpiadas internacionales

53

de los segmentos AE y CD, y suponga que el triangulo CF E es equilatero. Cuanto


mide el a ngulo ACB?
(a) 60

(b) 75

(c) 90

(d) 105

(e) 120

Solucion. La respuesta es (c).


Vease la solucion del problema 14 del AMC 10.
Problema 9. Las aristas de un cubo solido tienen 3 pulgadas de longitud. Se hace un
agujero cuadrado de 2 pulgadas por 2 pulgadas en el centro de cada cara del cubo. Las
aristas de cada corte son paralelas a las aristas del cubo, y cada agujero atraviesa totalmente el cubo. Cual es el volumen del solido resultante?
(a) 7

(b) 8

(c) 10

(d) 12

(e) 15

Solucion. La respuesta es (a).


Vease la solucion del problema 17 del AMC 10.
Problema 10. Los primeros cuatro terminos de una sucesion aritmetica son p, 9, 3p q
y 3p + q. Cual es el termino 2010-esimo de esta sucesion?
(a) 8041

(b) 8043

(c) 8045

(d) 8047

(e) 8049

Solucion. La respuesta es (a).


Los terminos consecutivos en una sucesion aritmetica tienen diferencia comun d. Luego, (3p+q)(3pq) = 2q = d. Como el segundo termino es igual a p+d, tenemos que
p+d = 9, y el tercer termino es igual a p+2d, de modo que p+2d = 3pq. Resolviendo el sistema formado por estas tres ecuaciones, obtenemos que p = 5, q = 2 y d = 4.
Por lo tanto, el termino 2010-esimo de la sucesion es p+2009d = 5+2009(4) = 8041.
Problema 11. La solucion de la ecuacion 7x+7 = 8x puede ser expresada en la forma
x = logb 77 . A que es igual b?
(a)

7
15

(b)

7
8

(c)

8
7

(d)

15
8

(e)

15
7

Solucion. La respuesta es (c).


Si x = logb 77 , entonces bx = 77 . Luego,
(7b)x = 7x bx = 7x+7 = 8x .
Como x > 0, se sigue que 7b = 8, de donde b = 78 .
Solucion alternativa. Tomando logaritmos de ambos lados de la ecuacion 7x+7 = 8x ,
tenemos que (x + 7) log 7 = x log 8. Resolviendo esta ecuacion, tenemos x log 7 +
77
7 log 7 = x log 8, de donde x(log 8 log 7) = 7 log 7 y de aqu x = log
. Como
log 8
7

x = logb 77 , se sigue de la formula del cambio de base para logaritmos que b = 78 .

54

Olimpiadas internacionales

Problema 12. En un pantano magico hay dos especies de anfibios parlantes: sapos,
que siempre dicen la verdad, y ranas, quienes siempre mienten. Cuatro anfibios, Brian,
Chris, LeRoy y Mike viven juntos en este pantano y cada uno dice lo siguiente:
Brian:
Chris:
LeRoy:
Mike:

Mike y yo somos de especies diferentes.


LeRoy es una rana.
Chris es una rana.
De los cuatro de nosotros, al menos dos son sapos.

Cuantos de estos anfibios son ranas?


(a) 0

(b) 1

(c) 2

(d) 3

(e) 4

Solucion. La respuesta es (d).


Vease la solucion del problema 15 del AMC 10.
Problema 13. Para cuantos valores enteros de k resulta que los graficos de x2 + y 2 =
k 2 y xy = k no se intersectan?
(a) 0

(b) 1

(c) 2

(d) 4

(e) 8

Solucion. La respuesta es (c).


Cuando k = 0, los graficos de x2 + y 2 = 0 y xy = 0 consisten del u nico punto (0, 0)
y la union de las rectas x = 0 y y = 0, respectivamente, de modo que los dos graficos
se intersectan. Cuando k 6= 0, el grafico de x2 + y 2 = k 2 es un crculo de radio k con
centro en el origen, y el grafico de xy = k es una hiperbola equil
atera con
centro en el
k,

k) si k > 0 o
origen. Los vertices
de
la
hip
e
rbola,
localizados
en
los
puntos
(

en los puntos ( k, k) si k < 0, son los puntos mas cercanos sobre el grafico
al origen. Si |k| 2, entonces
 2  2
|k| +
|k| = 2|k| k 2 ,
de modo que los graficos se intersectan. Si |k| = 1, entonces

 2  2
|k| +
|k| = 2 > 1 = k 2 ,

y los graficos no se intersectan. Por lo tanto, los graficos no se intersectan para k = 1


o k = 1.
Problema 14. Las longitudes de los lados del triangulo ABC son numeros enteros y
ninguno de sus a ngulos mide 0 . Sea D un punto sobre el lado AC tal que BD es
bisectriz del a ngulo ABC, AD = 3, y DC = 8. Cual es el menor valor que el
permetro de dicho triangulo puede tener?
(a) 30

(b) 33

(c) 35

(d) 36

(e) 37

Olimpiadas internacionales

55

Solucion. La respuesta es (b).


Vease la solucion del problema 16 del AMC 10.
Problema 15. Se altera una moneda de tal manera que la probabilidad de que caiga en
cara es menor que 21 y cuando se arroja la moneda cuatro veces, la probabilidad de que
se obtenga un numero igual de caras y sellos es 61 . Cual es la probabilidad de que la
moneda caiga en cara?

6 6 6+2
153
(a) 6
(b)
(c) 21
(d) 36 3
(e) 31
12
2
2
Solucion. La respuesta es (d).
Sea p la probabilidad buscada. Si la moneda searroja cuatro veces, la probabilidad de
que caiga dos veces cara y dos veces sello es 42 p2 (1 p)2 = 16 , y como 0 p 1, se

sigue que p(1 p) = 61 . Resolviendo para p obtenemos p = 16 (3 3). Como p < 12 ,

concluimos que p = 61 (3 3).


Problema 16. Bernardo elige aleatoriamente 3 numeros distintos del conjunto
{1, 2, 3, 4, 5, 6, 7, 8, 9}
y los ordena en orden descendente para formar un numero de 3 dgitos. Silvia elige
aleatoriamente 3 numeros distintos del conjunto
{1, 2, 3, 4, 5, 6, 7, 8}
y tambien los ordena en orden descendente para formar un numero de 3 dgitos. Cual
es la probabilidad que el numero de Bernardo sea mayor que el numero de Silvia?
(a)

47
72

(b)

37
56

(c)

2
3

(d)

49
72

(e)

39
56

Solucion. La respuesta es (b).


Vease la solucion del problema 18 del AMC 10.
Problema 17. Los lados del hexagono equiangular ABCDEF tienen longitudes AB =
CD = EF = 1 y BC = DE = F A = r. El a rea del triangulo ACE es el 70 % del
a rea del hexagono. Cual es la suma de todos los valores que r puede tener?
(a)

4 3
3

(b)

10
3

(c) 4

(d)

17
4

(e) 6

Solucion. La respuesta es (e).


Vease la solucion del problema 19 del AMC 10.
Problema 18. Un camino que consta de 16 pasos va de (4, 4) a (4, 4) de tal forma
que con cada paso se incrementa en 1 o bien la coordenada en x o bien la coordenada
en y. Cuantos de estos caminos son tales que permanecen en el exterior o en el borde
del cuadrado 2 x 2, 2 y 2 en todos los pasos?

56
(a) 92

Olimpiadas internacionales
(b) 144

(c) 1, 568

(d) 1, 698

(e) 12, 800

Solucion. La respuesta es (d).


Cada uno de tales caminos intersecta a la recta y = x en exactamente uno de los
puntos (4, 4), (3, 3) o (2, 2). Para j = 0, 1 y 2, el numero de caminos
desde

el punto (4, 4) hacia alguno de los puntos ((4 j), (4 j)) es 8j , y el numero
de caminos hacia el punto (4, 4) desde alguno de los puntos ((4 j), (4 j)) es el
mismo. Por lo tanto, el numero de caminos que cumplen lo requerido es igual a
2 2 2 !
8
8
8
2
+
+
= 2(12 + 82 + 282 ) = 1, 698.
0
1
2
Problema 19. Cada una de 2010 cajas alineadas contiene una sola canica roja, y para
1 k 2010, la caja en la posicion k-esima contiene tambien k canicas blancas.
Isabella comienza en la primera caja y extrae sucesivamente en orden una sola canica
aleatoriamente de cada caja. Ella se detiene cuando ella extrae por primera vez una
canica roja. Sea P (n) la probabilidad de que Isabella se detenga despues de extraer
1
?
exactamente n canicas. Cual es el menor valor de n para el cual P (n) < 2010
(a) 45

(b) 63

(c) 64

(d) 201

(e) 1005

Solucion. La respuesta es (a).


Vease la solucion del problema 23 del AMC 10.
Problema 20. Las progresiones aritmeticas (an ) y (bn ) tienen terminos enteros con
a1 = b1 = 1 < a2 b2 y an bn = 2010 para algun n. Cual es el mayor valor que n
puede tener?
(a) 2

(b) 3

(c) 8

(d) 288

(e) 2009

Solucion. La respuesta es (c).


Ya que an = 1 + (n 1)d1 y bn = 1 + (n 1)d2 para algunos enteros d1 y d2 , se sigue
que n 1 es un factor del maximo comun divisor de an 1 y bn 1. Luego, la pareja
ordenada (an , bn ) debe ser alguna de las parejas (2, 1005), (3, 670), (5, 402), (6, 335),
(10, 201), (15, 134) o (30, 67). Para cada una de estas parejas, excepto la sexta pareja,
los numeros an 1 y bn 1 son primos relativos, de modo que n = 2. Para la sexta
pareja, el maximo comun divisor de 151 y 1341 es 7. Como 7 es un numero primo,
tenemos que n = 8, y las progresiones definidas por an = 2n 1 y bn = 19n 18
satisfacen las condiciones del problema.
Problema 21. El grafico de y = x6 10x5 + 29x4 4x3 + ax2 yace en la parte del
plano por encima de la lnea recta y = bx + c excepto en tres valores de x, donde el
grafico y la lnea recta se intersectan. Cual es el mayor de estos tres valores?
(a) 4

(b) 5

(c) 6

(d) 7

(e) 8

Olimpiadas internacionales

57

Solucion. La respuesta es (a).


Supongamos que los tres puntos de interseccion tienen coordenada en x a los numeros
p, q y r, y sea f (x) = x6 10x5 + 29x4 4x3 + ax2 bx c. Entonces, f (p) =
f (q) = f (r) = 0, y f (x) 0 para todo x, de modo que
f (x) = ((x p)(x q)(x r))2 = (x3 Ax2 + Bx C)2 ,
donde A = p + q + r, B = pq + qr + rp y C = pqr.
El coeficiente de x5 es 10 = 2A, de donde A = 5. El coeficiente de x4 es 29 =
A2 + 2B = 25 + 2B, de donde B = 2. El coeficiente de x3 es 4 = 2C 2AB =
2C 20, de donde C = 8. Luego, f (x) = (x3 5x2 + 2x + 8)2 . Como las sumas
de los coeficientes de las potencias pares e impares de x son iguales, tenemos que 1
es una raz de f (x). Factorizando obtenemos que
f (x) = ((x + 1)(x2 6x + 8))2 = ((x + 1)(x 2)(x 4))2 ,
y el mayor valor de las tres races es 4.
Problema 22. Cual es el valor mnimo de f (x) = |x 1| + |2x 1| + |3x 1| +
+ |119x 1|?
(a) 49

(b) 50

(c) 51

(d) 52

Solucion. La respuesta es (a).


Observemos que

(x 1) (2x 1) (119x 1)

(x 1) (2x 1) ((m 1)x 1)+


f (x) =
+(mx 1) + + (119x 1)

(x 1) + (2x 1) + + (119x 1)

(e) 53

1
,
si x 119
1
1
si m x m1
2 m 119,
si x 1.

1
, un rayo
El grafico de f (x) consiste de un rayo con pendiente negativa para x 119
1
con pendiente positiva para x 1, y para 119 x 1 una sucesion de segmentos
de recta cuyas pendientes aumentan conforme x aumenta. El valor mnimo de f (x)
ocurre en el punto final (punto de la derecha) del intervalo que esta mas a la derecha
en el cual el grafico tiene una pendiente menor o igual que cero. La pendiente en el
1
1
intervalo [ m
, m1
] es
119
X

k=m

m1
X
k=1

k=

119
X

k=1

k2

m1
X
k=1

k = 7140 (m 1)(m).

La desigualdad 7140 + m m2 0 se satisface en el intervalo [84, 85] y la igualdad


1
1
, 84
] el grafico
se cumple en los puntos finales 84 y 85. Por lo tanto, en el intervalo [ 85
de f (x) tiene pendiente 0 y un valor constante de (84)(1) + (119 84)(1) = 49.
Problema 23. Sea n el numero formado por los dos u ltimos dgitos diferentes de cero
de 90!. A que es igual n?

58
(a) 12

Olimpiadas internacionales
(b) 32

(c) 48

(d) 52

(e) 68

Solucion. La respuesta es (a).


Vease la solucion del problema 24 del AMC 10.
Problema 24. Sea f (x) = log10 (sen(x) sen(2x) sen(3x) sen(8x)). La
interseccion del dominio de f (x) con el intervalo [0, 1] es la union de n intervalos
abiertos disjuntos. Cual es el valor de n?
(a) 2

(b) 12

(c) 18

(d) 22

(e) 36

Solucion. La respuesta es (b).


Sea g(x) = sen(x) sen(2x) sen(3x) sen(8x). El dominio de f (x) es la
union de todos los intervalos en los cuales g(x) > 0. Note que sen(n(kx)) =
(1)k+1 sen(nx), de modo que g(kx) = g(x). Como g( 21 ) = 0, es suficiente considerar los subintervalos de (0, 12 ) en los cuales g(x) > 0. En este intervalo las soluciones
distintas de la ecuacion g(x) = 0 son los numeros nk , donde 2 n 8, 1 k < n2 , y k
y n son primos relativos. Para n = 2, 3, 4, 5, 6, 7 y 8 hay, respectivamente, 0, 1, 1, 2, 1, 3
y 2 valores de k. Luego, hay 1 + 1 + 2 + 1 + 3 + 2 = 10 soluciones de g(x) = 0
en el intervalo (0, 12 ). El signo de g(x) cambia en nk a menos de que un numero par
de factores de g(x) sean cero en nk , es decir, a menos de que haya un numero par de
formas de representar a nk como un numero racional con denominador positivo menor
o igual que 8. Por lo tanto, el signo de g(x) cambia excepto en 41 = 28 y 31 = 26 .
Supongamos que las soluciones de g(x) = 0 en el intervalo (0, 21 ) son x1 , x2 , . . . , x10
en orden creciente, y sean x0 = 0 y x11 = 12 . Es facil verificar que x5 = 14 y x7 = 31 ,
de modo que para 0 j 10, el signo de g(x) cambia en xj excepto para j = 5 y 7.
Como 5 y 7 tienen la misma paridad y g(x) > 0 en (x0 , x1 ), la solucion de g(x) > 0 en
(0, 21 ) consiste de 6 intervalos abiertos disjuntos: (x0 , x1 ), (x2 , x3 ), (x4 , x5 ), (x5 , x6 ),
(x8 , x9 ), (x10 , x11 ). La solucion de g(x) > 0 en ( 12 , 1) tambien consiste de 6 intervalos
abiertos disjuntos, de modo que el numero de intervalos buscado es 12.
Problema 25. Se considera que dos cuadrilateros son iguales si uno puede ser obtenido
del otro por una rotacion y/o una traslacion. Cuantos cuadrilateros convexos cclicos
diferentes hay con lados enteros y permetro igual a 32?
(a) 560

(b) 564

(c) 568

(d) 1498

(e) 2255

Solucion. La respuesta es (c).


Supongamos que un cuadrilatero con lados a b c d y con permetro 32 existe.
Por la desigualdad del triangulo tenemos que a < b + c + d = 32 a, de donde a 15.
Recprocamente, si (a, b, c, d) es una cuarteta de enteros positivos cuya suma es igual
a 32, y cuya maxima entrada es a 15, entonces b + c + d = 32 a 17 > a,

y la desigualdad del triangulo se verifica. Esta


es la u nica condicion que se requiere
para garantizar la existencia de un cuadrilatero convexo con lados de longitudes dadas.
Mas aun, si se especifica el orden cclico de los lados, entonces hay exactamente uno
de tales cuadrilateros cclicos.

XII Olimpiada Centroamericana y del Caribe

59

El problema se reduce a contar todas las cuartetas (a, b, c, d) de enteros positivos con
a + b + c + d = 32 y max{a, b, c, d} 15, donde dos cuartetas se consideran la misma
si generan el mismo cuadrilatero, es decir, si una es una permutacion cclica de la otra.
Por ejemplo, (12, 4, 5, 11) y (5, 11, 12, 4) generan el mismo cuadrilatero.
El numero de cuartetas (a, b, c, d) con a + b + c + d = 32 se pueden contar como
sigue: consideremos 31marcas sobre una lnea que seran llenadas con 28 dgitos 1
y 3 signos +. Hay 31
formas de escoger las posiciones de los signos +, y cada
3
eleccion esta en correspondencia uno a uno con la cuarteta (a , b , c , d ), donde cada
entrada indica el numero de dgitos 1 que hay entre signos + consecutivos. Haciendo
(a, b, c, d) = (1, 1, 1, 1) + (a , b , c , d ), obtenemos precisamente todas las cuartetas
donde a, b, c, d 1 y a + b + c + d = 32. Para contar aquellas donde la maxima
entrada es mayor o igual que 16, consideremos 13 dgitos 1 y 3 signos +. Hay 16
3
cuartetas (a , b , c , d ) donde a , b , c , d 0 y a + b + c + d = 13. Hay 4 formas
de elegir una de las coordenadas, digamos a , para ser la mnima. Entonces, la cuarteta
(a, b, c, d) = (16,1, 1, 1) + (a , b , c , d ) satisface nuestras condiciones. Luego, hay
exactamente 4 16
3 cuartetas (a, b, c, d) donde a, b, c, d 1, a + b + c + d = 32, y
max(a, b, c, d) 16. En consecuencia hay


31
16
4
(7)
3
3
cuartetas (a, b, c, d) donde a, b, c, d son mayores o iguales que 1, a + b + c + d = 32 y
max(a, b, c, d) 15.
Si la cuarteta (a, b, c, d) tiene todos sus valores distintos, entonces tiene exactamente 4
permutaciones cclicas. Lo mismo ocurre si solo dos de sus valores son iguales entre
s, o tres de sus valores son iguales entre s y el valor restante es distinto. Si la cuarteta (a, b, c, d) tiene dos pares de valores iguales entre s ordenados como (a, a, b, b),
entonces dicha cuarteta tiene 4 permutaciones cclicas, pero si estan ordenados como
(a, b, a, b) entonces tiene 2 permutaciones cclicas. Finalmente, si todos los valores son
iguales entonces solo hay una permutacion cclica.
Hay exactamente 7 cuartetas de la forma (a, b, a, b) con a < b y a + b = 16, y hay
exactamente una cuarteta (a, a, a, a) = (8, 8, 8, 8) con cuatro valores iguales. Si sumamos a la ecuacion (7) dos veces el numero de cuartetas de la forma (a, b, a, b) mas 3
veces el numero de cuartetas de la forma (a, a, a, a), garantizamos que todas las clases
de equivalencia bajo permutaciones cclicas son contadas exactamente 4 veces. Por lo
tanto, el numero de cuadrilateros cclicos buscado es,

31
16
1
1
(31 5 29 32 5 14 + 17)
4
+ 14 + 3
=
4
4
3
3
1
(5 451 + 7) = 568.
=
4

XII Olimpiada Centroamericana y del Caribe


Del 24 de mayo al 1 de junio de 2010, se celebro en Mayaguez, Puerto Rico, la XII
Olimpiada Matematica de Centroamerica y el Caribe. Mexico ocupo el primer lugar,

60

XII Olimpiada Centroamericana y del Caribe

con 108 puntos, de entre los 14 pases que participaron. Barbados y Cuba no pudieron
asistir, y Jamaica, Trinidad y Tobago, e Islas Vrgenes Americanas participaron por
primera vez. En total participaron 41 estudiantes. La delegacion mexicana estuvo integrada por los alumnos: Diego Alonso Roque Montoya (Nuevo Leon), Julio Cesar Daz
Calderon (Oaxaca) y Fernando Josafath Anorve Lopez (Nuevo Leon).
De las 3 medallas de oro, 7 de plata y 11 de bronce que se entregaron en el certamen,
Diego obtuvo medalla de oro, y Julio Cesar y Fernando medalla de plata. Ademas
se otorgo el reconocimiento de Solucion Creativa a Diego Alonso por su solucion
del problema 5, reconocimiento que solo se ha otorgado 2 veces, hasta ahora, en una
Olimpiada Matematica de Centroamerica y el Caribe.
A continuacion presentamos los problemas con sus soluciones de la XII Olimpiada
Centroamericana y del Caribe. Los alumnos tuvieron dos sesiones de cuatro horas y
media cada una para resolverlos. Agradecemos de una manera muy especial a Leonardo Martnez por habernos proporcionado el archivo TEX con las soluciones de los
alumnos participantes.

Problema 1. Si S(n) denota la suma de los dgitos de un numero natural n, encuentre


todas las soluciones de
n(S(n) 1) = 2010
mostrando que son las u nicas.
Solucion de Julio Cesar Daz Calderon. Sea n = an 10n +an1 10n1 + +a1 101 +
a0 donde las ai s son sus dgitos. As, S(n) = a0 + a1 + + an .
Supongamos que n es tal que n(S(n) 1) = 2010. Como n es natural, S(n) es entero,
y como queremos que n(S(n) 1) = 2010, necesitamos S(n) 1 6= 0, as que
S(n) 6= 1, de modo que S(n) 2. Como 2010 = 2 3 5 67, tenemos S(n) 2
y queremos n(S(n) 1) = 2010, entonces n 3 5 67 = 1005, de modo que n
tiene a lo mas 4 dgitos, y por tanto n = a3 103 + a2 102 + a1 10 + a0 , de modo que
S(n) = a3 + a2 + a1 + a0 4 9 = 36, y as S(n) 1 35.
Pero 67 divide a 2010 = n(S(n) 1) y 67 es primo, as que 67 divide a n o 67 divide
a (S(n) 1). Si 67 divide a (S(n) 1), entonces 67 S(n) 1, pero no se puede
pues 0 < S(n) 35 < 67. As que 67 divide a n. Pero 2010 = n(S(n) 1), con
n 1005, as que n solo tiene 7 valores posibles: 67, 67 2, 67 3, 67 5, 67 2 3,
67 2 5 y 67 5 3. Tenemos entonces la siguiente tabla.
n
67
134
201
335
402
670
1005

S(n)
13
8
3
11
6
13
6

S(n) 1
12
7
2
10
5
12
5

XII Olimpiada Centroamericana y del Caribe

61

Al fijarnos en n(S(n) 1), descartamos los primeros 3 ya que su producto no puede


ser 2010 pues ningun factor tiene 5. El cuarto caso y los u ltimos dos los descartamos
pues en los productos correspondientes se repite el factor 2 o el factor 5 al menos 2
veces, y 2010 solo tiene un factor 2 y un factor 5. Finalmente, vemos que n = 402 y
S(n) 1 = 5 tienen producto igual a 2010. As, e ste es el u nico caso posible.
Problema 2. Dado el triangulo ABC, sean L, M y N los puntos medios de BC, CA y
AB, respectivamente. Se traza una tangente al circuncrculo del triangulo ABC en A,
siendo P y Q las intersecciones respectivas de las rectas LM y LN con dicha tangente.
Demuestre que CP es paralela a BQ.

Solucion de Fernando Josafath Anorve


Lopez. Supongamos que BAC = ,
ABC = , ACB = . Ahora, como L y M son puntos medios de BC y CA,
respectivamente, tenemos que LM ||AB. Por lo tanto, CLM = . Como P Q es
tangente al circuncrculo de ABC, CAP es semi-inscrito a la circunferencia, luego
CAP = CBA = . Entonces sabemos que CLP = CAP = . Es decir,
P ALC es cclico.

L
C

Como L y N son puntos medios de CB y AB, tenemos que LN ||AC, luego BLN =
BCA = . Sabemos que como P Q es tangente entonces BAQ es semi-inscrito
y BAQ = BCA = , y sabemos que BLQ = , de modo que BAQ =
BLQ = y entonces BQAL es cclico.
Ahora, usando que BLAQ y CLAP son cclicos, tomemos AQB = . Luego,
ALB = 180 , de donde CLA = y por lo tanto CP A = 180 .
Como AQB = y CP A = 180 , se sigue que BQ||CP .
Problema 3. Un jugador coloca una ficha en una casilla de un tablero de mn, dividido
en casillas de tamano 11. El jugador mueve la ficha de acuerdo a las siguientes reglas:

62

XII Olimpiada Centroamericana y del Caribe


En cada movimiento, el jugador cambia la ficha de la casilla en la que e sta se
encuentra a una de las casillas que tienen un lado en comun con ella.
El jugador no puede ubicar la ficha en una casilla que e sta ha ocupado previamente.
Dos movimientos consecutivos no pueden tener la misma direccion.

El juego termina cuando el jugador no puede mover la ficha. Determine todos los valores de m y n para los cuales el jugador puede colocar la ficha en alguna casilla tal que
e sta haya ocupado todas las casillas al terminar el juego.
Solucion de Diego Alonso Roque Montoya. Sin perder generalidad supongamos que
n m. Consideramos los siguientes casos:
a) m = 1, n 2;

cumple.

...
A
B
Para ir de A a B tienes que avanzar
b) m = 1, n > 2;
mnimo 2, y como no puedes moverte a los lados (ya que no existen) entonces esos
2 movimientos son consecutivos hacia una misma direccion, lo cual sera ilegal.
...
c) m = 2, n 2;

cumple.

d) m, n > 2. En este caso no hay 2 esquinas colindantes. Coloreamos el tablero como


tablero de ajedrez (de blanco y negro). Hay un numero par (0, 2 o 4) de esquinas
blancas y un numero par de esquinas negras (4, 2 o 0), as que si las esquinas son
A, B, C, D (no necesariamente en orden cclico), podemos tomar parejas (A, B) y
(C, D) tales que A y B son del mismo color x y C y D son del mismo color y. Los
movimientos tras 2 turnos son
o
o rotaciones de estos,
entonces cada 2 turnos vuelves al mismo color. Si la primera casilla del color x que
pisas no es ni A ni B, entonces cuando entres a A (la primera de estas) te atoraras
y no podras entrar a B. De manera similar con C y D. Entonces, la primera casilla
del color x que pisas es A o B y la de y es C o D, pero si x = y eso es imposible
y si x 6= y entonces hay 2 esquinas colindantes, contradiciendo el caso en el que
estamos. Esto demuestra que con m, n > 2 es imposible.

Problema 4. Se desea embaldosar un patio cuadrado de lado n entero positivo. Se dispone de dos tipos de baldosas: cuadradas de 5 5 y rectangulares de 1 3. Determine
los valores de n para los cuales es posible hacerlo.
Nota: El patio debe quedar completamente cubierto sin que las baldosas se sobrepongan.

Solucion de Fernando Josafath Anorve


Lopez. Demostraremos que si tenemos un
rectangulo y uno de sus lados es multiplo de 3, puede llenarse con baldosas de 3 1.

XII Olimpiada Centroamericana y del Caribe

63

Este resultado se usara posteriormente.


Si 3 divide al numero de filas, me tomo cada columna y la lleno de una por una. Ahora,
como tiene 3x cuadritos cada columna, puedo separarlos en grupos de 3 y llenar cada
uno, llenando as la columna. Analogamente si el numero de columnas es multiplo de
3, solo volteamos la figura y ya.
Para la siguiente parte del problema primero necesito cubrir los casos donde n 10.
n = 1 y n = 2 no se puede.
n = 3, n = 6, n = 9 s, pues 3|n.
n = 4 no se puede, pues no cabe una baldosa de 55 y no se puede con baldosas
de 3 1 pues 3 no divide a 16 = 4 4.
n = 5 s, pues lo cubrimos con una de 5 5.
n = 7 s, pues se llena enmedio con un 5 5 y las orillas se pueden cubrir con 8
de 3 1.
n = 8 s, siguiendo el procedimiento de la siguiente parte de la solucion.
Para los valores de n mayores o iguales que 10:
Si n 0 (mod 3) entonces ya vimos que s se puede.
Si n 2 (mod 3) entonces ponemos una baldosa de 5 5 en la esquina superior
izquierda, dejando rectangulos de 5(n5) y (n5)n. Como n 2 (mod 3),
tenemos n 5 0 (mod 3), y entonces 3|n 5, de modo que los 2 rectangulos
de ah pueden ser llenados. Notemos que el caso n = 8 entra aqu, completando
los casos n 10.
Si n 1 (mod 3) entonces ponemos 4 baldosas de 5 5 cubriendo la esquina
superior izquierda (pues n 10). Como n 1 (mod 3), tenemos n 10
0 (mod 3), as 3|n 10. As que los rectangulos que quedan pueden llenarse
porque tienen medidas 10 (n 10) y (n 10) n.
Problema 5. Sean p, q y r numeros racionales distintos de cero tales que
p
p
p
3
pq 2 + 3 qr2 + 3 rp2

es un numero racional distinto de cero. Pruebe que

1
1
1
p
+ p
+ p
3
3
3
2
2
pq
qr
rp2

tambien es un numero racional.

64

XII Olimpiada Centroamericana y del Caribe

Solucion de Diego Alonso Roque Montoya. 5 Comenzamos demostrando un lema:

3
entonces3 b tambi
Lema. Si a, b, c y a 3 b + c b2son racionales,
en es racional.

3 2
3 2
3
3
b
+
c
b
.
Entonces,
c
b
+
a
b
t = 0, de modo
Demostraci
o
n.
Tomemos
t
=
a

que 3 b es solucion
de la ecuacion cuadratica cx2 + ax t = 0, as que por la formula

2
general, 3 b = a 2ca +4ct .

b = ( b) =

a2 + 4ct
2c

a3 3a2 a2 + 4ct + 3a(a2 + 4ct) a2 + 4ct(a2 + 4ct)


=
8c3

Despejando a2 + 4ct yusando que a, b, c, t son racionales, obtenemos que a2 + 4ct


es racional, y por tanto 3 b tambien. Con e sto queda demostrado el lema.
Ahora, tomemos A =

p
p
p
3
1
pq 2 + 3 qr2 + 3 rp2 , B =
3

pq2

1
+
3

qr 2

1
+
,a =
3
2
rp

pq + qr + rp, b = pqr y c = p + q + r. Por hipotesisdel problema,


A, a, b, c son

3 2
3
racionales. As, A3 = (pq 2 + qr2 + rp2 + 6pqr) +
a
b
+
c
b
tambi
en y como

3 2
3
(pq 2 + qr2 + rp2 + 6pqr) es racional,
entonces
a
b
+
c
b
tambi
e
n
es
racional.

Entonces, el Lema nos dice que u = 3 b = 3 pqr es racional.


Basta verificar que AB = 3 + up + up + uq + uq + ur + ur , y como u, p, q, r son racionales,
entonces AB lo es. As, como A 6= 0 es racional, tenemos que B tambien lo es.
Problema 6. Sean y 1 dos circunferencias tangentes internamente en A, de centros
O y O1 , y radios r y r1 (r > r1 ), respectivamente. Sea B el punto diametralmente
opuesto a A en la circunferencia , y C un punto en tal que BC es tangente a 1 en
P . Sea A el punto medio de BC. Si se cumple que O1 A es paralela a AP , determine
la razon rr1 .
Solucion de Julio Cesar Daz Calderon. Como las circunferencias son tangentes en
A, los puntos B, O, O1 y A son colineales. Como A es punto medio de BC y O es
punto medio de BA (pues es un diametro), por el teorema de Thales tenemos A O||CA,
y as BA O = BCA = 90 . Luego, por el criterio de semejanza AA, tenemos que
los triangulos BA O y BCA son semejantes.
Tenemos que BP O1 = 90 por ser BC tangente a 1 en P . As, A O||P O1 , de
modo que por el teorema de Thales,
BO
A O
=
BO1
P O1

(8)

5 Esta soluci
on recibio Premio Especial por ser una solucion creativa. El premio fue otorgado por la forma
de abordar el problema, la cual no se le ocurrio a nadie previamente, y por la demostracion elemental del
lema (existen otras demostraciones que utilizan resultados de la Teora de Campos).

XII Olimpiada Centroamericana y del Caribe

65

Por hipotesis, O1 A ||AP , as, A O1 O = P AO1 y A OO1 = P O1 A, de modo


que por el criterio AA, tenemos que los triangulos A OO1 y P O1 A son semejantes.
De aqu tenemos,
A O1
A O
OO1
=
=
O1 A
P O1
PA

(9)

C
P
A

A
b
b

O1

Usando el teorema de Thales con A O1 ||AP , y las ecuaciones (8) y (9) tenemos:
BO1
BO
A O1
A O
=
,
=
=
BA
PA
P O1
BO1

concluyendo que BO1 = BO BA = r 2r = 2r.


Finalmente, ya que BO1 = 2r r1 , tenemos que 2r r1 = 2r, de donde obtenemos
que rr1 = 212 .

66

XII Olimpiada Centroamericana y del Caribe

Una conversacion con


Emilio Lluis Riera
Por Emilio Lluis Puebla

Introduccion
Uno de los matematicos mexicanos de mayor prestigio internacional, es sin duda, Emilio Lluis Riera. Nacio en Espana en 1925, realizo sus estudios de licenciatura, maestra
y doctorado en matematicas en la Universidad Nacional Autonoma de Mexico. Durante
su formacion trabajo con Solomon Lefschetz, uno de los matematicos mas destacados
del siglo XX. Tambien fue discpulo de Pierre Samuel, Aleksandr Kurosh, Igor Shafarevich, Guillermo Torres y Alberto Barajas. Es investigador del Instituto de Matematicas
de la UNAM desde 1951.
Ha publicado mas de veinte textos de Matematicas, que van desde nivel primaria hasta
posgrado. En cuanto a la investigacion matematica, es autor de numerosos artculos
publicados en revistas especializadas de prestigio internacional, siendo muy relevantes
sus aportaciones en Geometra Algebraica, Homolga y Cohomologa de grupos.
Tambien la docencia ha sido una de sus actividades prioritarias. Muchas generaciones
de matematicos han asistido a sus clases y su valiosa participacion en diversos programas de formacion docente ha dejado profundas huellas en varias generaciones de
profesores. En 1990 recibio el Premio Universidad Nacional en el a rea de Docencia en
ciencias exactas y en 1997 fue nombrado Profesor Emerito, el maximo galardon que
ofrece dicha institucion.
A continuacion presentamos una entrevista que Emilio Lluis Puebla, hijo de Lluis Riera y tambien un destacado matematico mexicano, hizo a su padre en exclusiva para la
revista Tzaloa.

68

Una conversacion con Emilio Lluis Riera

La entrevista
A finales de marzo del presente ano recib un correo electronico de Anne Alberro,
solicitandome que escribiera un artculo sobre mi padre. Anne fue durante algunos
anos ayudante de mi papa en la Facultad de Ciencias de la UNAM y su mama Pierrette
Semerena fue mi alumna estrella de tesis en 1986-87. Cre que una forma adecuada
para realizar este artculo sera mediante una entrevista grabada. Agradezco a Anne
que lo haya transcrito palabra por palabra. Esto nos proporciono una base sobre la cual
pudimos editar la siguiente conversacion.
LP.- Emilio Lluis Puebla

LR.- Emilio Lluis Riera

LP- Dinos primero, donde naciste?


LR- Nac en Roquetas. Queda pegadito a Tortosa la cual esta en la desembocadura del
Ebro al Mediterraneo, practicamente entre Cataluna y Valencia.
LP- En Cataluna?
LR- En Cataluna desde luego, pero muy cerca de Valencia.
LP- Ah naciste porque tus papas estaban dando clases en Roquetas?
LR- Los dos fueron maestros en Roquetas. Ellos ganaron plazas por oposicion y les dio
muchsimo gusto de que les dieran trabajo a los dos, de maestros, en el mismo lugar.
LP- Ellos se graduaron en Barcelona, en la Escuela Normal de Maestros.
LR- S, fue en la Escuela Normal de Maestros. Eran los u nicos maestros pues era un
pueblo muy chiquito.
LP- En las escuelas de aquel entonces los alumnos cursaban todos los grados juntos.
Comenzaste a estudiar la escuela en Roquetas o en Barcelona?
LR- La escuela la empece en Roquetas. Simplemente me met a los cursos que daban
mis papas donde vea a los ninos. Estudie igual que ellos. Ademas, cuando ya estaba
mas avanzado en primaria ayudaba a mis papas cuando tenan alumnos que haba que
darles un poco mas de apoyo. Me gustaba darles ayuda. Era muy bonito y muy divertido.
LP- Ah podra decirse que comenzo tu actividad docente.
LR- S, completamente. Haba muchachos mucho mayores que yo. Algunos ya estaban
terminando la primaria, pero que andaban mal, pues a veces en los pueblitos asisten a la
escuela cuando se puede. Primero trabajan el campo y despues van a la escuela. Haba
algunos alumnos bastante mayores que yo y a esos les pona especial atencion porque
me decan mis papas que haba que ensenarles rapidamente porque tenan que terminar
pronto, pues ya estaban grandes.
LP- Entonces tu te inicias propiamente en la docencia desde chiquito.
LR- S, desde chiquito, me gustaba dar clases.
LP- Es as como a menudo los hijos de carpinteros se hacen carpinteros, hijos de maestros...
LR- S, a mi me gustaba mucho ir a la ferretera que quedaba cerca de nuestra casa
en Roquetas, ah vi que los hijos de los ferreteros iban tambien a la ferretera desde
chiquitos.
LP- Por eso muchas veces los hijos de profesionistas de cierta disciplina se encaminan
a esa disciplina.
LR- Y si se aprende desde chico se hace mucho mejor.

Una conversacion con Emilio Lluis Riera

69

LP- Y despues te vas a estudiar a Barcelona.


LR- Mis papas se fueron a Barcelona cuando les dieron los cursos en las escuelas de
esa ciudad. Me fui tambien, claro, y ah continue la primaria y despues la secundaria.
LP- Y tus encuentros con la matematica fueron a traves de tus papas.
LR- S, simplemente lo que se ensena a todos los ninos.
LP- Pero e l tena especial interes en la matematica.
LR- Le gustaba mucho, sobre todo, las demostraciones, la geometra y esas cositas
donde hay que pensar. No era nada mas aprender de memoria las tablas de multiplicar
sino el ensenar a razonar.
LP- Despues, hay un periodo de inestabilidad poltica en Espana y muchos tienen que
salir de Espana. Cuentanos como fue eso, estabas estudiando la secundaria o preparatoria quizas?
LR- No, mucho menos, tena 10 u 11 anos.
LP- En la primaria?
LR- S, finales de la primaria o principios de la secundaria.
LP- Y luego se vieron forzados a salir por cuestiones de peligro.
LR- De peligro, de bombardeos constantes que estaban haciendo en Barcelona, que
te pescaban en la calle, y al da siguiente era igual. Entonces llego un momento que
era demasiado peligroso vivir donde vivamos. Era un piso comun y corriente en un
edificio grande. Estabamos en el tercer o quinto piso, no se. Y era peligroso vivir ah.
Entonces mis papas decidieron irse a otra parte.
LP- Yo me acuerdo que mi abuelo me conto que, estando en una reunion, cayo una
bomba atras de e l y no exploto, quedo ah clavada, hizo un agujero.
LR- Conocimos a muchos que desaparecieron durante los bombardeos. Estabamos expuestos a bombazos y por eso se decidio salir.
LP- Me contaron que haba un barco y que en ese barco se fueron con los ninos, toda
una escuela.
LR- No, primero fuimos en tren a Francia, no recuerdo el lugar, y ah tomamos un
barco sovietico pues ellos invitaron para que salieramos de la guerra con los 300 ninos.
LP- Y ese barco a donde los llevo?
LR- A Leningrado, y de ah nos llevaron un poco mas abajo de Moscu, donde hicieron
una escuela precisamente para ninos, para esos ninos espanoles que estabamos ah.
LP- Eran como 300?
LR- S, yo creo que de ese orden.
LP- Y haba mas profesores ademas de tus papas...
LR- S, s, claro. Haba mas profesores.
LP- ...y entonces ah en la URSS tu cursaste la secundaria y la preparatoria...
LR- El u ltimo ano de la primaria y luego la secundaria. La secundaria ya no lo hice en
la Casa de Ninos en espanol. La secundaria ya la hice en ruso en una escuela sovietica.
LP- Y, hay el equivalente a preparatoria?
LR- Secundaria y preparatoria, en total son cinco anos.
LP- Durante este tiempo estalla la segunda guerra mundial...
LR- Estalla la segunda guerra mundial, cuando Hitler invade la URSS yo estaba en la
preparatoria. A las pocas semanas que entro Hitler a la URSS, nos empacaron a todos
los de la Casa de Ninos y nos llevaron a una casa cerca de Saratov, en el Ro Volga.
LP- Y ah cuanto tiempo estuvieron?

70

Una conversacion con Emilio Lluis Riera

LR- Estuvimos durante la guerra, unos cuatro anos.


LP- Y tu ya habas ido a estudiar a Moscu la licenciatura de matematica?
LR- No, eso fue despues. Justito acabando la guerra, me fui cerca de Moscu y de ah iba
a tomar unos cursillos para poder entrar a la Universidad, cuyo ingreso era a base de
examenes que tenas que aprobar. Para entrar tuvimos que pasar unos examenes pavorosos, pues haba mucha gente que quera entrar. Moscu es la numero uno, entonces
tuvimos que hacer unos examenes realmente difciles, pero con ganas! Tanto Emiliano
Aparicio como yo, lo pasamos muy bien.
LP- Quienes fueron tus maestros?
LR- Shafarievich, entre otros. Haba matematicos de primera: Kuroch, etc.
LP- ...y luego..., viene la posibilidad de venir a Mexico invitados por la hermana de tu
mama que viva en Mexico.
LR- Gracias a eso pudieron dar permiso a mi mama para salir, porque en general en
ese tiempo era muy difcil que alguien saliera de la URSS sin razon de nada, pero en
ese caso como una hermana la solicito, le dieron permiso.
LP- Y entonces, vino toda la familia a Mexico en el ano 1947, me parece, y llegando
aqu, como es que te inscribes a estudiar matematica?
LR- Ah! Ese fue mi problema. Quera estudiar matematica y me decan que fuera a ingeniera. Nadie saba que ya exista una escuela de matematica. Por fin encontre alguien
que me dijo: mira, ve alla y habla con el director, como se llama? ... Napoles Gandara. Entonces fui a verlo. Inmediatamente me explico que ya tenamos en la U.N.A.M.
la carrera de matematico. Me trato muy bien e ingrese a la carrera.
LP- Y entonces pudiste continuar tus estudios que ya habas comenzado en la URSS y
terminaste tu licenciatura. Cuando aparecio Lefschetz?
iba a dar confeLR- Lefschetz aparecio desde que yo estaba en los primeros anos. El
rencias y cursillos y eso me animo muchsimo. Era una persona que saba matematica,
pero, de los primeros del mundo. Hablaba ruso, ingles, espanol. Ruso de nacimiento.
Despues lo llevaron a Italia, luego paso a Francia donde hizo el doctorado. Despues lo
pescaron en E.U., pues era de primer orden.
LP- Lefschetz fue uno de los mas destacados matematicos y cientficos. Recuerdo que
cuando vena cada verano tu trabajabas con e l y te propuso un tema de tesis doctoral.
Te sugirio que fueras a trabajar con Pierre Samuel a Francia.
LR- Me recomendo, que un tema como ese, debe discutirse con matematicos de a de
veras, y que si poda ir a Francia con Pierre Samuel, que saba de esas cosas. Pues,
encantado de la vida sal, con muchos problemas y con muchas necesidades.
LP- En 1954?, mas o menos.
LR- De ese orden.
LP- S, yo me acuerdo que te deje de ver una e poca.
LR- S, s, muy duro estuvo, no tena dinero, muy difcil todo, ademas el problema
no era tan facil y cuando vi que la cosa ya iba bien, me regrese. Trabaje en ClermontFerrand, donde Pierre Samuel era maestro de la Universidad.
LP- Y tu trabajo fue, quizas, uno de los primeros, si no es que el primer artculo publicado por un matematico, mexicano o espanol, en el Annals of Mathematics. Fue un
artculo muy exitoso, muy importante...
LR- ...es que el problema era bonito, era un problema interesante.
LP- Inmersion de Variedades Algebraicas.

Una conversacion con Emilio Lluis Riera

71

LR- S. Si la variedad tiene cierta dimension, cual es la mas pequena dimension donde
se puede meter, dicho de manera informal. Estaba muy bonito el tema y a Pierre Samuel le encanto.
LP- Ese resultado aparece en muchsimos lados, inclusive me acuerdo que en alguna
ocasion vino Robin Hartshorne, que tiene el libro de Geometra Algebraica, y ah viene
alguno de tus resultados.
LR- S, yo conoc bien a Hartshorne.
LP- Me acuerdo que me toco de nino conocer a grandes matematicos. Oscar Zariski,
entre otros.
LR- Zariski, nada menos.
LP- Varias veces vino a Mexico a trabajar aqu contigo y a m me tocaba convivir con
e l en la parte no matematica puesto que yo tena 9 o 10 anos. Tambien vino Pierre
Samuel en 1955 mas o menos. Me acuerdo porque traa a su hijo y se cayo en la fuente
de la casa de Manuel Dublan y se mojo la ropa. Me pediste que trajera algo mo, un
sueter, para que se lo pusiera... Luego, trabajaste en Berkeley.
LR- S, estuve dos semestres invitado por ellos, para hacer matematica, y estaba Tarski,
Sidenberg, Maxwell Rosenlich. Y el otro...
LP- ...el de geometra algebraica?
LR- No, e l haca geometra en general.
LP- Me acuerdo que te acompanamos un semestre, en 1962, donde conviviste y trabajaste con matematicos de Berkeley. Despues regresamos a Mexico y seguiste trabajando
en Geometra Algebraica y Cohomologa de Grupos. Durante ese periodo de los anos
60 me acuerdo que estabas trabajando mucho tambien en la escritura de textos.
LR- Tambien, eso era muy importante para maestros.
LP- Exactamente, y tambien en los anos 70.
LR- De los de primaria, escrib todos los anos, los 6 de primaria.
LP- Te acuerdas como fue la cuestion, te los pidio la Secretara de Educacion Publica?, o como estuvo?
LR- No, no fue por pedido, fue por gusto, por hacerlos, y se interesaron algunas editoriales.
LP- Y tambien hiciste los de secundaria?
LR- De secundaria tambien, una vez que estas escribiendo, pues ya escribes todo...
hasta de facultad tambien tengo uno, no?
LP- Varios. Me acuerdo de los de secundaria. Tenas un equipo con 6 maestros, que
trabajaban en eso, Cardenas y tu, por supuesto.
LR- Matematicos e ramos Cardenas y yo, nada mas y los demas eran maestros que
haba que ensenarles la parte matematica ya superior, y ellos se pegaban a escribir tambien.
LP- Muy entusiastas, Villar era uno de ellos.
LR- S, s, ah esta el libro.
LP- Colaboraste muchsimos anos en la creacion de textos de matematica para la Secretara...
LR- Y mas.
LP- Claro, es una de las labores mas importantes que hay que hacer.
LR- A m siempre me parecio muy importante la escritura de textos, mas para los
maestros que para los alumnos. Lo que necesitaban los maestros es tener, por escrito,

72

Una conversacion con Emilio Lluis Riera

lo que deban ensenar. Les di mil conferencias, ...muchsimas.


LP- S, durante toda esa e poca ibas a muchas ciudades del pas, ibas a Monterrey donde
fundaste junto con otros la facultad de matematicas, a Yucatan y tambien a Queretaro.
LR- S mucho a Queretaro... y a Toluca tambien mucho, a Morelia...
LP- Es decir, estuviste apoyando a muchsimas universidades donde se ensenaba matematica, para tener lo que ahora tenemos.
LR- Fu muchsimo a Monterrey y crecio rapidamente.
LP- A Sonora tambien...
LR- S, s,
LP- Es decir, fuiste a muchos lugares de la Republica. Yo me acuerdo que a cada rato
te bamos a dejar a la estacion del tren, viajabas toda la noche para amanecer alla y dar
clase, en fines de semana muchas veces.
LR- S, en fines de semana, daba clases sabado y domingo y regresaba el domingo en
la noche.
LP- Eso en la e poca de los 60, y despues ibas manejando a muchos lugares.
LR- Yucatan, muchas veces iba en coche a Yucatan.
LP- Uno de tus cumpleanos lo pasaste en Merida dando conferencias y participando
en un congreso.
LR- Tambien a Oaxaca fui mucho.
LP- Hay un famoso Symposium del 56, que vino Atiyah, Hilton, Kan, Cartan, Eilenberg, Chern y Hurevich quien perdio la vida en una de las piramides pues al no ver bien
se cayo. Vino Jean Pierre Serre, tambien estuvo Lefschetz. Tu colaboraste en la organizacion de ese famossimo Symposium que hizo historia en la matematica mundial.
LR- Es que eso jalo a la gente que le gustaba trabajar en matematica y crecio el grupo.
LP- Tambien recuerdo que dirigiste muchsimas tesis de todos tipos.
LR- Ah claro, s,...es que no haba gente para hacerlo, tena que meterse uno siempre.
LP- Ibas en la calle caminando o a cualquier tienda y todo mundo te conoca, porque de
alguna u otra manera haban sido tus alumnos o asistentes a alguna de tus conferencias.
Colaboraste muchsimos anos con Humberto Cardenas.
LR- S, s, muchsimos, y ahorita seguiramos, lo que pasa es que esta en Morelia, si
no, estaramos escribiendo artculos.
LP- Pero has seguido trabajando con e l todo el tiempo y han hecho un grupo de trabajo
que los ha llevado por distintas a reas de la matematica y han publicado muchsimos
artculos.
LR- S, porque Cardenas nunca estuvo en Geometra Algebraica, estaba en otras cosas.
LP- Tambien me acuerdo que vino Steenrod a Mexico.
LR- Y estaba muy contento con nosotros, trabajaba muy bien.
LP- Y a traves de todos estos anos has visto crecer el ambiente matematico y no nada
mas en la Ciudad de Mexico, sino en todo el pas.
LR- S, hemos visitado todo el pas, muchas veces.
LP- Del 60 al 62 fuiste presidente de la Sociedad Matematica Mexicana...
LR- S, s, me pescaron, y ni modo de decir que no.
LP- Y te toco organizar alguno de los congresos de la Sociedad, antes no era cada
ano...
LR- No era obligatorio, pero mas o menos.
LP- Cada presidencia es de dos anos de duracion. La Sociedad Matematica Mexicana

Una conversacion con Emilio Lluis Riera

73

obviamente crecio enormemente.


LR- Bastante, porque fuimos a todas partes. No nos quedabamos quietos.
LP- En Saltillo hay una placa conmemorativa de la Sociedad Matematica Mexicana
porque ah se fundo. Cuando a m me toco ser presidente, 40 anos despues de ti, en el
2001, se coloco tambien una placa alusiva a la Sociedad Matematica Mexicana, en el
mismo Ateneo Fuente de Saltillo...
LR- ...y esos congresos sirvieron mucho para entusiasmar a los maestros de matematicas de todos los niveles, sobre todo los de primaria, secundaria y desde luego los mas
avanzados. Eso ha servido mucho para la mejor formacion de los maestros.
LP- Entusiasmar a la gente... hay gente que no conoce nada de matematica y en cuanto
oye hablar de matematica se entusiasma...
LR- ...y aprende que las matematicas no son una serie de formulas y tablas sino que es
el arte de razonar correctamente, nada mas, y todo lo demas, sale sobrando.
LP- Muchas personas ven a la matematica desde distintos puntos de vista, algunos ven
a la matematica como una herramienta para hacer algo...
LR- ...la utilidad...
LP- la utilidad de..., pero los matematicos la vemos como una disciplina de estudio por
s misma.
LR- Formacion mental...
LP- ...y la cual es una bella arte...
LR- ...s, s una bella arte....
LP- Y en cuanto al numero de matematicos que existen en Mexico, que puedes decir.
LR- Pues me parece que todava somos muy pocos. Cuantos mas matematicos hay,
mas avanzado es el pas. Eso es importantsimo. Una cosa es saberlas y otra cosa es
utilizarlas, y si no la saben, no pueden utilizarla. Para que un pas este muy bien, debe
tener buena matematica en muchas partes.
LP- S, en alguna ocasion le que habra que multiplicar por 16 el numero de matematicos en Mexico...
LR- ...posiblemente...
LP- Por que razon diras que hay que estudiar matematicas?
LR- Por que razon? Por gusto, que les guste, que les guste. En general le gusta pensar bien a la gente, pero da mucho trabajo pensar bien y cuando se dan cuenta que
s pueden, se ponen contentsimos. En cuanto descubren una cosa de matematicas que
la pueden razonar y deducir, les da mucho gusto a los chamacos. Eso es lo que hay que
tratar de hacer: entusiasmar a la gente. Una cosa muy particular de la matematica es
que la matematica es totalmente internacional, muchas de las disciplinas que se estudian van dirigidas a ciertos grupos a ciertos lugares, pero la matematica es totalmente
de todo el mundo.
Y como ya se haca tarde, decidimos parar por el momento para retomar esta conversacion mas adelante.

74

Una conversacion con Emilio Lluis Riera

Informacion Olmpica
A continuacion presentamos las actividades programadas por el comite organizador de
la Olimpiada Mexicana de Matematicas, de octubre a diciembre de 2010.
Del 21 al 27 de noviembre en Ensenada, Baja California
24 Concurso Nacional de la Olimpiada Mexicana de Matematicas.
Del 9 al 19 de diciembre en Cuernavaca, Morelos
Entrenamiento para los seleccionados nacionales.

76

Informacion Olmpica

Apendice
Teorema 1 (Factorizacion en primos) Todo entero n mayor que 1 puede expresarse
como un producto de primos (con, tal vez, solamente un factor).
Ver [5, 7].
Definicion 2 (Congruencias) Dados dos numeros enteros a, b, y un entero positivo m,
decimos que a es congruente con b modulo m, si a b es multiplo de m. En este caso
escribimos a b (mod m).
Ver [9].
teorema de Fermat) Si p es un numero primo y a es un entero
Teorema 3 (Pequeno
primo relativo con p, entonces
ap1 1 (mod p).
Ver [5, 7].
Teorema 4 (Desigualdad media aritmetica - media geometrica) Si x1 , x2 , . . . , xn son
numeros reales positivos, entonces

x1 + x2 + + xn
n x1 x2 xn
n
y la igualdad se cumple si y solo si x1 = x2 = = xn .
Ver [3].
Definicion 5 (Sucesion de Fibonacci) La sucesion de Fibonacci Fn se define como
sigue:
F1 = 1, F2 = 1, y Fn = Fn1 + Fn2 si n > 2.
Ver [4].
Teorema 6 (Teorema del binomio) Si a y b son numeros reales y n es un entero positivo, entonces
n
X
n k nk
(a + b)n =
a b
,
k
k=0

78

Apendice


donde nk =
Ver [6].

n!
k!(nk)! .

Teorema 7 (Formulas de a rea)


1. El a rea de un rectangulo de lados a y b es a b.
2. El a rea de un triangulo es igual a 21 hl, donde l es la medida de un lado y h es la
medida de la altura sobre dicho lado.
3. El a rea de un crculo de radio r es igual a r2 .
Ver [1, 2].
Teorema 8 (Suma de los a ngulos internos de un triangulo) La suma de los a ngulos
internos de un triangulo es 180 .
Ver [1, 2].
Teorema 9 (Desigualdad del triangulo) Dado un triangulo cuyas longitudes de sus
lados son a, b y c, entonces se cumplen las siguientes relaciones
a + b > c, b + c > a, a + c > b.
El recproco tambien es verdadero, es decir, si a, b y c son numeros positivos que
satisfacen las relaciones anteriores, entonces se puede formar un triangulo cuyos lados
tienen longitudes a, b y c.
Ver [2].
Teorema 10 (Teorema de Pitagoras) En un triangulo rectangulo, el cuadrado de la
hipotenusa es igual a la suma de los cuadrados de los catetos.
Ver [1, 2, 8].
Definicion 11 (Congruencia de triangulos) Los triangulos ABC y A B C son congruentes si los a ngulos y los lados del triangulo ABC son iguales a los a ngulos y los
lados del triangulo A B C .
Ver [1, 2].
Criterio 12 (Criterio de congruencia ALA) Un criterio de congruencia de triangulos nos dice que si tenemos dos triangulos con un lado igual y dos a ngulos adyacentes
iguales, entonces son congruentes. A este criterio de congruencia se le conoce como
a ngulo-lado-angulo y lo denotamos como ALA.
Ver [1, 2].
Criterio 13 (Criterio de congruencia LLL) Un criterio de congruencia de triangulos nos dice que si tenemos dos triangulos con sus tres lados correspondientes iguales,
entonces son congruentes. A este criterio de congruencia se le conoce como lado-ladolado y lo denotamos como LLL.
Ver [1, 2].

Apendice

79

Definicion 14 (Semejanza de triangulos) Los triangulos ABC y A B C son semejantes, si sus a ngulos respectivos son iguales, es decir,
ABC = A B C
ACB = A C B
BAC = B A C
y sus lados homologos son proporcionales, esto es
Ver [1, 2].

AB
A B

BC
B C

CA
C A .

Criterio 15 (Criterio de semejanza AA) Si dos pares de a ngulos correspondientes


de los triangulos ABC y A B C son iguales, entonces los triangulos son semejantes.
A este criterio de semejanza le llamamos a ngulo-angulo y lo denotamos como AA.
Ver [1, 2].
Criterio 16 (Criterio de semejanza LAL) Si dos triangulos tienen dos lados homologos proporcionales y el a ngulo entre dichos lados igual, entonces los triangulos son
semejantes. A este criterio de semejanza se le conoce como lado-angulo-lado y lo denotamos como LAL.
Ver [1, 2].
Teorema 17 Si trazamos dos rectas tangentes a una circunferencia desde un mismo
punto P , entonces los segmentos de recta desde P a los puntos de tangencia son iguales
y el centro de la circunferencia yace en la bisectriz del a ngulo entre las rectas.
Ver [2].
Teorema 18 (Medida del a ngulo inscrito) La medida de un a ngulo inscrito en una
circunferencia es igual a la mitad del arco comprendido entre sus lados, es decir, la
mitad del a ngulo central que subtiende el mismo arco.
Ver [1, 2].
Definicion 19 (Cuadrilatero cclico) Un cuadrilatero es cclico si sus cuatro vertices
estan sobre una misma circunferencia.
Ver [2].
Teorema 20 (Cuadrilatero cclico) Un cuadrilatero convexo ABCD es cclico si y
solo si la suma de los a ngulos opuestos es igual a 180, es decir, si y solo si
DAB + BCD = ABC + CDA = 180 .
Ver [2].
Definicion 21 (Figuras en perspectiva) Dos figuras estan en perspectiva si todas las
rectas que unen puntos correspondientes de las dos figuras son concurrentes. El punto
por el cual pasan estas rectas se llama centro de perspectiva.
Ver [2].
Teorema 22 (Teorema de Desargues) Dos triangulos estan en perspectiva si y solo
si los puntos de interseccion de lados correspondientes son colineales.
Ver [2].

80

Apendice

Bibliografa
[1] A. Baldor. Geometra plana y del espacio. Publicaciones Cultural, Mexico, 1999.
[2] R. Bulajich Manfrino, J. A. Gomez Ortega. Geometra. Cuadernos de Olimpiadas
de Matematicas. Instituto de Matematicas de la UNAM, 2002.
[3] R. Bulajich Manfrino, J. A. Gomez Ortega, R. Valdez Delgado. Desigualdades. Cuadernos de Olimpiadas de Matematicas. Instituto de Matematicas de la
UNAM. Tercera edicion, 2007.
[4] R. Grimaldi, Matematicas Discreta y Combinatoria. Addison Wesley Longman,
Pearson. Tercera edicion, 1998.
[5] I. Niven, H. Zuckerman. Introduccion a la Teora de los Numeros. Limusa-Wiley,
Mexico 1972.
[6] M. L. Perez Segu. Combinatoria. Cuadernos de Olimpiadas de Matematicas.
Instituto de Matematicas de la UNAM, 2000.
[7] M. L. Perez Segu. Teora de Numeros. Cuadernos de Olimpiadas de Matematicas.
Instituto de Matematicas de la UNAM, 2003.
[8] A. Rechtman Bulajich. Algunas demostraciones del teorema de Pitagoras. Revista de la Olimpiada Mexicana de Matematicas, Tzaloa No. 1, 2010.
[9] A. Rechtman Bulajich, C.J. Rubio Barrios. Divisibilidad y congruencias. Revista
de la Olimpiada Mexicana de Matematicas, Tzaloa No. 2, 2009.
[10] N. Vilenkin. De cuantas formas? (Combinatoria). Editorial Mir, Moscu 1972.

82

Directorio

Directorio del Comite Organizador de la OMM


Anne Alberro Semerena
Facultad de Ciencias, UAEM
Av. Universidad 1001
62210, Cuernavaca, Morelos
Tel. (777) 3 81 03 80
Fax (777) 3 29 70 40
aalberro@uaem.mx

Gerardo Arizmendi Echegaray


Centro de Investigacion en Matematicas
Callejon Jalisco s/n, Mineral de Valenciana
36240, Guanajuato, Guanajuato
Tel. (473) 7 32 71 55
gerardo@cimat.mx

Ignacio Barradas Bribiesca


Universidad de Guanajuato
L. de Retana #5, Centro
36000, Guanajuato, Guanajuato
Tel. (473) 7 32 00 06 ext 2006
barradas@quijote.ugto.mx

Radmila Bulajich Manfrino


Facultad de Ciencias, UAEM
Av. Universidad 1001
62210, Cuernavaca, Morelos
Tel. (777) 3 29 70 20
Fax (777) 3 29 70 40
bulajich@uaem.mx

Gabriela Campero Arena


Facultad de Ciencias, UNAM
Av. Universidad 3000
04510, Mexico, D.F.
Tel. (55) 56 22 48 67
Fax (55) 56 22 48 66
gabriela@matematicas.unam.mx

Fernando Campos Garca

1a de Angel
Rico 85
AU.H. Vicente Guerrero
09200, Iztapalapa, Distrito Federal
Tel. (55) 34 63 75 43
fermexico89@hotmail.com

Jose Antonio Climent Hernandez


Facultad de Ciencias, UNAM
Av. Universidad 3000
04510, Mexico, D.F.
Tel. (55) 56 24 59 22
Fax (55) 56 22 48 59
jach@fciencias.unam.mx

Jose Alfredo Cobian Campos


Facultad de Ciencias, UNAM
Av. Universidad 3000
04510, Mexico, D.F.
Tel. (55) 56 22 49 25
Fax. (55) 56 22 48 59
cobian@matematicas.unam.mx

84
David Cosso Ruiz
Universidad Autonoma de Ciudad Juarez
Instituto de Ingeniera y Tecnologa
Departamento de Fsica y Matematicas
Av. del Charro 450 Nte.
CP 32310, Cd. Juarez, Chihuahua
Tel. (656) 688 48 87
Fax. (656) 688 48 13
sirio11@gmail.com

Directorio

Luis Cruz Romo


UPIITA, IPN
Av. Instituto Politecnico Nacional 2580
Col. Barrio la Laguna Ticoman
07340, Mexico, D.F.
lucruz@ipn.mx

Marco Antonio Figueroa Ibarra


Facultad de Matematicas
Universidad de Guanajuato
Callejon Jalisco s/n, Mineral de Valencia
36240, Guanajuato, Guanajuato
Tel. (473) 7 32 01 40
marcant@cimat.mx

Jesus
Jeronimo Castro
CIMAT
Apartado Postal 402
36000, Guanajuato, Guanajuato
Tel. (473) 7 32 71 55
Fax (473) 7 32 57 49
jeronimo@cimat.mx

Leonardo Ignacio Martnez Sandoval


Primera Cerrada de Alfalfares 41-2
Rinconada Coapa Primera Sec, Tlalpan
14330, Mexico, D.F.
Tel. (55) 26 52 23 29
ssbmplayer@gmail.com

Carlos Jacob Rubio Barrios


Universidad Autonoma de Yucatan
Periferico norte tablaje 13615
97119, Merida, Yucatan
Tel. (999) 942-3140 al 49
Fax (999) 942-31-40
carlos.rubio@uady.mx
jacob.rubio@gmail.com

Elena Ruiz Velazquez


Altair 12
Col. Lomas de Palmira
62550, Cuernavaca, Morelos
Tel. (777) 320 54 39
Cel. (777) 133 39 83
eleniux@gmail.com
A00375640@itesm.mx
Carmen Sosa Garza
Facultad de Ingeniera, UAQ
Cerro de las Campanas s/n
Queretaro, Queretaro
Tel. (442) 1 92 12 64 ext. 121 o 136
Fax (442) 1 92 12 646
carsg@uaq.mx

Pablo Soberon Bravo


Circuito Interior no. 830
Fracc. La Herradura
62303, Cuernavaca, Morelos
Cel. (777) 134 55 49
bandrak@hotmail.com

Rogelio Valdez Delgado


Facultad de Ciencias, UAEM
Av. Universidad 1001
62210, Cuernavaca, Morelos
Tel. (777) 3 29 70 20
Fax (777) 3 29 70 40
rogelio@matcuer.unam.mx

Directorio

Eduardo Velasco Barreras


Universidad de Sonora
Calle Yucas 16, Vista Bella
83170, Hermosillo, Sonora
Tel. (662) 2 19 10 07
hamsteritokeweb@hotmail.com

85
Hugo Villanueva Mendez
Instituto de Matematicas, UNAM
Cub. 4 de Becarios,
Circuito Exterior, Ciudad Universitaria
Coyoacan 04510,
Mexico, D.F.
Tel (55) 56 22 45 32
vill hugo@hotmail.com
hvillan@matem.unam.mx

86

Directorio
Direccion Postal de la Olimpiada Mexicana de Matematicas:
Cubculo 201, Departamento de Matematicas.
Circuito Exterior, Facultad de Ciencias.
Universidad Nacional Autonoma de Mexico.
Ciudad Universitaria.
Colonia Copilco, C.P. 04510.
Delegacion Coyoacan.
Mexico, Distrito Federal.
Telefono: (55) 5622-4864.
Fax: (55) 5622-5410.
Email: omm@fciencias.unam.mx
Pagina oficial de la Olimpiada Mexicana de Matematicas:
http://www.omm.unam.mx/

Das könnte Ihnen auch gefallen